EMT Final 1

¡Supera tus tareas y exámenes ahora con Quizwiz!

non-rebreather (NRB) masks

"high flow" oxygen flow rate: 10-15 LPM delivers up to 90% oxygen

An infant with a total blood volume of 800 mL would start showing signs of shock when as little as ______ of blood is lost.

100 ml

The recommended dimensions for a helicopter landing zone are: 50 × 50 feet. 75 × 75 feet. 100 × 100 feet. 150 × 150 feet.

100 × 100 feet.

Typically, medivac helicopters fly at speeds between:

130 to 150 miles an hour

Which of the following patients would have the HIGHEST treatment priority at the scene of a mass-casualty incident? 24-year-old man who is unconscious, has snoring respirations, and severe burns 32-year-old woman who is pulseless and apneic with an abdominal evisceration 29-year-old woman who is in full cardiac arrest with massive open chest trauma 32-year-old man with an open head injury, exposed brain matter, and no carotid pulse

24-year-old man who is unconscious, has snoring respirations, and severe burns

normal blood glucose levels

80-120 mg/dL

Your patient has a Glasgow Coma Scale (GCS) score of 13, a systolic blood pressure of 80 mm Hg, and a respiratory rate of 8 breaths/min. His Revised Trauma Score (RTS) is: A. 10. B. 9. C. 8. D. 11.

9

The pulse rate of a toddler is: 80 to 140 beats/min 90 to 150 beats/min 90 to 180 beats/min 100 to 160 beats/min

90 to 150 beats/min

A "hip" fracture is actually a fracture of the: A. proximal femur. B. pubic symphysis. C. femoral shaft. D. pelvic girdle.

A

In which of the following patients would the head tilt-chin lift maneuver be the MOST appropriate method of opening the airway? A. A 37-year-old female who is found unconscious in her bed B. A 24-year-old male who is found unconscious at the base of a tree C. A 45-year-old male who is semiconscious after falling 20 feet D. A 50-year-old male who is unconscious following head trauma

A

Skeletal muscle is attached to the bone by tough, ropelike, fibrous structures called: A. tendons. B. cartilage. C. ligaments. D. fascia

A

D. remove the battery from the monitor and then remove the vest.

A patient in cardiac arrest is wearing an external defibrillator vest, which is interfering with effective chest compressions. The EMT should: Select one: A. leave the battery attached to the monitor and remove the vest. B. remove the battery from the monitor and leave the vest in place. C. perform ventilations only and allow the vest device to defibrillate. D. remove the battery from the monitor and then remove the vest.

Which of the following is an example of a psychiatric emergency? A. A person violently attacking family members B. A person who is depressed and no longer caring for himself C. A person going on a week-long "bender" after losing a job D. A person who is experiencing a panic attack

A person violently attacking family members

National guidelines for EMS care are intended to __________. Select one: A. provide more consistent delivery of EMS care across the United States B. reduce expenses at the local and state levels C. facilitate a national EMS labor group D. unify EMS providers under a single medical director

A. Provide more consistent delivery of EMS care across the US

A 66-year-old woman experienced a sudden onset of difficulty breathing. She has a history of type 2 diabetes and deep vein thrombosis (DVT). On the basis of her medical history, which of the following should the EMT suspect? Select one: A. Pulmonary embolism B. Severe hypoglycemia C. Diabetic ketoacidosis D. Congestive heart failure

A. Pulmonary embolism

Which section of the heart receives deoxygenated blood? A. Right B. Atria C. Ventricles D. Left

A. Right Correct B. Atria C. Ventricles D. Left

Which of your senses can be safely used to identify a HazMat incident? Select one: A. Sight and sound B. Sound and smell C. Smell and sight D. Sight and touch

A. Sight and sound

C. family history.

Risk factors for AMI that cannot be controlled include: Select one: A. excess stress. B. hyperglycemia. C. family history. D. lack of exercise.

Which of the following diseases is potentially drug resistant and is thought to be transmitted by coughing? Tuberculosis Croup Diphtheria Epiglottitis

Tuberculosis

Breathing is controlled by an area in the: lungs. brain stem. spinal cord. diaphragm.

brain stem.

hypoventilation

breathing too slow or too shallow

heat exhaustion

caused by a combination of heat exposure and hypovolemia

hemorrhagic strokes

caused by bleeding within the brain

seizures

caused by disorganized electrical activity within the brain

appendicitis

caused by inflammation of the appendix can lead to life-threatening infection and septic shock

peritonitis

caused by inflammation of the peritoneum (membrane lining the abdominal organs and cavity) signs/symptoms: nausea, vomiting, loss of appetite, diarrhea, fever

acute coronary syndrome (ACS)

caused by myocardial ischemia (poor blood supply) includes angina pectoris and acute myocardial infarction

septic shock (distributive shock)

caused by severe infection damages blood vessels, increases plasma loss out of the vascular space

neurogenic shock (distributive shock)

caused by spinal cord injury (in c-spine) leads to massive vasodilation below level of injury signs/symptoms: *warm/pink skin color* because in shock, your skin should be cool, pale and moist

referred pain

causes pain in an area of the body other than the source

Interruption of cerebral blood flow may result from all of the following, EXCEPT: A. a thrombus. B. cerebral vasodilation. C. an acute arterial rupture. D. an embolism.

cerebral vasodilation

alpha radiation

dense, slow-moving radiation travels short distances

Your patient believes he has hepatitis and is now exhibiting signs of cirrhosis of the liver. He most likely has: hepatitis A. hepatitis B. hepatitis C. hepatitis D.

hepatitis C.

A patient with a head injury, hypertension, and bradycardia and biot respiration indicate

herniation of the brain stem

type II diabetes

non-insulin-dependent diabetes mellitus do not typically require supplemental insulin

Bacterial vagenoses is a condition that occurs when

normal bacteria in the vagina is replaced by an overgrowth of other bacterial forms

upper airway

nose/mouth nasopharynx oropharynx larynx epiglottis

epistaxis

nosebleeds

A patient with an altered mental status is: A. not thinking clearly or is incapable of being aroused. B. typically alert but is confused as to preceding events. C. completely unresponsive to all forms of stimuli. D. usually able to be aroused with a painful stimulus.

not thinking clearly or is incapable of being aroused.

The EMT should assess a patient's tidal volume by: observing for adequate chest rise. assessing the facial area for cyanosis. counting the patient's respiratory rate. measuring the patient's oxygen saturation.

observing for adequate chest rise.

You have sealed the open chest wound of a 40 year old male who was stabbed in the anterior chest. Your assessment reveals that he is experiencing increasing respiratory distress and tachycardia, and is developing cyanosis. You should: A. begin rapid transport at once. B. call for a paramedic ambulance. C. begin ventilatory assistance. D. partially remove the dressing.

partially remove the dressing.

epicardium

outermost layer of the heart and innermost layer of the pericardium

scope of practice

outlines the actions a provider is legally allowed to perform based on his or her license or certification level

damage

patient experienced damage or injury recognized by the legal system as worthy of compensation

tonic-clonic phase of seizure

patient experiences uncontrolled muscle contraction and relaxation patient may be apneic

The left atrium of the heart receives ___________ blood from the ___________. oxygenated; lungs deoxygenated; body oxygenated; body deoxygenated; lungs

oxygenated; lungs

the pediatric head is larger in proportion to the body

padding should be placed being the shoulders in supine position to maintain alignment of airway

When assessing a patient with abdominal pain, you should: A. visually assess the painful area of the abdomen, but avoid palpation because this could worsen his or her condition. B. ask the patient to point to the area of pain or tenderness and assess for rebound tenderness over that specific area. C. observe for abdominal guarding, which is characterized by sudden relaxation of the abdominal muscles when palpated. D. palpate the abdomen in a clockwise direction, beginning with the quadrant after the one the patient indicates is painful.

palpate the abdomen in a clockwise direction, beginning with the quadrant after the one the patient indicates is painful.

Insulin is produced in the: liver. pancreas. thyroid gland. adrenal glands.

pancreas.

The effects of epinephrine are typically observed within _________ following administration. A. 30 minutes B. 30 seconds C. 1 minute D. 1 hour

1 minute

From a mental health standpoint, an abnormal or disturbing pattern of behavior is a matter of concern if it lasts for at least: A. 1 month. B. 1 week. C. 2 weeks. D. 3 weeks.

1 month.

You see an infant capable of reaching out to people and drooling. She is most likely: 2 months of age. 3 months of age. 4 months of age. 5 months of age.

4 months of age.

Which of the following is a good guideline for physical examination of any patient? A. Examine extremities first. B. Complete the physical exam first, then check the ABCs. C. Always work toe-to-head. D. Avoid touching without permission.

Avoid touching without permission.

Which of the following incidents does NOT require a report to be filed with local authorities? Select one: A. Animal bites B. Cardiac arrest C. Spousal abuse D. Gunshot wounds

B. Cardiac arrest

What should an EMT do to limit errors in the field? Select one: A. Contact medical direction before initiating any treatments. B. Follow the agency's written protocols. C. Carry an EMT text for reference at all times. D. Deviate from established standards when necessary.

B. Follow the agency's written protocols

The EMT is legally obligated to protect a patient's privacy according to _____________. Select one: A. DCAP B. HIPAA C. CQI D. APGAR

B. HIPAA

The nasopharyngeal airway is MOST beneficial because it: A. can effectively stabilize fractured nasal bones if it is inserted properly. B. effectively maintains the airway of a patient in cardiopulmonary arrest. C. can maintain a patent airway in a semiconscious patient with a gag reflex. D. is generally well tolerated in conscious patients with an intact gag reflex.

C

Which of the following is the MOST reliable indicator of adequately performed bag-valve mask ventilations in an apneic adult with a pulse? A. Decreased compliance when squeezing the bag B. Consistently increasing heart rate C. Adequate rise of the chest when squeezing the bag D. Twenty breaths/min being delivered to the adult

C

_____ is responsible for properly securing and stabilizing the vehicle and providing a safe entrance and access to the patient. Select one: A. Law enforcement B. The rescue team C. The EMS service D. The hazardous materials unit

B. The rescue team

A 75-year-old male with type 1 diabetes presents with chest pain and a general feeling of weakness. He tells you that he took his insulin today and ate a regular meal approximately 2 hours ago. You should treat this patient as though he is experiencing: Select one: A. hyperglycemia. B. hypoglycemia. C. a heart attack. D. an acute stroke.

C. a heart attack.

In preconventional reasoning, children: Select one: A. make decisions based on their conscience. B. blame their actions on what they have observed in older children. C. act almost purely to avoid punishment and to get what they want. D. look for approval from their peers and society.

C. act almost purely to avoid punishment and to get what they want.

Your female patient complains of vaginal bleeding. She is alert, denies trauma, and states she is not pregnant. You should: A. expose the patient and assess for signs of trauma. B. advise the patient to refuse treatment. C. assess the patient for signs of hypoperfusion. D. call for law enforcement and wait to begin treatment.

C. assess the patient for signs of hypo perfusion.

You are delivering oxygen to a patient with a nasal cannula at 4 L/min when he begins to complain of a burning sensation in his nose. You should: A. remove the nasal cannula. B. apply a nonrebreathing mask. C. attach an oxygen humidifier. D. increase the flow rate to 6 L/min.

C. attach an oxygen humidifier.

complex access to patient

requires the use of special tools and training

inspiratory/expiratory reserve volume

the amount of air you can still inhale or exhale after a normal breath

respiration

the exchange of oxygen and CO2

perfusion

the flow of blood throughout the body

A dissecting aortic aneurysm occurs when:

the inner layers of the aorta become separated

The Adam's apple is: A. the small indentation in between the thyroid and cricoid cartilages. B. the upper part of the larynx that is formed by the thyroid cartilage. C. the lower part of the larynx that is formed by the cricoid cartilage. D. below the thyroid cartilage and forms the upper part of the trachea.

the upper part of the larynx that is formed by the thyroid cartilage. Correct

When transporting a patient with a facial injury, it is MOST important to be as descriptive as possible with the hospital regarding the patient's injuries because: A. they must make arrangements for an ICU bed. B. they may need to call a specialist to see the patient. C. most patients with facial trauma will need surgery. D. it saves time on repeat assessments at the hospital.

they may need to call a specialist to see the patient. Correct

Urinary tract infections are more common in ____________. A. women B. active adults C. men D. sedentary adults

women

Force acting over a distance is the definition of: A. work. B. potential energy. C. latent energy. D. kinetic energy.

work

late indications of hypoxia

AMS, severe dyspnea, cyanosis, bradycardia

You should use a rigid ______ called a stokes litter to carry a patient across uneven terrain from a remote location

Basket stretcher

Your primary assessment of an elderly woman reveals that she is conscious and alert, but is experiencing difficulty breathing. She has a history of emphysema, hypertension, and congestive heart failure. As you assess the patient's circulatory status, you should direct your partner to:

Administer oxygen with the appropriate device

Why should you be concerned about a 16-year-old patient who seems depressed? She could be exhibiting rebellious behavior. She has just finished her growth spurt and may be disappointed in the results. Adolescents are at a higher risk for suicide. She may be having unprotected sex.

Adolescents are at a higher risk for suicide.

Which of the following is considered an organic brain syndrome? A. Anxiety conditions B. Depression C. Alzheimer dementia D. Schizophrenia

Alzheimer dementia

Which of the following is NOT a component of continuous quality improvement (CQI)? Periodic review of run reports Discussion of needs for improvement Negative feedback to those who make mistakes while on a call Remedial training as deemed necessary by the medical director

Negative feedback to those who make mistakes while on a call

A seizure patient is having what kind of medical emergency? Respiratory Cardiovascular Neurologic Immunologic

Neurologic

implied consent

allows assumption of consent for emergency care from an unresponsive or incompetent patient can be used if a patient initially refused care but then loses consciousness

complex partial seizure

altered LOC, isolated twitching, and sensory changes possible

In what area of the lungs does respiration occur?

alveoli

medical direction

always contact medical direction to know the standing orders/protocols for a state or agency

residual volume

amount of air in the lungs after completely exhaling keeps lungs open

tidal volume

amount of air inhaled or exhaled in one breath

oxygen tanks

amount of oxygen is measured in psi full cylinder --> 2000 psi cylinder should be refilled at 200 psi

mass casualty incident (MCI)

an incident that takes the locally viable resources or requires a multi-jurisdictional response

Which of the following interventions is the MOST critical to the outcome of a patient with multisystem trauma? A. Rapid transport to a trauma center B. Elevation of the lower extremities C. Intravenous fluid administration D. Early administration of oxygen

Rapid transport to a trauma center

You and your partner enter the residence of an elderly couple both of whom are found unconscious in there bed, there is no evidence of trauma. As you begin your assessment you and your partner notice the smell of natural gas in the residence. Which of the following is the most appropriate action?

Rapidly remove the patients from their residence using the Clothes or blanket drag

Early bruising following abdominal trauma often manifests as:

Red areas of skin

When assessing your patient's pain, he says it started in his chest but has spread to his legs. This is an example of what part of the OPQRST mnemonic? Onset Quality Region/radiation Severity

Region/radiation

"PASTE" is an alternate assessment tool for ___________.

Respiratory patients

uremia

Severe kidney failure resulting in the buildup of waste products within the blood. Eventually brain functions will be impaired.

What type of terrorist group would MOST likely bomb an abortion clinic? Single-issue group Technology terrorists Extremist political group Violent religious group

Single-issue group

kidney stones

Solid crystalline masses formed in the kidney, resulting from an excess of insoluble salts or uric acid crystallizing in the urine; may become trapped anywhere along the urinary tract.

Which of the following helps filter the blood and has no digestive function? A. Spleen B. Stomach C. Small intestine D. Large intestine

Spleen

Which of the following sounds indicates swelling of the upper airway? A. Rales B. Rhonchi C. Wheezing D. Stridor

Stridor

You are caring for a 70 year old female with signs and symptoms of an acute stroke. She is conscious, has secretions in her mouth, breathing at a normal rate with adequate depth and has oxygen sats of 90%. What should you do?

Suction and transport immediately

_____ connect muscles to bones. Ligaments Cartilage Tendons Joints

Tendons

You are treating a patient with an apparent emotional crisis. After the patient refuses treatment, you tell him that you will call the police and have him restrained if he does not give you consent. Your actions in this case are an example of: assault. battery. negligence. abandonment.

assault.

You have been dispatched to a call for an unresponsive patient. What is the MOST important information that you should obtain from the dispatcher initially? The callback number of the caller The severity of the patient's problem Whether the patient is breathing The exact physical location of the patient

The exact physical location of the patient

hernia

The protrusion of an organ or tissue through an abnormal body opening.

Which of the following statements regarding gunshot wounds is correct? A. High-velocity bullets will cause less severe internal injuries. B. The size of a bullet has the greatest impact on the injury produced. C. Low-velocity bullets will cause the greatest amount of trauma. D. The speed of a bullet has the greatest impact on the injury produced.

The speed of a bullet has the greatest impact on the injury produced

Neurogenic shock is caused by

a radical change in the size of the vascular system

When caring for a patient with documented hypoglycemia, you should be MOST alert for: A. a seizure. B. respiratory distress. C. an acute stroke. D. a febrile convulsion.

a seizure

The foreign substance responsible for causing an allergic reaction is called a(n):

allergen

Physical changes that typically occur in early adults include an: Select one: A. increase in height because of spinal disc expansion. B. increase in muscle strength and reflexes. C. increase in fatty tissue, which leads to weight gain. D. increase in respiratory rate due to increased metabolism.

C. increase in fatty tissue, which leads to weight gain.

A 16-year-old female complains of vaginal bleeding and abdominal cramping that began several hours ago. During your assessment interview, you should: Select one: A. avoid asking questions that she will feel uncomfortable answering. B. recall that patients in this age group prefer not to be treated as adults. C. inquire about the possibility of pregnancy in private, if possible. D. obtain the majority of your information from one of her parents.

C. inquire about the possibility of pregnancy in private, if possible.

You are treating a 40-year-old male with a documented blood sugar reading of 480 mg/dL. The patient is semiconscious and breathing shallowly, and is receiving assisted ventilation from your partner. You should recognize that definitive treatment for this patient includes: Select one: A. glucagon. B. dextrose. C. insulin. D. oxygen.

C. insulin.

Before administering oral glucose to your diabetic patient, you should ensure that the patient: A. has a blood glucose level (BGL) of less than 80 B. has a blood glucose level (BGL) of less than 60 C. is conscious and can swallow D. is completely unresponsive

C. is conscious and can swallow

If a patient's chest barely moves during inhalation, even if the patient's respiratory rate is normal, you should suspect that: A. expiratory reserve volume is decreased. B. inspiratory reserve is increased. C. minute volume is decreased. D. overall tidal volume is increased.

C. minute volume is decreased. Correct

The FIRST step in the START triage system is to: Select one: A. focus on the patients who are unconscious. B. scan the area for patients with severe bleeding. C. move all walking patients to a designated area. D. get a quick head count of all the patients involved.

C. move all walking patients to a designated area.

When preparing a pregnant patient for delivery, you should position her: A. in a sitting position with her hips elevated 12 inches. B. on her left side with the right leg elevated. C. on a firm surface with her hips elevated 2 to 4 inches. D. in a supine position with her legs spread.

C. on a firm surface with her hips elevated 2 to 4 inches.

When immobilizing a patient with a kyphotic spine to a long backboard, the EMT would MOST likely have to: Select one: A. use a scoop stretcher instead of a log roll. B. force the head into a neutral alignment. C. place blankets behind the patient's head. D. secure the patient's head before the torso.

C. place blankets behind the patient's head.

Which of the following statements regarding febrile seizures is correct? A. Febrile seizures are the most dangerous type of seizure for pediatric patients. B. Febrile seizures are usually the result of trauma. C. Febrile seizures typically occur due to hypoglycemia. D. Febrile seizures pose little risk of permanent injury.

D. Febrile seizures pose little risk of permanent injury. usually caused by high fever

While providing care to a patient, blood got onto the ambulance stretcher. Because the stretcher was not properly cleaned afterward, a virus was transmitted to another emergency medical technician (EMT) several days later. Which route of transmission does this scenario describe? Select one: A. Airborne transmission B. Direct contact C. Vector-borne transmission D. Indirect contact

D. Indirect contact

Which of the following organs would MOST likely bleed profusely when injured? A. Stomach B. Intestine C. Bladder D. Liver

D. Liver

A 12-year-old male jumped approximately 12 feet from a tree and landed on his feet. He complains of pain to his lower back. What injury mechanism is MOST likely responsible for his back pain? A. Direct trauma to the spinal column B. Lateral impact to the spine C. Secondary fall after the initial impact D. Energy transmission to the spine

Energy transmission to the spine

Which of the following techniques is considered to be an emergency move? Extremity lift Supine transfer Firefighter's drag Direct ground lift

Firefighter's drag

A 39 year old male sustained a large laceration to his leg during an accident with a chainsaw and is experiencing signs and symptoms of shock. What should you do first?

Follow appropriate standard precaution

______ is the fuel that makes the body run. Sleep Exercise Food Work/life balance

Food

What should you do if you are exposed to a patient who is found to have pulmonary tuberculosis? Get the BCG vaccine. Get a tuberculin skin test. Undergo serious therapy. No precautions need to be taken.

Get a tuberculin skin test.

Emergency patient care occurs in progressive phases. What occurs first? Activation of EMS Initial prehospital care The patient receives definitive care Incident recognition

Incident recognition

diverticulitis

Inflammation in small pockets at weak areas in the muscle walls

What is the route of administration for the EpiPen auto-injector? Select one: A. Intravenous B. Intraosseous C. Intramuscular D. Sublingual

Intramuscular

guarding

Involuntary muscle contractions (spasm) of the abdominal wall to minimize the pain of movement and protect the inflamed abdomen; a sign of peritonitis.

The scene size-up at a motor vehicle crash or other incident:

Is an ongoing process until the incident is terminated

The crichoid cartilage is

Is the only complete circular cartilage of the trachea

breech birth

occurs when the baby's butt or legs are the first presenting part in the birth canal transport immediately

You are attending to a 26-year-old female who is 34 weeks pregnant with her first child. Your patient has been having lower abdominal pains and cramping for the past two hours. In placing your patient on the stretcher and preparing for transport, you should place her: A. in the Fowler position. B. in a position of comfort. C. on her left side. D. supine with her legs elevated.

on her left side.

A spinal cord injury at the level of C7 would MOST likely result in: A. immediate cardiac arrest. B. paralysis of all the respiratory muscles. C. paralysis of the intercostal muscles. D. paralysis of the diaphragm.

paralysis of the intercostal muscles

exhalation

passive process (doesn't require energy) exhaled air has 16% oxygen

associated symptoms

patient complaints in addition to the chief complaint

What is the most important intervention for a flail chest with respiratory compromise?

positive pressure ventilation and oxygen

By what mechanism is a person injured when he or she falls from a significant height

potential energy turned into kinetic energy

The stinger from a honeybee should be: A. B. irrigated with copious amounts of water. C. scraped away from the skin. D. squeezed with tweezers and removed.

scraped away from the skin

delayed patients

second patient priority yellow tagged

Functions of the liver include:

secretion of bile and filtration of toxic substances

A 52 year old male presents with fever of 102.5 and a severe headache. as you assess him you note the presence of multiple blisters on his face and chest which are all identical in shape and size. This patients clinical presentation is most consistent with

smallpox

trauma in geriatric patients

the risk of death from all forms of trauma is greater in geriatric population pelvic and femur fractures are extremely dangerous

triage

the sorting of patients based on the severity of injury

The term "pharmacology" is MOST accurately defined as: Select one: A. the study of how medications affect the brain. B. the study of drugs that are produced illegally. C. the study of drugs and their actions on the body. D. the study of drug excretion from the human body.

the study of drugs and their actions on the body

foreign body airway obstruction

the tongue is the most common airway obstruction

uterine rupture

the uterus thins as it grows, increasing risk of rupture presents as abdominal pain and vaginal bleeding

Children are often "belly breathers" because _________? A. they are routinely hypoxic B. their diaphragm is not functional C. their intercostal muscles are not developed D. they are consciously controlling ventilations

their intercostal muscles are not developed

injury to solid organs

they bleed primary risk is hemorrhagic shock (loss of >20% of blood volume)

injury to hollow organs

they spill their contents primary risk is infection

myocardium

think muscular wall of the heart

minor patients

third patient priority "walking wounded" green tagged

A terrorist would MOST likely use a secondary explosive device: to ensure that a structure is completely destroyed. in case the primary explosive device fails to detonate. as a means of dispersing a biologic or chemical agent. to injure rescuers and gain maximum public attention.

to injure rescuers and gain maximum public attention.

causes of airway obstruction

tongue fluid swelling foreign bodies

artificial ventilation

used for any patients with inadequate spontaneous breathing leading to severe respiratory distress/failure example: BVM

oropharyngeal airway (OPA)

used to prevent the tongue from obstructing the airway indication: unresponsive patient without a gag reflex CONTRAINDICATION: conscious patients or any patient with a gag reflex

nasopharyngeal airway (NPA)

used to prevent the tongue from obstructing the airway in patient who cannot protect their own airway indication: unresponsive patients without a gag reflex, patients with decreased LOC but have a gag reflex CONTRAINDICATION: conscious patients with a gag reflex, severe head injury or facial trauma

urgent move

used when the patient has potentially life-threatening injuries and must be moved immediately

emergency move

used when the scene is dangerous and the patient must be move quickly before providing patient care

When requesting medical direction for a patient who was involved in a major car accident, the EMT should avoid: using radio codes to describe the situation. questioning an order that seems inappropriate. relaying vital signs unless they are abnormal. the use of medical terminology when speaking.

using radio codes to describe the situation.

Febrile seizures are

usually benign but should be evaluated

two important systemic effects of cold on the body

vasoconstriction (to conserve heat) eventual slowing of metabolic rate

two important systemic effects of heat on the body

vasodilation (to shed excess heat) an increase in metabolic rate

mobile radios

vehicle-mounted transmitters and receivers have a greater range than portable radios but distance is limited unless used with a repeater

If your patient swallows blood following facial trauma, there is an increased risk of ________. A. altered LOC B. vomiting C. GI trauma D. hypotension

vomiting Correct

hematemesis

vomiting blood

he sebaceous glands produce sebum, a material that: A. pulls the hair erect when you are cold. B. discharges sweat onto the skin's surface. C. facilitates shedding of the epidermis. D. waterproofs the skin and keeps it supple.

waterproofs the skin and keeps it supple. Correct

Signs of compensated shock include all of the following, EXCEPT: restlessness or anxiety. pale, cool, clammy skin. a feeling of impending doom. weak or absent peripheral pulses.

weak or absent peripheral pulses.

esophageal varices

weakening of the blood vessels lining the esophagus signs/symptoms: vomiting large amounts of bright red blood, history of alcohol abuse

abdominal aortic aneurysm

weakening of the wall of the aorta in the abdominal bleeding seen in geriatric males signs/symptoms: tearing back pain, possible pulsating abdominal mass

When caring for a patient with an open facial injury, the EMT's immediate priority should be to: A. consider the mechanism of injury. B. closely assess the patient's airway. C. wear gloves and facial protection. D. manually stabilize the patient's head.

wear gloves and facial protection. Correct

While en route to a call for a major motor vehicle collision, the MOST important safety precaution(s) that you and your partner can take is/are: adhering to standard precautions. ensuring that the fire department arrives before you. using lights and siren and being aware of other drivers. wearing seat belts and shoulder harnesses at all times.

wearing seat belts and shoulder harnesses at all times.

type I/III ambulance

what you normally think of when you think of an ambulance

type II ambulance

what you rode in at EasCare

entrapment

when a person is trapped in an enclosed space

limb presentation in birth

when a single arm or leg is the first presenting part in the birth canal transport immediately, do not attempt delivery

carina

where the trachea branches into left and right mainstream bronchi

A 47-year-old male presents with severe abdominal pain of 3 hours' duration. His abdomen is distended and guarded. Your MOST important consideration for this patient should be to: A. assess his blood pressure to determine perfusion adequacy. B. transport him in a supine position. C. be alert for signs and symptoms of shock. D. determine the exact location and cause of his pain.

be alert for signs and symptoms of shock.

Nitroglycerin is contraindicated in patients: with a systolic blood pressure less than 100 mm Hg. with chest pain of greater than 30 minutes duration. who are currently taking antibiotics for an infection. who are younger than 40 years of age and have diabetes.

with a systolic blood pressure less than 100 mm Hg.

The cartilaginous tip of the sternum is called the: costal arch. manubrium. angle of Louis. xiphoid process.

xiphoid process.

The index of suspicion is MOST accurately defined as: A. the detection of less obvious life-threatening injuries. B. the way in which traumatic injuries occur. C. a predictable pattern that leads to serious injuries. D. your awareness and concern for potentially serious underlying injuries.

your awareness and concern for potentially serious underlying injuries

Everyone exhibits some signs and symptoms of mental illness at some point in life ____________. A. but that does not mean a person is mentally ill B. and should be physically restrained for their own safety and the safety of the EMT C. and needs medication administered to control them D. because mental illness affects everyone

but that does not mean a person is mentally ill

The phrenic nerves control the diaphragm and exit the spinal chord at

c3,c4,c5

managing shock

control bleeding place patient in Trendelenburg position prevent loss of body heat with blanket rapid transport to nearest hospital

brain stem

controls essential body functions, such as breathing and consciousness

cerebellum

coordinates voluntary movement, fine motor function, and balance

Signs of a pulmonary blast injury include:

coughing up blood

aerobic metabolism

creation of cellular energy with use of oxygen waste products are water and CO2, both are excreted through urinary and respiratory systems

anaerobic metabolism

creation of energy without use of oxygen byproducts are lactic acid

When health care providers force their cultural values onto their patients because they believe their values are better, they are displaying: ethnocentrism. proxemics. nonverbal communication. cultural imposition.

cultural imposition.

kyphosis

curvature in the spine that leads to a rounded back

You respond to a plastic factory, where numerous people present with shortness of breath, flushed skin, and altered mental status. One of the patients tells you he smelled almonds before he started feeling sick. These people were MOST likely exposed to: sarin. bleach. cyanide. chlorine.

cyanide.

acute myocardial infarction

death to an area of the myocardial muscle due to lack of oxygenated blood flow though the coronary arteries MI pain does not go away in a new minutes

stroke

death to brain tissue due to an interruption in blood flow

Kussmaul respirations

deep, rapid breaths seen in diabetic ketoacidosis patients

A patient with cardiac arrest secondary to ventricular fibrillation has the greatest chance for survival if: CPR is initiated within 10 minutes. oxygen and rapid transport are provided. defibrillation is provided within 2 minutes. paramedics arrive at the scene within 5 minutes.

defibrillation is provided within 2 minutes.

A common cause of shock in an infant is

dehydration from vomiting or diarrhea

oxygenation

delivery of oxygen to the blood

spontaneous abortion / miscarriage

delivery of the fetus before it is capable of surviving presents as cramping, lower abdominal pain, vaginal bleeding, and passage of tissue/clots

simple triage and rapid treatment (START)

developed for east, rapid triage of patients at an MCI uses RPM (respirations, perfusions, mental status) approach

frostnip

develops when body parts get very cold but not yet frozen

diverticulitis

develops when small pouches (diverticula) along the wall of the intestine fill with feces and become inflamed/infected signs/symptoms: LLQ pain, fever, weakness, nausea, vomiting, *bleeding not common*

Signs of excited delirium include: A. subdued behavior, crying, and suicidal thoughts. B. pallor, hypotension, and constricted pupils. C. diaphoresis, tachycardia, and hallucinations. D. slurred speech, bradycardia, and a high fever.

diaphoresis, tachycardia, and hallucinations.

five basic interventions for external bleeding with signs and symptoms of shock

direct pressure high flow oxygen prevent heat loss Trendelenburg position high priority transport

Bleeding from soft-tissue injuries to the face is MOST effectively controlled with: A. direct pressure using dry, sterile dressings. B. pressure dressings and chemical ice packs. C. ice packs and elevation of the patient's head. D. digital pressure to an adjacent pulse point.

direct pressure using dry, sterile dressings. Correct

During the primary assessment of a semiconscious 70-year-old female, you should: A. ensure a patent airway and support ventilation as needed. B. ask family members if the patient has a history of stroke. C. immediately determine the patient's blood glucose level. D. insert a nasopharyngeal airway and assist ventilations.

ensure a patent airway and support ventilation as needed

partial thickness (second degree) burns

epidermal and partial dermal injury painful, blisters present

superficial (first degree) burns

epidermal damage only painful, red, no blisters

What layer of the skin forms a watertight, protective seal for the body?

epidermis

An acute bacterial infection that results in swelling of the flap that covers the larynx during swallowing is called: croup. laryngitis. epiglottitis. diphtheria.

epiglottitis.

Key hormones of the sympathetic nervous system include:

epinephrine and nor epinephrine

A 33-year-old woman presents with a generalized rash, facial swelling, and hypotension approximately 10 minutes after being stung by a hornet. Her BP is 70/50 mm Hg and her heart rate is 120 beats/min. In addition to high-flow oxygen, this patient is in MOST immediate need of: epinephrine. rapid transport. an antihistamine. IV fluids.

epinephrine.

All of the following are responsibilities of the EMS medical director, EXCEPT: evaluating patient insurance information. serving as liaison with the medical community. ensuring that the appropriate standards are met by EMTs. ensuring appropriate EMT education and continuing training.

evaluating patient insurance information.

postpartum hemorrhage

excessive bleeding following delivery blood loss > 500mL is abnormal

polyphagia

excessive hunger due to cell starvation

polyuria

excessive peeing due to excess glucose in the urine

polydipsia

excessive thirst due to dehydration

With regard to pharmacology, the term "action" refers to the: ability of a drug to cause harm. ability of a drug to produce side effects. amount of time it will take the drug to work. expected effect of a drug on the patient's body.

expected effect of a drug on the patient's body.

While caring for a trauma patient, the EMT has blood splashed into her eyes. This is an example of: infection. exposure. indirect contact. transmission.

exposure.

You arrive at a scene where an older woman is complaining of chest pain. In assessing her, she holds her arm out for you to take her blood pressure. This is an example of: implied consent. informed consent. expressed consent. emergency consent.

expressed consent.

syncope

fainting caused by temporary loss of blood flow to the brain

The unauthorized confinement of a person is called: assault. battery. false imprisonment. slander.

false imprisonment.

pericardium

fibrous sac surrounding the heart

A person with bilateral femur fractures has: fractured one of his or her femurs. fractured both of his or her femurs. one femur fractured in two places. fractured the lateral aspect of the femur.

fractured both of his or her femurs.

gestational development

full term pregnancy is 9 months or 40 weeks

A large tanker truck has overturned on a highway. When you arrive, you see a clear liquid leaking from the rear of the tanker. The driver, who appears to be unconscious, is still in the vehicle and is bleeding heavily from the face. You should: immediately notify law enforcement for traffic control. fully assess the situation and request the appropriate assistance. put on gloves, a gown, and a mask and quickly remove the driver. go to the rear of the tanker and determine what type of fluid is leaking.

fully assess the situation and request the appropriate assistance.

simple access to patient

gaining access to patient without any tools or the need to break glass

Artificial ventilation may result in the stomach becoming filled with air, a condition called: gastric distention. vomitus. abdominal-thrust maneuver. acute abdomen.

gastric distention.

offline orders

general protocols, do not need to call med control

cystic fibrosis

genetic disorder leading to thick mucus production and chronic lung infections

portable radios

hand=held transmitter/receiver with a very limited range, unless used with a repeater

The MOST effective way of preventing the spread of disease is: handwashing. keeping your immunizations up-to-date. placing a HEPA respirator on the patient. wearing goggles, gloves, a gown, and a mask.

handwashing.

The MOST significant risk factor for a hemorrhagic stroke is: A. heavy exertion. B. hypertension. C. diabetes mellitus. D. severe stress.

hypertension

Cushing's response

hypertension, bradycardia, altered respiratory pattern

common medical conditions in geriatric patients

hypertension, heart disease, diabetes

Which of the following conditions would be the LEAST likely to mimic the signs and symptoms of a stroke? A. A postictal state B. Hypovolemia C. Intracranial bleeding D. Hypoglycemia

hypovolemia

A 20-year-old man was kicked numerous times in the abdomen during an assault. His abdomen is rigid and tender, his heart rate is 120 beats/min, and his respirations are 30 breaths/min. You should treat this patient for: a lacerated liver. a ruptured spleen. respiratory failure. hypovolemic shock.

hypovolemic shock.

Advil, Nuprin, and Motrin are trade names for the generic medication: Select one: A. acetaminophen. B. aspirin. C. nitrostat. D. ibuprofen.

ibuprofen

When obtaining a sample history from a patient with diabetes it would be most important to determine

if he or she has had a recent illness or excessive stress

left ventricular failure (CHF)

if the left ventricle pumps ineffectively, blood backs up into the lungs (pulmonary edema)

right ventricular failure (CHF)

if the right ventricle pumps ineffectively, blood backs up into the venous system that feeds into the right heart signs: JVD, pedal edema (swelling in feet)

cholecystitis

inflammation of the gall bladder due to gall stones most often occurs in females 30-50 years old signs/symptoms: RUQ pain, increased pain at night, increased pain after eating fatty foods, referred pain to the shoulder, nausea and vomiting

croup

inflammation of the pharynx, larynx, and trachea usually seen in children up to 3 years of age

inhalation route

inhaled into the lungs rapid onset Ex. oxygen, metered-dose inhaler (MDI) meds, small volume nebulizers

avulsion (open injury)

injury caused by a flap of skin being torn partially or completely loose

The MOST serious consequences of a poorly planned or rushed patient move is: A. injury to you or your patient. B. unnecessarily wasting time. C. confusion among team members. D. causing patient anxiety or fear.

injury to you or your patient.

Most patients with abdominal pain prefer to: A. sit in a semi-Fowler position with their knees slightly bent. B. lie on their side with their knees drawn into the abdomen. C. lie in a supine position with their knees in a flexed position. D. sit fully upright because it helps relax the abdominal muscles.

lie on their side with their knees drawn into the abdomen.

anaphylactic shock (distributive shock)

life threatening form of severe allergic reaction caused by massive vasodilation, widespread fluid leakage in vessels, and bronchoconstriction

The proper technique for using the power grip is to: A. position your hands about 6² apart. B. rotate your palms down. C. hold the handle with your fingers. D. lift with your palms up.

lift with your palms up.

parietal pleura

lines the inside surface of the chest cavity

Assessment of an unconscious patient's breathing begins by: inserting an oral airway. manually positioning the head. assessing respiratory rate and depth. clearing the mouth with suction as needed.

manually positioning the head.

sternum

manubrium --> upper portion xiphoid process --> inferior tip

abduction

movement away from the midline

dislocation

movement of a bone out of its normal position in a joint

adduction

movement toward the midline

ventilation

moving of air in and out of lungs

Which of the following questions is of LEAST pertinence when determining whether a mother will deliver her baby within the next few minutes? A. "Have you had a sonogram?" B. "Do you feel the need to push?" C. "When are you due?" D. "Is this your first baby?"

A. "Have you had a sonogram?"

Which of the following suffixes mean "two"? Select one: A. "dipl-" and "bi-" B. "primi-" and "bi-" C. "bi-" and "null-" D. "primi-" and "dipl-"

A. "dipl-" and "bi-"

From what internal female organ is the fetus expelled during delivery? A. Uterus B. Perineum C. Vagina D. Cervix

A. Uterus

The EMT should suspect left-sided heart failure in the geriatric patient who presents with: Select one: A. fever and a cough that produces green sputum. B. tachypnea and paroxysmal nocturnal dyspnea. C. swelling of the lower extremities and weakness. D. jugular venous distention and peripheral edema.

B. tachypnea and paroxysmal nocturnal dyspnea.

The central nervous system is composed of the: A. brain and sensory nerves. B. motor and sensory nerves. C. brain and spinal cord. D. spinal cord and sensory nerves.

C. brain and spinal cord. Correct

"Myo" in the word myocarditis means: Select one: A. vein. B. bone. C. muscle. D. cartilage.

C. muscle.

The physical act of moving air into and out of the lungs is called: A. diffusion. B. respiration. C. oxygenation. D. ventilation.

D

An infant's blood pressure typically increases with age because: Select one: A. as the infant gets older, his or her blood vessels dilate. B. the infant's total blood volume decreases with age. C. his or her normal heart rate usually increases with age. D. blood pressure directly corresponds to body weight.

D. blood pressure directly corresponds to body weight.

The first month of life after birth is referred to as the: A. toddler period. B. premature phase. C. start of infancy. D. neonatal period.

D. neonatal period.

Which of the following is required for an EMT to administer a drug to a patient? Select one: A. A patient with an altered level of consciousness B. An alert adult patient C. Medical direction approval D. A patient with stable vital signs

Medical direction approval

The EMT scope of practice within local response area is defined by

Medical director

The Messentary is what

Membranous fold that attaches the intestines to the walls of the body

The simplest, yet most effective method of preventing the spread of an infectious disease is to: Select one: A. undergo an annual physical examination. B. ensure that your immunizations are up-to-date. C. undergo annual testing for tuberculosis and hepatitis. D. wash your hands in between patient contacts.

D. wash your hands in between patient contacts.

Which of the following are found in the retroperitoneal space? Liver Spleen Kidneys Stomach

Kidneys

Which of the following is the MOST appropriate device to use when immobilizing a patient with a suspected spinal injury? A. Scoop stretcher B. Long backboard C. Portable stretcher D. Wheeled stretcher

Long backboard.

Which of the following may help reduce your patient's nausea? A. Cricoid pressure B. Oral glucose C. Low-flow oxygen D. Positive-pressure ventilation

Low-flow oxygen

Why do finances become an issue during middle adulthood? Middle adults are often supporting both their children and their parents. Middle adults make less money than early adults. Their deteriorating health makes it difficult to get to work. Hearing loss prevents them from working.

Middle adults are often supporting both their children and their parents.

ileus

Paralysis of the bowel, arising from any one of several causes; stops contractions that move material through the intestine.

A patient who requires cardiac monitoring in the field would require, at a minimum, which level of EMS provider?. EMR EMT Paramedic AEMT

Paramedic

Which of the following terms applies to a state of delusion in which the patient is out of touch with reality? A. Psychosis B. Schizophrenia C. Suicidal D. Agitated delirium

Psychosis

An air embolism associated with diving occurs when:

The diver holds his or her breath during a rapid ascent

Which of the following is a specific example of MIH

The paramedic administers a patients flu vaccination

Which of the following statements BEST describes a mass-casualty incident? At least half of the patients are dead. Either a bus or an airplane has crashed. You have more than two critical patients. The patient count exhausts your resources.

The patient count exhausts your resources.

sudden infant death syndrome (SIDS)

an unexplained, unexpected death of a patient under one year of age

febrile seizures

common cause of seizures in pediatric patients caused by high fevers that develop rapidly

congestive heart failure in geriatric patients

complain of weakness, dyspnea (especially during exertion), and difficulty breathing when lying down

myocardial infarction in geriatric patients

complain of weakness, dyspnea, abdominal pain, or epigastric pain (pain just below ribs) instead of chest pain

A two-door passenger car struck a tree while driving approximately 50 mph. The doors are badly damaged and jammed, and the driver appears to be unconscious inside the vehicle. Entering the vehicle by breaking the back window is an example of: simple access. complex access. technical rescue. disentanglement.

complex access.

sunken fontanelle in pediatric patients

can indicate hypovolemia

Medications encased in a gelatin shell that are taken by mouth are called: Select one: A. pills. B. caplets. C. capsules. D. tablets.

capsules

While palpating the radial pulse of a 56-year-old man with chest pain, you note that the pulse rate is 86 beats/min and irregular. This indicates: pain. fear. anxiety. dysrhythmia.

dysrhythmia.

Gastric distention will MOST likely occur: Select one: a. when you deliver minimal tidal volume. b. c. in patients who are intubated. d. when the airway is completely obstructed. e. if you ventilate a patient too quickly.

e. if you ventilate a patient too quickly.

gamma radiation (x-ray)

easily penetrates the body travels long distances

All of the following are causes of acute dyspnea, EXCEPT: asthma. emphysema. pneumothorax. pulmonary embolism.

emphysema.

The EMTS first priority is

personal safety

The two basic categories of diagnosis that a physician will use for behavioral crises or psychiatric emergencies are ____________. A. organic brain syndrome and altered mental status B. functional and nonfunctional C. physical and psychological D. All of these answers are correct.

physical and psychological

The condition characterized by reexperiencing an event and overresponding to stimuli that recall the event is called: acute stress reaction. delayed stress reaction. cumulative stress reaction. posttraumatic stress disorder (PTSD).

posttraumatic stress disorder (PTSD).

head tilt-chin lift

preferred manual method of opening the airway indication: patient with AMS, patient with suspected airway obstruction, patient requiring suctioning CONTRAINDIATION: suspected c-spine injury

abruptio placenta

premature separation of the placenta from the uterine wall leading to bleeding *painful* vaginal bleeding

widened blood pressure

pulse pressure above 50% of systolic indicates possible head injury example: 210/100

Epinephrine stimulates the ________ response, increasing blood pressure and relieving bronchospasm. A. respiratory B. parasympathetic C. sympathetic D. cardiac

sympathetic

angina pectoris

temporary chest pain caused by a lack of oxygen to the heart muscle usually caused by atherosclerosis in the coronary arteries does not cause permanent cardiac damage

anatomical position

the body is in the standing position, arms at the sides, with palms forward (thumbs on the outside)

You are dispatched to a bombing along with 15 other ambulances. Upon arriving at the scene, you should stage your ambulance: as close to the bombing site as possible. upwind and uphill from the bombing site. downwind and uphill from the bombing site. upwind and downhill from the bombing site.

upwind and uphill from the bombing site.

Upon arrival at a scene where hazardous materials are involved, you should park the ambulance: upwind from the scene. with the warning lights off. downhill from the scene. at least 50 feet from the scene.

upwind from the scene.

rapid extrication

urgent move used in patients in a motor vehicle

You are en route to an emergency call when you approach a slow-moving vehicle on a two-way road. You can see oncoming traffic in the other lane. The driver has his windows up and does not realize that you are behind him. You should:

remain at a safe distance until it is safe to pass

bradycardia in pediatric patients

should be treated as a sign of hypoxia

After taking diphenhydramine (Benadryl) for an allergic reaction, a person begins experiencing drowsiness and a dry mouth. These findings are an example of a(n): Select one: A. untoward effect. B. unpredictable effect. C. side effect. D. therapeutic effect.

side effect

A type _____ ambulance features a conventional, truck cab-chassis with a modular ambulance body that can be transferred to a newer chassis as needed. Select one: A. I B. II C. III D. IV

A. I

Which of the following will help improve radio communications? Select one: A. Wait 1 second after pressing the transmit button before speaking B. Hold the radio at least 6 inches from your mouth. C. Answer questions with "yes" or "no." D. Use codes to speed communication.

A. Wait 1 second after pressing the transmit button before speaking

A person is said to be obese when he or she is ________ over his or her ideal weight. A. 20% or more B. 30% or more C. 10% or more D. 40% or more

B. 30% or more

Internal bleeding into a fractured extremity is MOST often controlled by: A. keeping the patient warm. B. applying a tourniquet. C. applying chemical ice pack. D. splinting the extremity.

A. keeping the patient warm. B. applying a tourniquet. C. applying chemical ice pack. D. splinting the extremity. Correct

Signs and symptoms of meningitis in the infant or child include all of the following, EXCEPT: A. sunken fontanelles. B. headache and fever. C. a stiff or painful neck. D. altered mental status.

A. sunken fontanelles.

Burns associated with lightning strikes are typically: A. superficial. B. full-thickness. C. third-degree. D. partial-thickness

A. superficial. Correct B. full-thickness. C. third-degree. D. partial-thickness

Which of the following drugs is commonly referred to as "roofies"? A. GHB B. Ketamine C. Rohypnol D. MDMA

C. Rohypnol

Which of the following is a physical change that typically occurs in the adolescent age group? Select one: A. The systolic blood pressure decreases. B. The normal pulse rate steadily increases. C. Secondary sexual development begins. D. Muscle and bone growth both decrease.

C. Secondary sexual development begins.

Vector-borne transmission of an infectious organism occurs via: Select one: A. smoke or dust. B. inanimate objects. C. animals or insects. D. direct contact.

C. animals or insects.

General guidelines for safe ambulance driving include all of the following, EXCEPT: Select one: A. avoiding one-way streets whenever possible. B. assuming that other drivers will not see you. C. regularly using the siren as much as possible. D. avoiding routes with heavy traffic congestion.

C. regularly using the siren as much as possible.

As an EMT, the standards of emergency care are often partially based on: Select one: A. the wishes of the general public. B. a consensus among paramedic supervisors. C. locally accepted protocols. D. the priorities of the medical director.

C. locally accepted protocols.

Sedentary behavior while healing from a hip fracture would MOST likely predispose the older patient to: Select one: A. ischemic stroke. B. heart failure. C. pneumonia. D. osteoporosis.

C. pneumonia.

When arriving at the scene of an overturned tractor-trailer rig, you note that a green cloud is being emitted from the crashed vehicle. The driver is still in the truck; he is conscious but bleeding profusely from the head. After notifying the hazardous materials team, you should: Select one: A. park downhill from the scene. B. ask the driver to exit the vehicle. C. position the ambulance upwind. D. quickly gain access to the patient.

C. position the ambulance upwind.

In relation to the chest, the back is: Select one: A. ventral. B. anterior. C. posterior. D. inferior.

C. posterior.

Maintaining the chain of evidence at the scene of a crime should include: Select one: A. making brief notes at the scene and then completing them later. B. placing the patient in a private area until the police arrive. C. not cutting through holes in clothing that were caused by weapons. D. quickly moving any weapons out of the patient's sight. Feedback

C. not cutting through holes in clothing that were caused by weapons.

While triaging patients at the scene of a motor-vehicle crash, you encounter a 5-year-old child who is unresponsive and apneic. After positioning his airway, you should: A. categorize him as deceased. B. deliver 5 rescue breaths. C. palpate for a carotid pulse. D. categorize him as immediate.

C. palpate for a carotid pulse.

You are encouraging a patient to go to the hospital by ambulance. You know that a transport will keep you on duty several hours after your regular quitting time. This is an example of __________. Select one: A. abuse of authority B. undue hardship C. patient advocacy D. scope of practice

C. patient advocacy

A 17-year-old female dislocated her patella while playing soccer. Her knee is flexed and she complains of severe pain. You should: A. flex her knee slightly more and assess for distal circulation. B. gently straighten her knee and apply a padded board splint. C. make one attempt to return the patella to its normal position. D. keep her knee flexed and secure it with padded board splints.

D

A supracondylar or intercondylar fracture is also known as a fracture of the: A. olecranon process. B. proximal radius. C. radial head. D. distal humerus.

D

The MOST significant hazard associated with splinting is: A. aggravation of the injury or worsened pain. B. reduction in circulation distal to the injury site. C. compression of nerves, tissues, and vasculature. D. delaying transport of a critically injured patient.

D

Anaphylaxis caused by stinging insects is typically an allergic reaction to ____________ rather than the bite or sting itself. A. injected poison B. irritating toxin C. deadly venom D. All of these answers are correct

D

Your patient is unresponsive following blunt trauma to the head. Which of the following is the EMT's first priority for an unresponsive patient? A. assess respiratory rate B. open the airway C. perform a SAMPLE history D. determine if CPR is indicated

D. determine if CPR is indicated if unresponsive, do circulation (where CPR would be found) first

Placards and labels on a storage container are intended to: Select one: A. advise responders of the appropriate neutralizing measures. B. broadly classify chemicals as being explosive or nonexplosive. C. provide specific information about the chemical being carried. D. give a general idea of the hazard inside that particular container.

D. give a general idea of the hazard inside the particular container.

The main function of the endocrine system is to _________. Select one: A. regulate blood flow B. regulate glucose and calcium C. produce glandular secretions D. maintain homeostasis

D. maintain homeostasis

In contrast to bleeding caused by external trauma to the vagina, bleeding caused by conditions such as polyps or cancer: A. is typically not as severe. B. can be controlled in the field. C. often presents with acute pain. D. may be relatively painless.

D. may be relatively painless.

While staged at the scene of a structure fire, the EMT should _________. Select one: A. locate the safety officer B. stay with the incident commander C. assess firefighters for signs of fatigue D. remain with the ambulance

D. remain with the ambulance.

Your partner is performing one-rescuer CPR on a middle-aged woman in cardiac arrest. When you apply the AED pads, you note that she has a medication patch over the same area where one of the AED pads will be placed. You should: Select one: A. move the patch to another area of the patient's chest and then properly apply the AED pads. B. apply the AED pad at least 1 inch away from the medication patch to avoid skin burns. C. continue CPR until you can determine the name of the medication contained in the patch. D. remove the medication patch, wipe away any medication residue, and apply the AED pads.

D. remove the medication patch, wipe away any medication residue, and apply the AED pads.

Unlike viral agents, bacterial agents: A. are far less infectious. B. do not replicate in the body. C. are usually not treatable. D. respond to antibiotics.

D. respond to antibiotics. Correct

Nitroglycerin relieves the squeezing or crushing pain associated with angina by what

Dilating the arteries to increase oxygen supply

Which of the following would you NOT detect while determining your initial general impression of a patient? Cyanosis Gurgling respirations Severe bleeding Rapid heart rate

Rapid heart rate

A utility worker was trimming branches and was electrocuted when he accidentally cut a high power line. He fell approximately 20 feet and is lying unresponsive on the ground. The power line is lying across his chest. You should what

Rapidly asses the patient after insuring the power line is not live

A patient has a blood pressure of 130/70 mm Hg. The "130" in this measurement represents: A. ventricular relaxation. B. ventricular filling. C. atrial contraction. D. ventricular contraction.

D. ventricular contraction. Correct

End-tidal carbon dioxide (ETCO2) monitoring is clearly indicated for patients who present with:

Respiratory distress

Which of the following conditions would MOST likely affect the entire brain? A. Blocked cerebral artery in the frontal lobe B. Reduced blood supply to the left hemisphere C. Respiratory failure or cardiopulmonary arrest D. Ruptured cerebral artery in the occipital lobe

Respiratory failure or cardiopulmonary arrest

Which of the following clinical signs would necessitate the administration of naloxone (Narcan)? Select one: A. Tachycardia B. Hypertension C. Slow respirations D. Extreme agitation

Slow respirations

When EMS responds to a disaster, as part of their response within the ICS, EMS would start with a scene size-up. What is the next step for the first-responding units? Communicating with additional units Establishing command Caring for any injuries Stabilizing the incident

Establishing command

A 2 month old infant was found unresponsive in his crib by his mother. When you arrive you determine that the infant is aepnic and pulse-less. His skin is pale and cold and his arms are stiff. You should:

Inform the child's mother that her son is deceased

You are dispatched to a residence for a 66 year old male who according to family members has suffered a massive stroke. Your primary assessment reveals the patient is unresponsive, apenic and pulse less. What should you do?

Initiate CPR and attach the aed as soon as possible

B. who have experienced a head injury

Nitroglycerin is contraindicated in patients: Select one: A. who have taken up to two doses. B. who have experienced a head injury. C. with a history of an ischemic stroke. D. with a systolic blood pressure less than 120 mm Hg.

Where does oral glucose go?

Place between the cheek and gum

What is the first phase of extrication? Arrival Preparation Scene size-up Gaining access

Preparation

B. dry the chest if it is wet.

Prior to attaching the AED to a cardiac arrest patient, the EMT should: Select one: A. contact medical control. B. dry the chest if it is wet. C. perform CPR for 30 seconds. D. assess for a pulse for 20 seconds.

When testing a mechanical suctioning unit, you should turn on the device, clamp the tubing, and ensure that it generates a vacuum pressure of more than: A. 100 mm Hg. B. 200 mm Hg. C. 400 mm Hg. D. 300 mm Hg.

D

A 52-year-old unrestrained female struck the steering wheel with her face when her truck collided with another vehicle. She has obvious swelling to her face and several dislodged teeth. A visual exam of her mouth reveals minimal bleeding. She is conscious and alert with a blood pressure of 130/80 mm Hg, a pulse of 110 beats/min, and respirations of 22 breaths/min with adequate tidal volume. You should: A. fully immobilize her spine, attempt to locate the dislodged teeth, suction as needed, and transport. B. apply oxygen via a nonrebreathing mask, suction her airway as needed, disregard the dislodged teeth, and transport. C. fully immobilize her spine, irrigate her empty tooth sockets, attempt to locate the dislodged teeth, and transport. D. assist ventilations with a BVM device, immobilize her spine, suction her oropharynx for 30 seconds, and transport.

fully immobilize her spine, attempt to locate the dislodged teeth, suction as needed, and transport.

Patients with autism

have extreme difficulty with complex tasks that require many steps.

chronic obstructive pulmonary disease (COPD)

a slow, chronic disease process that obstructs and damages the lower airways and alveoli includes chronic bronchitis and emphysema

delirium

a sudden change in cognitive function or mental status

base station

a transmitter/receiver in a fixed location that is in contact with all other components in the radio system

repeater

a type of base station that receivers low-power transmissions from portable or mobile radios and rebroadcasts at higher power to improve range

The EMT has a legal duty to act if he or she is: off duty and witnesses a major car accident. a volunteer, is on duty, and is dispatched on a call. paid for his or her services, but is not on duty. out of his or her jurisdiction and sees a man choking.

a volunteer, is on duty, and is dispatched on a call.

The process by which medications travel through body tissues until they reach the bloodstream is called: adsorption. onset of action. absorption. transformation.

absorption.

pulmonary edema

accumulation of fluid in the lungs

inhalation

active process (requires energy) atmospheric air has 21% oxygen

Movement of the arm toward midline is referred to as: flexion. extension. adduction. abduction.

adduction.

a 4 year old, 15 kilogram male ingested a large quantity of Tylenol, the childs mother states she does not known when ingestion occurred. the child is alert and in no apparent distress. The EMT should

administer 15 grams of activated charcoal

While assisting a paramedic in the attempted resuscitation of a 55-year-old male in cardiac arrest, you should expect the paramedic to: Select one: A. give the patient nitroglycerin to increase his blood pressure. B. withhold drug therapy until an intraosseous catheter is in place. C. give the patient activated charcoal to rule out a drug overdose. D. administer drugs via the IV route to achieve the fastest effect.

administer drugs via the IV route to achieve the fastest effect.

A 39-year-old male was struck in the head by a baseball during a game. He is confused and has slurred speech. He has a large hematoma in the center of his forehead and cannot remember the events preceding the injury. After manually stabilizing his head and assessing his airway, you should: A. perform a neurologic exam. B. palpate his radial pulses. C. apply ice to the hematoma. D. administer high-flow oxygen.

administer high-flow oxygen. Correct

normal respiratory rate

adults (12-20 BPM) pediatrics (15-30 BPM) infants (25-50 BPM)

correct rates of artificial ventilation for apneic patients with a pulse

adults --> one breath every 5-6 seconds (10-12 BPM) infants/children --> one breath every 3-5 seconds (12-20 BPM) newborns --> one breath every 1-1.5 seconds (40-60 BPM)

If an injured patient needs to be moved but is not in immediate danger from fire or building collapse, you should first: order the equipment you need for extrication. check the patient's airway, breathing, and circulation. remove the patient with the rapid extrication technique. determine the number of people you will need to move the patient.

check the patient's airway, breathing, and circulation.

The plural form of the word bronchus is: bronchae. bronches. bronchices. bronchi.

bronchi.

What part of the nervous system controls the body's voluntary activities? A. Central B. Sensory C. Autonomic D. Somatic

A. Central B. Sensory C. Autonomic D. Somatic Correct

Treatment with continuous positive airway pressure (CPAP) would MOST likely be contraindicated in which of the following situations? -Shortness of breath and a blood pressure of 76/56 mm Hg. -Pulmonary edema, history of hypertension, and anxiety. -Difficulty breathing, two-word dyspnea, and tachycardia. -Conscious and alert patient with an oxygen saturation of 85%.

-Shortness of breath and a blood pressure of 76/56 mm Hg. Page 615, Treatment of Specific Conditions

A 59-year-old male with history of emphysema complains of an acute worsening of his dyspnea and pleuritic chest pain following a forceful cough. Your assessment reveals that he has a barrel-shaped chest, unilaterally diminished breath sounds, and tachycardia. What is the MOST likely cause of this patient's condition? -Rupture of the diaphragm -Exacerbatuon of his COPD -Spontaneous pneumotorax -Acute pulmonary embolism

-Spontaneous pneumothorax Page 599, Causes of Dyspnea

Which of the following conditions is NOT categorized as a psychiatric condition? -Depression -Alzheimer disease -Substance abuse -Schizophrenia

-Substance abuse Page 561, Types of Medical Emergencies

In which of the following situations would it be MOST appropriate to utilize an air medical transportation service? -29-year-old woman who is 18 weeks pregnant, has light vaginal bleeding, and stable vital signs. -61-year-old man with signs and symptoms of a stroke and a ground transport time of 50 minutes. -50-year-old conscious woman with severe nausea and vomiting, fever, and chills of 3 days' duration. -43-year-old man experiencing a heart attack, and the closest appropriate hospital is 15 minutes away.

-61-year-old man with signs and symptoms of a stroke and a ground transport time of 50 minutes. Page 568, Management, Transport, and Destination

Which of the following patients is at greatest risk for complications caused by the influenza virus? -68-year-old woman with type 2 diabetes. -50-year-old woman moderate obesity. -39-year-old man with mild hypertension. -12-year-old child with a fractured arm.

-68-year-old woman with type 2 diabetes. Page 569, Common or Serious Communicable Diseases

As an EMT, you may be authorized to administer aspirin to a patient with chest pain based on: Select one: A. medical director approval. B. the transport time to the hospital. C. the patient's condition. D. an order from a paramedic.

A. Medical director approval

Which of the following scenarios does NOT involve the presence of any symptoms? -A 44-year-old male with abdominal pain and severe dizziness. -A 61-year-old female who si unconscious with facial cyanosis. -A 55-year-old with a severe headache and 2 days of nausea. -A 49-year-old female with blurred vision and ringing in the ears.

-A 61-year-old female who is unconscious with facial cyanosis. Page 315, Introduction

While ascultating an elderly woman's breath sounds, you hear low-pitched "rattling" sounds at the bases of both of her lungs. This finding is MOST consistent with which of the following conditions? -Aspiration pneumonia -Early pulmonary edema -Acute asthma attack -Widespread atelectasis

-Aspiration pneumonia Page 618, Patient Assessment

Which of the following injuries would MOST likely cause obstructive shock? -Simple pneumothorax. -Liver laceration. -Spinal cord injury. -Cardiac tamponade.

-Cardiac tamponade. Page 492, Types of Shock

As you approach a patient lying at the side of the roadway, you observe severe bleeding from the leg. What should your first action be? -Administer oxygen. -Check for a pulse. -Open the airway. -Control the bleeding.

-Control the bleeding. -Page 497-498, Patient Assessment for Shock

Which of the following is a genetic disorder that predisposes the patient to repeated lung infections? -Severe acute respiratory syndrome -Cystic fibrosis -Celiac sprue -Multiple sclerosis

-Cystic fibrosis Page 618, Treatment of Specific Conditions

Which of the following statements regarding the mechanism of injury (MOI) is correct? -The MOI may allow you to predict the severity of a patient's injuries. -The exact location of a patient's injuries can be determined by the MOI. -A significant MOI always results in patient death or permanent disability. -A nonsignificant MOI rules out the possibility of serious trauma.

-The MOI may allow you to predict the severity of a patient's injuries. Page 319, Scene Size-Up

Which of the following conditions would be LEAST likely to result in hypoxia? -Severe anxiety -Pulmonary edema -Pleural effusion -Narcotic overdose

-Severe anxiety Page 589-590, Causes of Dyspnea

Which of the following statements regarding oxygen is correct? A. Oxygen supports the combustion process and may cause a fire. B. Oxygen is flammable and may explode if under high pressure. C. Oxygen is most safely administered in an enclosed environment. D. Oxygen cylinders must always remain in an upright position.

A

Non cardiovascular causes of shock include respiratory insufficiency and

Anaphalaxis

Kussmal Respirations are an indication that the body is

Attempting to eliminate acid from the blood

melena

Black, foul-smelling, tarry stool containing digested blood.

B. acute myocardial infarction.

Cardiogenic shock can occur within 24 hours of a(n): Select one: A. hypertensive emergency. B. acute myocardial infarction. C. aortic aneurysm. D. unstable angina attack.

A man has been sucked inside the bin of a grain silo and is trapped. Which of the following rescue teams is the MOST appropriate to request? Trench rescue High-angle rescue Local fire department Confined space rescue

Confined space rescue

When performing a full body scan, you should assess for ________. A. OPQRST B. DCAP-BTLS C. SAMPLE D. AVPU

DCAP-BTLS Correct

The EMT is legally obligated to protect a patients privacy according to

HIPPA

You receive a call for a domestic dispute. When you arrive at the scene, you find a young male standing on the front porch of his house. You notice that an adjacent window is broken. The patient has a large body, is clenching his fists, and is yelling obscenities at you. Which of the following findings is LEAST predictive of this patient's potential for violence? A. The broken window B. His clenched fists C. His large body size D. His shouting of obscenities

His large body size

What is Beck's triad and what does it indicate?

JVD, muffled heart sounds, narrowing pulse pressure indicates possible cardiac tamponade

Albuterol, a beta-2 agonist, is the generic name for: Alupent. Metaprel. Brethine. Ventolin.

Ventolin.

rectus abdominis

abdominal muscles

The process of binding or sticking to a surface is called: Select one: A. digestion. B. suspension. C. adsorption. D. absorption.

adsorption

pectoralis

anterior chest muscle

secondary triage

assessment is done once the patient arrives in appropriate treatment area designated by tag

Bronchospasm is MOST often associated with: asthma. bronchitis. pneumonia. pneumothorax.

asthma.

white diamond on Hazmat sign

displays symbols indicating special hazards

blue diamond on Hazmat sign

health hazard

type I diabetes

insulin-dependent diabetes mellitus inject supplemental insulin

cyanide (blood agent)

interfere with body's ability to deliver oxygen to the cells, eating to sever hypoxia and death

The most common and often most serious ambulance crashes occur at/on: stop lights. intersections. highways. stop signs.

intersections.

yellow diamond on Hazmat sign

reactivity hazard

A 60-year-old woman presents with a BP of 80/60 mm Hg, a pulse rate of 110 beats/min, mottled skin, and a temperature of 103.9°F. She is MOST likely experiencing: septic shock. neurogenic shock. profound heart failure. a severe viral infection.

septic shock.

bleeding from veins

steady flow of dark red blood

ilium

upper portion of pelvis

Upon arrival at the residence of a young male with an apparent emotional crisis, a police officer tells you that the man is acting bizarrely. You find him sitting on his couch

he is conscious, but confused. He takes medications, but cannot remember why. His skin is pale and diaphoretic, and he has noticeable tremors to his hands. You should FIRST rule out: hypoglycemia. suicidal thoughts. severe depression. schizophrenia. ;Answer: A Rationale: Numerous physical problems can cause bizarre behavior, such as hypoglycemia, hypoxemia, and brain tumors, among others. The EMT should rule out an underlying medical cause first. The patient's pallor, diaphoresis, and motor tremors suggest hypoglycemia. The EMT should assess the patient's blood glucose level, if trained to do so, and consider administering oral glucose. Psychiatric illnesses, such as clinical depression and schizophrenia, cannot be ruled in or out in the field. Upon arrival at the residence of a young male with an apparent emotional crisis, a police officer tells you that the man is acting bizarrely. You find him sitting on his couch; he is conscious, but confused. He takes medications, but cannot remember why. His skin is pale and diaphoretic, and he has noticeable tremors to his hands. You should FIRST rule out: hypoglycemia. Rationale: Correct answer suicidal thoughts. Rationale: This is a symptom, something that the patient tells you. It does not produce visible signs. Upon arrival at the residence of a young male with an apparent emotional crisis, a police officer tells you that the man is acting bizarrely. You find him sitting on his couch; he is conscious, but confused. He takes medications, but cannot remember why. His skin is pale and diaphoretic, and he has noticeable tremors to his hands. You should FIRST rule out: severe depression. Rationale: Depression cannot be ruled out in the prehospital setting. schizophrenia. Rationale: Schizophrenia cannot be ruled out in the field.

Rule of Nines (child)

head (front and back), chest, back = 18% each right arm, left arm = 9% each right leg, left leg = 13.5% each genital area = 1%

Rule of Nines (adult)

head (front and back), right arm, left arm = 9% each chest, back, right leg, left leg = 18% each genital area = 1%

The statement "the lungs are superior to the bladder" indicates that the lungs are closer to the: feet. surface of the skin. head. trunk.

head.

All of the following are common signs and symptoms of cardiac ischemia, EXCEPT: headache. chest pressure. shortness of breath. anxiety or restlessness.

headache.

stridor

high-pitched sounds indicating partial upper airway obstruction heard in upper airway and not in lower lung fields

wheezing lung sounds

high-pitched sounds usually heard during exhalation

immediate patients

highest patient priority red tagged

Urticaria is the medical term for: A. a wheal. B. burning. C. swelling. D. hives.

hives

gall bladder

hollow organ that collects and stores bile from liver, releases bile into intestine to aid with digestion

For a patient with a gastrointestinal complaint, it is MOST important for the EMT to _________. A. identify whether the patient requires rapid transport B. avoid transporting the patient if the condition is minor C. determine the cause of the patient's complaint D. perform all interventions prior to transport

identify whether the patient requires rapid transport

You respond to a residence for a 40 year old female who was assaulted by her husband, the scene has been secured by law enforcement. Upon your arrival, you find the patient lying supine on the floor in the kitchen. She is semiconscious with severely labored breath sounds. As your partner is supporting her ventilations, you should: A. obtain a set of baseline vital signs. B. immediately request ALS support. C. insert an oropharyngeal airway. D. perform a focused secondary exam.

immediately request ALS support

meconium

presence of fetal stool in the amniotic fluid

Pain that may be perceived at a distant point on the surface of the body, such as the back or shoulder, is called: A. referred pain. B. remote pain. C. radiating pain. D. visceral pain.

referred pain.

systemic vascular resistance (SVR)

resistance to blood flow throughout the body

In infants who have signs and symptoms of an airway infection, you should not waste time trying to dislodge a foreign body. You should intervene only if signs of ____________ develop, such as a weak, ineffective cough; cyanosis; stridor; absent air movement; or a decreasing level of consciousness. sudden infant death syndrome child abuse bronchitis severe airway obstruction

severe airway obstruction

Potential causes of cardiogenic shock include all of the following, EXCEPT: inadequate heart function. disease of muscle tissue. severe bacterial infection. impaired electrical system.

severe bacterial infection.

parietal pain (abdomen)

severe, localized pain --> usually sharp and constant pain will often cause the patient to curl up with knees to chest

beta radiation

slow moving radiation travels only a few feet

bleeding from capillaries

slow oozing of dark red blood may be mixed with clears fluid

dementia

slow, progressive deterioration on cognitive function

endocardium

smooth, thin lining on the inside of the heart

A laceration located on the plantar surface is on the: sole of the foot. palm of the hand. back of the body. front of the body.

sole of the foot.

pancreas

solid organ that aids in digestion, produces insulin and helps regulate blood glucose levels

spleen

solid organ that filters the blood

liver

solid organ that helps break down fats, filters toxins, and produces cholesterol

kidneys

solid organs that control fluid balance, filter wastes and control pH balance

Pinpoint pupils, vomiting, bradycardia, and excessive salivation are signs of exposure to: lewisite. soman. cyanide. phosgene.

soman.

errors of commission

something incorrect was included on the PCR

errors of omission

something that should be in a PCR but was left out

When treating a potentially hostile patient, you should try to diffuse the situation by: assuming an aggressive posture. staring at the patient. speaking calmly, confidently, and slowly. verbally threatening the patient.

speaking calmly, confidently, and slowly.

Significant trauma to the face should increase the EMT's index of suspicion for a(n): A. spinal column injury. B. airway obstruction. C. basilar skull fracture. D. displaced mandible.

spinal column injury. Correct

When treating a trauma patient who is in shock, LOWEST priority should be given to: spinal protection. thermal management. splinting fractures. notifying the hospital.

splinting fractures.

bleeding from arteries

spurting, bright red blood

The rapid extrication technique is a: nonurgent technique to remove a patient from a vehicle. technique used to transfer a patient from a bed to a stretcher. technique used to lift a patient with no suspected spinal injury onto a stretcher. technique used to quickly remove a patient from a vehicle and onto a backboard.

technique used to quickly remove a patient from a vehicle and onto a backboard.

When a woman presents with abdominal pain or other vague symptoms the EMT will have trouble figuring out the problem until

the EMT can collect patient history information

pneumothorax

the accumulation of air in the pleural space lungs sounds may be diminished or absent over injured area

dead space

the amount of air in the respiratory system not including the alveoli

Anaphylactic shock is typically associated with: urticaria. bradycardia. localized welts. a severe headache.

urticaria.

Deformity caused by a fracture would MOST likely be masked by:

swelling

You are dispatched to the scene of a trench collapse. Upon arriving at the scene, your ambulance should be parked at least _____ feet from the incident. 250 500 750 1000

500

Brain damage is very likely in a brain that does not receive oxygen for: 0-1 minutes. 0-4 minutes. 4-6 minutes. 6-10 minutes.

6-10 minutes.

The normal resting adult heart rate is: 50 to 70 beats/min. 60 to 100 beats/min. 80 to 110 beats/min. 110 to 120 beats/min.

60 to 100 beats/min.

The suffix "-pathy" means: Select one: A. disease. B. specialist. C. study of. D. enlargement.

A. disease.

"Gastro" in the word "gastroenteritis" means: Select one: A. tongue. B. stomach. C. intestine. D. bowel.

B. stomach.

An epidural hematoma is MOST accurately defined as:

Bleeding between the skull and dura matter

An integral part of rapid scan is evaluation using the mnemonic

DCAPBTLS

Appendicitis

Inflammation or infection of the appendix.

incision (open injury)

a sharp, clean cut

respiratory changes in pregnancy

patient is at risk for developing hypoxia rapidly

The proper hand placement for CPR is accomplished by

placing the heel of one hand on the sternum and along the nipple line

extrication

removal of the patient from entrapment

shock/Trendelenburg position

supine with legs elevated

In patients with deeply pigmented skin, changes in color may be apparent only in certain areas, such as the: -back of the neck. -forehead and face. -dorsum of the hand. -lips or oral mucosa.

-lips or oral mucosa. Page 331, Primary Assessment

The adult epinephrine auto-injector delivers ______ mg of epinephrine, and the infant-child auto-injector delivers ______ mg. A. 0.3; 0.15 B. 0.03; 0.3 C. 0.1; 0.01 D. 0.01; 0.1

0.3; 0.15

Your patient opens his eyes, moans, and pulls away from you when you pinch his trapezius muscle. You should assign a Glasgow Coma Scale (GCS) score of: A. 7. B. 8. C. 9. D. 6.

8

C. obtain vital signs and a SAMPLE history.

A 49-year-old male presents with an acute onset of crushing chest pain and diaphoresis. You should: Select one: A. administer up to 324 mg of baby aspirin. B. administer up to three doses of nitroglycerin. C. obtain vital signs and a SAMPLE history. D. assess the adequacy of his respirations.

Which of the following destinations is most appropriate for a 41-year-old male patient who was involved in a rollover motor vehicle collision and is unconscious and unresponsive, assuming that travel times to each is equal? A. Only a Level I trauma center B. A Level IV or Level III trauma center C. Any designated trauma center is acceptable. D. A Level I or Level II trauma center

A Level I or Level II trauma center

urinary tract infection (UTI)

A bacterial infection, usually of the lower urinary tract (urethra and bladder) that occurs when normal flora bacteria enter the urethra and grow.

Which of the following patients is the BEST candidate for oral glucose? A conscious patient who is showing signs of hypoglycemia An unconscious diabetic patient with a documented low blood sugar A conscious diabetic patient suspected of being hyperglycemic A semiconscious patient with signs and symptoms of low blood sugar

A conscious patient who is showing signs of hypoglycemia

Organic brain syndrome is MOST accurately defined as:

A dysfunction of the brain caused by abnormal physical or physiological function

In which of the following patients would a nasopharyngeal airway be contraindicated? A semiconscious patient with a gag reflex An unconscious patient with an intact gag reflex A patient who fell 20 feet and landed on his or her head An unconscious patient who gags when you insert an oral airway

A patient who fell 20 feet and landed on his or her head

Which of the following MOST accurately describes a simple partial seizure? A. A seizure that causes the patient to stare blankly B. A seizure that is not preceded by an aura C. A generalized seizure without incontinence D. A seizure that begins in one extremity

A seizure that begins in one extremity

Which of the following scenarios would warrant an interruption in CPR procedures? An hysterical family member trying to gain access to the unconscious patient A vehicle honking its horn anxious to pass by the scene on a blocked road A small set of steps leading to the exit of the building, on the way to the ambulance Being tired from trying to resuscitate a patient

A small set of steps leading to the exit of the building, on the way to the ambulance

A raised, swollen well defined area on the skin that is a result of an insect bite or sting is called

A wheal

The ___________ is made up of the maxilla and zygoma, as well as the frontal bone of the cranium. A. orbit B. sphenoid C. mastoid D. occiput

A. orbit Correct

A 37-year-old male is having a severe allergic reaction to penicillin. He does not have an epinephrine auto-injector and your protocols do not allow you to carry epinephrine on the ambulance. How should you proceed with the treatment of this patient? A. Remain at the scene with the patient and request a paramedic ambulance. B. Ask the patient if he has any diphenhydramine tablets that you can administer. C. Quickly determine if there are any bystanders who may carry epinephrine. D. Administer oxygen, transport at once, and request a paramedic intercept.

Administer oxygen, transport at once, and request a paramedic intercept

B. immediately resume CPR.

After the AED has delivered a shock, the EMT should: Select one: A. assess for a carotid pulse. B. immediately resume CPR. C. re-analyze the cardiac rhythm. D. transport the patient at once.

relaying patient info

Age Sex History Interventions Condition ETA

The main advantage(s) of the AED is: it provides quick delivery of a shock. it is easier than performing CPR. there is no need for ALS providers to be on scene. All of the above.

All of the above.

Which of the following is an example of abandonment? An EMT leaves the scene after a competent adult has refused care. An EMT transfers care of a patient to an emergency department nurse. An AEMT transfers care of a patient to a paramedic. An AEMT transfers patient care to an EMT.

An AEMT transfers patient care to an EMT.

Which of the following statements regarding an emergency patient move is correct? A. The patient is dragged against the body's long axis during an emergency move. B. The spine must be fully immobilized prior to performing an emergency move. C. An emergency move is performed before the primary assessment and treatment. D. It is not possible to perform an emergency move without injuring the patient.

An emergency move is performed before the primary assessment and treatment.

An infant who is repeatedly rejected experiences what type of attachment? Secure attachment Anxious-avoidant attachment Trust and mistrust Rejection attachment

Anxious-avoidant attachment

A patient is bleeding severely from a severed femoral artery high in the groin area region. Which of the following would most likely control the bleeding

Apply a topical homeostatic agent and apply direct pressure

Care for a victim of an immunologic emergency who is severely hypotensive should include which of the following? A. Initiate basic life support measures, including the use of an automated external defibrillator, if necessary. B. Position the patient's airway and initiate positive-pressure ventilations. C. Apply high-flow oxygen therapy, place the patient in a shock position, and help maintain the patient's body temperature. D. Routine spinal immobilization due to the potential for traumatic injury

Apply high-flow oxygen therapy, place the patient in a shock position, and help maintain the patient's body temperature

You are assessing a 45-year-old female who is severely depressed. She states that it seems as though her entire world is crashing down around her. She further states that she has had frequent thoughts of suicide, but is not sure if she can actually go through with it. How should you manage this situation? A. Ask the patient if she has developed a suicidal plan. B. Have law enforcement place her in protective custody. C. Encourage the patient to remain quiet during transport. D. Leave the scene and have a neighbor check in on her.

Ask the patient if she has developed a suicidal plan.

During your assessment of a patient with blunt chest trauma you know that the patient has shallow breathing and paradoxical breathing of the left chest wall. What should you do?

Assist ventilation with a BVM

Which of the following respiratory diseases causes obstruction of the lower airway? Croup Asthma Epiglottitis Laryngitis

Asthma

A. Provide high-flow oxygen.

At 0500 hours, you respond to the home of a 76-year-old man complaining of chest pain. Upon arrival, the patient states that he had been sleeping in the recliner all night due to indigestion, when the pain woke him up. He also tells you he has taken two nitroglycerin tablets. His vital signs are as follows: respirations, 16 breaths/min, pulse, 98 beats/min, blood pressure, 92/76 mm Hg. He is still complaining of chest pain. What actions should you take to intervene? Select one: A. Provide high-flow oxygen. B. Administer a third nitroglycerin tablet. C. Apply an AED. D. Begin chest compressions.

When dealing with a hazardous materials incident, where should you set up your decontamination area? Inside the hazard zone Inside the treatment area Between the hazard zone and treatment area Between the treatment and transportation zone

Between the hazard zone and treatment area

Proper protective equipment will vary depending on the hazards encountered. Which piece of equipment should be utilized during all patient contacts? Turnout gear Helmets Blood- and fluid-impermeable gloves Goggles

Blood- and fluid-impermeable gloves

A 19-year-old female is found unconscious by her roommate. Your primary assessment reveals that her breathing is inadequate. As you insert an oropharyngeal airway, she begins to gag violently. You should: A. select a smaller oropharyngeal airway and attempt to insert it. B. continue to insert the airway as you suction her oropharynx. C. remove the airway and be prepared to suction her oropharynx. D. insert the airway no further but leave it in place as a bite block.

C

Which of the following statements regarding crush syndrome is correct? A. Tissue damage that occurs in crush syndrome is severe, but kidney injury is unlikely because toxins are quickly eliminated from the body. B. With crush syndrome, massive blood vessel damage occurs following severe soft-tissue injuries, such as amputation of an extremity. C. Compromised arterial blood flow leads to crush syndrome and can occur when an area of the body is trapped for longer than 4 hours. D. Provided that a patient with a crush injury is freed from entrapment within 6 hours, the amount of tissue damaged is generally minimal.

Compromised arterial blood flow leads to crush syndrome and can occur when an area of the body is trapped for longer than 4 hours. Correct

Ethnocentrism

Considering your own cultural values to be more important when interacting with people of a different culture

Which of the following findings would be LEAST suggestive of the presence of high-energy trauma? A. Intrusion into the vehicle B. Steering wheel deformity C. Deployment of the air bag D. Dismounted seats

Deployment of the air bag

breech of duty

EMT failed to assess, treat, or transport patient according to the standard of care

duty to act

EMT has an obligation to respond and provide care

Why might EMTs encounter a larger proportion of violent patients than the population at large? A. EMTs respond with law enforcement. B. It is a common misperception; EMTs do NOT encounter a larger proportion of violent patients than the population at large. C. EMTs have to restrain patients. D. EMTs respond to patients who, by definition, are having an emergency.

EMTs respond to patients who, by definition, are having an emergency.

Which of the following sequences of events describes the AHA's chain of survival? Early access, integrated post-arrest care, early advanced care, early CPR, early defibrillation Early advanced care, early defibrillation, integrated post-arrest care, early CPR, early access Early access, early CPR, early defibrillation, early advanced care, integrated post-arrest care Early access, early riser, early CPR, early advanced care

Early access, early CPR, early defibrillation, early advanced care, integrated post-arrest care

Which of the following areas of the body has the thinnest skin? A. Ears B. Scalp C. Back D. Soles of the feet

Ears Correct

What is the most common misconception surrounding mental illness? A. All persons with mental disorders are physically violent and dangerous. B. Everyone has some form of mental illness. C. Feeling "bad" or "depressed" means that you must be "sick." D. Many mental illnesses stem from drug or alcohol abuse.

Feeling "bad" or "depressed" means that you must be "sick."

Which of the following statements regarding glucose is correct? Select one: A. Glucose is given to patients who are suspected of being hyperglycemic. B. Glucose is usually administered by the EMT via the intravenous route. C. Glucose is a complex sugar that rapidly absorbs into the bloodstream. D. Glucose is a simple sugar that is readily absorbed by the bloodstream.

Glucose is a simple sugar that is readily absorbed by the bloodstream.

A patient who is possibly experiencing a stroke is NOT eligible for thrombolytic (fibrinolytic) therapy if he or she: A. is older than 60 years of age. B. has bleeding within the brain. C. has had a prior heart attack. D. has a GCS score that is less than 8.

Has bleeding within the brain

A 30-year-old male experienced a generalized (tonic-clonic) seizure, which stopped before you arrived at the scene. The patient is conscious, is answering your questions appropriately, and refuses EMS transport. Which of the following would be the MOST compelling reason to disagree with his refusal of transport? A. He has experienced seizures since he was 20 B. His wife states that this was his "usual" seizure C. His Glasgow Coma Scale (GCS) score is 15 D. He is currently not prescribed any medications

He is currently not prescribed any medications

The Department of Homeland Security posts a daily advisory system to keep the public aware of the current terrorist threat level. What does an orange level indicate? Low: Low risk of terrorist attacks High: High risk of terrorist attacks Severe: Severe risk of terrorist attacks Elevated: Significant risk of terrorist attacks

High: High risk of terrorist attacks

If your medical patient is not in critical condition, how long should you spend on scene? 10 minutes or less 30 minutes 2 hours However long it takes to gather as much information as possible

However long it takes to gather as much information as possible

What is Cushing's response and what does it indicate?

Hypertension, bradycardia, altered respiratory pattern indicates possible closed head injury with increased intracranial pressure

What does sulfur mustard do to the cells within the body? It makes the cells retain water until they explode. It causes the cells to release all their energy, which then causes cellular death. It makes the cells mutate, which damages and changes the cells and causes cellular death. It makes the cells release all their fluids and causes severe dehydration until cellular death occurs.

It makes the cells mutate, which damages and changes the cells and causes cellular death.

Which of the following is a professional responsibility of the EMT? Telling the family of a dying patient that everything will be OK Maintaining only the skills that the EMT feels uncomfortable with Maintaining a professional demeanor even under the most stressful situations Advising an emergency department nurse that patient reports are given only to a physician

Maintaining a professional demeanor even under the most stressful situations

Which of the following statements regarding medications is FALSE? Many medications are known by different names. Some medications affect more than one body system. Over-the-counter drugs must be prescribed by a physician. EMTs should ask about any herbal remedies or vitamins that the patient may be taking.

Over-the-counter drugs must be prescribed by a physician.

Which of the following patient populations typically require a modified drug dose? Select one: A. Females B. Pediatric patients C. Patients with asthma D. Middle adults

Pediatric patients

referred pain

Pain felt in an area of the body other than the area where the cause of pain is located.

How should you determine the pulse in an unresponsive 8-year-old patient? Palpate the radial pulse at the wrist. Palpate the brachial pulse inside the upper arm. Palpate the radial pulse with your thumb. Palpate the carotid pulse in the neck.

Palpate the carotid pulse in the neck.

Which of the following organs lies in the retroperitoneal space? A. Spleen B. Liver C. Pancreas D. Gallbladder

Pancreas

Which of the following is a severe burn in a 65-year-old patient? A. Second-degree burn covering 10% of the BSA B. Full-thickness burn to 1% of the BSA C. Partial-thickness burn to 20% of the BSA D. Superficial burn to 30% of the BSA

Partial-thickness burn to 20% of the BSA Correct

Which of the following statements regarding dialysis is correct? A. The purpose of dialysis is to help the kidneys retain salt and water. B. Acute hypertension is a common adverse effect of dialysis. C. Hemodialysis is effective but carries a high risk of peritonitis. D. Patients who miss a dialysis treatment often present with weakness.

Patients who miss a dialysis treatment often present with weakness.

lung expansion

during inhalation, as the chest expands, the parietal pleura pull the visceral pleura, which pull the lungs

You are called to a construction site where a 27 year old male worker has fallen from the 2nd floor. He landed on his back and is drifting in and out of consciousness. A quick assessment reveals no bleeding or blood loss. His BP is 90/60, heart rate 110. His airway is open and breathing is within normal limits. You realize the patient is in shock. This patients shock is due to an injury to the:

Spinal chord

An injured hang glider is trapped at the top of a large mountain and must be evacuated to the ground. The terrain is very rough and uneven. Which of the following devices would be the safest and most appropriate to use? Stair chair Stokes basket Scoop stretcher Long backboard

Stokes basket

All of the following are examples of standard patient transfer equipment, EXCEPT: Stokes baskets. long backboards. wheeled stair chairs. wheeled ambulance stretchers.

Stokes baskets.

Which of the following is an example of a rules-based medication error? Select one: A. The EMT administers a drug that is not approved by the medical director. B. The EMT administers the correct drug, but gives it by the wrong route. C. The EMT accidentally gives a higher drug dose than what is indicated. D. The EMT administers a drug that is contraindicated for the patient.

The EMT administers a drug that is not approved by the medical director

Which of the following is an example of care that is provided using standing orders? Medical control is contacted by the EMT after a patient with chest pain refuses EMS care. The EMT defibrillates a cardiac arrest patient, begins CPR, and then contacts medical control. A physician gives the EMT an order via radio to administer oral glucose to a diabetic patient. Following an overdose, the EMT contacts the medical director for permission to give activated charcoal.

The EMT defibrillates a cardiac arrest patient, begins CPR, and then contacts medical control.

C. pediatric pads and an energy-reducing device are available.

The EMT should use an AED on a child between 1 month and 8 years of age if: Select one: A. he or she is not breathing and has a weakly palpable pulse. B. his or her condition is rapidly progressing to cardiac arrest. C. pediatric pads and an energy-reducing device are available. D. special pads are used and the child has profound tachycardia.

You are caring for a 56 year old male patient complaining of abdominal pain, Your service has recently switched to an electronic PCR system when completing a PCR it is important to be aware that

The PCR allows patient information to be transmitted directly to the receiving hospitals computer

Which of the following statements about red blood cells is FALSE? They contain iron. They carry oxygen. They help to fight infection. They give color to the blood.

They help to fight infection.

C. sinoatrial node.

The electrical impulse generated by the heart originates in the: Select one: A. bundle of His. B. coronary sinus. C. sinoatrial node. D. atrioventricular node.

Which of the following would MOST likely provide clues regarding the source of a patient's allergic reaction? A. The patient's family history B. The environment in which the patient is found C. The patient's general physical appearance D. The time of year in which the exposure occurred

The environment in which the patient is found

A patient whose speech is slurred and difficult to understand is experiencing: A. dysarthria. B. paraplegia. C. dysphagia. D. aphasia.

dysarthria

Which of the following processes occurs during the inflammation phase of the healing process? A. The blood vessels in and around the injury site constrict, which forces bacteria and other microorganisms away, thus preventing significant infection. B. The immune system releases histamines, which cause vasodilation and increased capillary permeability, resulting in local redness and swelling. C. White blood cells are forced away from the injury site, thus allowing an increase in the flow of red blood cells, resulting in increased blood flow. D. The veins and arteries at the injury site constrict and platelets aggregate, which stops bleeding and causes a temporary increase in the size of the wound.

The immune system releases histamines, which cause vasodilation and increased capillary permeability, resulting in local redness and swelling. Correct

You are caring for a driver that has struck a lightpole, she admits to drinking alcohol but orders you not to tell. You should report the information to:

The incoming nurse or doctor

A patient with atherosclerotic heart disease expediences chest pain during exertion because

The lumen of the coronary artery is narrowed and cannot accommodate increased blood flow

B. pulmonary veins.

The only vein(s) in the body that carry oxygenated blood is/are the: Select one: A. external jugular veins. B. pulmonary veins. C. subclavian veins. D. inferior vena cava.

Once you begin CPR in the field, you must continue until one of the following events occurs: The patient stops breathing and has no pulse The patient is transferred to another person who is trained in BLS, to ALS-trained personnel, or to another emergency medical responder You are out of gas in the ambulance A police officer assumes responsibility for the patient and gives direction to discontinue CPR

The patient is transferred to another person who is trained in BLS, to ALS-trained personnel, or to another emergency medical responder

Which of the following pieces of patient information is of LEAST pertinence when giving a verbal report to a nurse or physician at the hospital? The patient's name and age The patient's family medical history Vital signs that may have changed Medications that the patient is taking

The patient's family medical history

Which of the following statements regarding anterior nosebleeds is correct? A. They usually originate from the septum area and bleed slowly. B. They are usually severe and require aggressive treatment to control. C. They cause blood to drain into the posterior pharynx. D. They are usually caused by a fracture of the basilar skull.

They usually originate from the septum area and bleed slowly. Correct

Which of the following signs/symptoms are indicative of respiratory involvement of an allergic reaction? A. Flushed, itching, or burning skin B. A sense of impending doom C. Tightness in the chest or throat D. All of these answers are correct.

Tightness in the chest or throat

Which of the following situations MOST likely involves a hazardous material? Milk truck that overturned and is leaking fluid Tractor trailer rig that is emitting a visible cloud Moving van that collided head-on with a small car Pickup truck from the gas company that struck a tree

Tractor trailer rig that is emitting a visible cloud

________ is used to identify a body part that is on the "belly side" or anterior surface of the body. Deep Superficial Dorsal Ventral

Ventral

emesis

Vomiting.

You are attending to a 32-year-old male patient. The patient's wife tells you that he returned from Afghanistan last year. While he initially seemed fine, lately he has become withdrawn and distanced himself from his family and friends. He does not talk about it, but she knows that he has been having terrible nightmares that wake him up. The most appropriate question to ask regarding his experience in Afghanistan is: A. Have you ever been diagnosed with PTSD? B. Were you in combat? C. Can you tell me about your experience? D. Were you shot at or under fire?

Were you shot at or under fire?

C. on the thighs or ankles

When preparing to obtain a 12-lead ECG, the "LL" and "RL" electrodes should be placed: Select one: A. on the lower abdomen. B. anywhere on the arms. C. on the thighs or ankles. D. on either side of the chest.

A. A rapid heart rate

Which of the following is NOT a common sign or symptom associated with malfunction of an implanted cardiac pacemaker? Select one: A. A rapid heart rate B. Syncope or dizziness C. Heart rate less than 60 beats/min D. Generalized weakness

Your patient is having respiratory difficulty and is not responding to your treatment. What is the best method of transport? Without lights and sirens, to the closest hospital With lights and sirens, to the closest hospital Air transport, to a special facility located 30 miles away The patient does not need to be transported.

With lights and sirens, to the closest hospital

Which of the following components of a medical term conveys its essential meaning? Prefix Suffix Word root Combining vowels

Word root

A. begin high-quality CPR and apply the AED as soon as possible.

You and your partner arrive at the scene of a middle-aged man who collapsed about 5 minutes ago. He is unresponsive, apneic, and pulseless. Bystanders are present, but have not provided any care. You should: Select one: A. begin high-quality CPR and apply the AED as soon as possible. B. have your partner perform CPR while you question the bystanders. C. perform two-rescuer CPR for 5 minutes and request ALS backup. D. immediately apply the AED pads and analyze his cardiac rhythm

Which of the following statements about records and reports is FALSE? Legally, if it was not documented, it was not performed. A complete, accurate report is an important safeguard against legal problems. An incomplete or untidy patient care report is evidence of incomplete or inexpert emergency medical care. Your patient care report does not become a part of the patient's hospital record because your treatment was provided outside the hospital.

Your patient care report does not become a part of the patient's hospital record because your treatment was provided outside the hospital.

subcutaneous emphysema

a "crackling" sensation upon palpitation due to air escaping into the fatty tissue

deep vein thrombosis (DVT)

a blood clot in a large vein, usually in the leg a loose clot (embolus) can cause a pulmonary embolism (blockage of artery in lungs)

pregnancy-induced hypertension (PIH)

a blood pressure in a pregnant patient above 140/90 at least twice at 6 hours apart

Following blunt trauma to the face, a 21-year-old male complains of a severe headache and decreased ability to move his eyes. This patient's clinical presentation is MOST consistent with: A. a ruptured eyeball. B. a lacerated globe. C. a blowout fracture. D. optic vessel compression.

a blowout fracture. Correct

shaken baby syndrome

a form of abuse caused by violent shaking of a pediatric patient

closed fracture

a fracture where the skin is not broken

open fracture

a fracture with an associated open soft tissue injury

laceration (open injury)

a jagged cut

The only indications for placing your gloved fingers in the vagina during delivery are:

breech presentation and prolapsed umbilical chord.

While jogging, a 19 year old male experienced an acute onset of breath and pleuritic chest pain. He is conscious and alert with stable vital signs. Your assessment reveals that he has diminished breath sounds over the left side of the chest. You should: A. administer oxygen and transport to the hospital. B. circumferentially tape a dressing around his chest. C. immediately perform a rapid head-to-toe exam. D. recognize that he needs a needle decompression.

administer oxygen and transport to the hospital

A 59-year-old woman presents with chest pressure. She is conscious and alert, but her skin is cool, pale, and clammy. Your first step in providing care (treatment) should be: apply the AED. administer oxygen. ask her if she takes nitroglycerin. take a complete set of vital signs.

administer oxygen.

tension pneumothorax (obstructive shock)

air enters the chest cavity due to lung injury or a open sucking chest wound

An open pneumothorax occurs when: A. a fractured rib perforates the tissue of the lung. B. air enters the pleural space from a perforated lung. C. extreme pleural pressure causes the lung to rupture. D. air enters the pleural space from outside the body.

air enters the pleural space from outside the body.

The MOST significant complication associated with facial injuries is: A. damage to the eyes. B. mandibular immobility. C. airway compromise. D. cervical spine injury.

airway compromise. Correct

While caring for a 5-year-old boy with respiratory distress, you should: avoid direct eye contact with the child, as this may frighten him. avoid letting the child hold any toys, as this may hinder your care. avoid alerting the child prior to a painful procedure. allow a parent or caregiver to hold the child if the situation allows.

allow a parent or caregiver to hold the child if the situation allows.

You are assessing a 27-year-old woman with a heat-related emergency. Her skin is flushed, hot, and moist

and her level of consciousness is decreased. After moving her to a cool environment, managing her airway, and administering oxygen, you should: give her ice water to drink. place her in the recovery position. cover her with wet sheets and fan her. take her temperature with an axillary probe. ;Answer: C Rationale: This patient is experiencing heat stroke. After moving her to a cooler area, managing her airway, and administering oxygen, the single most important treatment for her involves rapid cooling. Turn on the AC in the back of the ambulance, cover her with a wet sheet, and begin fanning her. Consider applying chemical ice packs to her groin and axillae (follow local protocols). Untreated heat stroke almost always results in death due to brain damage. You are assessing a 27-year-old woman with a heat-related emergency. Her skin is flushed, hot, and moist; and her level of consciousness is decreased. After moving her to a cool environment, managing her airway, and administering oxygen, you should: give her ice water to drink. Rationale: Give the patient nothing by mouth. place her in the recovery position. Rationale: Place the patient in the shock position. You are assessing a 27-year-old woman with a heat-related emergency. Her skin is flushed, hot, and moist; and her level of consciousness is decreased. After moving her to a cool environment, managing her airway, and administering oxygen, you should: cover her with wet sheets and fan her. Rationale: Correct answer take her temperature with an axillary probe. Rationale: The core temperatures are the most accurate.

Which of the following is most likely to cause immediate death? A. Aortic rupture B. Pulmonary contusion C. Aortic dissection D. Myocardial contusion

aortic rupture

When communicating with an older patient, you should: approach the patient slowly and calmly. step back to avoid making the patient uncomfortable. raise your voice to ensure that the patient can hear you. obtain the majority of your information from family members.

approach the patient slowly and calmly.

rules of splinting

check CMS before and after splinting immobilize above and below injury attempt to regain deformed injuries with absent distal pulses do not delay transport

warm zone

area between hot and cold only life-threatening conditions are treated here everyone must be decontaminated before entering cold zone

When assessing for arm drift of a patient with a suspected stroke, you should: A. ask the patient to hold his or her arms up with the palms down. B. observe movement of the arms for approximately 2 minutes. C. ask the patient to close his or her eyes during the assessment. D. expect to see one arm slowly drift down to the patient's side.

ask the patient to close his or her eyes during the assessment

You have applied a dressing and roller-gauze bandage to a laceration on the arm of a young female. During transport, she begins to complain of numbness and tingling in her hand. You should: A. carefully manipulate her arm until the symptoms subside. B. assess distal circulation and readjust the bandage as needed. C. remove the gauze bandage and replace it with an elastic one. D. remove the bandage and dressing and apply another one.

assess distal circulation and readjust the bandage as needed. Correct

Pediatric Assessment Triangle (PAT)

assess for TICLS (Tone, Interactivity, Consolability, Look, Speech) work of breathing circulation of skin

While the EMT is in a vehicle assessing the patient, the rescue team should be: assessing exactly how the patient is trapped and determining the safest way to extricate. awaiting further instructions from the EMT regarding how to proceed with the extrication. actively extricating the patient using whichever rescue method is deemed necessary by the rescue leader. preparing for a simple extrication process, as the EMT has obviously gained access to the patient.

assessing exactly how the patient is trapped and determining the safest way to extricate.

A 62-year-old male is seen with crushing chest pain, which he describes as being the same kind of pain that he had with a previous heart attack. He has prescribed nitroglycerin but states that he has not taken any. After administering supplemental oxygen if needed and contacting medical control, you should: Select one: A. administer up to three doses of nitroglycerin before assessing his blood pressure. B. administer the nitroglycerin unless he has taken Viagra within the past 72 hours. C. begin immediate transport and request a rendezvous with a paramedic unit. D. assist him with his nitroglycerin unless his systolic blood pressure is less than 100 mm Hg

assist him with his nitroglycerin unless his systolic blood pressure is less than 100 mm Hg

A patient is found unconscious after falling from a third floor window. His respirations are slow and irregular. You should: place him in the recovery position. apply oxygen via a nonrebreathing mask. suction his airway for up to 15 seconds. assist his breathing with a bag-valve mask.

assist his breathing with a bag-valve mask.

If a "frequent flier" calls 9-1-1 because of a suspected head injury, you should NEVER: take the call seriously; don't waste your time or resources on such a caller. perform a primary assessment; he called for a head injury last week, and it wasn't serious. assume you know what the problem is; every case is different, and you don't want to miss a potentially serious problem. treat the patient with respect; he is probably lying.

assume you know what the problem is; every case is different, and you don't want to miss a potentially serious problem.

A general impression of the patient is formed by an EMT: A. when the patient is in the ambulance. B. speaking with family members or bystanders first. C. speaking privately with the patient. D. at a distance when the patient is first seen.

at a distance when the patient is first seen.

bag valve mask (BVM) device

at about 15 LPM, patient gets almost 100% oxygen

The pulse check should take: 1 second. at least 1 second but no more than 5 seconds. at least 10 seconds. at least 5 seconds but no more than 10 seconds.

at least 5 seconds but no more than 10 seconds.

As the bronchus divides into smaller bronchioles, the terminal ends of these smaller passages form the:

aveoli

A young female experienced a laceration to her left eyeball from flying glass when her boyfriend broke a soda bottle. There is moderate bleeding and the patient states that she cannot she out of the injured eye

avoid applying pressure to the globe when you are covering the eye

An EMT may inquire his or her back, even if it is straight, if the: A. hands are held close to the legs. B. back is bent forward at the hips. C. shoulder is aligned over the pelvis. D. force is exerted straight down the spine.

back is bent forward at the hips.

bundle of His

backup of backup pacemaker generates electrical impulses between 20-40 times per minute

atrioventricular (AV) node

backup pacemaker generates electrical impulses between 40-60 times per minute

Upon arriving at a scene in which the incident command system has been activated, you should expect to: be passed from sector to sector, as needed, in between assignments. report directly to the incident commander in between assignments. be assigned a specific responsibility for the duration of the incident. be given general directions and allowed to function independently.

be passed from sector to sector, as needed, in between assignments.

second stage of labor

begins with full cervical dilation and ends with delivery of baby

first stage of labor

begins with onset of contractions and ends with full cervical dilation (10 cm)

flexion

bending the joint (decreasing the angle at the joint)

Activated charcoal is indicated for patients who have ingested certain drugs and toxins because it: acts as a direct reversal agent for most medications. induces vomiting before the chemical can be digested. detoxifies the drug before it can cause harm to the patient. binds to chemicals in the stomach and delays their absorption.

binds to chemicals in the stomach and delays their absorption.

The major side affect associated with activated charcoal is

black stool

subdural hematoma

bleeding above the brain often caused by venous bleeding following a cerebral contusion

epidural hematoma

bleeding beneath the skull but above the dura mater signs/symptoms: patient experiences brief LOC, wakes up, then LOC deteriorates

hemothorax

bleeding into the pleural space

pulmonary embolism

blockage of a pulmonary artery due to a blood clot of other obstruction

ischemic strokes

blood flow to the brain is compromised due to a blockage usually due to atherosclerosis

hypoglycemia

blood glucose levels below 60 mg/dL can lead to altered LOC, seizures, coma, brain death

hyperglycemia

blood glucose levels higher than 120 mg/dL

preeclampsia

blood poisoning by toxins from a local bacterial infection presents with high BP, edema in hands/feet, protein in urine

systolic blood pressure

blood pressure exerted during contraction of the left ventricle

diastolic blood pressure

blood pressure in between contractions (relaxation period)

hematochezia (melena)

bloody stool

Closed chest injuries are typically caused by ________? A. flying debris B. blunt trauma C. penetrating trauma D. high-velocity weapons

blunt trauma

heatstroke

body loses the ability to regulate body heat

Epinephrine is given to patients with anaphylactic shock because of its effects of: bronchodilation and vasodilation. bronchodilation and vasoconstriction. vasodilation and bronchoconstriction. bronchoconstriction and vasoconstriction.

bronchodilation and vasoconstriction.

cerebral contusion

bruising of brain often accompanied with edema or concussion injury

A 40-year-old male intentionally cut his wrist out of anger after losing his job. Law enforcement has secured the scene prior to your arrival. As you enter the residence and visualize the patient, you can see that he has a towel around his wrist and a moderate amount of blood has soaked through it. You should: A. calmly identify yourself to the patient. B. approach the patient with caution. C. tell the patient that you want to help. D. quickly tend to the bleeding wound.

calmly identify yourself to the patient.

clavicle and rib fractures

can be associated with pneumothorax a fracture to one of the first several ribs indicates a serious MOI

congestive heart failure (CHF)

cardiac emergency in which the heart does not pump effectively, pleasing to a backup of fluid and pulmonary edema

cardiovascular changes in pregnancy

cardiac workload increases blood volume increases

The term "shock" is MOST accurately defined as: a decreased supply of oxygen to the brain. cardiovascular collapse leading to inadequate perfusion. decreased circulation of blood within the venous circulation. decreased function of the respiratory system leading to hypoxia.

cardiovascular collapse leading to inadequate perfusion.

When assessing an elderly male who complains of nausea and generalized weakness, you find that he takes atorvastatin (Lipitor) and amlodipine (Norvasc). These medications suggest a history of: Select one: A. cardiovascular disease. B. non-insulin-dependent diabetes. C. reactive airway disease. D. bacterial infection.

cardiovascular disease

biological agents

cause disease

nerve agents

cause excessive parasympathetic nervous system stimulation signs/symptoms: Salivation/Seizures, Lacrimation, Urination, Defecation, Gastric upset, Emesis, Miosis (pupillary constriction) (SLUDGEM)

pulmonary/choking agents

cause lung injury

vesicants (blistering agents)

cause pain, burns, and blisters to exposed skin, eyes and respiratory tract

zygomatic bone

cheekbones

Instead of the abdominal-thrust maneuver, use ___________ for women in advanced stages of pregnancy and patients who are obese. chest thrusts Sellick maneuver basic life support DNR orders

chest thrusts

Your patient complains of abdominal pain that occurs mostly at night or after eating fatty foods. You should suspect ____________. A. kidney stones B. cholecystitis C. a hernia D. appendicitis

cholecystitis

Major controllable risk factors for an AMI include: older age. family history. cigarette smoking. male sex.

cigarette smoking.

Which of the following MOST accurately describes what the patient will experience during the postictal state that follows a seizure? A. Hyperventilation and hypersalivation B. Confusion and fatigue C. A rapidly improving level of consciousness D. A gradually decreasing level of consciousness

confusion and fatigue

A 30-year-old female presents with redness, inflammation, and pain to her left eye. During your assessment, you note that she is having difficulty keeping her eyes open. You should suspect that she is experiencing: A. acute retinitis. B. a detached retina. C. conjunctivitis. D. a corneal abrasion.

conjunctivitis. Correct

ligaments

connect bone to bone

tendons

connect bone to muscle

minor consent

consent is required from a parent or legal guardian not required for emancipated minors (minor who is a parent, married, in the military

dermis

consists of blood vessels, nerve endings, sweat glands, and hair follicles

axial skeleton

consists primarily of the skull, spinal column, and rib cage

hot zone

contaminated area patient care does not take place here

An 8-year-old boy was struck by a car, is unconscious, and is bleeding from the mouth. A police officer tells you that he is unable to contact the child's parents. You should: continue to treat the child and transport as soon as possible. cease all treatment until the child's parents can be contacted. continue with treatment only if authorized by medical control. provide only airway management until the parents are contacted.

continue to treat the child and transport as soon as possible.

diaphragm

contracts during inhalation moves down and expands the size of the thoracic cavity

A closed soft-tissue injury characterized by swelling and ecchymosis is called a(n): A. abrasion. B. hematoma. C. crush injury. D. contusion.

contusion. Correct

Which of the following statements about the patient care report is correct? Select one: a. EMTs are not liable for any actions that are accurately documented. b. Incomplete reports are common and accepted in EMS. c. Patient care cannot be discredited based on poor documentation. d. It is difficult to prove actions were performed if they are not included on the report.

d. It is difficult to prove actions were performed if they are not included on the report.

It is essential that you ______ your equipment to prevent the spread of disease. A. incinerate B. properly store C. throw out D. decontaminate

decontaminate

dilation of blood vessels

decreases SVR and causes decrease in BP

Upon arriving at the scene of a motor vehicle crash, you find a single patient still seated in his car. There are no scene hazards. As you approach the vehicle, you note that the patient is semiconscious and has a large laceration to his forehead. You should: A. slide a long backboard under his buttocks and lay him sideways on the board. B. apply a cervical collar and quickly remove the patient with a clothes drag. C. direct your partner to apply manual in-line support of the patient's head. D. apply a vest-style extrication device before attempting to move the patient.

direct your partner to apply manual in-line support of the patient's head.

lower GI bleeds

due to diverticulitis

upper GI bleeds

due to ulcers

visceral pain (abdomen)

dull, diffuse pain that is difficult to locate frequently associated with nausea and vomiting

midline plane

divides body left and right

transverse plane

divides body top and bottom at belly bottom

primary triage

done quickly and on the spot to determine the patient's basic needs/condition involves "tagging"

The amount of medication that is given is known as the _________. Select one: A. indication B. side effect C. contraindication D. dose

dose

tachypnea

fast breathing

subcutaneous tissue

fatty tissue

An elderly patient may understate or minimize symptoms of his or her illness due to

fear of hospitalization

hypovolemic shock

fluid problem loss of blood, dehydration due to vomiting, diarrhea, or burns can cause hypovolemic shock

Peritonitis may result in shock because: A. fluid shifts from the bloodstream into body tissues. B. abdominal distention impairs cardiac contractions. C. intra-abdominal hemorrhage is typically present. D. severe pain causes systemic dilation of the vasculature.

fluid shifts from the bloodstream into body tissues.

generalized seizures

grand mal seizures patient is unresponsive and experiences full-body convulsions

You are assessing a 59-year-old male and note that his pupils are unequal. He is conscious and alert. When obtaining his medical history, it is MOST pertinent to ask him if he: A. regularly sees a family physician. B. has a history of eye surgeries. C. is allergic to any medications. D. noticed the change during a meal.

has a history of eye surgeries. Correct

transient (temporary) ischemic attack

has same presentation as a stroke but signs/sympotoms seem to correct itself within 24 hours with no brain damage usually a warning sign of an impending stroke

The primary purpose of a "jump kit" is to: ensure that you have immediate access to the AED. have available all of the equipment that you will use in the entire call. have easy access to manage patients with severe uncontrolled bleeding. have available all of the equipment that will be used in the first 5 minutes.

have available all of the equipment that will be used in the first 5 minutes.

hypoxia

inadequate delivery of oxygen to the cells

shock / hypoperfusion

inadequate tissue perfusion

In an otherwise healthy individual, the primary stimulus to breathe is a(n): increased level of oxygen in the blood. decreased level of oxygen in the blood. increased level of carbon dioxide in the blood. decreased level of carbon dioxide in the blood

increased level of carbon dioxide in the blood.

All of the following are potential causes of impaired tissue perfusion EXCEPT: increased number of red blood cells. pump failure. low fluid volume. poor vessel function.

increased number of red blood cells.

You are attending to a 34-year-old male patient who requires transport to the hospital for assessment of his chronic back pain. The patient weighs over 750 pounds. Your bariatric stretcher has a wider surface area to allow for: A. increased stability and leverage when lifting with more than two providers. B. increased patient comfort and dignity. \ C. better stability when moving the patient on uneven ground. D. increased lifting capacity and patient weight load.

increased patient comfort and dignity.

In contrast to typical wheeled ambulance stretchers, features of a bariatric stretcher include: A. weight capacity of up to 650 lb. B. a collapsible undercarriage. C. two safety rails on both sides of the stretcher. D. increased stability from a wider wheelbase.

increased stability from a wider wheelbase.

pneumonia

infection of the lungs

Deoxygenated blood from the abdomen, pelvis, and lower extremities is returned to the right atrium via the:

inferior vena cava

full thickness (third degree) burns

injury completely through dermal layer dry, leathery skin, no pain

sprain

injury to a ligament usually involve the shoulder, knee or ankle joints

causation

injury to patient was in part due to EMT's breech of duty

Following a blunt injury to the head, a 22-year-old female is confused and complains of a severe headache and nausea. On the basis of these signs and symptoms, you should be MOST concerned with the possibility of: A. spinal cord injury. B. a fracture of the skull. C. airway compromise. D. intracranial bleeding.

intracranial bleeding

A factory worker was splashed in the eyes with a strong acid chemical. He complains of intense pain and blurred vision. Your ambulance does not carry bottles of sterile saline or water. You should: A. neutralize the acid chemical in his eye with an alkaline chemical. B. mix baking soda with water and irrigate his eyes with the solution. C. flush both eyes with an alcohol-based solution and transport. D. irrigate both eyes continuously for 20 minutes with plain water.

irrigate both eyes continuously for 20 minutes with plain water. Correct

A simple pneumothorax: A. is caused by penetrating chest trauma. B. is commonly caused by blunt chest trauma. C. often has a nontraumatic cause. D. heals on its own without any treatment.

is commonly caused by blunt chest trauma

When a light is shone into the pupil: A. it should become smaller in size. B. both pupils should dilate together. C. the opposite pupil should dilate. D. it should become larger in size.

it should become smaller in size. Correct

The superficial temporal artery can be palpated: A. over the mastoid process. B. just anterior to the tragus. C. at the angle of the jaw. D. slightly above the ear.

just anterior to the tragus. Correct

To minimize the risk of injuring yourself when lifting or moving a patient, you should: A. flex at the waist instead of the hips. B. avoid the use of log rolls or body drags. C. keep the weight as close to your body as possible. D. use a direct carry whenever possible.

keep the weight as close to your body as possible.

French catheter

known as "whistle tip" flexible catheter best suited for suctioning the nose stoma, or inside an advanced airway device

You arrive at the scene of a major motor vehicle crash. The patient, a 50 year old female, was removed form her vehicle prior to your arrival. Bystanders who removed her state that she was not wearing a seatbelt. The patient is unresponsive, tachycardic, and diaphoretic. Your assessment reveals bilaterally clear and equal breath sounds, a midline trachea, and collapsed jugular veins. You should be MOST suspicious that this patient has experienced a: A. pericardial tamponade. B. tension pneumothorax. C. laceration of the aorta. D. massive hemothorax.

laceration of the aorta

visceral pleura

lines the outer surface of the lungs

bruising to the right upper quadrant of the abdomen following blunt trauma is most suggestive of injury to the

liver

The ______________ is a circumferential chest compression device composed of a constricting band and backboard. mechanical piston device load-distributing band impedance threshold device cardiopulmonary resuscitation

load-distributing band

evaporation (body losing heat)

loss of heat through evaporation of water from the skin ex. getting out of the pool or shower

convection (body losing heat)

loss of heat to passing air ex. standing a cold breeze

ischium

lower portion of pelvis

You come upon an unresponsive patient who is not injured and is breathing on her own with a normal rate and an adequate tidal volume. What would be the advantage of placing her in the recovery position?

maintaining a clear airway

What is the lower jawbone called

mandible

People at higher risks for suicide include all the following except

married males older than 30

The upper jawbones are called the: A. mastoid. B. maxillae. C. zygoma. D. mandible.

maxillae. Correct

depressed skull fracture

may be noticeable among palpitation

When a person is lying supine at the end of exhalation, the diaphragm: A. is less prone to penetrating trauma. B. contracts and flattens inferiorly. C. descends below the level of the navel. D. may rise as high as the nipple line.

may rise as high as the nipple line

A body part that lies closer to the midline when compared to another is considered to be: medial. distal. lateral. proximal.

medial.

As an EMT, you may be authorized to administer aspirin to a patient with chest pain based on:

medical director approval

After administering 0.3 mg of epinephrine via auto-injector to a 22-year-old woman with an allergic reaction, you note improvement in her breathing and dissipation of her hives. However, she is still anxious and tachycardic. You should: A. contact medical control and obtain authorization to administer another 0.3 mg of epinephrine. B. monitor her closely but recall that anxiety and tachycardia are side effects of epinephrine. C. consider administering 0.15 mg of epinephrine to completely resolve her allergic reaction. D. transport her rapidly, as it is obvious that she is having a severe reaction to the epinephrine.

monitor her closely but recall that anxiety and tachycardia are side effects of epinephrine

ingested toxins

most common route of exposure

linear skull fracture

most skull fractures are this do not present with deformity or depression

cold zone

most treatment is performed here

polypharmacy

multiple medications usually geriatric patients have this

tonic phase of seizure

muscle rigidity, possible incontinence patient may be apneic

While en route to a major motor vehicle crash, an on-scene police officer advises you that a 6-year-old male who was riding in the front seat is involved. He further states that the child was only wearing a lap belt and that the air bag deployed. On the basis of this information, you should be MOST suspicious that the child has experienced: A. blunt trauma to the head. B. lower extremity fractures. C. open abdominal trauma. D. neck and facial injuries.

neck and facial injuries

insulin

needed to efficiently move glucose out of the bloodstream

Failure of the EMT to provide the same care as another EMT with the same training is called: libel. slander. negligence. abandonment.

negligence.

During the normal wound healing process, bleeding may occur from even a minor injury because:

new capillaries that stem from intact capillaries are delicate and take time to become as stable as the pre-existing ones

simple partial seizure

no change in LOC, possible twitching or sensory changes, but no full-body convulsions

apnea

no spontaneous breathing

supine hypotensive syndrome

occurs when the fetus compresses the inferior vena cava

eclampsia

occurs when the mother seizes following preeclampsia

placenta previa

occurs when the placenta attaches to the uterus over the cervical opening and as the cervix dilates, the placenta tears and bleeds *painless* vaginal bleeding

As you approach a young male who was involved in an industrial accident, you note that his eyes are closed and that he is not moving. You can see several large contusions to his arms, a laceration to his forehead with minimal bleeding, and a closed deformity to his right leg. You should: A. assess his pulse for rate, regularity, and quality. B. open his airway and assess his breathing status. C. apply high-flow oxygen and assess his injuries. D. perform an immediate head-to-toe assessment.

open his airway and assess his breathing status. Correct

ulcers

open wounds along the digestive tract, often the stomach

foramen magnum

opening in the occipital bone where brain connects to spinal cord

enteral route

oral (by mouth) slow onset of action Ex. aspirin, activated charcoal, oral glucose

You respond to a construction site and find a worker lying supine in the dirt. He has been hit by a heavy construction vehicle and flew more than 15 feet (4.6 m) before landing in his current position. There is discoloration and distention of his abdomen about the right upper quadrant. He is unconscious and his respirations are 10 breaths/min and shallow, with noisy gurgling sounds. What method will you use to keep his airway open?

oropharyngeal airway

Self-concept is: our perception of ourselves. how we feel about ourselves. how we fit in with peers. how we react to certain situations.

our perception of ourselves.

recovery position

patient positioned on their side reduces risk of aspiration

Burns to pediatric patients are generally considered more serious than burns to adults because: A. most burns in children are the result of child abuse. B. pediatric patients are more prone to hyperthermia. C. pediatric patients have a proportionately larger volume of blood. D. pediatric patients have more surface area relative to total body mass.

pediatric patients have more surface area relative to total body mass. Correct

Your patient complains of chronic "burning" stomach pain that improves after eating. You should suspect: A. peptic ulcer disease. B. kidney stones. C. aortic aneurysm. D. pneumonia.

peptic ulcer disease.

A 30-year-old semiconscious man is pinned by the steering wheel of his badly wrecked vehicle. Once access has been gained to the patient, the EMT should: have the fire department disentangle the patient and quickly remove him from the vehicle. immediately apply high-flow oxygen to the patient and then allow extrication to begin. perform a primary assessment and provide any needed emergency care prior to extrication. ensure that the patient is not bleeding significantly before allowing the extrication process to commence.

perform a primary assessment and provide any needed emergency care prior to extrication.

Your primary assessment of an elderly woman who fell reveals an altered level of consciousness and a large hematoma to her forehead. After protecting her spine and administering oxygen, you should: reassess your interventions. perform a rapid exam. transport the patient immediately. perform a focused assessment of her head.

perform a rapid exam.

informed consent

required from all patients who are alert and competent

distributive shock

pipe problem occurs due to widespread vasodilation, causes blood pooling and relative hypovolemia

If a patient with an implanted pacemaker is in cardiac arrest, the EMT should: avoid defibrillation with the AED and transport at once. not apply the AED until he or she contacts medical control. place the AED pads away from the pacemaker. apply the AED pads directly over the implanted pacemaker.

place the AED pads away from the pacemaker.

When moving a conscious, weak patient down a flight of stairs, you should: A. place the wheeled stretcher at the bottom of the stairs and carry the patient down the stairs with a stair chair. B. secure the patient to a scoop stretcher and carry him or her headfirst down the stairs to the awaiting stretcher. C. collapse the undercarriage of the wheeled stretcher and carefully carry the patient down the stairs on the stretcher. D. assist the patient in walking down the stairs and place him or her on the wheeled stretcher at the bottom of the stairs.

place the wheeled stretcher at the bottom of the stairs and carry the patient down the stairs with a stair chair.

When lifting a stretcher using the power lift, you should: bend at the hips, knees, back, and arms. bend at the waist and keep your back straight. place your hands palms up on the litter handle. place your hands palms down on the litter's side bars.

place your hands palms up on the litter handle.

In order to avoid exacerbating a patients injury, it is especially important to use extreme caution when providing positive-pressure ventilation to patients with a: A. pneumothorax. B. flail chest. C. cardiac tamponade. D. myocardial contusion.

pneumothorax

When two EMTs are lifting a patient on a long backboard, they should: lift the patient from the sides of the board. make every attempt to lift with their backs. position the strongest EMT at the foot of the board. position the strongest EMT at the head of the board.

position the strongest EMT at the head of the board.

biceps femoris

posterior femur, part of hamstrings

A 28 year old male was struck in the chest with a baseball bat during an altercation. He is conscious and alert and complains of severe chest pain. Your assessment reveals a large area of ecchymosis over the sternum and a rapid, irregular pulse. In addition to providing supplemental oxygen, you should: A. determine if he has cardiac problems. B. apply an AED and assess his BP. C. apply bulky dressings to the sternum. D. prepare for immediate transport.

prepare for immediate transport

Immediately life-threatening chest injuries must be found and managed during the _________? A. secondary assessment B. patient history C. scene size-up D. primary assessment

primary assessment

Findings such as inadequate breathing or an altered level of consciousness should be identified in the: primary assessment. focused assessment. secondary assessment. reassessment.

primary assessment.

When assessing a patient who experienced a blast injury, it is important to remember that: A. primary blast injuries are typically the most obvious. B. solid organs usually rupture from the pressure wave. C. secondary blast injuries are usually the least obvious. D. primary blast injuries are the most easily overlooked.

primary blast injuries are the most easily overlooked

carbon dioxide drive

primary mechanism of breathing control for most people high CO2 levels (monitored by brain stem) will stimulate an increase in respiration

sinoatrial (SA) node

primary power plant of heart generates electrical impulses between 60-100 times per minute

germinal layer of epidermis

produces new cells and pushes them to the surface

status epilepticus

prolonged seizure (about 30 min) or recurring seizures without regained consciousness

expressed consent

requires that patient be alert and competent and can be given verbally or nonverbally not in-depth

An important aspect in the treatment of a patient with severe abdominal pain is to: A. encourage the patient to remain in a supine position. B. administer analgesic medications to alleviate pain. C. give 100% oxygen only if signs of shock are present. D. provide emotional support en route to the hospital.

provide emotional support en route to the hospital.

All of the following are functions of the emergency medical dispatcher, EXCEPT: alerting the appropriate EMS response unit. screening a call and assigning it a priority. providing emergency medical instructions to the caller. providing medical direction to the EMT in the field.

providing medical direction to the EMT in the field.

A man called EMS 12 hours after injuring his chest. Your assessment reveals a flail segment to the right side of the chest. The patient is experiencing respiratory distress and his oxygen saturations is 78%. His breath sounds are equal bilaterally and his jugular veins are normal. You should suspect: A. massive hemothorax. B. traumatic asphyxia. C. tension pneumothorax. D. pulmonary contusion.

pulmonary contusion

When the speed of a motor vehicle doubles, the amount of kinetic energy: A. quadruples. B. triples. C. is not affected. D. doubles.

quadruples

When an allergic reaction proceeds to life-threatening anaphylaxis, it will usually do so __________. A. slowly, over more than 30 minutes. B. quickly, within 30 seconds. C. slowly, over several hours. D. quickly, within 30 minutes.

quickly, within 30 minutes

A 6-year-old female was riding her bicycle and struck a clothesline with her throat. She is breathing, but with obvious difficulty. Your assessment reveals a crackling sensation in the soft tissues of her neck and facial cyanosis. In addition to the appropriate airway management, the intervention that will MOST likely improve her chance of survival is: A. quickly immobilizing her spinal column. B. rapidly transporting her to the hospital. C. requesting a paramedic ambulance. D. careful monitoring her vital signs.

rapidly transporting her to the hospital. Correct

The kidneys help to regulate blood pressure by

removing sodium and water from the body.

After receiving an order from medical control over the radio, the EMT should: carry out the order immediately. disregard the order if it is not understood. obtain the necessary consent from the patient. repeat the order to the physician word for word.

repeat the order to the physician word for word.

You are managing a 62-year-old woman who complains of crushing chest pain. Her blood pressure is 84/64 mm Hg and her heart rate is 110 beats/min. Medical control advises you to assist her in taking her prescribed nitroglycerin. After receiving this order, you should: reassess the patient's heart rate and then assist with the nitroglycerin. repeat the patient's blood pressure to the physician and confirm the order. wait 10 minutes, reassess the blood pressure, and then give the nitroglycerin. administer the nitroglycerin to the patient and then reassess her blood pressure.

repeat the patient's blood pressure to the physician and confirm the order.

The process in which oxygen and carbon dioxide are exchanged in the lungs is called: respiration. ventilation. metabolism. inhalation.

respiration.

three key assessments for newborns

respirations, heart rate, and color get the heart rate above 100 beats/min

In most cases, cardiac arrest in children results from

respiratory arrest

respiratory syncytial virus (RSV)

respiratory infection common in infants and children signs/symptoms: childlike symptoms, poor fluid intake, signs of dehydration

minute volume

respiratory rate x tidal volume

A semiconscious patient pushes your hand away when you pinch his earlobe. You should describe his level of consciousness as: alert. unresponsive. responsive to painful stimuli. responsive to verbal stimuli.

responsive to painful stimuli.

early indications of hypoxia

restlessness, anxiety, irritability, dyspnea, tachycardia

A 38-year-old male with a history of schizophrenia is reported by neighbors to be screaming and throwing things in his house. You are familiar with the patient and have cared for him in the past for unrelated problems. Law enforcement officers escort you into the residence when you arrive. The patient tells you that he sees vampires and is attempting to ward them off by screaming and throwing things at them. He has several large lacerations to his forearms that are actively bleeding. The MOST appropriate way to manage this situation is to: A. request that the police officers arrest him and take him to the hospital. B. approach the patient and calm him by placing your hand on his shoulder. C. restrain the patient with appropriate force in order to treat his injuries. D. try to gain the patient's trust by telling him that you see the vampires too.

restrain the patient with appropriate force in order to treat his injuries.

You arrive at the scene of an "injured person." As you exit the ambulance, you see a man lying on the front porch of his house. He appears to have been shot in the head and is lying in a pool of blood. You should: immediately assess the patient. proceed to the patient with caution. quickly assess the scene for a gun. retreat to a safe place and wait for law enforcement to arrive.

retreat to a safe place and wait for law enforcement to arrive.

Upon arriving at the scene of a domestic dispute, you hear yelling and the sound of breaking glass from inside the residence. You should: immediately gain access to the patient. carefully enter the house and then call the police. retreat to a safe place until the police arrive. tell the patient to exit the residence so you can provide care.

retreat to a safe place until the police arrive.

As you approach an unconscious patient who is still in her wrecked vehicle, you note that there is a power line entangled in the wreckage of the vehicle. You should: retreat until the power line has been removed or the power is shut off. carefully gain access to the patient without touching any metal objects. don a pair of rubber gloves and carefully disentangle and remove the power line. call for a tow truck to lift the vehicle off of the power line and then access the patient.

retreat until the power line has been removed or the power is shut off.

six "rights" of drug administration

right patient right drug right route right amount right time right documentation

The ______ is both the mechanical weight-bearing base of the spinal column and the fused central posterior section of the pelvic girdle. A. thorax B. sacrum C. ischium D. coccyx

sacrum

abrasion (open injury)

scrape to the skin due to surface friction

A 15-year-old female was struck by a small car while riding her bicycle. She was wearing a helmet and was thrown to the ground, striking her head. In addition to managing problems associated with airway, breathing, and circulation, it is MOST important for you to: A. obtain baseline vital signs. B. inspect the helmet for cracks. C. stabilize her entire spine. D. leave her bicycle helmet on.

stabilize her entire spine

Protective measures that prevent health care workers from coming into contact with germs are referred to as: exposure. standard precautions. transmission. PPE.

standard precautions.

homeostasis

state of balance or equilibrium within the body

You are attending to a 27-year-old patient with a history of depression. The patient's family tells you that she has been openly talking about harming herself and suicide, and they got scared when she tried to overdose on some medications. The patient did not take the pills and is alert and oriented. Despite all of your best efforts to convince her, the patient refuses to go to hospital for treatment. Based on this information, you should: A. call for the police to attend the scene and, once they arrive, release responsibility for patient transport to them B. restrain the patient and transport her to the hospital as she represents a threat to her safety. C. ensure that the patient is competent and completes the required documentation, and leave her in the care of her family. D. stay with the patient while you arrange for other transport options as a potentially life-threatening emergency exists.

stay with the patient while you arrange for other transport options as a potentially life-threatening emergency exists.

When caring for a patient experiencing excited delirium, the EMT should remember that: A. most patients will have low blood pressure and hyperglycemia. B. excited delirium is worsened by nervous system depressant drugs. C. lights and siren are effective in redirecting the patient's behavior. D. sudden death can occur if the patient's violence is not controlled.

sudden death can occur if the patient's violence is not controlled.

A _____ is an opening that connects the trachea directly to the skin

stoma

As you and your partner are carrying a stable patient down a flight of stairs in a stair chair, you feel a sudden, sharp pain in your lower back. You should: A. stop the move and have the patient walk down the stairs. B. reposition your hands and continue to move the patient. C. stop the move and request additional lifting assistance. D. guide your partner while moving the chair backwards.

stop the move and request additional lifting assistance.

extension

straightening the joint (increasing the angle at the joint)

strain

stretching injury to a muscle or tendon

Cardiac output is affected by

stroke volume and heart rate

pharmacodynamics

study of drug's effects on the body

Pharmacology is defined as the: study of cells and tissues. study of drugs and medications. effects of medications in the lungs. distribution of drugs to the body's tissues.

study of drugs and medications.

pharmacokinetics

study of how drugs enter the body and are metabolized/eliminated

sublingual route

under the tongue faster onset than oral Ex. nitroglycerin

A burn that is characterized by redness and pain is classified as a: A. second-degree burn. B. partial-thickness burn. C. superficial burn. D. full-thickness burn.

superficial burn. Correct

For CPR to be effective, the patient must be on a firm surface, lying in the ______________ position. Fowler's prone supine recovery

supine

Activated charcoal is an example of a(n): Select one: A. elixir. B. suspension. C. solution. D. gel.

suspension

A. have chronic hypertension.

symptoms of a hypertensive emergency would MOST likely be delayed in patients who: Select one: A. have chronic hypertension. B. regularly take illegal drugs. C. have had a stroke in the past. D. are older than 40 years of age.

Patients with chest injuries will often present with _______? A. Cheyne-Stokes respirations B. tachypnea C. agonal respirations D. Kussmaul respirations

tachypnea

It is impractical to apply a vest-type extrication device on a critically injured patient to remove him or her from a wrecked vehicle because it: takes too long to correctly apply. does not fully immobilize the spine. cannot be used on patients who are in their car. does not provide adequate stabilization.

takes too long to correctly apply.

The term "weaponization" is defined as: the period of time that a chemical agent will remain on a given surface before it evaporates. the method or mechanism by which a terrorist or terrorist group delivers a chemical or biologic agent. the cultivation, synthesization, and/or mutation of an agent in order to maximize the target population's exposure. the detonation of an explosive device utilizing an item that is inconspicuous, such as a briefcase or suitcase.

the cultivation, synthesization, and/or mutation of an agent in order to maximize the target population's exposure.

standard of care

the degree of care a reasonable person with similar training would provide in a similar situation

compensated shock

the early stage of shock body can still compensate for the hypovolemic state by increasing heart rate and vasoconstriction

To extract a patient from the basement of a building, you must transport the patient up a flight of stairs. In doing this, you must ensure that: the elevated head of the backboard goes first. the backboard with the elevated foot end goes first. the backboard is slightly tilted to the left to distribute weight. the patient's feet are higher than his or her head, whichever end is carried first.

the elevated head of the backboard goes first.

nuchal cord

the umbilical cord is wrapped around the baby's neck remove it from neck if able to

According to the START triage system, which of the following patients should be triaged as an immediate priority

unresponsive with res rate of 34 bpm

According to the Association of Air Medical Services, you should consider air medical transport of a trauma patient if: A. ground transport will take your ambulance out of service for an extended period of time, regardless of the severity of the patient's injuries. B. he or she was involved in a motor vehicle crash in which another occupant in the same vehicle was killed, even if your patient's injuries are minor. C. traffic conditions hamper the ability to get the patient to a trauma center by ground within the ideal time frame for the best clinical outcome. D. the patient requires advanced life support care and stabilization, and the nearest ALS-ground ambulance is more than 5 to 10 minutes away.

traffic conditions hamper the ability to get the patient to a trauma center by ground within the ideal time frame for the best clinical outcome

radiation (body losing heat)

transfer of radiant heat ex. entering a walk-in freezer

A 4-year-old female has a peanut lodged in the external auditory canal of her right ear. You should: A. transport her to the emergency department. B. thoroughly flush her ear with sterile saline. C. remove the peanut with a cotton-tipped swab. D. use tweezers to try to remove the object.

transport her to the emergency department. Correct

You respond to a residence for a child who is having a seizure. Upon arrival at the scene, you enter the residence and find the mother holding her child, a 2-year-old male. The child is conscious and crying. According to the mother, the child had been running a high fever and then experienced a seizure that lasted approximately 3 minutes. You should: A. transport the child to the hospital and reassure the mother en route. B. cover the child with wet towels and give oxygen via nasal cannula. C. call medical control and request permission to give the child aspirin. D. advise the mother to take her child to the doctor the following day.

transport the child to the hospital and reassure the mother en route

If an injury distracts an EMT from assessing a more serious underlying illness, the EMT has suffered from: tunnel vision. index of suspicion. virulence. a trauma emergency.

tunnel vision.

pleura

two tine, smooth layers of tissue with thin film of fluid in between to allow frictionless movement across one another

C. the inner layers of the aorta become separated.

when: Select one: A. all layers of the aorta suddenly contract. B. a weakened area develops in the aortic wall. C. the inner layers of the aorta become separated. D. the aorta ruptures, resulting in profound bleeding.

advance directives

written instructions, signed by the patient, specifying their wishes regarding treatment and resuscitation efforts

pubis

anterior portion of pelvis

hypoxic drive

backup system to the CO2 drive low oxygen levels stimulate breathing

Most terrorist attacks:

require multiple terrorists working together

nasal cannula

"low flow" oxygen flow rate: 1-6 LPM delivers 24-44% oxygen

psychogenic shock (distributive shock)

"pseudo-shock" caused by sudden, temporary vasodilation that leads to syncope (fainting)

rales

"wet" or "crackling" sounds

Which of the following is MOST characteristic of adequate breathing? -20 breaths/min with shallow movement of the chest wall and pallor -24 breaths/min with bilaterally equal breath sounds and pink skin -22 breaths/min with an irregular pattern of breathing and cyanosis -30 breaths.min with supraclavicular retractions and clammy skin

-24 breaths/min with bilaterally equal breath sounds and pink skin Page 587, Pathophysiology

The rapid exam of a patient that occurs following the primary assessment should take no longer than: -30 seconds. -90 to 120 seconds. -60 to 90 seconds. -120 to 180 seconds.

-60 to 90 seconds. Page 332, Primary Assessment

Which of the following conditions would be LEAST likely to cause an altered level of consciousness? -Poisoning. -Drug overdose. -Acute anxiety. -Inadequate perfusion.

-Acute anxiety. Page 324, Primary Assessment

In what area of the lungs does respiration occur? -Bronchi -Alveoli -Trachea -Capillaries

-Alveoli Page 585, Anatomy of the Respiratory System

Which of the following is the ONLY action that can prevent eventual death from a tension pneumothorax? -Early administration of high-flow oxygen. -Positive-pressure ventilation with a bag-valve mask. -Decompression of the injured side of the chest. -Rapid administration of intravenous fluids.

-Decompression of the injured side of the chest. Page 502, Emergency Medical Care for Shock

When should nonlifesaving interventions be performed for your multisystem trauma patient? -Prior to transport. -En route to the hospital. -During the primary assessment. -Immediately after the injuries are discovered.

-En route to the hospital. Page 498, Patient Assessment for Shock

What maneuver should be used to open the airway of an unresponsive patient with suspected trauma? -Head tilt-chin lift. -Head tilt-neck left. -Tongue-jaw lift. -Jaw-thrust maneuver.

-Jaw-thrust maneuver. Page 327, Primary Assessment

Which of the following must be assessed in every respiratory patient? -Blood glucose levels -Lung sounds -Orthostatic vital signs -Distal pulse, motor, sensation

-Lung sounds Page 604, Patient Assessment

Which of the following is bacterium resistant to most antibiotics and causes akin abscesses? -MRSA -Avian flu -Whooping cough -H1N1

-MRSA Page 575, Common or Serioud Communicaable Diseases

When assessing a patient with a medical complaint, which of the following would MOST likely reveal the case of the problem? -Medical history. -Primary assessment. -Baseline vital signs. -Index of suspicion.

-Medical history. Page 564, Patient Assessment

An injured patient is assigned a total score of 9 on the GCS. He is assigned a score of 2 eye opening, a score of 3 for verbal response, and a score of 4 for motor response. Which of the following clinical findings is consistent with his GCS score? -Eyes remain closed, makes incomprehensible sounds, exhibits abnormal extension. -Opens eyes in response to pain, uses inappropriate words, withdraws from pain. -Opens eyes in response to voice, makes incomprehensible sounds, localizes pain. -Opens eyes spontaneously, is confused when spoken to, exhibits abnormal flexion.

-Opens eyes in response to pain, uses inappropriate words, withdraws from pain. Page 364, Secondary Assessment

A 25-year-old unrestrained female struck the steering wheel with her chest when her car hit a tree while traveling at a high rate of speed. She has signs and symptoms of chock, which you suspect are the result of intrathoracic bleeding. Which of the following interventions will provide this patient with the greatest chance for survival? -High-flow oxygen administration. -Intravenous fluid administration. -Full immobilization of her spine. -Rapid transport to a trauma center.

-Rapid transport to a trauma center. Page 498, Patient Assessment for Shock

Which of the following actions would NOT be performed during the scene size-up? -Notifying the dispatcher to send fire personnel. -Noting the position of a crashed motor vehicle. -Asking a neighbor to secure the patient's dog. -Rapidly assessing a patient's respiratory status.

-Rapidly assessing a patient's respiratory status. Page 317, Scene Size-Up

Which of the following statements regarding the secondary assessment is correct? -During the secondary assessment, the EMT's primary focus should be on taking the patient's vital signs and obtaining a SAMPLE history. -The secondary assessment should be performed en route to the hospital, regardless of the severity of the patient's condition. -A secondary assessment should always be performed, even if yo must continually manage life threats that were identified in the primary assessment. -The secondary assessment should focus on a certain area or region of the body as determined by the chief complaint.

-The secondary assessment should focus on a certain area or region of the body as determined by the chief complaint. Page 348, Secondary Assessment

Most patients with an infectious disease will have ______. -seizures. -a fever. -a low blood glucose level. -abdominal pain.

-a fever. -Page 596, Infectious Diseases

An infectious disease is MOST accurately defined as: -a medical condition caused by the growth and spread of small, harmful organisms within the body. -any disease that enters the body via the bloodstream and renders the immune system nonfunctional. -the invasion of the human body by a bacterium that cannot be destroyed by antibiotics or other drugs. -a disease that can be spread from one person or species to another through a number of mechanisms.

-a medical condition caused by the growth and spread of small, harmful organisms within the body. Page 569, Infectious Diseases

When auscultating the lungs of a patient with respiratory distress, you hear adventitious sounds. This means that the patient has: -abnormal breath sounds -normal breath sounds -diminished breath sounds -an absence of breath sounds

-abnormal breath sounds Page 604, Patient Assessment

Most treatments provided in the prehospital setting are intended to ______. -address the patient's symptoms. -correct the patient's underlying problem. -reduce the need for transport to the hospital. -conform the patient's diagnosis.

-address the patient's symptoms. Page 567, Management, Transport, and Destination

The determination of whether a medical patient is a high-priority or low-priority transport is typically made: -as soon as the patient voices his or her chief complaint. -upon completion of a detailed secondary assessment. -once the patient's baseline vital signs are known. -after the primary assessment has been completed.

-after the primary assessment has been completed. Page 568, Management, Transport, and Destination

Crackles (rales) are caused by ______. -severe bronchoconstriction -air passing through fluid -narrowing of the upper airways -mucus in the larger airways

-air passing through fluid Page 604, Patient Assessment

An elderly patient has fallen and hit her head. Your initial care should focus on: -airway, breathing, and circulation. -gathering medical history data. -providing immediate transport. -obtaining baseline vital signs.

-airway, breathing, and circulation. Page 324, Primary Assessment

Treatment and transport priorities at the scene of a mass-casualty incident should be determined after: -area hospitals have been notified. -all the patients have been triaged. -a physician arrives at the scene. -the number of patients is known.

-all the patients have been triaged. Page 321, Scene Size-Up

Foods, medications, and insiects are common causes of: -anaphylactic shock. -neurogenic shock. -psychogenic shock, -septic shock.

-anaphylactic shock. Page 494, Types of Shock

A 20-year-old male has a large laceration to his wrist. He is holding a blood-soaked towel over the wound, but it continues to bleed rapidly. You should: -administer high-flow supplemental oxygen. -wrap the towel with pressure bandages. -apply a tourniquet proximal to the wrist. -apply pressure to the brachial artery.

-apply a tourniquet proximal to the wrist. Page 504, Emergency Medical Care for Shock

A 39-year-old male sustained a stab would to the groin during an altercation at a bar. As you approach the patient, you note that he is conscious. He is screaming in pain and is attempting to control the bleeding, which is bright red and spurting from his groin area. You should: -apply direct pressure to the wound. -elevate his legs and keep him warm. -administer 100% supplemental oxygen. -ensure that his airway is patent.

-apply direct pressure to the wound. Page 332, Primary Assessment

When you shine a light into one pupil, the normal reaction of the other pupil should be to: -dilate. -become larger. -become smaller. -not react.

-become smaller. Page 364-365, Secondary Assessment

During an EMS call, you should take standard precautions: -immediately after completion of your primary assessment. -after it has been determined that the patient is bleeding. -before you load the patient into the ambulance. -before exiting the ambulance and before actual patient contact.

-before exiting the ambulance and before actual patient contact. Page 320, Scene Size-Up

When assessing a patient with signs and symptoms of shock, it is important to remember that: -blood pressure may be the last measurable factor to change in shock. -irreversible chock often responds well to a prompt blood transfusion. -the patient's respirations are deep during the early stages of shock. -multiple fractures are the most common cause of hypovolemic shock.

-blood pressure may be the last measurable factor to change in shock. Page 496, The Progression of Shock

His parents tell you that their son has had a chest infection for the past two days and when they took him to their family doctor, they were told it was likely due to the respiratory syncytial virus (RSV). They have kept him well hydrated, but the infection seems to have gotten worse. On auscultation, you hear decreased air entry bilaterally with fine expiratory wheezes and the occasional coarse wet crackle. Based on this information, your patient is most likely suffering from: -bronchiolitis -pertussis -croup -epiglottis

-bronchiolitis Page 591-592, Causes of Dyspnea

Your patient has a decreased cardiac output and poor myocardial contractility. This will likely lead to ______. -septic shock. -cardogenic shock. -hypovolemic shock. -neurogenic shock.

-cardiogenic shock. Page 491, Types of Shock

Pulmonary edema and impaired ventilation occur during: -neurogenic shock. -septic shock. -anaphylactic shock. -cardiogenic shock.

-cardogenic shock. Page 491, Types of Shock

When treating an 80-year-old patient who is in shock, it is important to remember that: -changes in gastric motility may delay gastric emptying, which increases the risk for vomiting. -compensation from the respiratory system usually manifests with increased tidal volume. -medications older patients take for hypertension often cause an unusually hast heart rate. -the older patient's central nervous system usually more briskly to compensate for shock.

-changes in gastric motility may delay gastric emptying, which increases the risk for vomiting. Page 505, Emergency Medical Care for Shock

You are attempting to a 27-year-old male driver of a car. According to his passenger, the patient had been acting strangely while driving, then slumped forward against wheel, apparently unconscious. The car drove off the road and struck a telephone pole. The patient remains unconscious, and physical assessment reveals only a large hematoma on his right forehead with no other physical signs. Your patient is a diabetic who had been under a lot of stress lately and may have missed meals. This is an example of a: -combination of a psychiatric and trauma emergency. -trauma emergency. -combination of a medical and trauma emergency. -medical emergency.

-combination of a medical and trauma emergency. Page 561, Introduction

Your patient is in shock, but the body's defense mechanisms are currently able to maintain adequate circulation. This is called ______. -decompensated shock. -late shock. -compensated shock. -irreversible shock.

-compensated shock. Page 496, The Progression of Shock

When approaching a 32-year-old male who is complaining of traumatic neck pain, you should: -ensure that the patient can see you approaching him. -stand behind him and immediately stabilize his head. -approach him from behind and ask him not to move. -assess his mental status by having hi move his head.

-ensure that the patient can see you approaching him. Page 323, Primary Assessment

A 30-year-old male presents with acute shortness of breath, widespread hives, and facial swelling. He denies any past medical history and takes no medications. During your assessment, you hear wheezing over all the lung fields. His blood pressure is 90/50 mm Hg and his heart rate 110 beats/min. In addition to giving him high-flow oxygen, the MOST important treatment for this patient is: -a beta-antagonist -an antihistamine -albuterol -epinephrine

-epinephrine Page 598, Causes of Dyspnea

A pleural effusion is MOST accurately defined as: -fluid accumulation outside the lung -diffuse collapsing of the alveoli -a bacterial infection of the lung tissue -a unilaterally collapsed lung

-fluid accumulation outside the lung Page 599, Causes of Dyspnea

The BEST way to prevent infection from whooping cough is to: -get vaccinated against diphtheria, tetanus, and pertussis. -ask all patients if they have recently traveled abroad. -wear a HEPA mask when treating any respiratory patient. -routinely place a surgical mask on all respiratory patients.

-get vaccinated against diphtheria, tetanus, and pertussis. Page 575, Common or Serious Communicable Diseases

Hepatitis B is more virulent than hepatitis C, which means that it: -leads to chronic infection after exposure. -is less resistant to treatment. -has a greater ability to produce disease. -is a more contagious type of disease.

-has a greater ability to produce disease. Page 573, Common or Serious Communicable Diseases

You should suspect that a patient is experiencing respiratory failure if he or she: -is anxious, tachycardic, and leaning forward. -has an increased heart rate and retractions. -is restless and is working hard to breathe. -has bradycardia and diminished muscle tone.

-has bradycardia and diminished muscle tone. Page 329, Primary Assessment

What are the three components of the "perfusion triangle"? -arteries, veins, capillaries. -plasma, red blood cells, platelets. -heart, brain, lungs. -heart, blood vessels, blood.

-heart, blood vessels, blood. Page 488, Pathophysiology

In an acute injury setting, neurogenic chock is commonly accompanied by: -tachycardia. -hypothermia. -hypovolemia. -diaphoresis.

-hypothermia. Page 493, Types of Shock

Your patient has a chronic respiratory condition. His stimulus to breathe is triggered by low oxygen levels in the blood. This is known as the ______. -alternate drive -hypoxic drive -CO2 drive -COPD drive

-hypoxic drive Page 589, Pathophysiology

Asthma is caused by a response of the: -respiratory system -endocrine system -immune system -cardiovascular system

-immune system Page 595, Causes of Dyspnea

You are assessing an asthma patient with his prescribed metered-dose inhaler. After the patient takes a deep breath and depresses the inhaler, you should: -instruct him to hold his breath for as long as he comfortably can -allow him to breathe room air and assess his oxygen saturation -advise him to exhale forcefully to ensure medication absorption

-instruct him to hold his breath for as long as he comfortably can Page 612, Emergency Medical Care

When caring for a patient with an altered mental status and signs of circulatory compromise, you should: -limit your time at the scene to 10 minutes or less, if possible. -transport immediately and begin all emergency treatment en route to the hospital. -have a paramedic unit respond to the scene if it less than 15 minutes away. -perform a detailed secondary assessment prior to transporting the patient.

-limit your time at the scene to 10 minutes or less, if possible. Page 567, Management, Transport, and Destination

You and your EMT partner arrive at the residence of a 50-year-old man who complains of weakness. Your primary assessment reveals that he is critically ill and will require aggressive treatment. The closest hospital is 25 miles away. You should: -load him into the ambulance, begin transport, and perform all treatment en route to the hospital. -perform a detailed secondary assessment, assess his vital signs, and then transport rapidly. -manage all threats to airway, and circulation and consider requesting an ALS unit. -administer oxygen via nonbreathing mask and obtain as much of his medical history as possible.

-manage all threats to airway, breathing, and circulation and consider requesting an ALS unit. Page 564, Patient Assessment

The chief complaint is MOST accurately defined as the: -most life-threatening condition that you discover. -most serious thing the patient is concerned about. -gross physical signs that you detect on assessment. -condition that exacerbates an underlying problem.

-most serious thing that the patient is concerned about. Page 338, History Taking

Assessment of the medical patient is usually focused on the ______. -medical history. -field diagnosis. -nature of illness. -associated symptoms.

-nature of illness. Page 562, Patient Assessment

It is especially important to assess pulse, sensation, and movement in all extremities as well as pupillary reactions in patients with a suspected ______ problem. -cardiac. -respiratory. -neurologic. -endocrine.

-neurologic. Page 565, Patient Assessment

In order for efficient pulmonary gas exchange to occur: -the percentage of inhaled carbon dioxide must exceed the percentage of inhaled oxygen. -the pulmonary capillaries must be completely constricted and the alveoli must be collapsed. -there must be low quantities of pulmonary surfactant to allow for full alveolar expansion. -oxygen and carbon dioxide must be able to freely diffuse across the alveolar-capillary membrane.

-oxygen and carbon dioxide must be able to freely diffuse across the alveolar-capillary membrane. Page 586, Physiology of Respiration

In 2009, the H1N1 virus accounted for over 200,000 deaths worldwide in the form of the swine flu. In 1919, a similar outbreak of the H1N1 occured in the form of the Spanish flu. Starting in Kansas City, the virus spread rapidly worldwide, claiming up to 50 million lives. These are both examples of: -epidemics. -uncontrolled outbreaks. -parasitic infection. -pandemics.

-pandemics. Page 569, Infectious Disease

You suspect your patient is in shock. You note the patient's skin is pale. This is likely due to ______. -an increased heart rate. -hypothermia. -peripheral vasodilation. -peripheral vasoconstriction.

-peripheral vasoconstriction. Page 490, Pathophysiology

Hypovolemic shock caused by severe burns is the result of a loss of: -plasma. -red blood vessels. -whole blood. -platelets.

-plasma. Page 495, Types of Shock

You are assessing a 72-year-old man with abdominal pain. The patient is sitting in a chair; he is conscious, alert, and calm. As you are talking to the patient, your partner discreetly directs your attention to a handgun, which is located on a a nearby table. You should: -document the presence of the weapon, including its specific location, and continue your assessment of the patient. -position yourself in between the patient and the gun and ask your partner to request law enforcement assistance. -direct your partner to move the gun to a safe area and then advise the patient that his weapon has been secured. -immediately cease all patient care, carefully back out of the residence, and request law enforcement assistance.

-position yourself in between the patient and the gun and ask your partner to request law enforcement assistance. Page 318, Scene Size-Up

A palpable pulse is created by: -pressure waves through the arteries caused by cardiac contraction -electrical conduction in the heart producing ventricular contraction -the pressure of circulating blood against the walls of the arteries -the pressure that is caused when venous blood returns to the heart

-pressure waves through the arteries caused by cardiac contraction. Page 329, Primary Assessment

"PASTE" is an alternate assessment tool for ______. -seizure patients -respiratory patients -stroke patients -cardiac patients

-respiratory patients Page 607, Patient Assessment

Shock due to severe infection is called: -septic shock. -neurogenic shock. -anaphylactic shock. -hypovolemic shock.

-septic shock. Page 493, Types of Shock

Inadequate circulation of blood throughout the body is called ______. -hypoxia. -shock. -hypotension. -perfusion.

-shock. Page 487, Introduction

After sizing up the scene of a patient with a possible infectious disease, your next priority should be to: -take standard precautions. -contact medical control. -notify law enforcement. -quickly access the patient.

-take standard precautions. Page 569, Infectious Diseases

When forming your general impression of a patient with a medical complaint, it is important to remember that: -the conditions of many medical patients may not appear serious at first. -it is during the general impression that assessment of the ABCs occurs. -most serious medical conditions do not present with obvious symptoms. -the majority of medical patients you encounter are also injured.

-the conditions of many medical patients may not appear serious at first. Page 563, Patient Assessment

Reassessment is performed to determine all of the following, EXCEPT: -the nature of any newly identified problems. -the reason why the patient called EMS. -whether or not the patient is deteriorating. -the patient's response to your treatment.

-the reason why the patient called EMS. Page 375, Reassessment

To protect vital organs, the body compensates by directing blood flow away from organs that are more tolerant of low flow, such as: -the heart. -the brain. -the skin. -the lungs.

-the skin. Page 488, Pathophysiology

You are attending to a 54-year-old female patient in a homelss shelter. The patient tells you that she had the flu a couple of weeks ago, and she has not gotten over it. She has been tired and keeps waking up at night, sweating. She has been coughing up green sputum occasionally and has been experiencing episodes of chest pain that get worse when she breathes. Based on this information, your patient is most likely suffering from: -influenza Type A -chronic obstructive pulmonary disease (COPD) -pneumonia -tuberculosis

-tuberculosis Page 593, Causes of Dyspnea

You are attending to a 3-year-old male patient who is presenting with severe shortness of breath. His parents report that he has had a cough and cold with a low grade fever for the past two days. They became worried today, as his level of distress has increased dramatically. On assessment, the patient is sitting upright and making high-pitched noises with each breath. Based on this information, the patient is most likely suffering from: -inflammation of the lower respiratory tract and bronchospasm. -inflammation of the bronchioles. -viral infection of the upper respiratory tract. -bacterial infection of the epiglottis.

-viral infection of the upper respiratory tract. Page 590-591, Causes of Dyspnea

Distributive shock occurs when: -severe bleeding causes tachycardia in order to distribute blood to the organs faster. -an injury causes restriction of the heart muscle and impairs its pumping function. -widespread dilation of the blood vessels causes blood to pool in the vascular beds. -temporary but severe vasodilation causes a decrease in blood supply to the brain.

-widespread dilation of the blood vessels causes blood to pool in the vascular beds. Page 493, Types of Shock

Ten days after treating a 34-year-old patient with tuberculosis, you are given a tuberculin skin test, which yields a positive result. This MOST likely indicates that: -you contracted the disease by casual contact instead of exposure to secretions. -you are actively infected with tuberculosis and should be treated immediately. -you were exposed to another infected person prior to treating the 34-year-old patient. -the disease is dormant in your body, but will probably never cause symptoms.

-you were exposed to another infected person prior to treating the 34-year-old patient. Page 574, Common or Serious Communicable Diseases

Suctioning the oral cavity of an adult should be accomplished within

15 seconds

When pulling a patient, you should extend your arms no more than ______ in front of your torso. A. 15 to 20 inches B. 20 to 30 inches C. 10 to 15 inches D. 5 to 10 inches

15 to 20 inches

A 21-year-old male was working in an auto repair shop and sustained radiator burns to the anterior aspect of both arms and to his anterior chest. According to the rule of nines, this patient has burns that cover _____ of his BSA. A. 27% B. 45% C. 36% D. 18%

18%

Which of the following patients is breathing adequately? 36-year-old man with cyanosis around the lips and irregular respirations 29-year old woman with respirations of 20 breaths/min, who is conscious and alert 22-year-old man with labored respirations at a rate of 28 breaths/min and pale skin 59-year-old woman with difficulty breathing, whose respirations are rapid and shallow

29-year old woman with respirations of 20 breaths/min, who is conscious and alert

At what speed will the ambulance begin to hydroplane when there is water present on the roadway? 25 mph 30 mph 40 mph 50 mph

30 mph

An open fracture is MOST accurately defined as a fracture in which: A. the overlying skin is no longer intact. B. bone ends protrude through the skin. C. a large laceration overlies the fracture. D. a bullet shatters the underlying bone.

A

Crepitus and false motion are: A. positive indicators of a fracture. B. most common with dislocations. C. indicators of a severe sprain. D. only seen with open fractures.

A

During your assessment of a 29-year-old female with significant deformity to her left elbow, you are unable to palpate a radial pulse. Your transport time to the hospital is approximately 40 minutes. You should: A. apply gentle manual traction in line with the limb and reassess for a pulse. B. make two or three attempts to restore distal circulation by manipulating the elbow. C. splint the elbow in the position of deformity and transport immediately. D. carefully straighten the injured arm and secure it with padded board splints.

A

What occurs when a patient is breathing very rapidly and shallowly? A. Air moves primarily in the anatomic dead space and does not participate in pulmonary gas exchange. B. Air is forcefully drawn into the lungs due to the negative pressure created by the rapid respirations. C. Minute volume increases because of a marked increase in both tidal volume and respiratory rate. D. The majority of tidal volume reaches the lungs and diffuses across the alveolar-capillary membrane.

A

Which of the following structures is NOT found in the upper airway? A. Bronchus B. Larynx C. Oropharynx D. Pharynx

A

A. administer oxygen, give her 324 mg of aspirin, and assess her further.

A 66-year-old female with a history of hypertension and diabetes presents with substernal chest pressure of 2 hours' duration. Her blood pressure is 140/90 mm Hg, her pulse is 100 beats/min and irregular, her respirations are 22 breaths/min, and her oxygen saturation is 92%. The patient does not have prescribed nitroglycerin, but her husband does. You should: Select one: A. administer oxygen, give her 324 mg of aspirin, and assess her further. B. obtain a SAMPLE history and contact medical control for advice. C. give her high-flow oxygen, attach the AED, and transport at once. D. give her one nitroglycerin and reassess her systolic blood pressure.

gastroesophageal reflux disease [GERD]

A condition in which the sphincter between the esophagus and the stomach opens, allowing stomach acid to move up into the esophagus, usually resulting in a burning sensation within the chest; also called acid reflux.

acute abdomen

A condition of sudden onset of pain within the abdomen, usually indicating peritonitis; immediate medical or surgical treatment is necessary.

Which of the following patients is competent and can legally refuse EMS care? A confused young female who states that she is the president A man who is staggering and states that he drank only three beers A conscious and alert woman who is in severe pain from a broken leg A diabetic patient who has slurred speech and is not aware of the date

A conscious and alert woman who is in severe pain from a broken leg

positional asphyxia

Restriction of chest wall movements and/or airway obstruction; can rapidly lead to sudden death.

In which of the following patients should you remove an impaled object? A. A semiconscious patient with an ice pick impaled in the chest B. A pulseless and apneic patient with a knife impaled in the back C. An apneic patient with a shard of glass impaled in the abdomen D. A conscious and alert patient with a fishhook impaled in the eye

A pulseless and apneic patient with a knife impaled in the back Correct

The simplest effective disinfectant solution is prepared by mixing A. 1 part bleach to 10 parts water B. 2 parts bleach to 10 parts water C. 10 parts bleach to 1 part water D. 10 parts bleach to five parts water

A. 1 part bleach to 10 parts water

Your patient has a partial thickness burn to his anterior chest. It is about the size of the palm of his hand. What is the approximate total body surface area of the burn? A. 1% B. 3% C. 5% D. 7%

A. 1% a hand is about 1% of body surface area

Significant vital sign changes will occur if the typical adult acutely loses more than ______ of his or her total blood volume. A. 20% B. 15% C. 10% D. 5%

A. 20% Correct B. 15% C. 10% D. 5%

An appropriate interval between reassessments of your unconscious patient might be: A. 3 minutes B. 6 minutes C. 9 minutes D. 11 minutes

A. 3 minutes

What is the correct compression-to-ventilation ratio for adult CPR? Select one: A. 30:2 B. 5:1 C. 30:1 D. 3:2

A. 30:2

A patient with a head injury presents with abnormal flexion of his extremities. What numeric value should you assign to him for motor response? A. 4 B. 2 C. 3 D. 5

A. 4 B. 2 C. 3 Correct D. 5

Hypothermia occurs when the core body temperature falls below: A. 95°F (35°C). B. 98°F (37°C). C. 90°F (32°C). D. 88°F (31°C).

A. 95°F (35°C). Correct B. 98°F (37°C). C. 90°F (32°C). D. 88°F (31°C).

In which of the following situations does a legal duty to act clearly exist? Select one: A. A call is received 15 minutes prior to shift change. B. The EMT hears of a cardiac arrest after his or her shift ends. C. A bystander encounters a victim who is not breathing. D. The EMT witnesses a vehicle crash while off duty. Feedback

A. A call is received 15 minutes prior to shift change.

Which of the following statements regarding a patient refusal is correct? Select one: A. A mentally competent adult has the legal right to refuse EMS care and transport. B. Advice given to a patient who refuses EMS treatment should not be documented. C. A patient who consumed a few beers will likely be able to refuse EMS treatment. D. Documentation of proposed care is unnecessary if the patient refuses treatment.

A. A mentally competent adult has the legal right to refuse EMS care and transport.

Which of the following is probably NOT a multiple-casualty incident? Select one: A. A motor vehicle crash with two critically injured patients and two ambulances B. A loss of power to a hospital or nursing home with ventilator-dependent patients C. An apartment fire with one patient and the possibility of others trapped in the building D. A rollover crash of a school bus with eight children who have injuries of varying severity

A. A motor vehicle crash with two critically injured patients and two ambulances.

Which of the following is an effective stress management strategy? Select one: A. Developing a social network outside of EMS B. Frequently reflecting on troublesome calls C. Requesting overtime to occupy your mind D. Modestly increasing caffeine consumption Feedback

A. Developing a social network outside of EMS

Which of the following definitions of a mass-casualty incident is correct? Select one: A. Any call that involves three or more patients B. Any situation that meets the demand of equipment or personnel C. Any incident that does not require mutual aid response D. Any call that involves at least one motor vehicle

A. Any call that involves three or more patients

Which organ or organ system has the greatest tolerance for lack of perfusion (shock)? A. Brain B. Gastrointestinal system C. Kidneys D. Skeletal muscle

A. Brain B. Gastrointestinal system Correct C. Kidneys D. Skeletal muscle

According to the "E" in the DOPE mnemonic, which of the following actions should you perform to troubleshoot inadequate ventilation in a patient with a tracheostomy tube? A. Check the mechanical ventilator for malfunction. B. Attempt to pass a suction catheter into the tube. C. Look for blood or other secretions in the tube. D. Listen to breath sounds to assess for a pneumothorax.

A. Check the mechanical ventilator for malfunction.

What mechanism(s) does the body use to control bleeding? A. Clotting B. Coagulation C. Vasoconstriction D. All of these answers are correct.

A. Clotting B. Coagulation C. Vasoconstriction D. All of these answers are correct. Correct

Which of the following statements regarding interaction with the caregiver of a child or adult with special health care needs is correct? A. Communication with the patient's caregiver or family members is important because they are the most familiar with the patient's condition. B. In general, the EMT should only speak with a certified home health care provider because he or she is the expert on the patient's illness. C. Before performing an assessment of the patient's ABCs, the EMT should ask the caregiver about the patient's medical condition. D. In most cases, it is more appropriate for the EMT to contact medical control prior to speaking with the patient's primary caregiver.

A. Communication with the patient's caregiver or family members is important because they are the most familiar with the patient's condition.

Which of the following findings is LEAST suggestive of child abuse? A. Consistency in the method of injury reported by the caregiver B. An unexplained delay in seeking medical care after the injury C. Burns to the hands or feet that involve a glove distribution D. Evidence of alcohol consumption or drug use at the scene

A. Consistency in the method of injury reported by the caregiver

Which of the following courses requires about 150 hours of training? Select one: A. EMT B. EMR C. AEMT D. Paramedic

A. EMT

Which of the following MOST accurately describes hyperthermia? A. Heat evaporates a significant amount of body water. B. The body is exposed to more heat than it can lose. C. The body eliminates more heat than it can generate. D. The core body temperature exceeds 99.5°F (37°C).

A. Heat evaporates a significant amount of body water. B. The body is exposed to more heat than it can lose. Correct C. The body eliminates more heat than it can generate. D. The core body temperature exceeds 99.5°F (37°C).

Which of the following statements regarding hemophilia is correct? A. Hemophiliacs take aspirin to enhance blood clotting. B. Patients with hemophilia may bleed spontaneously. C. Hemophilia is defined as a total lack of platelets. D. Approximately 25% of the population has hemophilia.

A. Hemophiliacs take aspirin to enhance blood clotting. B. Patients with hemophilia may bleed spontaneously. Correct C. Hemophilia is defined as a total lack of platelets. D. Approximately 25% of the population has hemophilia.

Which of the following breathing patterns is MOST indicative of increased intracranial pressure? A. Increased rate and depth with the distinct odor of acetone on the patient's breath B. Irregular rate, pattern, and volume of breathing with intermittent periods of apnea C. Slow, shallow, occasional gasps that progress to prolonged periods of apnea D. Increased rate with a normal inspiratory time and a prolonged expiratory time

A. Increased rate and depth with the distinct odor of acetone on the patient's breath B. Irregular rate, pattern, and volume of breathing with intermittent periods of apnea Correct C. Slow, shallow, occasional gasps that progress to prolonged periods of apnea D. Increased rate with a normal inspiratory time and a prolonged expiratory time

Which of the following is NOT a common factor that would affect a 75-year-old patient's vital signs? Select one: A. Increased weight B. Medical conditions C. Medications D. Overall health

A. Increased weight

Which of the following statements regarding a "dedicated line" is correct? Select one: A. It is a constantly open line of communication that cannot be accessed by outside users. B. It is a frequency that is used exclusively by EMTs to communicate with one another in the field. C. It is a designated frequency on a portable radio that provides direct access to medical control. D. It is a constantly open line of communication that is under exclusive control of a single user.

A. It is a constantly open line of communication that cannot be accessed by outside users.

Which of the following is NOT a function of the skin? A. Metabolic coordination B. Temperature regulation C. Sensory reception D. Pressure and pain perception

A. Metabolic coordination Correct

Which portion of the blood carries oxygen to and wastes away from body tissues? A. Plasma B. White blood cells C. Red blood cells D. Platelets

A. Plasma B. White blood cells C. Red blood cells Correct D. Platelets

Which aspect of the Health Insurance Portability and Accountability Act (HIPAA) MOST affects EMS personnel? Select one: A. Protecting patient privacy B. Ensuring access to insurance C. Controlling insurance costs D. Preventing insurance fraud

A. Protecting patient privacy

Which of the following statements regarding secondary brain injury is correct? A. Signs are often present immediately after an impact to the head. B. Hypoxia and hypotension are the two most common causes of secondary brain injury. C. Because cerebral edema develops quickly, it is considered to be a primary brain injury. D. It results from direct brain trauma following an impact to the head.

A. Signs are often present immediately after an impact to the head. B. Hypoxia and hypotension are the two most common causes of secondary brain injury. Correct C. Because cerebral edema develops quickly, it is considered to be a primary brain injury. D. It results from direct brain trauma following an impact to the head.

Which of the following structures does NOT contain smooth muscle? A. Skeletal system B. Urinary system C. Gastrointestinal tract D. Blood vessels

A. Skeletal system Correct

Which of the following splinting devices would be MOST appropriate to use for a patient who has an open fracture of the forearm with external bleeding? A. Sling and swathe B. Vacuum splint C. Air splint D. Cardboard splint

A. Sling and swathe B. Vacuum splint C. Air splint Correct D. Cardboard splint

Which of the following is an early sign of pit viper envenomation? A. Syncope and bleeding at distal sites B. General weakness and diaphoresis C. Signs and symptoms of hypoperfusion D. Local swelling and ecchymosis

A. Syncope and bleeding at distal sites B. General weakness and diaphoresis C. Signs and symptoms of hypoperfusion D. Local swelling and ecchymosis Correct

Which of the following is true regarding the Emergency Response Guide (ERG): A. The ERG helps identify the evacuation distance for specific hazardous materials. B. The ERG provides off-line medical direction for emergency responders. C. The ERG identifies all additional resources needed for a hazardous materials incident. D. The ERG describes the triage protocol for a hazardous materials incident.

A. The ERG helps identify the evacuation distance for specific hazardous materials.

What happens when blood volume is lost from the body? A. The arteries contract to increase the blood pressure. B. Arterial blood is diverted to the skin and muscles. C. The veins dilate to increase systemic perfusion. D. Widespread vasodilation causes blood pressure to decrease.

A. The arteries contract to increase the blood pressure. Correct

What is an EMT's primary service area? Select one: A. The main area in which the EMS agency operates B. The area serviced by the EMT's medical director C. The only area the EMT is certified to provide patient care D. The location where the EMT provides essential care during a 9-1-1 call

A. The main area in which the EMS agency operates

Which of the following findings would be the MOST significant when assessing a patient with possible internal bleeding? A. The patient takes rivaroxaban (Xeralto). B. The patient has not eaten in 24 hours. C. The patient has a history of hypertension. D. The patient had a stroke 5 years prior.

A. The patient takes rivaroxaban (Xeralto). Correct B. The patient has not eaten in 24 hours. C. The patient has a history of hypertension. D. The patient had a stroke 5 years prior.

Which of the following statements regarding the different stages of the grieving process is correct? Select one: A. The stages of the grieving process may occur simultaneously. B. It is rare that people will jump back and forth between stages. C. The grieving process typically begins with severe depression. D. Bargaining is the most unpleasant stage of the grieving process.

A. The stages of the grieving process may occur simultaneously.

You arrive at the scene of a motor vehicle-versus-pedestrian accident. The patient, a 13-year-old male, is unconscious and has multiple injuries. As you are treating the child, a law enforcement officer advises you that the child's parents will be at the scene in approximately 15 minutes. What should you do? Select one: A. Transport the child immediately and have the parents meet you at the hospital. B. Treat the child at the scene and wait for the parents to arrive and give consent. C. Withhold treatment until the parents arrive and give you consent for treatment. D. Begin transport at once and have the parents meet you en route to the hospital.

A. Transport the child immediately and have the parents meet you at the hospital.

Which of the following would be the MOST practical method of communicating with a hearing-impaired patient until his or her hearing aids can be located? A. Using a piece of paper and writing utensil to ask questions B. Attempting to use body language to determine the problem C. Using a high-pitched voice while speaking directly into the ear D. Contacting dispatch and requesting a sign language interpreter

A. Using a piece of paper and writing utensil to ask questions

Spina bifida is defined as: A. a birth defect caused by incomplete closure of the spinal column. B. chronic pressure on the brain caused by excess cerebrospinal fluid. C. a birth defect in which the child is born without spinal vertebrae. D. congenital inflammation of the spinal cord, usually in the neck.

A. a birth defect caused by incomplete closure of the spinal column.

In older patients, the first indicator of nontraumatic internal bleeding may be: A. a low blood pressure. B. a heart rate over 120 beats/min. C. diaphoresis and pale skin. D. weakness or dizziness.

A. a low blood pressure. B. a heart rate over 120 beats/min. C. diaphoresis and pale skin. D. weakness or dizziness. Correct

Common signs and symptoms of a serious head injury include all of the following, EXCEPT: A. a rapid, thready pulse. B. decerebrate posturing. C. widening pulse pressure. D. CSF leakage from the ears.

A. a rapid, thready pulse. Correct B. decerebrate posturing. C. widening pulse pressure. D. CSF leakage from the ears.

If direct pressure fails to immediately stop severe bleeding from an extremity, you should apply: A. a tourniquet proximal to the injury. B. additional sterile dressings. C. a splint and elevate the extremity. D. digital pressure to a proximal artery.

A. a tourniquet proximal to the injury. Correct B. additional sterile dressings. C. a splint and elevate the extremity. D. digital pressure to a proximal artery.

You are transporting a 67-year-old female patient to the hospital for investigation of abdominal pain. The patient care record indicates that the patient has a history of AAA. Based on this information, the patient care record includes an abbreviation for: Select one: A. abdominal aortic aneurysm. B. against ambulance advice. C. acute abdominal assessment. D. acute abdominal aneurysm

A. abdominal aortic aneurysm.

An indicator of an expanding intracranial hematoma or rapidly progressing brain swelling is: A. acute unilateral paralysis following the injury. B. a progressively lowering blood pressure. C. a rapid deterioration of neurologic signs. D. an acute increase in the patient's pulse rate.

A. acute unilateral paralysis following the injury. B. a progressively lowering blood pressure. C. a rapid deterioration of neurologic signs. Correct D. an acute increase in the patient's pulse rate.

A 26-year-old female presents with heavy vaginal bleeding. She is conscious, but restless. Her blood pressure is 84/54 mm Hg, her pulse is 120 beats/min and weak, and her respirations are 22 breaths/min with adequate depth. She tells you that she inserted a tampon about 2 hours ago. You should: A. administer high-flow oxygen, place a sterile pad over her vagina, keep her warm, elevate her lower extremities, and transport without delay. B. administer high-flow oxygen, ask her to remove the tampon, perform a detailed secondary assessment, and transport promptly. C. assist her ventilations with a bag-valve mask, place one sterile dressing into her vagina, perform a rapid secondary assessment, and transport. D. administer high-flow oxygen, perform a detailed assessment of her vaginal area for signs of trauma, place her on her side, and transport.

A. administer high-flow oxygen, place a sterile pad over her vagina, keep her warm, elevate her lower extremities, and transport without delay.

A specific legal document that directs relatives and caregivers regarding the medical treatment that may be given to patients who cannot speak for themselves is called a(n): Select one: A. advance directive. B. physician directive. C. power of attorney. D. statute of care.

A. advance directive.

Medical control gives you an order that seems inappropriate for the patient's condition. After confirming that you heard the physician correctly, you should: Select one: A. advise the physician that the order is unclear and ask for clarification. B. state that you will not carry out the order because it is inappropriate. C. obtain consent from the patient and then carry out the order as usual. D. carry out the order and then carefully document it on the run form.

A. advise the physician that the order is unclear and ask for clarification.

When a child is struck by a car, the area of greatest injury depends MOSTLY on the: A. age of the child and the size of the car that struck him or her. B. speed at which the car was traveling when impact occurred. C. size of the child and the height of the bumper upon impact. D. height of the child and the speed at which the car was traveling.

A. age of the child and the size of the car that struck him or her.

When you are communicating with an older patient, it is important to remember that: Select one: A. age-related changes diminish the effectiveness of the eyes and ears. B. most older patients are confused due to a decrease in brain cells. C. deafness and blindness are a normal part of the process of aging. D. the majority of older patients experience a loss of low-frequency hearing.

A. age-related changes diminish the effectiveness of the eyes and ears.

The EMT must assume that any unwitnessed water-related incident is accompanied by: A. alcohol intoxication. B. an air embolism. C. cold-water immersion. D. possible spinal injury.

A. alcohol intoxication. B. an air embolism. C. cold-water immersion. D. possible spinal injury. Correct

The purpose of the pediatric assessment triangle (PAT) is to: A. allow you to rapidly and visually form a general impression of the child. B. gather critical data by performing a rapid hands-on assessment of the child. C. facilitate a rapid head-to-toe assessment of the child by visualization only. D. determine if the child's vital signs are within the age-appropriate limits.

A. allow you to rapidly and visually form a general impression of the child.

A history of pelvic inflammatory disease or tubal ligations increases a woman's risk for: A. an ectopic pregnancy. B. preeclampsia. C. placenta previa. D. gestational diabetes.

A. an ectopic pregnancy.

If applying a dressing to control the bleeding of a patient's arm, the EMT should ________. A. apply direct pressure first B. use large or small gauze pads or dressings depending upon the size of the wound C. cover the entire wound, above and below, with the dressing D. All of these answers are correct.

A. apply direct pressure first B. use large or small gauze pads or dressings depending upon the size of the wound C. cover the entire wound, above and below, with the dressing D. All of these answers are correct. Correct

A 31-year-old male was bitten on the leg by an unidentified snake. The patient is conscious and alert and in no apparent distress. Your assessment of his leg reveals two small puncture marks with minimal pain and swelling. In addition to administering oxygen and providing reassurance, further care for this patient should include: A. applying ice to the wound and transporting quickly. B. transporting only with close, continuous monitoring. C. supine positioning, splinting the leg, and transporting. D. elevating the lower extremities and giving antivenin.

A. applying ice to the wound and transporting quickly. B. transporting only with close, continuous monitoring. C. supine positioning, splinting the leg, and transporting. Correct D. elevating the lower extremities and giving antivenin.

Injuries to the external male genitalia _______. A. are rarely life-threatening B. are often life-threatening C. usually result in permanent damage D. frequently lead to hypovolemic shock

A. are rarely life-threatening

A 4-year-old boy had an apparent seizure. He is conscious and calm and is sitting on his mother's lap. His father is sitting in a nearby chair. The child's mother suddenly begins crying uncontrollably, which causes the child to start crying. You should: Select one: A. ask the father to hold the child so you can assess him while your partner tries to calm the mother. B. give the child a favorite toy or blanket to hold onto and perform your assessment to the best of your ability. C. reassure the child's mother that seizures in children are very common and that there is nothing to worry about. D. attempt to calm the child's mother, but avoid separating her from her child because this will increase her anxiety.

A. ask the father to hold the child so you can assess him while your partner tries to calm the mother.

A 30-year-old male was rescued after being lost in the woods for approximately 18 hours. The outside temperature is 30°F (-1°C). He is immediately placed in the warmed ambulance, where you perform a primary assessment. He is unresponsive, pale, and apneic. You should: A. assess for a carotid pulse for up to 60 seconds. B. apply an AED and assess his cardiac rhythm. C. open his airway and give two rescue breaths. D. apply chemical heat packs to his groin and axillae.

A. assess for a carotid pulse for up to 60 seconds. Correct B. apply an AED and assess his cardiac rhythm. C. open his airway and give two rescue breaths. D. apply chemical heat packs to his groin and axillae.

You and your partner respond to a park where several people were reportedly struck by lightning. When you arrive, you find three patients. The first patient is lying supine on the ground; he is unresponsive and does not appear to be breathing. The second patient is ambulatory, appears confused, and is holding his arm against his chest. The third patient is sitting on the ground holding the sides of his head. After calling for backup, you should: A. assess the unresponsive patient's pulse, begin CPR starting with chest compressions if he is pulseless, and attach the AED as soon as possible. B. focus your initial treatment efforts on the patients who are conscious because the unresponsive patient is likely in irreversible cardiac arrest. C. immediately begin CPR on the unresponsive patient, but cease resuscitation efforts if there is no response after 5 minutes of treatment. D. recognize that the patients who are conscious are at high risk for developing cardiac arrest and quickly assess them for potentially life-threatening injuries.

A. assess the unresponsive patient's pulse, begin CPR starting with chest compressions if he is pulseless, and attach the AED as soon as possible. Correct B. focus your initial treatment efforts on the patients who are conscious because the unresponsive patient is likely in irreversible cardiac arrest. C. immediately begin CPR on the unresponsive patient, but cease resuscitation efforts if there is no response after 5 minutes of treatment. D. recognize that the patients who are conscious are at high risk for developing cardiac arrest and quickly assess them for potentially life-threatening injuries.

A young male was shot in the abdomen by an unknown type of gun. He is semiconscious, has shallow breathing, and is bleeding externally from the wound. As you control the external bleeding, your partner should: A. assist the patient's ventilations. B. apply a nonrebreathing mask. C. perform a secondary assessment. D. obtain baseline vital signs.

A. assist the patient's ventilations. Correct B. apply a nonrebreathing mask. C. perform a secondary assessment. D. obtain baseline vital signs.

You and your paramedic partner are caring for a patient who ingested codeine, acetaminophen (Tylenol), and hydrocodone (Vicodin). The patient is unresponsive, his breathing is slow and shallow, and his pulse is slow and weak. Treatment for this patient should include: Select one: A. assisted ventilation and naloxone (Narcan). B. assisted ventilation and flumazenil (Romazicon). C. oxygen via a nasal cannula and atropine sulfate. D. high-flow oxygen via a nonrebreathing mask.

A. assisted ventilation and naloxone (Narcan).

You are assessing a 440-lb man who complains of shortness of breath and lower back pain. The patient is conscious and alert, his blood pressure is 148/98 mm Hg, and his heart rate is 120 beats/min. Your MOST immediate action should be to: A. avoid placing him in a supine position if possible and administer oxygen. B. perform a secondary assessment, focusing on his respiratory system and back. C. ask a member of your team to locate the best route to move him to the ambulance. D. notify the receiving facility and advise them of the patient's weight and status.

A. avoid placing him in a supine position if possible and administer oxygen.

Following proper decontamination, a 30-year-old male is brought to you. He is semiconscious and has rapid, shallow respirations. A quick visual assessment reveals no obvious bleeding. You should: Select one: A. begin some form of positive-pressure ventilation. B. ask a firefighter what the patient was exposed to. C. perform a rapid assessment to locate critical injuries. D. administer high-flow oxygen via a nonrebreathing mask.

A. begin some form of positive-pressure ventilation.

A patient has fractured both femurs. Anatomically, these injuries would be described as being: Select one: A. bilateral. B. medial. C. unilateral. D. proximal.

A. bilateral.

Whether you are using a commercial device or a stick and triangular bandage as a tourniquet, it is important to remember that: A. bulky dressings should be securely applied over the tourniquet to further assist in controlling the bleeding. B. the tourniquet should only be removed at the hospital because bleeding may return if the tourniquet is released. C. you should try to control the bleeding by applying pressure to a proximal arterial pressure point first. D. the tourniquet should be applied directly over a joint if possible because this provides better bleeding control.

A. bulky dressings should be securely applied over the tourniquet to further assist in controlling the bleeding. B. the tourniquet should only be removed at the hospital because bleeding may return if the tourniquet is released. Correct C. you should try to control the bleeding by applying pressure to a proximal arterial pressure point first. D. the tourniquet should be applied directly over a joint if possible because this provides better bleeding control.

The central nervous system (CNS) is composed of the: A. cerebrum and meninges. B. brain and spinal cord. C. cerebellum and brain. D. meninges and spinal cord.

A. cerebrum and meninges. B. brain and spinal cord. Correct C. cerebellum and brain. D. meninges and spinal cord.

The five sections of the spinal column, in descending order, are the: A. cervical, coccygeal, thoracic, sacral, and lumbar. B. cervical, thoracic, lumbar, sacral, and coccygeal. C. thoracic, cervical, lumbar, coccygeal, and sacral. D. coccygeal, sacral, lumbar, thoracic, and cervical.

A. cervical, coccygeal, thoracic, sacral, and lumbar. B. cervical, thoracic, lumbar, sacral, and coccygeal. Correct C. thoracic, cervical, lumbar, coccygeal, and sacral. D. coccygeal, sacral, lumbar, thoracic, and cervical.

Nerve agents and choking agents are examples of: A. chemical agents B. nuclear weapons C. biological agents D. radiological weapons

A. chemical agents biological agents cause diseases

Drug names you may find on medication packaging include all of the following EXCEPT: A. chemical name B. trade name C. generic name D. all of the above can be found

A. chemical name

The process of removing dirt, dust, blood, or other visible contaminants from a surface or equipment is called: Select one: A. cleaning. B. disinfection. C. sterilization. D. high-level disinfection.

A. cleaning.

Most of the serious injuries associated with scuba diving are caused by: A. cold water temperature. B. too rapid of a descent. C. alcohol consumption. D. too rapid of an ascent.

A. cold water temperature. B. too rapid of a descent. C. alcohol consumption. D. too rapid of an ascent. Correct

EMTs should have a strong working knowledge of medical terminology in order to: Select one: A. communicate effectively with other members of the health care team. B. perform and document more accurate patient assessments. C. determine and document more accurate patient diagnoses. D. more clearly explain to patients the nature of their condition.

A. communicate effectively with other members of the health care team.

The ability to understand others and have them understand you is known as __________. Select one: A. communication B. self-confidence C. scene leadership D. teamwork and diplomacy

A. communication

The use of special tools to remove an entrapped patient from a vehicle is known as _________. Select one: A. complex access B. simple access C. vehicle stabilization D. incident management

A. complex access

Ethnocentrism is defined as: Select one: A. considering your own cultural values to be more important when interacting with people of a different culture. B. suspecting that a person has an ulterior motive based on the tone of his or her voice when answering a question. C. subconsciously forcing your cultural values onto a patient because you feel that your own values are more acceptable. D. understanding that people from different cultural backgrounds respond to pain and stress differently.

A. considering your own cultural values to be more important when interacting with people of a different culture.

You are attending to a 23-year-old female patient who is 16 weeks pregnant with her second child. The patient has apparently fallen and sustained an injury to her upper right arm. When you ask about the incident history, the patient is reluctant to explain what happened and becomes very quiet. Based on this information, you should: A. continue your care in a private area, document any details regarding the scene and the incident, and reassure her as you provide care. B. ask more detailed questions and press the issue until you have a more detailed understanding of the incident. C. immediately remove the patient from the environment and transport. D. contact police and remain at the scene until they arrive and escort you to the hospital.

A. continue your care in a private area, document any details regarding the scene and the incident, and reassure her as you provide care.

Classic signs and symptoms of hypoglycemia include: Select one: A. cool, clammy skin; weakness; tachycardia; and rapid respirations. B. warm, dry skin; hunger; abdominal pain; and deep, slow respirations. C. warm, dry skin; irritability; bradycardia; and rapid respirations. D. cold, clammy skin; bradycardia; hunger; and deep, rapid respirations.

A. cool, clammy skin; weakness; tachycardia; and rapid respirations.

You are transporting a 28-year-old man with a frostbitten foot. The patient's vital signs are stable and he denies any other injuries or symptoms. The weather is treacherous and your transport time to the hospital is approximately 45 minutes. During transport, you should: A. cover his foot with chemical heat compresses. B. protect the affected part from further injury. C. rewarm his foot in 102°F to 104°F (38.9°C to 40°C) water. D. administer oxygen via a nonrebreathing mask.

A. cover his foot with chemical heat compresses. B. protect the affected part from further injury. Correct C. rewarm his foot in 102°F to 104°F (38.9°C to 40°C) water. D. administer oxygen via a nonrebreathing mask.

Sellick's maneuver involves applying pressure over the A. cricoid cartilage B. thyroid cartilage C. carina D. epiglottis

A. cricoid cartilage (surrounds trachea) Sellback's maneuver --> used in endotracheal intubation (also called cricoid maneuver) carina --> point in trachea where it branches into bronchi

When activated, the sympathetic nervous system produces all of the following effects, EXCEPT: A. dilation of the bronchiole smooth muscle. B. pupillary constriction. C. increase in heart rate. D. shunting of blood to vital organs.

A. dilation of the bronchiole smooth muscle. B. pupillary constriction. Correct C. increase in heart rate. D. shunting of blood to vital organs.

A 13-year-old child is on a home ventilator. The parents called because the mechanical ventilator is malfunctioning and the child has increasing respiratory distress. You should: A. disconnect the ventilator and apply a tracheostomy collar. B. attempt to troubleshoot the mechanical ventilator problem. C. reset the ventilator by unplugging it for 30 to 60 seconds. D. place a call to the home health agency treating this patient.

A. disconnect the ventilator and apply a tracheostomy collar.

Communicating with patients who are deaf or hard-of-hearing can be facilitated by doing all of the following, EXCEPT: Select one: A. elevating the tone of your voice and exaggerating word pronunciation. B. providing pen and paper if the patient prefers to write his or her response. C. shining a light on your face when you are in a darkened environment. D. placing yourself in a position to ensure that the patient can see your lips.

A. elevating the tone of your voice and exaggerating word pronunciation.

The rescue team is in the process of extricating a 40-year-old male from his truck. The patient's wife, who was uninjured in the crash, is calmly observing the extrication and asks you if her husband will be all right. You should: Select one: A. ensure that she is in a safe area, away from the scene. B. allow her to observe the extrication and keep her calm. C. ask her follow-up questions about the details of the crash. D. allow her to talk to her husband during the extrication

A. ensure that she is in a safe area, away from the scene.

Proper procedure for administering oral glucose to a patient includes all of the following, EXCEPT: Select one: A. ensuring the absence of a gag reflex. B. requesting permission from medical control. C. checking the medication's expiration date. D. assessing the patient's mental status.

A. ensuring the absence of a gag reflex.

The two main types of cells contained in blood are called _________. Select one: A. erythrocytes and leukocytes B. transport and clotting C. platelets and plasma D. hemoglobin A and S

A. erythrocytes and leukocytes

When a warm hand is immersed in water that is 70°F (21°C), heat is transferred from the hand to the water through a process called: A. evaporation. B. radiation. C. convection. D. conduction.

A. evaporation. B. radiation. C. convection. D. conduction. Correct

It is common for young females who experience their first menstrual period to: A. experience abdominal cramping, which may be misinterpreted. B. have a false positive home pregnancy test result. C. become so emotionally distraught that they contemplate suicide. D. lose up to 500 mL of blood within the first 24 hours.

A. experience abdominal cramping, which may be misinterpreted.

The Glasgow Coma Scale (GCS) is used to assess: A. eye opening, verbal response, and motor response. B. verbal response, eye opening, and mental status. C. mental status, eye opening, and respiratory rate. D. sensory response, pupil reaction, and heart rate.

A. eye opening, verbal response, and motor response. Correct B. verbal response, eye opening, and mental status. C. mental status, eye opening, and respiratory rate. D. sensory response, pupil reaction, and heart rate.

Hyperextension injuries of the spine are MOST commonly the result of: A. falls. B. hangings. C. compression. D. diving.

A. falls. B. hangings. Correct C. compression. D. diving.

The term used when individual units or different organizations make independent, and often inefficient, decisions regarding an incident is called: Select one: A. freelancing. B. undermining. C. logistical chaos. D. single command.

A. freelancing.

All of the following terms refer to a body part that is cold but not frozen, EXCEPT: A. frostbite. B. frostnip. C. immersion foot. D. trench foot.

A. frostbite. Correct B. frostnip. C. immersion foot. D. trench foot.

Following blunt trauma to the abdomen, a 21-year-old female complains of diffuse abdominal pain and pain to the left shoulder. Your assessment reveals that her abdomen is distended and tender to palpation. On the basis of these findings, you should be MOST suspicious of injury to the: A. gallbladder. B. spleen. C. pancreas. D. liver.

A. gallbladder. B. spleen. Correct C. pancreas. D. liver.

A 67-year-old male presents with weakness, dizziness, and melena that began approximately 2 days ago. He denies a history of trauma. His blood pressure is 90/50 mm Hg and his pulse is 120 beats/min and thready. You should be MOST suspicious that this patient is experiencing: A. gastrointestinal bleeding. B. an aortic aneurysm. C. acute appendicitis. D. intrathoracic hemorrhaging.

A. gastrointestinal bleeding. Correct B. an aortic aneurysm. C. acute appendicitis. D. intrathoracic hemorrhaging.

In the incident command system, organizational divisions may include sections, branches, divisions, and: Select one: A. groups. B. teams. C. platoons. D. squads.

A. groups.

Urine is transported from the kidneys to the urinary bladder via the: A. ureters. B. renal duct. C. prostate. D. urethra.

A. ureters. Correct

To minimize distractions and confusion when assessing an older patient, you should: Select one: A. have only one EMT speak to the patient at a time. B. dismiss the family members from the room or area. C. elevate your voice and speak directly to the patient. D. perform a physical exam and then talk to the patient.

A. have only one EMT speak to the patient at a time.

You and your partner arrive at the scene of a major motor vehicle crash. The driver, a young male, is severely entrapped in his car. He has an open head injury and massive facial trauma. He is unresponsive, is not breathing, and does not have a palpable carotid pulse. You should: Select one: A. have your partner check for a pulse to confirm that the patient is deceased. B. ventilate the patient for 5 minutes and then stop if there is no response. C. request the fire department to extricate the patient so you can begin CPR. D. stop any active bleeding and advise dispatch to send a paramedic crew.

A. have your partner check for a pulse to confirm that the patient is deceased.

Common signs and symptoms of heat exhaustion include all of the following, EXCEPT: A. hot, dry skin. B. tachycardia. C. nausea. D. headache.

A. hot, dry skin. Correct B. tachycardia. C. nausea. D. headache.

A man jumped from the roof of his house and landed on his feet. He complains of pain to his heels, knees, and lower back. This mechanism of injury is an example of: A. hyperflexion. B. axial loading. C. hyperextension. D. distraction.

A. hyperflexion. B. axial loading. Correct C. hyperextension. D. distraction.

Circulation of blood within an organ or tissue in adequate amounts to meet the cells' oxygen, nutritional, and waste-removal needs is termed _______. A. hypoperfusion B. perfusion C. coagulation D. hemorrhage

A. hypoperfusion B. perfusion Correct C. coagulation D. hemorrhage

A pediatric patient who is breathing 12 breaths/min would be categorized as: Select one: A. immediate. B. delayed. C. minimal. D. expectant.

A. immediate.

Abdominal pain, vomiting, and fever are most likely due to _______. A. infection B. evisceration C. hypovolemia D. hemorrhage

A. infection

Which assessment is performed at least once for every patient? A. initial assessment B. rapid trauma assessment C. detailed physical exam D. focused history and physical exam

A. initial assessment

Of the four avenues of poisoning, generally ___________ is the most worrisome in terms of treatment to the EMS provider. Select one: A. injection B. absorption C. ingestion D. inhalation

A. injection

In which order should the EMT perform the following steps when assessing the chest? A. inspection, palpation and auscultation B. auscultation, inspection, and palpation C. palpitation, percussion, and auscultation D. inspection, auscultation, and percussion

A. inspection, palpation and auscultation

Your patient is a 19 y.o. football player who was "speared" while being tackled approx 20 min prior to your arrival. He complains of left shoulder pain and looks pale. His radial pulse is 130 and weak. You suspect: A. internal bleeding B. shoulder dislocation C. heat exhaustion D. pneumothorax

A. internal bleeding

When caring for a known alcoholic patient with severe trauma to the chest and abdomen, you should be concerned that: Select one: A. internal bleeding may be profuse because prolonged alcohol use may impair the blood's ability to clot. B. delirium tremens (DTs) are commonly induced by physical trauma and can lead to life-threatening seizures. C. long bone fractures are likely because chronic alcohol consumption weakens the structure of the bones. D. signs and symptoms of shock may be masked by the stimulant effects produced by alcohol.

A. internal bleeding may be profuse because prolonged alcohol use may impair the blood's ability to clot.

In contrast to a cerebral concussion, a cerebral contusion: A. involves physical injury to the brain tissue. B. results from a laceration to the brain tissue. C. usually does not cause a loss of consciousness. D. does not cause pressure within the skull.

A. involves physical injury to the brain tissue. Correct B. results from a laceration to the brain tissue. C. usually does not cause a loss of consciousness. D. does not cause pressure within the skull.

You should be MOST suspicious that a patient has experienced a significant head injury if his or her pulse is: A. irregular. B. rapid. C. weak. D. slow.

A. irregular. B. rapid. C. weak. D. slow. Correct

A 37-year-old female with a history of diabetes presents with excessive urination and weakness of 2 days' duration. Her blood glucose level reads 320 mg/dL. If this patient's condition is not promptly treated, she will MOST likely develop: Select one: A. irreversible renal failure. B. severe insulin shock. C. hypoxia and overhydration. D. acidosis and dehydration.

A. irreversible renal failure.

Once a cervical collar has been applied to a patient with a possible spinal injury, it should not be removed unless: A. it causes a problem managing the airway. B. the patient adamantly denies neck pain. C. sensory and motor functions remain intact. D. lateral immobilization has been applied.

A. it causes a problem managing the airway. Correct B. the patient adamantly denies neck pain. C. sensory and motor functions remain intact. D. lateral immobilization has been applied.

In pediatric patients, the liver and spleen are _______. A. larger in proportion to the abdomen B. smaller in proportion to the abdomen C. more protected by the thorax compared to adults D. less likely to bleed when injured

A. larger in proportion to the abdomen

The impedance threshold device (ITD) may improve circulation during active compression-decompression CPR by: Select one: A. limiting the amount of air that enters the lungs during the recoil phase between chest compressions, which results in negative intrathoracic pressure and improved cardiac filling. B. maintaining increased intrathoracic pressure during the downward stroke of each chest compression, which forces more blood from both of the ventricles. C. drawing all of the air out of the lungs in between chest compressions, which causes positive intrathoracic pressure and a reduction of blood return to the right side of the heart. D. maximizing the amount of air in the lungs following chest recoil, which hyperinflates the lungs and forces more blood from the ventricle during each compression.

A. limiting the amount of air that enters the lungs during the recoil phase between chest compressions, which results in negative intrathoracic pressure and improved cardiac filling.

An overdose of acetaminophen, the active ingredient in Tylenol, will MOST likely cause: Select one: A. liver failure. B. kidney failure. C. gastric ulcers. D. CNS depression.

A. liver failure.

You are caring for a 10-year-old child who was an unrestrained passenger in a vehicle that was struck from the rear. The child complains of neck pain. Which of the following should be done first? A. manual cervical spine precautions. B. apply an appropriately sized cervical collar. C. assess pulse, motor, sensation in all extremities. D. perform a secondary assessment.

A. manual cervical spine precautions.

At the scene of an automobile crash, a utility pole has been broken and power lines are lying across the car. The patients inside the car are conscious. You should: Select one: A. mark off a danger zone around the downed lines. B. proceed with normal extrication procedures. C. advise the patients to carefully get out of the car. D. remove the lines with a nonconductive object.

A. mark off a danger zone around the downed lines

The EMT's scope of practice within his or her local response area is defined by the: Select one: A. medical director. B. EMS supervisor. C. local health district. D. state EMS office.

A. medical director.

When you are communicating with an older patient, it is important to remember that: Select one: A. most older people think clearly and are capable of answering questions. B. speaking loudly and distinctly will ensure that the patient can hear you. C. your questions should focus exclusively on the patient's obvious problem. D. hostility and confusion should be presumed to be due to the patient's age.

A. most older people think clearly and are capable of answering questions.

When approaching a helicopter, whether the rotor blades are moving or not, you should: Select one: A. never duck under the body or the tail boom because the pilot cannot see you in these areas. B. remember that the main rotor blade is flexible and can dip as low as 5¢ to 6¢ from the ground. C. carefully approach the aircraft from the rear unless a crew member instructs you to do otherwise. D. approach the aircraft from the side because this will make it easier for you to access the aircraft doors.

A. never duck under the body or the tail boom because the pilot cannot see you in these areas.

Which of the following medications is intended to decrease myocardial workload and increase myocardial blood flow? A. nitroglycerin B. activated charcoal C. aspirin D. epinephrine

A. nitroglycerin nitro is a vasodilator

When providing a patient report via radio, you should protect the patient's privacy by: Select one: A. not disclosing his or her name. B. withholding medical history data. C. using coded medical language. D. refraining from objective statements.

A. not disclosing his or her name.

Unlike a multiple-casualty incident, a natural disaster: Select one: A. often requires personnel to remain on scene for several days. B. exists when there are more than 100 critically injured patients. C. is typically short-lived and does not require as much manpower. D. usually does not require the incident command system process.

A. often requires personnel to remain on scene for several days.

A man finds his 59-year-old wife unconscious on the couch. He states that she takes medications for type 2 diabetes. He further tells you that his wife has been ill recently and has not eaten for the past 24 hours. Your assessment reveals that the patient is unresponsive. You should: Select one: A. open and maintain her airway and assess breathing. B. administer 100% oxygen via a nonrebreathing mask. C. assess for the presence of a medical identification tag. D. administer oral glucose between her cheek and gum.

A. open and maintain her airway and assess breathing.

Hypotension, hypoventilation, and pinpoint pupils would be expected following an overdose of: Select one: A. oxycodone (Percocet). B. ecstasy. C. amphetamine sulfate (Benzedrine). D. crack cocaine.

A. oxycodone (Percocet).

During your primary assessment of a 19-year-old unconscious male who experienced severe head trauma, you note that his respirations are rapid, irregular, and shallow. He has bloody secretions draining from his mouth and nose. You should: A. pack his nostrils to stop the drainage of blood. B. suction his oropharynx for up to 15 seconds. C. immobilize his spine and transport immediately. D. assist his ventilations with a BVM.

A. pack his nostrils to stop the drainage of blood. B. suction his oropharynx for up to 15 seconds. Correct C. immobilize his spine and transport immediately. D. assist his ventilations with a BVM.

You receive a call to a residence for a sick patient. Upon your arrival, you find the patient, a 53-year-old diabetic male, lying down on his front porch. His wife tells you that he had been mowing the lawn in the heat for the past 3 hours. The patient is confused and has hot, moist skin. His pulse is weak and thready, and his blood pressure is 90/50 mm Hg. You should: A. perform a head-to-toe assessment and look for signs of trauma. B. place him in a sitting position and have him drink 1 L of water. C. load him into the ambulance and begin rapid cooling interventions. D. administer one tube of oral glucose and reassess his mental status.

A. perform a head-to-toe assessment and look for signs of trauma. B. place him in a sitting position and have him drink 1 L of water. C. load him into the ambulance and begin rapid cooling interventions. Correct D. administer one tube of oral glucose and reassess his mental status.

When caring for a female patient who has been sexually assaulted, you should: A. place any bloodstained clothing or other articles in separate paper bags. B. advise her that she will not be allowed to shower or change her clothes. C. ask the patient for a concise, detailed report of what happened to her. D. allow law enforcement to take her statement before you begin treatment.

A. place any bloodstained clothing or other articles in separate paper bags.

A 43-year-old man is experiencing a severe nosebleed. His blood pressure is 190/110 mm Hg and his heart rate is 90 beats/min and bounding. Preferred treatment for this patient includes: A. placing a rolled 4² × 4² dressing between his lower lip and gum. B. having the patient pinch his own nostrils and then lie supine. C. pinching the patient's nostrils and having him lean forward. D. packing both nostrils with gauze pads until the bleeding stops.

A. placing a rolled 4² × 4² dressing between his lower lip and gum. B. having the patient pinch his own nostrils and then lie supine. C. pinching the patient's nostrils and having him lean forward. Correct D. packing both nostrils with gauze pads until the bleeding stops.

The development of an incident action plan is the responsibility of the: Select one: A. planning section. B. logistics section. C. operations section. D. finance section.

A. planning section.

The severity of bleeding should be based on all of the following findings, EXCEPT A. poor general appearance. B. clinical signs and symptoms. C. systolic blood pressure. D. the mechanism of injury.

A. poor general appearance. B. clinical signs and symptoms. C. systolic blood pressure. Correct D. the mechanism of injury.

You respond to a local lake where a diver complains of difficulty breathing that occurred immediately after rapidly ascending from a depth of approximately 30 feet. On assessment, you note that he has cyanosis around his lips and has pink froth coming from his nose and mouth. You should: A. position him supine with his head elevated 30°, suction his mouth and nose, hyperventilate him with a bag-valve mask, and contact medical control for further guidance. B. suction his mouth and nose, apply high-flow oxygen, monitor the patient's breath sounds for a pneumothorax, and contact medical control regarding transport to a recompression facility. C. place him in a semi-sitting position, suction his mouth and nose, apply a continuous positive airway pressure (CPAP) device, and transport to the closest emergency department. D. suction his mouth and nose, keep him supine and elevate his legs to prevent air bubbles from entering his brain, administer high-flow oxygen, and transport to a hyperbaric chamber.

A. position him supine with his head elevated 30°, suction his mouth and nose, hyperventilate him with a bag-valve mask, and contact medical control for further guidance. B. suction his mouth and nose, apply high-flow oxygen, monitor the patient's breath sounds for a pneumothorax, and contact medical control regarding transport to a recompression facility. Correct C. place him in a semi-sitting position, suction his mouth and nose, apply a continuous positive airway pressure (CPAP) device, and transport to the closest emergency department. D. suction his mouth and nose, keep him supine and elevate his legs to prevent air bubbles from entering his brain, administer high-flow oxygen, and transport to a hyperbaric chamber.

Common interventions used to stimulate spontaneous respirations in the newborn include all of the following, EXCEPT: A. positive-pressure ventilations. B. thorough drying with a towel. C. suctioning of the upper airway. D. some form of tactile stimulation.

A. positive-pressure ventilations.

The function of the National Incident Management System (NIMS) is to: Select one: A. prepare for, prevent, respond to, and recover from domestic incidents. B. facilitate a standard method of incident command for natural disasters. C. prepare for the potential of a nuclear attack against the United States. D. educate city and county governments regarding foreign terrorist attacks.

A. prepare for, prevent, respond to, and recover from domestic incidents.

Upon arrival at a search-and-rescue incident, the EMS crew should _________. Select one: A. prepare the equipment to carry to the patient B. begin searching for the patient C. split up and request a search grid D. provide the incident commander with the crew's names and certification level(s)

A. prepare the equipment to carry the patient

In contrast to Lyme disease, Rocky Mountain spotted fever: A. presents with flu-like symptoms and a bull's-eye rash. B. can cause paralysis and cardiorespiratory collapse. C. causes painful joint swelling after a few days or weeks. D. may be confused with rheumatoid arthritis.

A. presents with flu-like symptoms and a bull's-eye rash. B. can cause paralysis and cardiorespiratory collapse. Correct C. causes painful joint swelling after a few days or weeks. D. may be confused with rheumatoid arthritis.

Your patient is a construction worker who fell from a residential rooftop. He is responsive to pain. Breaths are shallow and irregular. You should: A. provide artificial ventilations with a BVM and high flow oxygen B. obtain an ox sat reading before determining if oxygen is necessary C. apply a BVM with high flow oxygen D. apply a nasal cannula and reassess the patient

A. provide artificial ventilations with a BVM and high flow oxygen BVM for big injuries

The body's natural cooling mechanism, in which sweat is converted to a gas, is called: A. radiation. B. evaporation. C. conduction. D. convection.

A. radiation. B. evaporation. Correct C. conduction. D. convection.

Coordinating efforts through a unified command system at a mass casualty incident will likely: A. reduce duplication of effort and freelancing B. make communications more difficult C. delay patient care and transport D. run efficiently even without prior practice

A. reduce duplication of effort and freelancing still needs practice tho

At a scene with downed electrical lines, the EMT should _________. Select one: A. remain outside the danger (hot) zone B. enter the danger (hot) zone for patient removal only C. stabilize the patient in the danger (hot) zone D. relocate the danger (hot) zone away from the patient

A. remain outside the danger (hot) zone.

The venom of a black widow spider is toxic to the: A. renal system. B. cardiovascular system. C. respiratory system. D. nervous system.

A. renal system. B. cardiovascular system. C. respiratory system. D. nervous system. Correct

When driving with lights and siren, you are _____ that drivers yield the right-of-way. Select one: A. requesting B. demanding C. offering D. None of these answers are correct

A. requesting

Quid pro quo, a type of sexual harassment, occurs when the harasser: Select one: A. requests sexual favors in exchange for something else. B. touches another person without his or her consent. C. makes rude remarks about a person's body parts. D. stares at certain parts of another person's anatomy.

A. requests sexual favors in exchange for something else.

You and your partner are both male and are attending to a 28-year-old female patient complaining of diffuse abdominal pain. The patient is 34 weeks pregnant with her first child. The patient refuses to allow you to examine her, and her husband informs you that their culture does not allow males to examine or care for pregnant women. You should: A. respect the patient's wishes, ensure that the appropriate documentation is completed, and transport the patient. B. call for the police to ensure that patient assessment is carried out. C. insist that the patient requires proper care and that requires an adequate physical assessment and that you cannot be responsible for the outcome. D. inform the patient that by calling for an ambulance, she is agreeing to the care provided and continue with your assessment and management.

A. respect the patient's wishes, ensure that the appropriate documentation is completed, and transport the patient.

You are caring for a 22 y.o. female who complains of respiratory distress after exercising. She is alert, speaking full sentences, and has a persistent cough. She is most likely experiencing: A. respiratory distress with adequate breathing B. respiratory distress with inadequate breathing C. respiratory paralysis D. respiratory arrest

A. respiratory distress with adequate breathing

A 19 y.o. female is found unconscious in her apt living room. While surveying the apt, your partner yells from the bedroom "she has an SVN machine in here". This suggests the patient has a history of: A. respiratory problems B. heart disease C. drug abuse D. sleep apnea

A. respiratory problems SVN machines are used to nebulize bronchodilator medications

You are called for a drowning victim at a family BBQ. Upon arrival, 2 family members approach you yelling, swearing, and demanding to know what took you so long. One of the family members pushes you. You should: A. retreat and request immediate assistance from law enforcement B. explain to the family where you responded from and why you were delayed C. order the family members to back off and take you to the patient D. tell the family that assaulting an EMS provider is a felony

A. retreat and request immediate assistance from law enforcement #scenesafety

Common complications associated with central venous catheters include all of the following, EXCEPT: A. rupture of a central vein. B. bleeding around the line. C. clotting of the line. D. a local infection.

A. rupture of a central vein

When assessing a patient with a head injury, you note the presence of thin, bloody fluid draining from his right ear. This indicates: A. rupture of the tympanic membrane following diffuse impact to the head. B. fractures to the internal structures of the ear following direct trauma. C. significant pressure and bleeding in between the skull and dura mater. D. a linear skull fracture and a significant increase in intracranial pressure.

A. rupture of the tympanic membrane following diffuse impact to the head. Correct B. fractures to the internal structures of the ear following direct trauma. C. significant pressure and bleeding in between the skull and dura mater. D. a linear skull fracture and a significant increase in intracranial pressure.

Because a tracheostomy tube bypasses the nose and mouth: A. secretions can build up in and around the tube. B. severe swelling of the trachea and bronchi can occur. C. the risk of a local infection is significantly high. D. bleeding or air leakage may occur around the tube.

A. secretions can build up in and around the tube

The risk of bleeding in the skull, which increases with age, is MOST directly related to: Select one: A. shrinkage of the brain. B. a decrease in neurons. C. meningeal deterioration. D. blood vessel dilation.

A. shrinkage of the brain.

When determining the frequency of contractions, you should time the contractions from the: A. start of one to the start of the next. B. start of one to the end of the next. C. end of one to the start of the next. D. end of one to the end of the next.

A. start of one to the start of the next.

The EMT should ensure that vaginal bleeding is _________. A. taken seriously and the patient is transported for gynecologic evaluation B. severe enough to warrant transport C. controlled directly by the patient D. not caused by traumatic injury

A. taken seriously and the patient is transported for gynecologic evaluation

At present, the likelihood of a nuclear attack against the United States is very low because: A. terrorist nations do not have the ability to deliver a nuclear weapon via missile or bomb. B. other than the United States, no other countries are currently in possession of nuclear weapons. C. the United States has an effective early warning system to detect an incoming nuclear missile. D. all nuclear devices or weapons that different countries are in possession of are currently accounted for.

A. terrorist nations do not have the ability to deliver a nuclear weapon via missile or bomb. Correct

The presence of meconium in the amniotic fluid indicates: A. that the baby's airway may be obstructed. B. that full newborn resuscitation will be needed. C. an expected finding in full-term infants. D. that the fetus is at least 4 weeks premature.

A. that the baby's airway may be obstructed.

Which of the following statements regarding the Americans With Disabilities Act (ADA) of 1990 is correct? Select one: A. The ADA prohibits employers from failing to provide full and equal employment to those who are disabled. B. The minimum number of hours required to successfully complete an EMT course is less for candidates who are disabled. C. The ADA applies only to individuals with a diagnosed and well-documented physical disability. D. According to the ADA, EMT candidates with a documented disability are exempt from taking the NREMT exam.

A. the ADA prohibits employers from failing to provide full and equal employment to those who are disabled

When caring for a patient with a behavioral emergency, remember: A. the behavior may be caused by a physiological condition. B. treatment is not needed if there are no physiological abnormalities. C. any conscious patient can refuse treatment and transport. D. the patient must be restrained if he/she verbalizes suicidal thoughts.

A. the behavior may be caused by a physiological condition. patients must be *competent* as well as conscious to refuse treatment

In contrast to younger patients, older patients are more prone to a decrease in blood pressure (BP) upon standing because: Select one: A. the body is less able to adapt the BP to rapid postural changes. B. any change in position causes blood to be shunted to the brain. C. their red blood cells are destroyed at a faster than normal rate. D. the aging process results in an overall increase in blood volume.

A. the body is less able to adapt the BP to rapid postural changes.

Large amounts of adenosine triphosphate (ATP) are generated when: A. the cells function with adequate oxygen. B. carbon dioxide levels in the blood are high. C. circulating blood glucose levels fall. D. the cells function without oxygen.

A. the cells function with adequate oxygen. Correct

The ideal procedure for moving an injured patient from the ground to a backboard is: A. the clothes drag. B. the direct patient carry. C. the four-person log roll. D. the use of a scoop stretcher.

A. the clothes drag. B. the direct patient carry. C. the four-person log roll. Correct D. the use of a scoop stretcher.

Hypovolemic shock occurs when: A. the clotting ability of the blood is enhanced. B. at least 10% of the patient's blood volume is lost. C. the body cannot compensate for rapid blood loss. D. the patient's systolic blood pressure is less than 100 mm Hg

A. the clotting ability of the blood is enhanced. B. at least 10% of the patient's blood volume is lost. C. the body cannot compensate for rapid blood loss. Correct D. the patient's systolic blood pressure is less than 100 mm Hg

De-oxygenated blood returns to the

Right atrium

When immobilizing a trauma patient's spine, the EMT manually stabilizing the head should not let go until: A. the head has been stabilized with lateral immobilization. B. the patient has been completely secured to the backboard. C. an appropriately sized cervical collar has been applied. D. the patient has been secured to the ambulance stretcher.

A. the head has been stabilized with lateral immobilization. B. the patient has been completely secured to the backboard. Correct C. an appropriately sized cervical collar has been applied. D. the patient has been secured to the ambulance stretcher.

A tight-fitting motorcycle helmet should be left in place unless: A. the helmet is equipped with a full face shield or visor. B. the patient complains of severe neck or back pain. C. the patient must be placed onto a long backboard. D. it interferes with your assessment of the airway.

A. the helmet is equipped with a full face shield or visor. B. the patient complains of severe neck or back pain. C. the patient must be placed onto a long backboard. D. it interferes with your assessment of the airway. Correct

Which of the following is true during inhalation of a spontaneously breathing patient? A. there is a drop in pressure within the thorax B. there is an increase in pressure within the thorax C. air is being pushed into the lungs D. the diaphragm and intercostal muscles are relaxed

A. there is a drop in pressure within the thorax

The MOST prominent symptom of decompression sickness is: A. tightness in the chest. B. dizziness and nausea. C. difficulty with vision. D. abdominal or joint pain.

A. tightness in the chest. B. dizziness and nausea. C. difficulty with vision. D. abdominal or joint pain. Correct

A person who routinely misuses a substance and requires increasing amounts to achieve the same effect is experiencing: Select one: A. tolerance. B. withdrawal. C. dependence. D. addiction.

A. tolerance.

A 70-year-old male complains of shortness of breath. During your assessment, you note that he has bilateral hearing aids. When you ask him questions related to his chief complaint, he does not answer you. You can hear a whistling sound coming from his hearing aids. You should: A. try repositioning the hearing aid or remove it and turn down the volume. B. remove both of his hearing aids and use pencil and paper to communicate. C. remove his hearing aids, turn up the volume, and replace them in his ears. D. recognize that the batteries in his hearing aids are probably depleted.

A. try repositioning the hearing aid or remove it and turn down the volume.

Drowning is MOST accurately defined as: A. water in the lungs following submersion in water. B. death from suffocation after submersion in water. C. temporary survival after submersion in water. D. death beyond 24 hours after submersion in water.

A. water in the lungs following submersion in water. B. death from suffocation after submersion in water. Correct C. temporary survival after submersion in water. D. death beyond 24 hours after submersion in water.

Gloves, a mask, eye protection, and a face shield should be used: Select one: A. when performing endotracheal intubation. B. during routine cleaning of the ambulance. C. while handling needles or other sharps. D. whenever you touch nonintact skin.

A. when performing endotracheal intubation

ABC vs CAB sequence

ABC (airway, breathing, circulation) is used in conscious patients CAB (circulation, airway, breathing) is used in unresponsive patients

Which of the following conditions or situations presents the MOST unique challenge to the EMT when immobilizing an elderly patient on a long backboard? A. Abnormal spinal curvature B. Joint flexibility C. Naturally deformed bones D. Patient disorientation

Abnormal spinal curvature.

Which of the following open soft-tissue injuries is limited to the superficial layer of the skin and results in the least amount of blood loss? A. Laceration B. Abrasion C. Avulsion D. Incision

Abrasion Correct

How does a disaster differ from a mass-casualty incident? Disasters may not involve personal injuries. In a disaster, EMS may be on the scene for days or weeks. Only an elected official can declare a disaster. All of the above

All of the above

Why do colds develop so easily in toddlers and preschoolers? They experience a loss of passive immunity. They do not have well-developed lung musculature. They are spending a lot of time around playmates and classmates. All of the above

All of the above

Why is breathing more labor intensive for the elderly? The size of the airway increases and the surface area of the alveoli decreases. The natural elasticity of the lungs decreases. The overall strength of the intercostal muscles and the diaphragm decreases. All of the above

All of the above

What is the purpose of the incident command system (ICS)? Ensuring responder and public safety Achieving incident management goals Ensuring the efficient use of resources All of the above.

All of the above.

When using abbreviations, acronyms, or symbols, an EMT should: be familiar with those used in your agency. use only those that are medically accepted. use them to shorten documentation. All of the above.

All of the above.

A behavioral crisis interferes with which of the following? A. Activities of daily living B. Behavior that is acceptable to the community C. Dressing, eating, or bathing D. All of these answers are correct.

All of these answers are correct.

When providing bag-mask ventilations to an infant, what is most important to remember? Blood pressure typically increases with age. An infant's lungs are fragile. An infant grows at a rate of about 30 g per day. An infant has a proportionately larger tongue than an adult.

An infant's lungs are fragile.

Which stage of grieving commonly results in blame? Denial Anger, hostility Bargaining Depression

Anger, hostility

A 40-year-old man, who was the unrestrained driver of a car that hit a tree at a high rate of speed, struck the steering wheel with his chest. He has a large bruise over the sternum and an irregular pulse rate of 120 beats/min. You should be MOST concerned that he: A. has injured his myocardium. B. has a collapsed lung and severe hypoxia. C. has extensive bleeding into the pericardial sac. D. is at extremely high risk for ventricular fibrillation.

Answer: A Rationale: A myocardial contusion, or bruising of the heart muscle, is usually the result of blunt trauma—specifically, to the center of the chest. In some cases, the injury may be so severe that it renders the heart unable to maintain adequate cardiac output; as a result, blood pressure falls. The pulse rate is often irregular; however, lethal cardiac dysrhythmias such as ventricular tachycardia and ventricular fibrillation are uncommon. A 40-year-old man, who was the unrestrained driver of a car that hit a tree at a high rate of speed, struck the steering wheel with his chest. He has a large bruise over the sternum and an irregular pulse rate of 120 beats/min. You should be MOST concerned that he: A. has injured his myocardium. - Rationale: Correct answer B. has a collapsed lung and severe hypoxia. - Rationale: This will produce an absence or decrease of breath sounds and unilateral chest wall expansion. C. has extensive bleeding into the pericardial sac. - Rationale: This will produce muffled heart sounds and decreased cardiac output. D. is at extremely high risk for ventricular fibrillation. - Rationale: Lethal dysrhythmias are uncommon. A. B. C. D.

The purpose of a shunt is to: A. minimize pressure within the skull. B. reroute blood away from the lungs. C. instill food directly into the stomach. D. drain excess fluid from the peritoneum.

Answer: A Rationale: A ventriculoperitoneal (VP) shunt—simply called a "shunt"—is a tube that extends from the ventricles (cavities) of the brain to the peritoneal cavity. VP shunts are used to drain excess fluid from the brain, thus preventing increased pressure within the skull. The purpose of a shunt is to: A. minimize pressure within the skull. - Rationale: Correct answer B. reroute blood away from the lungs. - Rationale: The shunt is connected from the brain to the abdomen. C. instill food directly into the stomach. - Rationale: The shunt drains excess cerebrospinal fluid from the brain. D. drain excess fluid from the peritoneum. - Rationale: The shunt drains excess cerebrospinal fluid from the brain.

While inspecting the interior of a wrecked automobile, you should be MOST suspicious that the driver experienced an abdominal injury if you find: A. a deformed steering wheel. B. that the airbags deployed. C. a crushed instrument panel. D. damage to the lower dashboard.

Answer: A Rationale: Airbags save lives when used in conjunction with properly worn seatbelts. Unfortunately, however, not all drivers wear their seatbelts. If unrestrained, the driver's abdomen may strike the steering wheel, resulting in significant trauma. Suspect this if you lift the airbag and note that the lower part of the steering wheel is deformed. While inspecting the interior of a wrecked automobile, you should be MOST suspicious that the driver experienced an abdominal injury if you find: A. a deformed steering wheel. - Rationale: Correct answer B. that the airbags deployed. - Rationale: Typically, the face and chest are impacted by airbags if safety belts are worn properly. C. a crushed instrument panel. - Rationale: This would indicate the possibility of leg and hip injuries. D. damage to the lower dashboard. - Rationale: This would indicate the possibility of leg and hip injuries.

Which of the following mechanisms of injury would MOST likely cause a crushing injury of the larynx and/or trachea? A. Attempted suicide by hanging B. Gunshot wound to the lateral neck C. Car crash involving lateral impact D. Patient whose head hits the windshield

Answer: A Rationale: Any crushing injury of the upper part of the neck is likely to involve the larynx or trachea. Examples include the anterior neck impacting a steering wheel, hanging (distraction) mechanisms, and clothesline injuries. Which of the following mechanisms of injury would MOST likely cause a crushing injury of the larynx and/or trachea? A. Attempted suicide by hanging - Rationale: Correct answer B. Gunshot wound to the lateral neck - Rationale: This would produce a penetrating injury. C. Car crash involving lateral impact - Rationale: This would produce an injury to the spine and possibly the head. D. Patient whose head hits the windshield - Rationale: This would produce an injury to the head or a compression injury to the spine.

A 22-year-old male was attacked by a rival gang and has a large knife impaled in the center of his chest. Your assessment reveals that he is apneic and pulseless. You should: A. carefully remove the knife, control any bleeding, begin CPR, and transport. B. stabilize the knife in place, provide rescue breathing, and transport at once. C. remove the knife and control any bleeding, apply the AED, and analyze his rhythm. D. begin CPR, control any external bleeding, and transport rapidly to a trauma center.

Answer: A Rationale: As a rule, impaled objects should be stabilized in place. However, if they interfere with the patient's breathing or your ability to perform CPR, they should be removed. You cannot perform CPR on a patient if a knife is impaled in the center of the chest. Carefully remove the knife, control any bleeding, begin CPR, and transport at once. The AED is not indicated for patients with traumatic cardiac arrest; their arrest is usually caused by massive blood loss, not a primary cardiac dysrhythmia. A 22-year-old male was attacked by a rival gang and has a large knife impaled in the center of his chest. Your assessment reveals that he is apneic and pulseless. You should: A. carefully remove the knife, control any bleeding, begin CPR, and transport. - Rationale: Correct answer B. stabilize the knife in place, provide rescue breathing, and transport at once. - Rationale: The knife must be removed to provide effective CPR. C. remove the knife and control any bleeding, apply the AED, and analyze his rhythm. - Rationale: An AED is not recommended in traumatic arrest, but CPR must be initiated. D. begin CPR, control any external bleeding, and transport rapidly to a trauma center. - Rationale: The impaled object must be removed prior to the initiation of chest compressions.

When caring for a female with trauma to the external genitalia, the EMT should: A. use local pressure to control bleeding. B. carefully pack the vagina to reduce bleeding. C. remove any impaled objects from the vagina. D. cover any open wounds with moist, sterile dressings.

Answer: A Rationale: Bleeding from the external genitalia should be controlled by applying a dry, sterile dressing and local direct pressure. Never pack anything into the vagina to try to control bleeding; this increases the risk of infection, and anything you place into the vagina will only need to be removed at the hospital. Impaled objects in the genitalia should be carefully stabilized in place, not removed. When caring for a female with trauma to the external genitalia, the EMT should: A. use local pressure to control bleeding.- Rationale: Correct answer B. carefully pack the vagina to reduce bleeding. - Rationale: Never pack anything into the vagina. C. remove any impaled objects from the vagina.- Rationale: Impaled objects are stabilized in place and are not removed. D. cover any open wounds with moist, sterile dressings. - Rationale: Apply dry, sterile dressings with local direct pressure.

When a small child falls from a significant height, the ______ MOST often strikes the ground first. A. head B. back C. feet D. side

Answer: A Rationale: Compared to adults, pediatric patients have proportionately larger heads. When they fall from a significant height, gravity usually takes them headfirst. This is why head trauma is the most common cause of traumatic death in the pediatric patient. When a small child falls from a significant height, the ______ MOST often strikes the ground first. A. head - Rationale: Correct answer B. back - Rationale: The head is heavier, and gravity tends to tilt the head in a downward direction. C. feet - Rationale: Adults will attempt to land feet first. D. side - Rationale: The head is heavier, and gravity tends to tilt the head in a downward direction.

A patient who experiences an immediate loss of consciousness followed by a lucid interval has a(n): A. epidural hematoma. B. subdural hematoma. C. concussion. D. contusion.

Answer: A Rationale: Epidural hematomas are caused by injury to an artery—usually the middle meningeal artery—that lies in between the skull and brain. Patients with an epidural hematoma typically experience an immediate loss of consciousness, followed by a brief period of consciousness (lucid interval), as intracranial pressure increases. Subdural hematomas are the result of injury to a vein; therefore, they tend to bleed slowly and usually cause a progressive decline in level of consciousness. Concussions and contusions may cause a loss of consciousness, but it is typically brief. A patient who experiences an immediate loss of consciousness followed by a lucid interval has a(n): A. epidural hematoma. - Rationale: Correct answer B. subdural hematoma. - A patient who experiences an immediate loss of consciousness followed by a lucid interval has a(n): C. concussion. - Rationale: Concussions may cause a loss of consciousness, but is typically brief. D. contusion. - Rationale: Contusions may cause a loss of consciousness, but is typically brief.

A 45-year-old convenience store clerk was shot in the right anterior chest during a robbery. Your assessment reveals that the wound has blood bubbling from it every time the patient breathes. Your MOST immediate action should be to: A. prevent air from entering the wound. B. cover the wound with a bulky dressing. C. assess the patient's back for an exit wound. D. transport the patient promptly to the closest trauma center.

Answer: A Rationale: Immediate treatment for a sucking chest wound (open pneumothorax) involves covering the wound with an occlusive dressing. This will prevent air from being drawn into the chest cavity. After covering the wound, assess for an exit wound, apply high-flow oxygen (if not already done), and transport promptly. A 45-year-old convenience store clerk was shot in the right anterior chest during a robbery. Your assessment reveals that the wound has blood bubbling from it every time the patient breathes. Your MOST immediate action should be to: A. prevent air from entering the wound. - Rationale: Correct answer B. cover the wound with a bulky dressing. - Rationale: You must use an occlusive dressing. C. assess the patient's back for an exit wound. - Rationale: Do this after the anterior chest wound is covered. D. transport promptly to the closest trauma center. - Rationale: Do this after the initial treatment of an open chest wound.

An unrestrained driver collided with a bridge pillar. Upon inspection of the interior of his vehicle, you note that the lower dashboard is crushed. During your assessment of the patient, you will MOST likely encounter: A. trauma to the pelvis. B. blunt abdominal trauma. C. a severe closed head injury. D. penetrating thoracic trauma.

Answer: A Rationale: Impact points are often obvious from a quick inspection of the vehicle's interior. During a frontal collision, the unrestrained occupant's knees often impact the lower dashboard. With this type of impact, energy is transferred from the knees, to the femurs, and then to the pelvis or hip. An unrestrained driver collided with a bridge pillar. Upon inspection of the interior of his vehicle, you note that the lower dashboard is crushed. During your assessment of the patient, you will MOST likely encounter: A. trauma to the pelvis. - Rationale: Correct answer B. blunt abdominal trauma. - Rationale: This is usually a result of striking the steering wheel. C. a severe closed head injury. - Rationale: This is usually the result of striking the windshield. D. penetrating thoracic trauma. - Rationale: This is usually caused by flying debris, collision with parts of the vehicle, or other movable objects.

Which of the following sets of vital signs is LEAST indicative of internal bleeding? A. BP, 140/90 mm Hg; pulse rate, 58 beats/min; respirations, 8 breaths/min B. BP, 100/50 mm Hg; pulse rate, 120 beats/min; respirations, 24 breaths/min C. BP, 98/60 mm Hg; pulse rate, 110 beats/min; respirations, 28 breaths/min D. BP, 102/48 mm Hg; pulse rate, 100 beats/min; respirations, 22 breaths/min

Answer: A Rationale: Internal hemorrhage typically reveals vital signs that are consistent with shock: hypotension, tachycardia, and tachypnea. Hypertension, bradycardia, and bradypnea (choice "A") is consistent with a closed head injury, not internal bleeding. Which of the following sets of vital signs is LEAST indicative of internal bleeding? A. BP, 140/90 mm Hg; pulse rate, 58 beats/min; respirations, 8 breaths/min - Rationale: Correct answer B. BP, 100/50 mm Hg; pulse rate, 120 beats/min; respirations, 24 breaths/min- Rationale: This is indicative of a progression to decompensated shock. C. BP, 98/60 mm Hg; pulse rate, 110 beats/min; respirations, 28 breaths/min- Rationale: This is indicative of a progression to decompensated shock. D. BP, 102/48 mm Hg; pulse rate, 100 beats/min; respirations, 22 breaths/min - Rationale: This is indicative of a progression to decompensated shock.

While assisting a woman in labor, you visualize her vaginal area and see an arm protruding from her vagina. She tells you that she feels the urge to push. You should: A. cover the arm with a sterile towel and transport immediately. B. encourage her to keep pushing as you prepare for rapid transport. C. insert your gloved fingers into the vagina and try to turn the baby. D. instruct the mother to keep pushing and give her high-flow oxygen.

Answer: A Rationale: Limb presentations do not deliver in the field—period! If the mother feels the urge to push, instruct her to stop; she should pant instead. Cover the protruding limb with a sterile towel, administer high-flow oxygen to the mother, and transport immediately. Delivery must take place in the hospital. While assisting a woman in labor, you visualize her vaginal area and see an arm protruding from her vagina. She tells you that she feels the urge to push. You should: A. cover the arm with a sterile towel and transport immediately. - Rationale: Correct answer B. encourage her to keep pushing as you prepare for rapid transport. - Rationale: EMS cannot successfully deliver such a presentation in the field. C. insert your gloved fingers into the vagina and try to turn the baby.- Rationale: You should only do this to create an airway for the infant in a breech presentation. D. instruct the mother to keep pushing and give her high-flow oxygen. - Rationale: EMS cannot successfully deliver such a presentation in the field.

Paradoxical chest movement is typically seen in patients with: A. a flail chest. B. a pneumothorax. C. isolated rib fractures. D. a ruptured diaphragm.

Answer: A Rationale: Paradoxical chest movement occurs when an area of the chest wall bulges out during exhalation and collapses during inhalation. This type of abnormal chest movement is seen in patients with a flail chest—a condition in which several adjacent ribs are fractured in more than one place, resulting in a free-floating segment of fractured ribs. Paradoxical chest movement is typically seen in patients with: A. a flail chest.- Rationale: Correct answer B. a pneumothorax. - Rationale: This will produce unilateral chest wall movement. C. isolated rib fractures. - Rationale: This will produce pain, but not irregular chest wall movement. D. a ruptured diaphragm. - Rationale: This typically occurs on the left side. You may hear bowel sounds over the lower chest area.

Which of the following drugs is commonly used to facilitate sexual assault? A. Rohypnol B. Heroin C. Cocaine D. Marijuana

Answer: A Rationale: Rohypnol is a sedative that is used by criminals to facilitate sexual assault by depressing the victim's central nervous system. Which of the following drugs is commonly used to facilitate sexual assault? A. Rohypnol - Rationale: Correct answer B. Heroin - Rationale: Heroin is not used to facilitate sexual assault. C. Cocaine - Rationale: Cocaine is not used to facilitate sexual assault. D. Marijuana - Rationale: Marijuana is not used to facilitate sexual assault.

A 70-year-old man is experiencing a severe nosebleed. When you arrive, you find him leaning over a basin, which contains an impressive amount of blood. He has a history of coronary artery disease, diabetes, and migraine headaches. His BP is 180/100 and his heart rate is 100 beats/min. Which of the following is the MOST likely contributing factor to his nosebleed? A. His blood pressure B. His history of diabetes C. The fact that he is elderly D. His heart rate of 100 beats/min

Answer: A Rationale: Several conditions can cause a nosebleed (epistaxis), including skull fractures, facial injuries, sinusitis (inflamed sinuses), high blood pressure, coagulation disorders (ie, hemophilia), and digital trauma (ie, nose picking). A BP of 180/100 indicates a significant amount of pressure on the arteries, which is no doubt the main contributing factor to this patient's nosebleed. A 70-year-old man is experiencing a severe nosebleed. When you arrive, you find him leaning over a basin, which contains an impressive amount of blood. He has a history of coronary artery disease, diabetes, and migraine headaches. His BP is 180/100 and his heart rate is 100 beats/min. Which of the following is the MOST likely contributing factor to his nosebleed? A. His blood pressure - Rationale: Correct answer B. His history of diabetes- Rationale: Diabetes can be a cause of hypertension and vascular problems, but typically is not a condition that will cause epistaxis. C. The fact that he is elderly- Rationale: Elderly patients are prone to hypertension, which can cause epistaxis, but age is not a factor. D. His heart rate of 100 beats/min - Rationale: His heart rate may be a result of his age or a compensatory mechanism dealing with blood loss.

The brain, a part of the central nervous system (CNS), is divided into the: A. cerebrum, cerebellum, and brain stem. B. cerebrum, brain stem, and spinal cord. C. cerebellum, cerebrum, and spinal cord. D. spinal cord, cerebrum, and cerebral cortex.

Answer: A Rationale: The brain and spinal cord comprise the central nervous system (CNS). The brain is divided into three major regions: the cerebrum (the largest portion), the cerebellum, and the brain stem. Each region of the brain carries out specific functions. The brain, a part of the central nervous system (CNS), is divided into the: A. cerebrum, cerebellum, and brain stem. - Rationale: Correct answer B. cerebrum, brain stem, and spinal cord. - The brain, a part of the central nervous system (CNS), is divided into the: C. cerebellum, cerebrum, and spinal cord. - Rationale: The spinal cord is not part of the brain. D. spinal cord, cerebrum, and cerebral cortex. - Rationale: The spinal cord is not part of the brain.

The first stage of labor ends when: A. the presenting part of the baby is visible. B. contractions are less than 10 minutes apart. C. the mother experiences her first contraction. D. the amniotic sac ruptures and labor pains begin.

Answer: A Rationale: The first stage of labor begins with the onset of contractions and ends when the cervix is fully dilated. However, since cervical dilation cannot be assessed in the field, the first stage of labor is considered over when the presenting part of the baby is visible at the vaginal opening (crowning). The first stage of labor ends when: A. the presenting part of the baby is visible. - Rationale: Correct answer B. contractions are less than 10 minutes apart. - Rationale: True labor is when the frequency and intensity of contractions increase and is part of the first stage of labor. C. the mother experiences her first contraction. - Rationale: This is the beginning of the first stage of labor. D. the amniotic sac ruptures and labor pains begin. - Rationale: This is considered to be a part of the first stage of labor.

Which of the following organs would be the MOST likely to bleed profusely if severely injured? A. Liver B. Kidney C. Stomach D. Gallbladder

Answer: A Rationale: The liver is a highly vascular solid organ, and contains approximately 40% of the body's total blood volume at any given time. If severely injured, bleeding from the liver would be profuse and rapid. Other solid organs, such as the spleen and kidneys, may also produce severe bleeding if injured, though not as rapidly as the liver. The stomach and gallbladder are hollow organs; if lacerated, they would spill their contents into the abdominal cavity, resulting in peritonitis. Which of the following organs would be the MOST likely to bleed profusely if severely injured? A. Liver- Rationale: Correct answer B. Kidney - Rationale: This will produce bleeding, but not as rapidly as the liver. C. Stomach- Rationale: This hollow organ will spill its contents into the abdominal cavity. D. Gallbladder - Rationale: This hollow organ will spill its contents into the abdominal cavity.

When using the mnemonic CHILD ABUSE to assess a child for signs of abuse, you should recall that the "D" stands for: A. delay in seeking care. B. divorced parents. C. dirty appearance. D. disorganized speech.

Answer: A Rationale: The mnemonic CHILD ABUSE stands for Consistency of the injury with the child's developmental age, History inconsistent with the injury, Inappropriate parental concerns, Lack of supervision, Delay in seeking care, Affect, Bruises of varying stages, Unusual injury patterns, Suspicious circumstances, and Environmental clues. A delay in care may happen when the parent or caregiver does not want the abuse noted by other people. When using the mnemonic CHILD ABUSE to assess a child for signs of abuse, you should recall that the "D" stands for: A. delay in seeking care. - Rationale: Correct answer B. divorced parents. - Rationale: Divorce may put the child at greater risk, but does not indicate the child is being abused. C. dirty appearance. - Rationale: This is something providers should be aware of. A potential for abuse exists, but this does not indicate that the child is being abused. D. disorganized speech. - Rationale: This may indicate a learning disability or handicap.

Which of the following statements regarding the "Adam's apple" is FALSE? A. It is inferior to the cricoid cartilage. B. It is formed by the thyroid cartilage. C. It is the uppermost part of the larynx. D. It is more prominent in men than in women.

Answer: A Rationale: The most obvious prominence in the center of the anterior neck is the Adam's apple. This prominence is the upper part of the larynx, formed by the thyroid cartilage. It is more prominent in men than in women. The other portion of the larynx is the cricoid cartilage, a firm ridge that is inferior to the thyroid cartilage. Which of the following statements regarding the "Adam's apple" is false? A. It is inferior to the cricoid cartilage. - Rationale: Correct answer. B. It is formed by the thyroid cartilage. - Rationale: This is true. C. It is the uppermost part of the larynx. - Rationale: This is true. D. It is more prominent in men than in women. - Rationale: This is true.

Which of the following musculoskeletal injuries has the GREATEST risk for shock due to blood loss? A. Pelvic fracture B. Posterior hip dislocation C. Unilateral femur fracture D. Proximal humerus fracture

Answer: A Rationale: The pelvic cavity can accommodate a large volume of blood. Shock in a patient with a pelvic injury is usually due to injury to femoral veins or arteries. Bilateral femur fractures can also cause severe blood loss (up to 1 liter per femur). Which of the following musculoskeletal injuries has the GREATEST risk for shock due to blood loss? A. Pelvic fracture - Rationale: Correct answer B. Posterior hip dislocation - Rationale: Unless the dislocation has injured the vascular system, bleeding will be contained and minimal. C. Unilateral femur fracture- Rationale: A unilateral femur fracture can lose 500 to 1,500 mL of blood. D. Proximal humerus fracture - Rationale: Nerves and blood vessels can be injured, and the blood loss could be 500 mL.

To effectively immobilize a fractured clavicle, you should apply a(n): A. sling and swathe. B. air splint over the entire arm. C. rigid splint to the upper arm, then a sling. D. traction splint to the arm of the injured side.

Answer: A Rationale: The quickest and most effective way to immobilize a fractured clavicle (collarbone) is to apply a sling and swathe. The sling will help minimize movement of the clavicle itself, while the swath will minimize movement of the arm on the affected side. To effectively immobilize a fractured clavicle, you should apply a(n): A. sling and swathe. - Rationale: Correct answer B. air splint over the entire arm. - Rationale: An air splint is not effective on a joint. C. rigid splint to the upper arm, then a sling.- Rationale: A sling will not prevent the movement of the shoulder. D. traction splint to the arm of the injured side. - Rationale: There is no traction splint for the arm.

When caring for a chemical burn to the eye, the EMT should: A. prevent contamination of the opposite eye. B. immediately cover the injured eye with a sterile dressing. C. avoid irrigating the eye, as this may cause further injury. D. irrigate both eyes simultaneously, even if only one eye is injured.

Answer: A Rationale: When irrigating a chemical burn to the eye, it is important to direct the stream away from the uninjured eye. If you do not, you will likely flush the chemical into the unaffected eye. After irrigating the eye for the appropriate amount of time, cover both eyes with a sterile dressing. When caring for a chemical burn to the eye, the EMT should: A. prevent contamination of the opposite eye. - Rationale: Correct answer B. immediately cover the injured eye with a sterile dressing. - Rationale: Irrigation of the eye must take place first. C. avoid irrigating the eye, as this may cause further injury. - Rationale: Irrigation of the affected eye must take place. Direct the stream away from the uninjured eye. D. irrigate both eyes simultaneously, even if only one eye is injured. - Rationale: Direct the stream of the contaminated eye away from the unaffected eye.

Whiplash injuries are MOST common following _________ impacts. A. Rear-end B. Rollover C. Frontal D. Lateral

Answer: A Rationale: Whiplash injuries of the neck are a common occurrence following rear-end collisions. As the vehicle is suddenly thrust forward, the occupant's head is thrust backward. Properly positioned headrests can minimize the severity of whiplash injuries. Whiplash injuries are MOST common following _________ impacts. A. rear-end - Rationale: Correct answer B. rollover - Rationale: This typically causes life-threatening injuries. C. frontal - Rationale: This typically causes chest, head, abdominal, and extremity injuries. D. lateral - Rationale: You should suspect lateral chest and abdominal injuries on the side of impact, as well as pelvic injuries.

If the cells do not receive glucose, they will begin to metabolize: fat. acid. sugar. ketones.

Answer: A Rationale: If the body's cells do not receive glucose, they will begin to metabolize the next most readily available substance—fat. Fat metabolism results in the production of ketoacids, which are released into the bloodstream (hence the term "ketoacidosis"). If the cells do not receive glucose, they will begin to metabolize: fat. Rationale: Correct answer acid. Rationale: Fatty acids are a by-product (waste product) of the metabolism of fat. If the cells do not receive glucose, they will begin to metabolize: sugar. Rationale: Sugar is glucose. ketones. Rationale: Ketones are a by-product (waste product) of the metabolism of fat.

A 45-year-old man with type 1 diabetes is found unresponsive. Which of the following questions is MOST important to ask his wife? "Did he take his insulin today?" "How long has he been a diabetic?" "Has he seen his physician recently?" "What kind of insulin does he take?"

Answer: A Rationale: All of these questions are important to ask the spouse of an unconscious diabetic. However, it is critical to ask if the patient took his insulin. This will help you differentiate hypoglycemic crisis from hyperglycemic crisis. For example, if the patient took his insulin and did not eat, or accidentally took too much insulin, you should suspect hypoglycemic crisis. If the patient did not take his insulin, you should suspect hyperglycemic crisis. A 45-year-old man with type 1 diabetes is found unresponsive. Which of the following questions is MOST important to ask his wife? "Did he take his insulin today?" Rationale: Correct answer "How long has he been a diabetic?" Rationale: This is useful SAMPLE history information. A 45-year-old man with type 1 diabetes is found unresponsive. Which of the following questions is MOST important to ask his wife? "Has he seen his physician recently?" Rationale: This is also important SAMPLE history information. "What kind of insulin does he take?" Rationale: This provides important information about a patient's medications.

Which one of the following is NOT an appropriate treatment for EMTs to provide to a patient who has a hematologic disorder? Analgesics for pain Support of symptoms High-flow oxygen therapy at 12 to 15 L/min Rapid transport

Answer: A Rationale: Although analgesics would benefit a patient suffering from a hematologic disorder, the administration of such medications is not in the scope of practice for the EMT. ALS providers would have to be present to provide this emergency care. Which one of the following is NOT an appropriate treatment for EMTs to provide to a patient who has a hematologic disorder? Analgesics for pain Rationale: Correct answer Support of symptoms Rationale: This is an appropriate treatment. Which one of the following is NOT an appropriate treatment for EMTs to provide to a patient who has a hematologic disorder? High-flow oxygen therapy at 12 to 15 L/min Rationale: This is an appropriate treatment. Rapid transport Rationale: This is an appropriate treatment.

A 71-year-old man with a history of hypertension and vascular disease presents with tearing abdominal pain. His blood pressure is 80/60 mm Hg, his heart rate is 120 beats/min, and his respirations are 28 breaths/min. Your assessment reveals that his abdomen is rigid and distended. Considering his medical history and vital signs, you should be MOST suspicious for a(n): aortic aneurysm. hemorrhagic stroke. acute myocardial infarction. infarction of the large intestine.

Answer: A Rationale: Arteriosclerosis is a vascular disease in which the arteries thicken, harden, and calcify. This places the patient at risk for stroke, heart disease, bowel infarction, and hypertension, among other conditions. Hypertension and vascular disease are significant risk factors for an aneurysm—a weakening in the wall of an artery. The patient's vital signs; abdominal pain; and rigid, distended abdomen should make you highly suspicious for a leaking abdominal aortic aneurysm. A 71-year-old man with a history of hypertension and vascular disease presents with tearing abdominal pain. His blood pressure is 80/60 mm Hg, his heart rate is 120 beats/min, and his respirations are 28 breaths/min. Your assessment reveals that his abdomen is rigid and distended. Considering his medical history and vital signs, you should be MOST suspicious for a(n): aortic aneurysm;Rationale: Correct answer hemorrhagic stroke. Rationale: This is when the patient complains of the worst headache of his life, loses the ability to speak, and eventually becomes difficult to arouse. It tends to worsen over time. A 71-year-old man with a history of hypertension and vascular disease presents with tearing abdominal pain. His blood pressure is 80/60 mm Hg, his heart rate is 120 beats/min, and his respirations are 28 breaths/min. Your assessment reveals that his abdomen is rigid and distended. Considering his medical history and vital signs, you should be MOST suspicious for a(n): acute myocardial infarction. Rationale: Although the patient history could predispose him to an acute MI, the symptoms would be pain in the chest or shoulder, nausea, vomiting, a feeling of shortness of breath, and sweating. A 71-year-old man with a history of hypertension and vascular disease presents with tearing abdominal pain. His blood pressure is 80/60 mm Hg, his heart rate is 120 beats/min, and his respirations are 28 breaths/min. Your assessment reveals that his abdomen is rigid and distended. Considering his medical history and vital signs, you should be MOST suspicious for a(n): infarction of the large intestine. Rationale: If the large intestine ruptures, it would present with signs of peritonitis.

A condition that clouds the lens of the eye is called: cataract. nystagmus. astigmatism. glaucoma.

Answer: A Rationale: As people get older, cataracts, or clouding of the lens of the eye, may interfere with vision. Glaucoma is a condition caused by increased intraocular pressure (IOP). Nystagmus is characterized by involuntary movement of the eyes. Astigmatism is an optical defect that causes blurred vision due to the inability of the eye to focus an object into a sharp, focused image on the retina. A condition that clouds the lens of the eye is called: cataract Rationale: Correct answer nystagmus. Rationale: This is a horizontal, involuntary movement of the eyes. astigmatism. Rationale: This is an optical defect that causes blurred vision. glaucoma. Rationale: This is a condition caused by increased intraocular pressure (IOP).

The medical term for inflammation of the urinary bladder is: cystitis. nephritis. cholecystitis. diverticulitis.

Answer: A Rationale: Cystitis is the medical term for inflammation of the urinary bladder. Nephritis is inflammation of the kidney. Cholecystitis is inflammation of the gallbladder. Diverticulitis is a condition in which small pouches in the colon (large intestine) become inflamed. The medical term for inflammation of the urinary bladder is: cystitis. Rationale: Correct answer nephritis. Rationale: Nephritis is the inflammation of the kidney. The medical term for inflammation of the urinary bladder is: cholecystitis. Rationale: Cholecystitis is the inflammation of the gallbladder. diverticulitis. Rationale: Diverticulitis is the inflammation of part of the large intestine.

A 56-year-old man experienced a sudden, severe headache and then became unresponsive. He has a history of high blood pressure. The MOST likely cause of his condition is a(n): hemorrhagic stroke. acute ischemic stroke. severe migraine headache. transient ischemic attack.

Answer: A Rationale: Hemorrhagic strokes are typically preceded by a sudden, severe headache (signals the rupture of a cerebral artery), after which the patient becomes unresponsive due to bleeding within the brain. Ischemic strokes and transient ischemic attacks generally do not cause a sudden, severe headache, and migraine headaches typically do not cause a loss of consciousness. A 56-year-old man experienced a sudden, severe headache and then became unresponsive. He has a history of high blood pressure. The MOST likely cause of his condition is a(n): hemorrhagic stroke. Rationale: Correct answer acute ischemic stroke. Rationale: This generally does not cause a sudden, severe headache. A 56-year-old man experienced a sudden, severe headache and then became unresponsive. He has a history of high blood pressure. The MOST likely cause of his condition is a(n): severe migraine headache. Rationale: This generally does not cause a loss of consciousness. transient ischemic attack. Rationale: This generally does not cause a sudden, severe headache.

A 30-year-old male, who ingested an unknown substance, begins to vomit. You should: collect the vomitus and bring it to the hospital. apply a bag-valve mask. analyze the vomitus and try to identify the poison. suction his oropharynx for no longer than 30 seconds.

Answer: A Rationale: If the patient vomits, examine the contents for pill fragments. Ensure that you are wearing proper personal protective equipment for this activity. Note and document anything unusual that you see. You should try to collect the vomitus in a separate plastic bag so that it can be analyzed at the hospital. A 30-year-old male, who ingested an unknown substance, begins to vomit. You should: collect the vomitus and bring it to the hospital. ;answer apply a bag-valve mask. Rationale: This will not help get rid of the vomitus. A 30-year-old male, who ingested an unknown substance, begins to vomit. You should: analyze the vomitus and try to identify the poison. Rationale: This should be left for the hospital to do. suction his oropharynx for no longer than 30 seconds. Rationale: You should suction for no longer than 15 seconds.

A diabetic patient presents with a blood glucose level of 310 mg/dL and severe dehydration. The patient's dehydration is the result of: excretion of glucose and water from the kidneys. a deficiency of insulin that causes internal fluid loss. an infection that often accompanies hyperglycemia. an inability to produce energy because of insulin depletion.

Answer: A Rationale: In severe hyperglycemia, the kidneys excrete excess glucose from the body. This process requires a large amount of water to accomplish; therefore, water is excreted with the glucose, resulting in dehydration. A diabetic patient presents with a blood glucose level of 310 mg/dL and severe dehydration. The patient's dehydration is the result of: excretion of glucose and water from the kidneys. Rationale: Correct answer A diabetic patient presents with a blood glucose level of 310 mg/dL and severe dehydration. The patient's dehydration is the result of: a deficiency of insulin that causes internal fluid loss. Rationale: A lack of insulin will cause the glucose level to rise, and it is the glucose that causes the fluid loss. A diabetic patient presents with a blood glucose level of 310 mg/dL and severe dehydration. The patient's dehydration is the result of: an infection that often accompanies hyperglycemia. Rationale: An infection is an invasion of the body by an organism—glucose is not a foreign element. A diabetic patient presents with a blood glucose level of 310 mg/dL and severe dehydration. The patient's dehydration is the result of: an inability to produce energy because of insulin depletion. Rationale: A body's inability to metabolize glucose does not cause a fever.

What is a wheal? A raised, swollen, well-defined area on the skin An area of localized swelling involving the lips, tongue, and larynx Generalized itching or burning that appears as multiple, small, raised areas on the skin An exaggerated immune response to any substance

Answer: A Rationale: Insect stings and bites can cause a wheal, which is a raised, swollen, well-defined area on the skin. There is no specific treatment for these injuries, although applying ice sometimes makes them less irritating. What is a wheal? A raised, swollen, well-defined area on the skin Rationale: Correct answer An area of localized swelling involving the lips, tongue, and larynx. Rationale: This is the definition of angioedema What is a wheal? Generalized itching or burning that appears as multiple, small, raised areas on the skin Rationale: This is the definition of urticaria. An exaggerated immune response to any substance Rationale: This is the definition of an allergic reaction.

An important part of the assessment process for a patient with special needs is to: interact with the caregiver. interact with the patient. talk to the manufacturer of the equipment being used. transport immediately.

Answer: A Rationale: Interaction with the caregiver of a child or adult with special needs will be extremely import. They are trained to use and troubleshoot problems with medical equipment. An important part of the assessment process for a patient with special needs is to: interact with the caregiver. Rationale: Correct answer interact with the patient. Rationale: Although this is important, it is more important to talk to the caregiver. An important part of the assessment process for a patient with special needs is to: talk to the manufacturer of the equipment being used. Rationale: The caregiver will be able to help you with the equipment. transport immediately. Rationale: It is more important to talk to the caregiver.

A 34-year-old woman with a recent history of pelvic inflammatory disease presents with acute severe abdominal pain. Her abdomen is distended and diffusely tender to palpation. Based on these findings, you should suspect: peritonitis. pancreatitis. appendicitis. cholecystitis.

Answer: A Rationale: Peritonitis—an inflammation of the thin membrane that lines the abdominal cavity—typically presents with acute abdominal pain. Causes of peritonitis include infection and blunt or penetrating abdominal trauma. The pain caused by peritonitis is typically diffuse (widespread), whereas appendicitis, pancreatitis, and cholecystitis (inflammation of the gallbladder) typically present with pain that is localized to a particular area. A 34-year-old woman with a recent history of pelvic inflammatory disease presents with acute severe abdominal pain. Her abdomen is distended and diffusely tender to palpation. Based on these findings, you should suspect: peritonitis. Rationale: Correct answer pancreatitis. Rationale: Pancreatitis is usually a localized pain (in one specific area). A 34-year-old woman with a recent history of pelvic inflammatory disease presents with acute severe abdominal pain. Her abdomen is distended and diffusely tender to palpation. Based on these findings, you should suspect: appendicitis. Rationale: Appendicitis is usually a localized pain (in one specific area). cholecystitis. Rationale: Cholecystitis is usually a localized pain (in one specific area).

Polypharmacy is a term used to describe a patient who takes: multiple medications. other people's medications. a medication more than once a day. medication only when he or she feels the need to.

Answer: A Rationale: Polypharmacy is a term used to describe a patient who takes multiple medications every day. The more medications a patient takes, the greater the risk of a negative drug interaction. Polypharmacy is a term used to describe a patient who takes: multiple medications Rationale: Correct answer other people's medication. Rationale: This is incorrect. a medication more than once a day. Rationale: Many medications are taken more than once a day. medication only when he or she feels the need to. Rationale: This is considered noncompliant.

Patients with diabetic ketoacidosis experience polydipsia because: they are dehydrated secondary to excessive urination. the cells of the body are starved due to a lack of glucose. fatty acids are being metabolized at the cellular level. hyperglycemia usually causes severe internal water loss.

Answer: A Rationale: Severe hyperglycemia—which leads to diabetic ketoacidosis—causes the body to excrete large amounts of glucose and water. As a result, the patient becomes severely dehydrated, which leads to excessive thirst (polydipsia). Patients with diabetic ketoacidosis experience polydipsia because: they are dehydrated secondary to excessive urination. Rationale: Correct answer the cells of the body are starved due to a lack of glucose. Rationale: True, but the lack of glucose does not cause thirst. Patients with diabetic ketoacidosis experience polydipsia because: fatty acids are being metabolized at the cellular level. Rationale: Fats are metabolized by the cells instead of glucose, which produces acids and ketones—thus the term ketoacidosis. hyperglycemia usually causes severe internal water loss. Rationale: This is false. It causes water loss due to glucose being excreted (externally) in the urine solution.

A 70-year-old man presents with an acute onset of severe, tearing abdominal pain that radiates to his back. His BP is 88/66 mm Hg, pulse rate is 120 beats/min, and respirations are 26 breaths/min. Treatment for this patient should include: rapid transport to the hospital. firm palpation of the abdomen. placing him in a sitting position. oxygen at 4 L/min via nasal cannula.

Answer: A Rationale: Severe, tearing abdominal pain that radiates to the back is typical of an abdominal aortic aneurysm (AAA); it commonly occurs in older patients—especially those with hypertension. Treatment includes high-flow oxygen and rapid transport. If the patient has signs of shock, place him or her supine. Do not vigorously palpate the patient's abdomen; doing so may cause the aneurysm to rupture. A 70-year-old man presents with an acute onset of severe, tearing abdominal pain that radiates to his back. His BP is 88/66 mm Hg, pulse rate is 120 beats/min, and respirations are 26 breaths/min. Treatment for this patient should include: rapid transport to the hospital. Rationale: Correct answer firm palpation of the abdomen. Rationale: A firm or vigorous palpation is contraindicated in patients with severe and sudden-onset abdominal pain. A 70-year-old man presents with an acute onset of severe, tearing abdominal pain that radiates to his back. His BP is 88/66 mm Hg, pulse rate is 120 beats/min, and respirations are 26 breaths/min. Treatment for this patient should include: placing him in a sitting position. Rationale: Hypotension is treated by elevating the patient's legs into the shock position. oxygen at 4 L/min via nasal cannula. Rationale: High-flow oxygen is indicated in the treatment of shock.

Shortly after ascending rapidly to the surface of the water while holding his breath, a 29-year-old diver begins coughing up pink, frothy sputum and complains of dyspnea and chest pain. You should suspect and treat this patient for: an air embolism. a pneumothorax. pneumomediastinum. decompression sickness.

Answer: A Rationale: Signs of an air embolism, which present after a person rapidly ascends to the surface of the water while holding his or her breath, include skin mottling, pink froth at the mouth or nose, muscle or joint pain, dyspnea and/or chest pain, dizziness, nausea or vomiting, visual impairment, paralysis or coma, and even cardiac arrest. Shortly after ascending rapidly to the surface of the water while holding his breath, a 29-year-old diver begins coughing up pink, frothy sputum and complains of dyspnea and chest pain. You should suspect and treat this patient for: an air embolism. Rationale: Correct answer a pneumothorax. Rationale: A pneumothorax is a rupture or perforation of the pleura, causing air to leak into the pleural sac. Shortly after ascending rapidly to the surface of the water while holding his breath, a 29-year-old diver begins coughing up pink, frothy sputum and complains of dyspnea and chest pain. You should suspect and treat this patient for: pneumomediastinum. Rationale: This is air found in the mediastinum, between the lungs. decompression sickness. Rationale: This is a condition marked by joint pain, nausea, loss of motion, and breathing difficulties.

The MOST reliable indicator of upper airway swelling during a severe allergic reaction is: stridor. anxiety. cyanosis. wheezing.

Answer: A Rationale: Stridor is a high-pitched sound that is most often heard during inhalation. It indicates swelling of the upper airway. Wheezing, a whistling sound, is caused by narrowed bronchioles; it indicates narrowing or swelling of the lower airway. Anxiety and cyanosis can occur from a variety of causes; they are not exclusive to airway swelling. The MOST reliable indicator of upper airway swelling during a severe allergic reaction is: stridor. Rationale: Correct answer anxiety. Rationale: This is typically a symptom of hypoxia or decreased oxygenation to the brain. The MOST reliable indicator of upper airway swelling during a severe allergic reaction is: cyanosis. Rationale: This is a sign of hypoxia and inadequate tissue perfusion. wheezing. Rationale: This is a sign of lower airway constriction or narrowing.

What does the DOPE mnemonic help you to recognize? Causes of airway obstruction Risk factors for patients using technology assistance Questions to ask patients with pacemakers A vagal nerve stimulator

Answer: A Rationale: The DOPE mnemonic helps you to recognize causes of airway obstruction in patients using technology assistance. What does the DOPE mnemonic help you to recognize? Causes of airway obstruction Rationale: Correct answer Risk factors for patients using technology assistance Rationale: This is not the correct answer. Questions to ask patients with pacemakers Rationale: This is not the correct answer. A vagal nerve stimulator Rationale: This is not the correct answer.

Your patient opens his eyes when you say his name, is making incomprehensible sounds, and withdraws when you pinch his earlobe. What is his GCS score? 9 8 11 12

Answer: A Rationale: The Glasgow Coma Scale gives a score of 3 to a patient who opens his or her eyes in response to speech. "Incomprehensible sounds" has a score of 2, and "withdraws to pain" has a score of 4. When added together, this patient's GCS score is 9. Your patient opens his eyes when you say his name, is making incomprehensible sounds, and withdraws when you pinch his earlobe. What is his GCS score? 9 Rationale: Correct answer 8 Rationale: This is not the right score. 11 Rationale: This is not the right score. 12 Rationale: This is not the right score.

A 41-year-old man presents with slow, irregular breathing; hypotension; and dilated pupils. These signs MOST likely indicate dysfunction of the: brain stem. hypothalamus. cerebrum. cerebellum.

Answer: A Rationale: The brain stem is responsible for functions such as breathing, blood pressure, and pupil constriction. Brain stem dysfunction would result in abnormal findings with these functions. A 41-year-old man presents with slow, irregular breathing; hypotension; and dilated pupils. These signs MOST likely indicate dysfunction of the: brain stem. Rationale: Correct answer hypothalamus. Rationale: The hypothalamus causes changes to occur in the heart rate, body temperature, and thirst. A 41-year-old man presents with slow, irregular breathing; hypotension; and dilated pupils. These signs MOST likely indicate dysfunction of the: cerebrum. Rationale: The cerebrum causes changes to occur in emotion, thought, touch, and movement. cerebellum. Rationale: The cerebellum causes changes to occur in muscle control, body coordination, standing, walking, and writing.

The mnemonic DUMBELS can be used to recall the signs and symptoms of a cholinergic drug poisoning. The "E" in DUMBELS stands for: emesis. erythema. ecchymosis. elevated blood pressure.

Answer: A Rationale: The mnemonic "DUMBELS," which can help you recall the signs and symptoms of cholinergic drug poisoning (ie, organophosphates), stands for excessive defecation, urination, miosis (pupillary constriction), bronchorrhea, emesis, lacrimation, and salivation. The mnemonic DUMBELS can be used to recall the signs and symptoms of a cholinergic drug poisoning. The "E" in DUMBELS stands for: emesis. ;answer erythema. Rationale: Erythema is a redness of the skin usually caused by fever or inflammation. The mnemonic DUMBELS can be used to recall the signs and symptoms of a cholinergic drug poisoning. The "E" in DUMBELS stands for: ecchymosis. Rationale: Ecchymosis is bleeding from broken blood vessels into surrounding tissue. elevated blood pressure. Rationale: Elevated blood pressure usually causes bradycardia.

A 30-year-old male, who has been playing softball all day in a hot environment, complains of weakness and nausea shortly after experiencing a syncopal episode. Appropriate treatment for this patient includes all of the following, EXCEPT: giving a salt-containing solution by mouth. moving him to a cooler environment at once. administering oxygen via nonrebreathing mask. placing him in a supine position and elevating his legs.

Answer: A Rationale: Treatment for heat exhaustion begins by moving the patient to a cooler environment. Remove excess clothing, administer oxygen as needed, and place the patient supine. Elevating the patient's legs may improve blood flow to the brain and prevent another syncopal episode. If the patient is not nauseated, give a salt-containing solution by mouth. Give nothing by mouth if the patient is nauseated; doing so increases the risks of vomiting and aspiration. A 30-year-old male, who has been playing softball all day in a hot environment, complains of weakness and nausea shortly after experiencing a syncopal episode. Appropriate treatment for this patient includes all of the following, EXCEPT: giving a salt-containing solution by mouth. Rationale: Correct answer moving him to a cooler environment at once. Rationale: This is an appropriate treatment for heat exhaustion. A 30-year-old male, who has been playing softball all day in a hot environment, complains of weakness and nausea shortly after experiencing a syncopal episode. Appropriate treatment for this patient includes all of the following, EXCEPT: administering oxygen via nonrebreathing mask. Rationale: This is an appropriate treatment for heat exhaustion. placing him in a supine position and elevating his legs. Rationale: This is an appropriate treatment for heat exhaustion.

Depression and schizophrenia are examples of: functional disorders. altered mental status. behavioral emergencies. organic brain syndrome.

Answer: A Rationale: Unlike an organic disorder, a functional disorder cannot be linked to any physical dysfunction or failure of an organ. Depression, schizophrenia, obsessive-compulsive disorder (OCD), and bipolar disorder are examples of functional disorders. They are usually caused by a chemical imbalance in the brain—not a structural or physical abnormality. Depression and schizophrenia are examples of: functional disorders. Rationale: Correct answer altered mental status. Rationale: Altered mental status is a common presentation in patients with a wide variety of medical problems. Depression and schizophrenia are examples of: behavioral emergencies. Rationale: These are emergencies that do not have a clear physical cause and that result in aberrant behavior. organic brain syndrome. Rationale: Organic brain syndrome is a psychiatric disorder caused by a permanent or temporary physical change in the brain.

When physically restraining a violent patient, the EMT should: continually talk to the patient as he or she is being restrained. check circulation in all extremities only if the patient is prone. remove the restraints if the patient appears to be calming down. use additional force if the restrained patient begins to yell at you.

Answer: A Rationale: When physically restraining a violent patient, the EMT or his or her partner should continually talk to the patient throughout the process. Treat the patient with dignity and respect—regardless of the situation. Once restraints are placed, they should not be removed, even if the patient appears to be calm. Circulation in all extremities should be monitored, regardless of the position in which the patient is restrained. When physically restraining a violent patient, the EMT should: continually talk to the patient as he or she is being restrained. Rationale: Correct answer check circulation in all extremities only if the patient is prone. Rationale: Always check the patient's extremity circulation frequently when physical restraints are applied. When physically restraining a violent patient, the EMT should: remove the restraints if the patient appears to be calming down. Rationale: Once restraints are applied, they should not be removed. use additional force if the restrained patient begins to yell at you. Rationale: Use only the force necessary to initially restrain a patient.

The signs and symptoms of an allergic reaction are caused by the release of: histamine. epinephrine. leukotrienes. both histamine and leukotrienes.

Answer: A Response: The two chief chemicals released by the body that result in the signs and symptoms of an allergic reaction are histamines and leukotrienes. Epinephrine (adrenaline) is used to treat allergic reactions. Glucagon is a hormone secreted by the pancreas that helps to control metabolism. The signs and symptoms of an allergic reaction are caused by the release of: histamine. Rationale: Histamine is a chemical that, along with leukotrienes, is released to cause an allergic reaction. epinephrine. Rationale: Epinephrine is used to treat anaphylaxis. The signs and symptoms of an allergic reaction are caused by the release of: leukotrienes. Rationale: Leukotrienes are a chemical that is released, along with histamine, to cause an allergic reaction. both histamine and leukotrienes. Rationale: Correct answer

When assessing a conscious and alert 9-year-old child, you should: A. isolate the child from his or her parent. B. allow the child to answer your questions. C. obtain all of your information from the parent. D. avoid placing yourself below the child's eye level.

Answer: B Rationale: A 9-year-old child is capable of answering questions. By allowing a child to answer your questions, you can gain his or her trust and build a good rapport, which facilitates further assessment and treatment. Do not isolate the child from his or her parent, yet do not allow the parent to do all the talking, unless the child is unable to communicate. When assessing any patient, you should place yourself at or slightly below the patient's eye level. This position is less intimidating and helps to minimize patient anxiety. When assessing a conscious and alert 9-year-old child, you should: A. isolate the child from his or her parent. - Rationale: Do not isolate a child from his or her parents. B. allow the child to answer your questions. - Rationale: Correct answer C. obtain all of your information from the parent. - Rationale: Some information from parents is useful, but allow the child to speak. D. avoid placing yourself below the child's eye level. - Rationale: Never tower over a child, instead maintain yourself at/or below eye level.

A 44-year-old man was struck in the back of the head and was reportedly unconscious for approximately 30 seconds. He complains of a severe headache and "seeing stars," and states that he regained his memory shortly before your arrival. His presentation is MOST consistent with a(n): A. contusion. B. concussion. C. subdural hematoma. D. intracerebral hemorrhage.

Answer: B Rationale: A concussion occurs when the brain is jarred around inside the skull. It may result in a brief loss of consciousness and, occasionally, amnesia. Seeing stars is a common finding following trauma to the back of the head (occiput), as this region is primarily responsible for vision. A concussion—the least severe of all closed head injuries—typically does not result in physical damage to the brain. Compared to a concussion, a contusion, subdural hematoma, and intracerebral hemorrhage are usually associated with a more prolonged loss of consciousness. A 44-year-old man was struck in the back of the head A. and was reportedly unconscious for approximately - 30 seconds. He complains of a severe headache and "seeing stars," and states that he regained his memory shortly before your arrival. His presentation is MOST consistent with a(n): B. contusion. - Rationale: This is when brain tissue is damaged and the patient presents with prolonged confusion and loss of consciousness. C. concussion. - A 44-year-old man was struck in the back of the head D. and was reportedly unconscious for approximately - 30 seconds. He complains of a severe headache and "seeing stars," and states that he regained his memory shortly before your arrival. His presentation is MOST consistent with a(n): subdural hematoma. Rationale: This is an accumulation of blood beneath the dura mater. intracerebral hemorrhage. Rationale: This is bleeding within the brain itself.

A young male was struck in the forearm with a baseball and complains of pain to the area. Slight swelling and ecchymosis are present, but no external bleeding. Which type of injury does this describe? A. Abrasion B. Contusion C. Hematoma D. Avulsion

Answer: B Rationale: A contusion (bruise) is caused by direct blunt force trauma. The epidermis remains intact, but small blood vessels in the dermis are injured. The depth of the injury varies, depending on the amount of energy absorbed. Pain and swelling occur as fluid and blood leak into the damaged area. The buildup of blood produces a characteristic blue and black discoloration called ecchymosis. A young male was struck in the forearm with a baseball and complains of pain to the area. Slight swelling and ecchymosis are present, but no external bleeding. Which type of injury does this describe? A. Abrasion - Rationale: An abrasion is a wound of the superficial layer of skin, caused by friction. B. Contusion - Rationale: Correct answer C. Hematoma - Rationale: A hematoma is blood that has collected within damaged tissue or in a body cavity, associated with large blood vessel damage. D. Avulsion - Rationale: An avulsion is an injury that separates various layers of tissue.

What purpose does a one-way "flutter valve" serve when used on a patient with an open pneumothorax? A. It prevents air escape from within the chest cavity. B. It allows a release of air trapped in the pleural space. C. It only prevents air from entering an open chest wound. D. It allows air to freely move in and out of the chest cavity.

Answer: B Rationale: A one-way flutter valve is used to treat patients with an open pneumothorax (sucking chest wound), and serves two purposes: It allows air trapped in the pleural space to escape during exhalation, and it prevents air from entering the pleural space during inhalation. These combined effects alleviate pressure on the affected lung, which allows it to reexpand. What purpose does a one-way "flutter valve" serve when used on a patient with an open pneumothorax? A. It prevents air escape from within the chest cavity.- Rationale: It allows air to exit the chest. B. It allows a release of air trapped in the pleural space. - Rationale: Correct answer C. It only prevents air from entering an open chest wound. - Rationale: It prevents air from entering and allows air to exit the chest. D. It allows air to freely move in and out of the chest cavity.- Rationale: It allows air to move out freely and prevents air from entering.

A newborn is considered to be "term" if it is born after ____ weeks and before ____ weeks. A. 34, 37 B. 37, 42 C. 38, 44 D. 39, 43

Answer: B Rationale: A term gestation ranges between 37 and 42 weeks. An infant who is born before 37 weeks gestation (or weighs less than 5 lb, regardless of gestational age) is considered premature. An infant born after 42 weeks is considered past due. A newborn is considered to be "term" if it is born after ____ weeks and before ____ weeks. A. 34, 37 - Rationale: A newborn is considered premature if he or she is born before 37 weeks. B. 37, 42 - Rationale: Correct answer C. 38, 44 - Rationale: A newborn is considered past due if he or she is born after 42 weeks. D. 39, 43 - Rationale: A newborn is considered past due if he or she is born after 42 weeks.

The primary reason for applying a sterile dressing to an open injury is to: A. prevent contamination. B. control external bleeding. C. reduce the risk of infection. D. minimize any internal bleeding.

Answer: B Rationale: Although prevention of contamination is an important reason for applying a sterile dressing to an open injury, the primary reason is to control the external bleeding associated with it. The primary reason for applying a sterile dressing to an open injury is to: A. prevent contamination. - Rationale: This is important, but not the primary reason. B. control external bleeding. - Rationale: Correct answer C. reduce the risk of infection. - Rationale: The prevention of contamination will reduce the potential for infection. D. minimize any internal bleeding. - Rationale: Internal bleeding is minimized by the application of a pressure bandage to an open wound.

As you are assessing a 24-year-old man with a large laceration to the top of his head, you should recall that: A. the scalp, unlike other parts of the body, has relatively fewer blood vessels. B. blood loss from a scalp laceration may contribute to hypovolemic shock in adults. C. any avulsed portions of the scalp should be carefully cut away to facilitate bandaging. D. most scalp injuries are superficial and are rarely associated with more serious injuries.

Answer: B Rationale: Although the scalp is highly vascular and tends to bleed heavily when injured, scalp injuries are rarely the sole cause of hypovolemic shock in adults. However, they can contribute to hypovolemia caused by injuries elsewhere in the body. Scalp lacerations, whether deep or superficial, should prompt you to look for more serious underlying injuries, such as a skull fracture. If the injury involves an avulsion, the avulsed flap of skin should be carefully replaced to its original position, not cut away. As you are assessing a 24-year-old man with a large laceration to the top of his head, you should recall that: A. the scalp, unlike other parts of the body, has relatively fewer blood vessels. - Rationale: The scalp is highly vascular. B. blood loss from a scalp laceration may contribute to hypovolemic shock in adults. - As you are assessing a 24-year-old man with a large laceration to the top of his head, you should recall that: C. cny avulsed portions of the scalp should be carefully cut away to facilitate bandaging. - Rationale: The avulsed flap should be carefully replaced to its original position. D. most scalp injuries are superficial and are rarely associated with more serious injuries. - Rationale: Deep or superficial scalp lacerations should prompt EMS providers to assess for more serious underlying injuries.

The term "hematuria" is defined as: A. blood in the stool. B. blood in the urine. C. vomiting up blood. D. urinary bladder rupture.

Answer: B Rationale: Blood in the urine is called hematuria. Following trauma, the presence of hematuria suggests injury to the urinary bladder or kidneys. Bright red blood in the stool is called hematochezia; dark, tarry stools are called melena. Vomiting up blood is called hematemesis. The term "hematuria" is defined as: A. blood in the stool.- Rationale: This is known as hematochezia or melena (dark stools). B. blood in the urine. - Rationale: Correct answer C. vomiting up blood.- Rationale: This is known as hematemesis. D. urinary bladder rupture. - Rationale: This will produce abdominal pain and eventually peritonitis due to a leaking into the abdominal cavity. It will also cause a lack of or a reduction of urinary output.

A young male experienced severe blunt chest trauma when his passenger car struck another vehicle head-on. During your inspection of the interior of his vehicle, you would MOST likely find: A. deployed airbags. B. steering wheel deformity. C. starring of the windshield. D. a crushed instrument panel.

Answer: B Rationale: Blunt chest injuries during a motor vehicle crash typically occur when the chest impacts the steering wheel. Therefore, your inspection of the vehicle's interior will most likely reveal a deformed steering wheel. A young male experienced severe blunt chest trauma when his passenger car struck another vehicle head-on. During your inspection of the interior of his vehicle, you would MOST likely find: A. deployed airbags. - Rationale: Typically, this will cause abrasions to the face, head, and arms. B. steering wheel deformity. - Rationale: Correct answer C. starring of the windshield. - Rationale: Typically, this indicates the presence of head, face, and neck injuries. D. a crushed instrument panel. - Rationale: Typically, this indicates the presence of leg and hip injuries.

Which of the following is NOT a component of the cardiovascular system? A. Heart B. Lungs C. Venules D. Plasma

Answer: B Rationale: Components of the cardiovascular system include the heart, blood vessels (arteries, arterioles, capillaries, venules, and veins), and blood (plasma and blood cells). The lungs are a component of the respiratory system. Which of the following is NOT a component of the cardiovascular system? A. Heart - Rationale: This is part of the cardiovascular system. B. Lungs- Rationale: Correct answer C. Venules- Rationale: This is part of the cardiovascular system. D. Plasma - Rationale: This is part of the cardiovascular system.

Controlling internal bleeding requires: A. applying a tourniquet. B. surgery in a hospital. C. positioning the patient in the sitting position. D. providing slow and considerate transport.

Answer: B Rationale: Controlling internal bleeding usually requires surgery that must be done in the hospital. To care for the patient in the field, administer high-flow oxygen and assist ventilations, if needed; control all obvious external bleeding; monitor and record the vital signs every 5 minutes; place the nontrauma patient in a shock position; keep the patient warm; and provide immediate transport. Controlling internal bleeding requires: A. applying a tourniquet. - Rationale: Never use a tourniquet to control bleeding from closed, internal, soft-tissue injuries. B. surgery in a hospital.- Rationale: Correct answer C. positioning the patient in the sitting position.- Rationale: You should place the patient in a supine position. D. providing slow and considerate transport. - Rationale: You should provide immediate transport.

What is the EMT's FIRST priority when dealing with a patient experiencing excessive vaginal bleeding? A. Determine the cause of the bleeding. B. Treat the patient for shock and transport. C. Determine if the bleeding is a result of sexual assault. D. Keep the patient warm and apply oxygen.

Answer: B Rationale: Determining the cause of the bleeding is less important than treating for shock and transporting the patient. EMTs can control the bleeding by using sanitary pads on the external genitalia. When treating for shock, the EMT must place the patient in the appropriate position, keep her warm, and apply oxygen. What is the EMT's FIRST priority when dealing with a patient experiencing excessive vaginal bleeding? A. Determine the cause of the bleeding. - Rationale: Determining the cause of the bleeding is less important than treating for shock and transporting the patient. B. Treat the patient for shock and transport. - Rationale: Correct answer C. Determine if the bleeding is a result of sexual assault. - Rationale: This information will be handled by the hospital staff and police. D. Keep the patient warm and apply oxygen. - Rationale: This step is only part of treating the patient for shock.

The quickest and MOST effective way to control external bleeding from an extremity is: A. a pressure bandage. B. direct pressure. C. a splint. D. a tourniquet.

Answer: B Rationale: Direct pressure is the quickest, most effective way to control external bleeding from an extremity. This will effectively control external bleeding in most cases. The quickest and MOST effective way to control external bleeding from an extremity is: A. a pressure bandage. - Rationale: This is done after direct pressure has controlled the bleeding. B. direct pressure.- Rationale: Correct answer C. a splint.- Rationale: Most cases of external bleeding can be controlled by direct pressure and elevation and do not require a splint. D. a tourniquet. - Rationale: This is the last method of controlling external bleeding.

Treatment for a semiconscious child who swallowed an unknown quantity of pills includes: A. administering 1 g/kg of activated charcoal and rapidly transporting. B. monitoring the child for vomiting, administering oxygen, and transporting. C. positioning the child on his left side, elevating his legs 6 inches, and transporting. D. contacting medical control and requesting permission to induce vomiting.

Answer: B Rationale: If a semi- or unconscious child has ingested pills, poisons, or any other type of harmful substance, closely observe for vomiting, give high-flow oxygen (assist ventilations if necessary), and rapidly transport to the emergency department. Do not give activated charcoal to any patient who is not conscious and alert enough to swallow. Induction of vomiting is not indicated for anyone—regardless of age. Treatment for a semiconscious child who swallowed an unknown quantity of pills includes: A. administering 1 g/kg of activated charcoal and rapidly transporting. - Rationale: Do not give anything by mouth to an individual who is not conscious and alert enough to swallow. B. monitoring the child for vomiting, administering oxygen, and transporting. - Rationale: Correct answer C. positioning the child on his left side, elevating his legs 6 inches, and transporting. - Rationale: Placing the child in the recovery position is acceptable if vomiting is possible, but the patient's legs should remain flat. D. contacting medical control and requesting permission to induce vomiting. - Rationale: Inducing vomiting is not indicated for anyone at any age.

Upon delivery of the baby's head, you note that the umbilical cord is wrapped around its neck. You should: A. immediately clamp and cut the cord. B. make one attempt to slide the cord over the head. C. keep the cord moist and transport as soon as possible. D. give the mother high-flow oxygen and transport rapidly.

Answer: B Rationale: If the umbilical cord is wrapped around the baby's neck (nuchal cord), you should make one attempt to gently remove the cord from around the baby's neck. If this is not possible, the cord should be clamped and cut. Keep the cord moist, administer high-flow oxygen to the mother, and transport at once. Upon delivery of the baby's head, you note that the umbilical cord is wrapped around its neck. You should: A. immediately clamp and cut the cord. - Rationale: Do this only after an attempt is made to slide the cord over the infant's head. B. make one attempt to slide the cord over the head. - Rationale: Correct answer C. keep the cord moist and transport as soon as possible. - Rationale: This is the treatment for deliveries where the cord presents and not the infant's head. D. give the mother high-flow oxygen and transport rapidly. - Rationale: Do this only after an attempt to slide the cord over the infant's head.

When applying a tourniquet to an amputated arm, the EMT should: A. use the narrowest bandage possible. B. avoid applying the tourniquet over a joint. C. cover the tourniquet with a sterile bandage. D. use rope to ensure that the tourniquet is tight.

Answer: B Rationale: If you must apply a tourniquet, never apply it directly over a joint. You should use the widest bandage possible and make sure it is secured tightly. Never use wire, rope, a belt, or any other narrow material, as it could cut the skin. The tourniquet should never be covered with a bandage; leave it open and in full view. When applying a tourniquet to an amputated arm, the EMT should: A. use the narrowest bandage possible. - Rationale: You should use the widest bandage possible. B. avoid applying the tourniquet over a joint.- Rationale: Correct answer C. cover the tourniquet with a sterile bandage.- Rationale: You should leave a tourniquet open and in plain view. D. use rope to ensure that the tourniquet is tight. - Rationale: Never use a wire, rope, belt, or any other narrow material, as it may cut or damage the extremity.

Kinetic energy is a calculation of: A. weight and size. B. weight and speed. C. mass and weight. D. speed and force.

Answer: B Rationale: Kinetic energy is a calculation of mass (weight) and velocity (speed). Energy cannot be destroyed, only converted. Kinetic energy is a calculation of: A. weight and size. - Rationale: Weight is part of the formula, but size would also mean weight. B. weight and speed. - Rationale: Correct answer C. mass and weight. - Rationale: Mass and weight are the same. D. speed and force. - Rationale: Force is the product of mass times acceleration, all part of Newton's second law.

When caring for a patient with severe epistaxis, the MOST effective way to prevent aspiration of blood is to: A. insert a nasopharyngeal airway and lean the patient back. B. tilt the patient's head forward while he or she is leaning forward. C. place the patient supine with his or her head in the flexed position. D. tilt the patient's head forward while he or she is leaning backward.

Answer: B Rationale: Leaning forward, with the head tilted forward, will stop blood from trickling down the throat. This decreases the risk that the patient will swallow the blood, which may cause vomiting, or aspirating the blood into the lungs. When caring for a patient with severe epistaxis, the MOST effective way to prevent aspiration of blood is to: A. insert a nasopharyngeal airway and lean the patient back. - Rationale: Never insert a nasopharyngeal airway into actively bleeding nares. B. tilt the patient's head forward while he or she is leaning forward.- Rationale: Correct answer C. place the patient supine with his or her head in the flexed position.- Rationale: Lying a patient supine with epistaxis will cause blood to be swallowed and may cause vomiting. D. tilt the patient's head forward while he or she is leaning backward. - Rationale: Tilt the patient's head forward, but the patient's body must also lean forward.

During immobilization of a patient with a possible spinal injury, manual stabilization of the head must be maintained until: A. an appropriate-size extrication collar has been placed. B. the patient is fully immobilized on a long backboard. C. a range of motion test of the neck has been completed. D. pulse, motor, and sensory functions are found to be intact.

Answer: B Rationale: Manual stabilization of the patient's head must be maintained until he or she is fully secured to the long backboard. This includes the application of an extrication collar, straps, and lateral immobilization (head blocks). Pulse, motor, and sensory functions must be checked before and after the immobilization process. Do not assess range of motion in a patient with a possible spinal injury; this involves moving the patient's neck and may cause further injury. During immobilization of a patient with a possible spinal injury, manual stabilization of the head must be maintained until: A. an appropriate-size extrication collar has been placed. - Rationale: This is only one small part of the total immobilization process. B. the patient is fully immobilized on a long backboard. - During immobilization of a patient with a possible spinal injury, manual stabilization of the head must be maintained until: C. c range of motion test of the neck has been completed. - Rationale: Do not assess the range of motion in a patient with a possible spinal injury. D. pulse, motor, and sensory functions are found to be intact. - Rationale: This is done before and after complete immobilization.

The onset of menstruation is called: A. menopause. B. menarche. C. ovulation. D. bleeding.

Answer: B Rationale: Menarche is the onset of menstruation, typically occurring between the ages of 11 and 16 years. The onset of menstruation is called: A. menopause. - Rationale: Menopause is when the cycle of ovulation and menstruation cease. B. menarche. - Rationale: Correct answer C. ovulation. - Rationale: Ovulation occurs each month when one ovum is released into the fallopian tube. D. premenstrual syndrome - Rationale: Premenstrual syndrome involves symptoms that typically occur before menstruation.

When caring for a patient with facial trauma, the EMT should be MOST concerned with: A. spinal trauma. B. airway compromise. C. associated eye injuries. D. severe external bleeding.

Answer: B Rationale: No airway, no patient! Injuries to the face often cause obstruction of the upper airway, either by clotted blood or associated swelling. Additionally, large amounts of blood can be swallowed, which increases the risks of vomiting and aspiration. Bleeding control, spinal trauma, and associated injuries are important factors and should be treated accordingly; however, the airway comes first. When caring for a patient with facial trauma, the EMT should be MOST concerned with: A. spinal trauma. - Rationale: This is a concern to be treated, but treating the airway is first. B. airway compromise. - Rationale: Correct answer C. associated eye injuries. - Rationale: This is a concern to be treated, but treating the airway is first. D. severe external bleeding. - Rationale: This is a concern to be treated, but treating the airway is first.

You are transporting a patient with possible peritonitis following trauma to the abdomen. Which position will he MOST likely prefer to assume? A. Sitting up B. Legs drawn up C. Legs outstretched D. On his right side

Answer: B Rationale: Patients with peritonitis often lie very still and tend to have their legs drawn up into the abdomen. This relieves strain on the abdominal muscles and may provide pain relief. A. You are transporting a patient with possible peritonitis following trauma to the abdomen. Which position will he MOST likely prefer to assume?- Sitting up B. Rationale: Lying very still with the legs drawn up will help relieve the patient's pain. - Legs drawn up C. Rationale: Correct answer- Legs outstretched D. Rationale: The patient's legs drawn up or flexed will help relieve pain. - On his right side Rationale: The patient's legs drawn up or flexed will help relieve pain.

A 23-year-old woman, who is 24 weeks pregnant with her first baby, complains of edema to her hands, a headache, and visual disturbances. When you assess her vital signs, you note that her blood pressure is 160/94 mm Hg. She is MOST likely experiencing: A. eclampsia. B. preeclampsia. C. a hypertensive crisis. D. chronic water retention.

Answer: B Rationale: Preeclampsia—also called pregnancy-induced hypertension—usually develops after the 20th week of gestation and most commonly affects primagravida (first pregnancy) patients. It is characterized by a headache, visual disturbances, edema of the hands and feet, anxiety, and high blood pressure. Preeclampsia can lead to eclampsia, a life-threatening condition that is characterized by seizures. A 23-year-old woman, who is 24 weeks pregnant with her first baby, complains of edema to her hands, a headache, and visual disturbances. When you assess her vital signs, you note that her blood pressure is 160/94 mm Hg. She is MOST likely experiencing: A. eclampsia. - Rationale: Eclampsia is a seizure that results from severe hypertension. B. preeclampsia. - Rationale: Correct answer C. a hypertensive crisis. - Rationale: This is a severe, sudden increase in blood pressure, typically greater than 110 diastolic, that can lead to a stroke. D. chronic water retention. - Rationale: This is a fluid imbalance usually caused by too much sodium in the body.

When treating an open extremity fracture, you should: A. apply a splint and then dress the wound. B. dress the wound before applying a splint. C. irrigate the wound before applying a dressing. D. allow the material that secures the splint to serve as the dressing.

Answer: B Rationale: Prior to splinting an open extremity fracture, you should cover the wound with a dry, sterile dressing. This will help control any bleeding and decreases the risk of infection. Irrigating an open fracture should be avoided in the field; this also increases the risk of infection—especially if foreign material is flushed into the wound. When treating an open extremity fracture, you should: A. apply a splint and then dress the wound. - Rationale: The dressing must come before the splint. B. dress the wound before applying a splint. - Rationale: Correct answer C. irrigate the wound before applying a dressing.- Rationale: Irrigation of an open fracture in the prehospital setting may increase the chance of infection. D. allow the material that secures the splint to serve as the dressing. - Rationale: The wound must be dressed separately from the splint and before splinting is done.

Signs and symptoms of a chest injury include all of the following, EXCEPT: A. hemoptysis. B. hematemesis. C. asymmetrical chest movement. D. increased pain with breathing.

Answer: B Rationale: Signs and symptoms of a chest injury include bruising to the chest, chest wall instability, increased pain with breathing, asymmetrical (unequal) chest movement if a pneumothorax is present, and hemoptysis (coughing up blood) if intrapulmonary bleeding is occurring. Hematemesis (vomiting blood) indicates bleeding in the gastrointestinal tract—usually the esophagus or stomach—not the chest cavity. Signs and symptoms of a chest injury include all of the following, EXCEPT: A. hemoptysis. - Rationale: Hemoptysis is coughing up blood or blood-tinged sputum. B. hematemesis. - Rationale: Correct answer C. asymmetrical chest movement. - Rationale: This may indicate a flail chest or pneumothorax. D. increased pain with breathing. - Rationale: A chest injury will cause the presence of pain during inspiratory or expiratory chest wall movement.

Skeletal muscle is also referred to as: A. smooth muscle. B. striated muscle. C. autonomic muscle. D. involuntary muscle.

Answer: B Rationale: Skeletal muscle, also called striated muscle because of its characteristic stripes (striations), attaches to the bones and usually crosses at least one joint, forming the major muscle mass of the body. This type of muscle is also called voluntary muscle because it is under direct voluntary control of the brain. Skeletal muscle is also referred to as: A. smooth muscle. - Rationale: This is found in the walls of most tubular structures, internal organs, and the cardiovascular system. B. striated muscle. - Rationale: Correct answer C. autonomic muscle.- Rationale: This is not a term associated with the muscles. D. involuntary muscle. - Rationale: Involuntary muscle is also called smooth muscle. It is not under voluntary control of the brain.

What is the narrowest portion of the uterus? A. Vagina B. Cervix C. Fallopian tubes D. Ovaries

Answer: B Rationale: The ovaries are the primary female reproductive organ. The developing embryo travels into the uterus through the fallopian tube. The embryo attaches to the uterine wall and continues to grow. The narrowest portion of the uterus is the cervix, which opens into the vagina. What is the narrowest portion of the uterus? A. Vagina - Rationale: The vagina is the outermost cavity of the woman's reproductive system. B. Cervix - Rationale: Correct answer C. Fallopian tube - Rationale: The fallopian tubes are not part of the uterus. They connect each ovary with the uterus. D. Ovaries - Rationale: The ovaries are located on each side of the abdomen and are not part of the uterus.

A patient injured her knee while riding a bicycle. She is lying on the ground, has her left leg flexed, is in severe pain, and cannot move her leg. Your assessment reveals obvious deformity to her left knee. Distal pulses are present and strong. The MOST appropriate treatment for her injury involves: A. wrapping her entire knee area with a pillow. B. splinting the leg in the position in which it was found. C. straightening her leg and applying two rigid board splints. D. straightening her leg and applying and inflating an air splint.

Answer: B Rationale: The patient likely has a dislocated knee. You should immobilize any joint injury in the position in which it was found—especially if distal pulses are present and strong. Attempting to straighten a dislocated joint may cause damage to the nerves and/or vasculature. A patient injured her knee while riding a bicycle. She is lying on the ground, has her left leg flexed, is in severe pain, and cannot move her leg. Your assessment reveals obvious deformity to her left knee. Distal pulses are present and strong. The MOST appropriate treatment for her injury involves: A. wrapping her entire knee area with a pillow. - Rationale: Providers can wrap the knee with a pillow to splint it. It is most important to splint the joint in the position it was found. B. splinting the leg in the position in which it was found. - Rationale: Correct answer C. straightening her leg and applying two rigid board splints.- Rationale: The straightening of a joint injury is contraindicated if the pulses are intact. D. straightening her leg and applying and inflating an air splint. - Rationale: Air splints are not effective on joint injuries that are flexed.

Which of the following signs is least indicative of a head injury? A. Asymmetrical pupils B. Pupillary constriction to bright light C. Both eyes moving in opposite directions D. Inability to look upward when instructed to

Answer: B Rationale: The pupils normally constrict in bright light and dilate in dim light. Suspect a head injury if the pupils do not react appropriately, are asymmetrical (unequal), do not move together, or if the patient is unable to look upward. Which of the following signs is LEAST indicative of a head injury? A. Asymmetrical pupils - Rationale: This may be an indication of a head injury. B. Pupillary constriction to bright light - Rationale: Correct answer C. Both eyes moving in opposite directions - Rationale: This may be an indication of a head injury. D. Inability to look upward when instructed to - Rationale: This may be an indication of a head injury.

When a person is looking at an object up close, the pupils should: A. dilate. B. constrict. C. remain the same size. D. dilate, and then constrict.

Answer: B Rationale: The pupils, which allow light to move to the back of the eye, constrict in bright light and dilate in dim light. The pupils should also constrict when looking at an object up close and dilate when looking at an object farther away; this is called pupillary accommodation. These pupillary adjustments occur almost instantaneously. When a person is looking at an object up close, the pupils should: A. dilate. - Rationale: The pupils will dilate when looking at objects far away. B. constrict. - Rationale: Correct answer C. remain the same size. - Rationale: The pupils will constrict when looking at objects that are close. D. dilate, and then constrict. - Rationale: The pupils will constrict first when looking at close objects.

The purpose of splinting a fracture is to: A. reduce the fracture if possible. B. prevent motion of bony fragments. C. reduce swelling in adjacent soft tissues. D. force the bony fragments back into anatomic alignment.

Answer: B Rationale: The purpose of splinting a fracture is to prevent motion of the bony fragments, thus minimizing the possibility of neurovascular damage. Splinting is not intended to force bony fragments into anatomic alignment, nor will it reduce swelling (ice reduces swelling). You should never try to reduce a fracture. The purpose of splinting a fracture is to: A. reduce the fracture if possible. - Rationale: Reduction of a suspected fracture is a medical procedure to be performed in the hospital. B. prevent motion of bony fragments. - Rationale: Correct answer C. reduce swelling in adjacent soft tissues.- Rationale: Splinting will not reduce swelling, but cold application will. D. force the bony fragments back into anatomic alignment. - Rationale: Splinting to immobilize a fracture site is not intended to force bony fragments back into alignment.

A patient experienced a severe compression to the chest when trapped between a vehicle and a brick wall. You suspect traumatic asphyxia due to the hemorrhage into the sclera of his eyes and which other sign? A. Flat neck veins B. Cyanosis in the face and neck C. Asymmetrical chest movement D. Irregular heart rate

Answer: B Rationale: The sudden increase in intrathoracic pressure results in a characteristic appearance, including distended neck veins, and hemorrhage into the sclera of the eyes, signaling the bursting of small blood vessels. A patient experienced a severe compression to the chest when trapped between a vehicle and a brick wall. You suspect traumatic asphyxia due to the hemorrhage into the sclera of his eyes and which other sign? A. Flat neck veins- Rationale: The neck veins would be distended. Flat neck veins may indicate a hemothorax. B. Cyanosis in the face and neck - Rationale: Correct answer C. Asymmetrical chest movement - Rationale: This is seen with a flail segment. D. Irregular heart rate- Rationale: This may be seen with a myocardial contusion.

A 5-year-old boy was burned when he pulled a barbecue grill over on himself. He has partial- and full-thickness burns to his anterior chest and circumferentially on both arms. What percentage of his body surface area has been burned? A. 18% B. 27% C. 36% D. 45%

Answer: B Rationale: Using the pediatric rules of nines, the anterior chest accounts for 9% of the BSA (the entire anterior trunk, which includes the chest and abdomen, account for 18% of the BSA), and each arm accounts for 9% of the BSA. Therefore, this child has experienced 27% BSA burns. A 5-year-old boy was burned when he pulled a barbecue grill over on himself. He has partial- and full-thickness burns to his anterior chest and circumferentially on both arms. What percentage of his body surface area has been burned? A. 18% - Rationale: 18% would indicate the patient's arms only. B. 27% - Rationale: Correct answer C. 36% - Rationale: The patient's chest is 9% and both arms are 18%. D. 45% - Rationale: The patient's chest is 9% and both arms are 18%.

A patient tripped, fell, and landed on her elbow. She is in severe pain and has obvious deformity to her elbow. You should: A. assess distal pulses. B. manually stabilize her injury. C. assess her elbow for crepitus. D. apply rigid board splints to her arm.

Answer: B Rationale: When caring for a patient with an orthopaedic injury, you should first manually stabilize the injury site; this will prevent further injury. You should then assess pulse, motor functions, and sensory functions distal to the injury. Splint the injury using the appropriate technique, and then reassess pulse, motor functions, and sensory functions. Do not intentionally assess for crepitus; this is a coincidental finding that you may encounter during your assessment and should not be elicited. A patient tripped, fell, and landed on her elbow. She is in severe pain and has obvious deformity to her elbow. You should: A. assess distal pulses. - Rationale: This is completed after the manual stabilization of the injury. B. manually stabilize her injury. - Rationale: Correct answer C. assess her elbow for crepitus.- Rationale: Do not intentionally assess for crepitus; this is a coincidental finding. D. apply rigid board splints to her arm. - Rationale: This is completed after manual stabilization of the injury.

When the chest impacts the steering wheel during a motor vehicle crash with rapid deceleration, the resulting injury, which often kills patients, usually within seconds, is: A. a hemothorax. B. aortic shearing. C. a pneumothorax. D. a ruptured myocardium.

Answer: B Rationale: When the chest impacts the steering wheel following rapid forward deceleration, aortic injuries (shearing or rupture) are the cause of death in nearly two thirds of patients. The aorta is the largest artery in the body; when it is sheared from its supporting structures or ruptures outright, exsanguination (bleeding to death) occurs—usually within a matter of seconds. When the chest impacts the steering wheel during a motor vehicle crash with rapid deceleration, the resulting injury, which often kills patients, usually within seconds, is: A. a hemothorax. - Rationale: This is a serious injury, but is not fatal in seconds. B. aortic shearing. - Rationale: Correct answer C. a pneumothorax. - Rationale: This is a serious injury, but is not fatal in seconds. D. a ruptured myocardium. - Rationale: This is a serious injury, but not common.

A behavioral crisis is MOST accurately defined as: a severe, acute psychiatric condition in which the patient becomes violent and presents a safety threat to self or to others. any reaction to events that interferes with activities of daily living or has become unacceptable to the patient, family, or community. A behavioral crisis is MOST accurately defined as: a normal response of a patient to a situation that causes an overwhelming amount of stress, such as the loss of a job or marital problems. a reaction to a stressful event that the patient feels is appropriate, but is considered inappropriate by the patient's family or the community.

Answer: B Rationale: A behavioral crisis is any reaction to events that interferes with the patient's activities of daily living or has become acceptable to the patient, his or her family, or the community. Not all patients with an emotional crisis are "psychotic," nor are all violent patients experiencing a psychiatric condition; these are common misconceptions. Various medical conditions can cause a behavioral crisis (eg, hypoglycemia, hypoxemia, brain tumors). A behavioral crisis is MOST accurately defined as: a severe, acute psychiatric condition in which the patient becomes violent and presents a safety threat to self or to others. Rationale: This could be considered a symptom of a mental disorder. any reaction to events that interferes with activities of daily living or has become unacceptable to the patient, family, or community. Rationale: Correct answer A behavioral crisis is MOST accurately defined as: a normal response of a patient to a situation that causes an overwhelming amount of stress, such as the loss of a job or marital problems. Rationale: This could be normal behavior or could progress to depression. a reaction to a stressful event that the patient feels is appropriate, but is considered inappropriate by the patient's family or the community. Rationale: This could be normal behavior.

Most poisonings occur via the _________ route. injection ingestion inhalation absorption

Answer: B Rationale: Approximately 80% of all poisonings occur by ingestion. Most poisonings occur via the _________ route. injection Rationale: Less than 20% of poisonings occur via injection. ingestion ;answer Most poisonings occur via the _________ route. inhalation Rationale: Less than 20% of poisonings occur via inhalation. absorption Rationale: Less than 20% of poisonings occur via absorption.

Which of the following is a developmental disorder characterized by impairment of social interaction? Down syndrome Autism Cerebral palsy Spina bifida

Answer: B Rationale: Autism is a developmental disability characterized by impairment of social interaction. Cerebral palsy and spina bifida are physical disabilities. Down syndrome is characterized by a genetic chromosomal defect. Which of the following is a development disorder characterized by impairment of social interaction? Down syndrome Rationale: Down syndrome is characterized by a genetic chromosomal defect. Autism Rationale: Correct answer Which of the following is a development disorder characterized by impairment of social interaction? Cerebral palsy Rationale: Cerebral palsy is a physical disability. Spina bifida Rationale: Spina bifida is a physical disability.

You are treating a woman who was stung numerous times by hornets. On assessment, you note that some of the stingers are still embedded in her skin. You should: leave the stingers in place. scrape the stingers from her skin. pull the stingers out with tweezers. cover the stings with tight dressings.

Answer: B Rationale: Because of the venom left in the sac located at the end of the stinger, you should not grab the stingers in an attempt to remove them. Instead, scrape them off with a rigid object such as a credit card. You are treating a woman who was stung numerous times by hornets. On assessment, you note that some of the stingers are still embedded in her skin. You should: leave the stingers in place. Rationale: A stinger will continue to inject venom even when the stinger is no longer attached to the insect. scrape the stingers from her skin. Rationale: Correct answer You are treating a woman who was stung numerous times by hornets. On assessment, you note that some of the stingers are still embedded in her skin. You should: pull the stingers out with tweezers. Rationale: Using tweezers may squeeze more venom into the patient. cover the stings with tight dressings. Rationale: Remove the stingers—do not leave them in place.

When a person is exposed to cold temperatures and strong winds for an extended period of time, he or she will lose heat mostly by: radiation. convection. conduction. evaporation.

Answer: B Rationale: Convection occurs when heat is transferred to circulating air, as when cool air moves across the body surface. A person wearing lightweight clothing and standing outside in cold, windy, weather is losing heat to the environment mostly by convection. When a person is exposed to cold temperatures and strong winds for an extended period of time, he or she will lose heat mostly by: radiation. Rationale: Radiation is the transfer of heat by radiant energy. convection. Rationale: Correct answer When a person is exposed to cold temperatures and strong winds for an extended period of time, he or she will lose heat mostly by: conduction. Rationale: Conduction is the direct transfer of heat by contact. evaporation. Rationale: Body moisture evaporates and cools the body.

The MOST important reason for promptly transporting a stroke patient to the hospital is because: a transient ischemic attack can be ruled out. medications may be given to reverse the stroke. the clot in the coronary artery may be dissolved. he or she needs close blood pressure monitoring.

Answer: B Rationale: Fibrinolytic medications (clot busters) have been shown to reverse the symptoms of a stroke by dissolving the clot that is blocking the cerebral artery. However, for the patient to be eligible for this therapy, it must be initiated within 3 hours after the onset of symptoms. For this reason, prompt transport of the stroke patient is critical. The MOST important reason for promptly transporting a stroke patient to the hospital is because: a transient ischemic attack can be ruled out. Rationale: In a TIA, stroke symptoms resolve (on their own) usually in less than 24 hours with no neurologic deficits. medications may be given to reverse the stroke. Rationale: Correct answer The MOST important reason for promptly transporting a stroke patient to the hospital is because: the clot in the coronary artery may be dissolved. Rationale: The coronary artery is in the heart and not the brain. he or she needs close blood pressure monitoring. Rationale: Monitoring of a patient's blood pressure is important, but dissolving the clot and stopping the progression of damage is more important.

A 19-year-old diabetic male was found unresponsive on the couch by his roommate. After confirming that the patient is unresponsive, you should: suction his oropharynx. manually open his airway. administer high-flow oxygen. begin assisting his ventilations.

Answer: B Rationale: Immediately after determining that a patient is unresponsive, your first action should be to manually open his or her airway (eg, head tilt-chin lift, jaw-thrust). Use suction as needed to clear secretions from the patient's mouth. After manually opening the airway and ensuring it is clear of obstructions, insert an nasal airway adjunct and then assess the patient's breathing. A 19-year-old diabetic male was found unresponsive on the couch by his roommate. After confirming that the patient is unresponsive, you should: suction his oropharynx. Rationale: After opening the airway, suction as needed to remove any secretions. manually open his airway. Rationale: Correct answer A 19-year-old diabetic male was found unresponsive on the couch by his roommate. After confirming that the patient is unresponsive, you should: administer high-flow oxygen. Rationale: After opening the airway, provide oxygen only as clinically indicated. begin assisting his ventilations. Rationale: After opening the airway, assist with ventilations if the patient's breathing is inadequate.

A 21-year-old male was found unconscious in an alley. Your initial assessment reveals that his respirations are slow and shallow, and his pulse is slow and weak. Further assessment reveals that his pupils are bilaterally constricted. His presentation is MOST consistent with an overdose of: cocaine. an opioid. a stimulant drug. methamphetamine.

Answer: B Rationale: Opioids are central nervous system depressant drugs; when taken in excess, they cause respiratory depression, bradycardia, and hypotension. Another common sign is miosis (constricted [pinpoint]) pupils. Cocaine, stimulant drugs (uppers), and methamphetamine have the opposite effect; they stimulate the central nervous system and cause tachycardia and hypertension. A 21-year-old male was found unconscious in an alley. Your initial assessment reveals that his respirations are slow and shallow, and his pulse is slow and weak. Further assessment reveals that his pupils are bilaterally constricted. His presen-tation is MOST consistent with an overdose of: cocaine. Rationale: This increases the vitals, heart rate, pulse, and breathing. an opioid. ;answer A 21-year-old male was found unconscious in an alley. Your initial assessment reveals that his respirations are slow and shallow, and his pulse is slow and weak. Further assessment reveals that his pupils are bilaterally constricted. His presen-tation is MOST consistent with an overdose of: a stimulant drug. Rationale: This increases the vitals, heart rate, pulse, and breathing. methamphetamine. Rationale: This increases the vitals, heart rate, pulse, and breathing.

Which of the following conditions makes the elderly patient prone to fractures from even minor trauma? Hypertension Osteoporosis Arteriosclerosis Rheumatoid arthritis

Answer: B Rationale: Osteoporosis, a decrease in bone density that causes the bones to become brittle, makes elderly patients prone to fractures, even from minor trauma. It is especially common in postmenopausal women. Which of the following conditions makes the elderly patient prone to fractures from even minor trauma? Hypertension Rationale: This is high blood pressure. ;Rationale: Correct answer Which of the following conditions makes the elderly patient prone to fractures from even minor trauma? Arteriosclerosis Rationale: This is the stiffening or hardening of the arteries. Rheumatoid arthritis Rationale: This is an inflammatory disorder that affects the entire body and leads to degeneration and deformation of joints.

All of the following are examples of passive rewarming techniques, EXCEPT: removing cold, wet clothing. administering warm fluids by mouth. turning up the heat inside the ambulance. covering the patient with warm blankets.

Answer: B Rationale: Passive rewarming involves allowing the patient's body temperature to rise gradually and naturally. Removing cold, wet clothing; turning up the heat in the ambulance; and covering the patient with warm blankets are examples of passive rewarming. Administering warmed fluids by mouth or intravenously is an example of active rewarming; this should be avoided in the uncontrolled prehospital setting. All of the following are examples of passive rewarming techniques, EXCEPT: removing cold, wet clothing. Rationale: This is passive rewarming. administering warm fluids by mouth. Rationale: Correct answer turning up the heat inside the ambulance. Rationale: This is passive rewarming. covering the patient with warm blankets. Rationale: This is passive rewarming.

Unlike an ischemic stroke, a transient ischemic attack is characterized by all of the following, EXCEPT: symptoms that resolve within 24 hours. symptoms that persist for longer than 24 hours. weakness or paralysis to one side of the body. an acute onset of confusion and slurred speech.

Answer: B Rationale: Signs and symptoms of a transient ischemic attack (TIA) are usually identical to those of an acute ischemic stroke (eg, hemiparesis, slurred speech, confusion, facial droop). Unlike the ischemic stroke, however, the symptoms of a TIA usually resolve within 24 hours. Unlike an ischemic stroke, a transient ischemic attack is characterized by all of the following EXCEPT: symptoms that resolve within 24 hours. Rationale: This is a sign of a transient ischemic attack. symptoms that persist for longer than 24 hours. Rationale: Correct answer Unlike an ischemic stroke, a transient ischemic attack is characterized by all of the following EXCEPT: weakness or paralysis to one side of the body. Rationale: This is a sign of a transient ischemic attack. an acute onset of confusion and slurred speech. Rationale: This is a sign of a transient ischemic attack.

Which of the following signs or symptoms would you be the LEAST likely to find in a patient with an acute abdomen? Rapid, shallow breathing Soft, nondistended abdomen Tachycardia and restlessness Constipation or diarrhea

Answer: B Rationale: Signs and symptoms of an acute abdomen include, but are not limited to, rapid and shallow breathing, a tense and distended abdomen, tachycardia, restlessness, and constipation or diarrhea. Which of the following signs or symptoms would you be the LEAST likely to find in a patient with an acute abdomen? Rapid, shallow breathing Rationale: This is a common sign of an acute abdomen. Soft, nondistended abdomen Rationale: Correct answer Which of the following signs or symptoms would you be the LEAST likely to find in a patient with an acute abdomen? Tachycardia and restlessness Rationale: These are common signs of an acute abdomen. Constipation or diarrhea Rationale: These are common signs of an acute abdomen.

The EMTALA act states that: patients should only be treated if they can pay for care. all patients must be treated regardless of their ability to pay for care. only those with serious injuries can be treated without payment for care. only certain facilities can treat patients who cannot pay for care.

Answer: B Rationale: The Emergency Medical Treatment and Active Labor Act (EMTALA) requires all facilities to assess and treat patients regardless of their ability to pay for care. The EMTALA act states that: patients should only be treated if they can pay for care. Rationale: This is not the correct answer. all patients must be treated regardless of their ability to pay for care. Rationale: Correct answer The EMTALA act states that: only those with serious injuries can be treated without payment for care. Rationale: This is not the correct answer. only certain facilities can treat patients who cannot pay for care. Rationale: This is not the correct answer.

According to the GEMS diamond, a person's activities of daily living are evaluated during the: SAMPLE history. social assessment. medical assessment. environmental assessment.

Answer: B Rationale: The GEMS diamond was created to help you remember what is unique to older people. During the social assessment (the "S" in the GEMS diamond), the patient's activities of daily living (eg, eating, dressing, bathing, toileting) are evaluated. Are these activities being provided? If so, by whom? Are there delays in obtaining food, medication, or other necessary items? According to the GEMS diamond, a person's activities of daily living are evaluated during the: SAMPLE history. Rationale: This is a mnemonic used when obtaining information during a focused history and physical exam. social assessment;Rationale: Correct answer According to the GEMS diamond, a person's activities of daily living are evaluated during the: medical assessment. Rationale: "M" is obtained by a thorough medical history. It is important and is completed before the social assessment. environmental assessment. Rationale: "E" is the assessment of the environment. It considers if the home is well kept, too hot or too cold, or poses any hazards.

The adult EpiPen system delivers _____ mg of epinephrine, and the infant-child system delivers _____ mg. 0.15, 0.3 0.3, 0.15 0.15, 0.5 0.5, 0.2

Answer: B Rationale: The adult EpiPen system delivers 0.3 mg of epinephrine via an automatic needle and syringe system; the infant-child system delivers 0.15 mg. 0.15, 0.3 Rationale: This is not the correct dosage. 0.3, 0.15 Rationale: Correct answer 0.15, 0.5 Rationale: This is not the correct dosage. 0.5, 0.2 Rationale: This is not the correct dosage.

What are the two main components of blood? Erythrocytes and hemoglobin Cells and plasma Leukocytes and white blood cells Platelets and neutrophils

Answer: B Rationale: The blood is made up of two main components: cells and plasma. The cells in the blood include red blood cells (erythrocytes), white blood cells (leukocytes), and platelets. These cells are suspended in a straw-colored fluid called plasma. What are the two main components of blood? Erythrocytes and hemoglobin Rationale: Erythrocytes are a type of blood cell, and hemoglobin is a chemical that is contained within blood cells. Cells and plasma Rationale: Correct answer What are the two main components of blood? Leukocytes and white blood cells Rationale: Leukocytes are white blood cells, which are a type of blood cell. Platelets and neutrophils Rationale: Platelets are a type of blood cell, and neutrophils are a type of white blood cell.

Which combination of factors would MOST likely cause a hypoglycemic crisis in a diabetic patient? Eating a meal and taking insulin Skipping a meal and taking insulin Eating a meal and not taking insulin Skipping a meal and not taking insulin

Answer: B Rationale: The combination that would most likely cause a hypoglycemic crisis is skipping a meal and taking insulin. The patient will use up all available glucose in the bloodstream and become hypoglycemic. Left untreated, hypoglycemic crisis may cause permanent brain damage or even death. Which combination of factors would MOST likely cause a hypoglycemic crisis in a diabetic patient? Eating a meal and taking insulin Rationale: This process will maintain the body's glucose level. Skipping a meal and taking insulin Rationale: Correct answer Which combination of factors would MOST likely cause a hypoglycemic crisis in a diabetic patient? Eating a meal and not taking insulin Rationale: Eating will cause the glucose levels to rise. Skipping a meal and not taking insulin Rationale: Glucose levels should remain the same but may be influenced by the patient's metabolic rate or physical activities. This does not cause a hypoglycemic crisis.

The ___________ lies in the retroperitoneal space. liver pancreas stomach small intestine

Answer: B Rationale: The pancreas, kidneys, and ovaries lie in the retroperitoneal space, which is behind the peritoneum, and are often the cause of acute abdominal pain. The liver, stomach, and small intestine are all found within the true (anterior) abdomen. The ___________ lies in the retroperitoneal space. liver Rationale: The liver is found in the anterior abdomen. pancreas Rationale: Correct answer The ___________ lies in the retroperitoneal space. stomach Rationale: The stomach is found in the anterior abdomen. small intestine Rationale: The small intestine is found in the anterior abdomen.

How much activated charcoal should you administer to a 55-pound child who swallowed a bottle of aspirin? 12.5 g 25 g 2 g/kg 50 g

Answer: B Rationale: The usual dose of activated charcoal for adults and children is 1 g of charcoal per kg of body weight. To convert a patient's weight from pounds to kilograms, simply divide the weight in pounds by 2.2. Therefore, a 55-pound child should receive 25 g of activated charcoal (55 [pounds] ÷ 2.2 = 25 [kg]). The average pediatric dosing range for activated charcoal is 12.5 to 25 g. How much activated charcoal should you administer to a 55-pound child who swallowed a bottle of aspirin? 12.5 g Rationale: 12.5 g at 1 g/kg dose would be given to a 27-lb child. 25 g ;answer How much activated charcoal should you administer to a 55-pound child who swallowed a bottle of aspirin? 2 g/kg Rationale: The standard dose is 1 g/kg. 50 g Rationale: 50 g at 1 g/kg dose would be given to a patient weighing 110 lbs.

Type 1 diabetes is a condition in which: too much insulin is produced. glucose utilization is impaired. too much glucose enters the cell. the body does not produce glucose.

Answer: B Rationale: Type 1 diabetes is a disease in which the pancreas fails to produce enough insulin (or produces none at all). Insulin is a hormone that promotes the uptake of sugar from the bloodstream and into the cells. Without insulin, glucose utilization is impaired because it cannot enter the cell. Type 1 diabetes is a condition in which: too much insulin is produced. Rationale: The body only produces the amount of insulin that is needed to enable glucose to enter cells. glucose utilization is impaired. Rationale: Correct answer Type 1 diabetes is a condition in which: too much glucose enters the cell. Rationale: An abnormally high blood glucose level is known as hyperglycemia. the body does not produce glucose. Rationale: Glucose is derived from the oral intake of carbohydrates. It is stored in different body structures and then metabolized by cells.

When administering epinephrine by auto-injector, the EMT should hold the injector in place for: 5 seconds. 10 seconds. 20 seconds. 30 seconds.

Answer: B Rationale: When administering epinephrine via auto-injector, push the injector firmly against the thigh until it activates. Hold the injector in place for 10 seconds to ensure that all the medication is injected. When administering epinephrine by auto-injector, the EMT should hold the injector in place for: 5 seconds. Rationale: The injector should be held in place for 10 seconds. 10 seconds. Rationale: Correct answer When administering epinephrine by auto-injector, the EMT should hold the injector in place for: 20 seconds. Rationale: The injector should be held in place for 10 seconds. 30 seconds. Rationale: The injector should be held in place for 10 seconds.

Which of the following signs is LEAST indicative of a patient's potential for violence? The patient appears tense and "edgy." The patient is 6'5" tall and weighs 230 lb. The patient is loud and shouting obscenities. The patient is facing you with clenched fists.

Answer: B Rationale: When assessing a patient's potential for violence, you should observe for suggestive physical activity, such as clenching of the fists; glaring eyes; shouting obscenities; and rapid, disorganized speech. There is no correlation between a patient's physical size and his or her potential for violence. Which of the following signs is LEAST indicative of a patient's potential for violence? The patient appears tense and "edgy." Rationale: This is a signal of possible physical aggression and anger. The patient is 6'5" tall and weighs 230 lb. Rationale: Correct answer Which of the following signs is LEAST indicative of a patient's potential for violence? The patient is loud and shouting obscenities. Rationale: This is a signal of possible physical aggression and anger. The patient is facing you with clenched fists. Rationale: This is a signal of possible physical aggression and anger.

General guidelines to follow when caring for a patient with a behavioral crisis include all of the following, EXCEPT: being honest and reassuring. rapidly transporting the patient. having a definite plan of action. avoiding arguing with the patient.

Answer: B Rationale: When caring for a patient with a behavioral crisis, the EMT must be prepared to spend extra time with the patient. It may take longer to assess and listen to the patient prior to transport. General guidelines to follow when caring for a patient with a behavioral crisis include all of the following, EXCEPT: being honest and reassuring. Rationale: This is part of proper treatment. rapidly transporting the patient. Rationale: Correct answer having a definite plan of action. Rationale: This is part of proper treatment. avoiding arguing with the patient. Rationale: This is part of proper treatment.

When caring for a patient with a surface contact poisoning, it is important to remember to: prevent contamination of the patient. avoid contaminating yourself or others. let the hospital remove the surface poison. immediately flush dry chemicals with water.

Answer: B Response: Emergency care for a patient with a surface contact poisoning includes avoiding contaminating yourself and others and removing the irritating or corrosive substance from the patient as rapidly as possible. Dry chemicals must be brushed from the body prior to flushing with water. When caring for a patient with a contact poisoning, it is important to remember to: prevent contamination of the patient. Rationale: The patient is already contaminated and you must try to minimize further contamination. avoid contaminating yourself or others. ;answer When caring for a patient with a contact poisoning, it is important to remember to: let the hospital remove the surface poison. Rationale: Remove irritating or corrosive substances as rapidly as possible. immediately flush dry chemicals with water. Rationale: You must brush off dry chemicals first before washing.

When caring for a patient with signs of a pneumothorax, your MOST immediate concern should be: A. hypovolemia. B. intrathoracic bleeding. C. ventilatory inadequacy. D. associated myocardial injury.

Answer: C Rationale: A pneumothorax occurs when air enters the pleural space and progressively collapses the lung. This impairs the ability of the lung to move air in and out (ventilate). As the lung collapses further, ventilatory efficiency decreases, resulting in hypoxemia; this should be your most immediate concern. Some patients with a pneumothorax may also experience intrathoracic bleeding and associated myocardial injury, depending on the mechanism of injury and the force of the trauma. When caring for a patient with signs of a pneumothorax, your MOST immediate concern should be: A. hypovolemia. - Rationale: This may be indicated by the signs and symptoms of shock. B. intrathoracic bleeding. - Rationale: The patient may experience this, but inadequate ventilation is your immediate concern. C. ventilatory inadequacy. - Rationale: Correct answer D. associated myocardial injury. - Rationale: The patient may experience this, but inadequate ventilation is your immediate concern.

Your patient is a 21-year-old male who has massive face and head trauma after being assaulted. He is lying supine, is semiconscious, and has blood in his mouth. You should: A. insert a nasal airway, assess his respirations, and give 100% oxygen. B. suction his airway and apply high-flow oxygen via a nonrebreathing mask. C. manually stabilize his head, log roll him onto his side, and suction his mouth. D. apply a cervical collar, suction his airway, and begin assisting his ventilations.

Answer: C Rationale: Blood or other secretions in the mouth place the airway in immediate jeopardy and must be removed before they are aspirated. At the same time, you must protect the patient's spine due the mechanism of injury. Therefore, you should manually stabilize the patient's head, log roll him onto his side (allows drainage of blood from his mouth), and suction his mouth for up to 15 seconds. After ensuring that his airway is clear, assess his breathing and give high-flow oxygen or assist his ventilations. Nasal airways should not be used in patients with severe facial or head trauma. Your patient is a 21-year-old male who has massive face and head trauma after being assaulted. He is lying supine, is semiconscious, and has blood in his mouth. You should: A. insert a nasal airway, assess his respirations, and give 100% oxygen. - Rationale: Nasal airways should not be used in patients with severe facial or head trauma or with suspected fractures. B. suction his airway and apply high-flow oxygen via a nonrebreathing mask. - Your patient is a 21-year-old male who has massive face and head trauma after being assaulted. He is lying supine, is semiconscious, and has blood in his mouth. You should: C. canually stabilize his head, log roll him onto his side, and suction his mouth. - Rationale: Correct answer D. apply a cervical collar, suction his airway, and begin assisting his ventilations. - Rationale: The cervical collar should be applied, but manual stabilization must take place first. There are no indications here that the patient's rate of respirations are inadequate and require assisted ventilations.

Signs of a cardiac tamponade include all of the following, EXCEPT: A. muffled heart tones. B. a weak, rapid pulse. C. collapsed jugular veins. D. narrowing pulse pressure.

Answer: C Rationale: Cardiac tamponade, which is almost always caused by penetrating chest trauma, occurs when blood accumulates in the pericardial sac. This impairs the heart's ability to contract and relax; as a result, the systolic blood pressure decreases and the diastolic blood pressure increases (narrowing pulse pressure). Because the heart cannot adequately eject blood, the blood backs up beyond the right atrium, resulting in jugular venous distention. In some cases, heart tones may be muffled or distant. Other signs include a weak, rapid pulse and hypotension. Signs of a cardiac tamponade include all of the following, EXCEPT: A. muffled heart tones. - Rationale: This is an assessment finding with cardiac tamponade. B. a weak, rapid pulse. - Rationale: This is an assessment finding with cardiac tamponade. C. collapsed jugular veins. - Rationale: Correct answer D. narrowing pulse pressure.- Rationale: This is an assessment finding with cardiac tamponade.

The MOST effective way to prevent cardiopulmonary arrest in a newborn is to: A. rapidly increase its body temperature. B. allow it to remain slightly hypothermic. C. ensure adequate oxygenation and ventilation. D. start CPR if the heart rate is less than 100 beats/min.

Answer: C Rationale: Cardiopulmonary arrest in infants and children (including newborns) is most often the result of respiratory arrest. Therefore, ensuring adequate oxygenation and ventilation at all times is critical. It is also important to maintain the infant's body temperature and to prevent hypothermia. The MOST effective way to prevent cardiopulmonary arrest in a newborn is to: A. rapidly increase its body temperature. - Rationale: It is important to maintain the infant's body temperature and prevent hypothermia. B. allow it to remain slightly hypothermic. - Rationale: Hypothermia and shivering will deplete the infant's glucose and cause hypoglycemia. C. ensure adequate oxygenation and ventilation. - Rationale: Correct answer D. start CPR if the heart rate is less than 100 beats/min. - Rationale: Start CPR when the heart rate is less than 60 beats/min and not increasing with adequate ventilations.

A 14-year-old baseball player was hit in the chest with a line drive. He is in cardiac arrest. Which of the following is the most likely explanation? A. Myocardial contusion B. Traumatic asphyxia C. Commotio cordis D. Hemothorax

Answer: C Rationale: Commotio cordis is a blunt chest injury caused by a sudden, direct blow to the chest that occurs only during the critical portion of a person's heartbeat. The result may be immediate cardiac arrest. The blunt force causes ventricular fibrillation that responds positively to defibrillation within the first 2 minutes after the injury. A 14-year-old baseball player was hit in the chest with a line drive. He is in cardiac arrest. Which of the following is the most likely explanation? A. Myocardial contusion- Rationale: This may cause an irregular heartbeat, but rarely leads to cardiac arrest. B. Traumatic asphyxia - Rationale: This is the result of a crushing injury, not a direct hit. C. Commotio cordis - Rationale: Correct answer D. Hemothorax - Rationale: This comes from bleeding around the rib cage or from a lung or great vessel, rather than from a direct hit.

The presence of subcutaneous emphysema following trauma to the face and throat is MOST suggestive of: A. esophageal injury. B. cervical spine fracture. C. crushing tracheal injury. D. carotid artery laceration.

Answer: C Rationale: Crushing injuries or fractures of the larynx or trachea can result in a leakage of air into the soft tissues of the neck. The presence of air in the soft tissues produces a characteristic crackling sensation called subcutaneous emphysema. The presence of subcutaneous emphysema following trauma to the face and throat is MOST suggestive of: A. esophageal injury. - Rationale: This will produce bleeding, which may be observed in the patient's mouth or through difficulty swallowing. B. cervical spine fracture. - Rationale: This may be indicated by pain and/or paralysis. C. crushing tracheal injury. - Rationale: Correct answer D. carotid artery laceration. - Rationale: This could be assessed by excessive swelling or the presence of a large hematoma in the neck area.

Which of the following statements regarding chemical burns is FALSE? A. Most chemical burns are caused by strong acids or alkalis. B. Fumes of strong chemicals can cause burns to the respiratory tract. C. Prior to removing a dry chemical, flush the area with sterile water. D. Do not attempt to neutralize an acid burn with an alkaline chemical.

Answer: C Rationale: Dry chemicals should be brushed off of the patient before irrigating the wound with sterile water or saline. Failure to do so may increase the burning process and cause further tissue damage. Which of the following statements regarding chemical burns is FALSE? A. Most chemical burns are caused by strong acids or alkalis. - Rationale: Chemical burns are caused by acids and alkalis. B. Fumes of strong chemicals can cause burns to the respiratory tract. - Rationale: Chemicals are in the fumes and will cause respiratory tract burns. C. Prior to removing a dry chemical, flush the area with sterile water. - Rationale: Correct answer D. Do not attempt to neutralize an acid burn with an alkaline chemical. - Rationale: It would take a chemist to perform this procedure. Too much alkaline would cause burning to the patient's skin.

Severe abrasion injuries can occur when motorcycle riders are slowed after a collision by road drag. Road drag is most often associated with which type of motorcycle impact? A. Head-on collision B. Angular collision C. Ejection D. Controlled crash

Answer: C Rationale: During an ejection, the rider will travel at high speed until stopped by a stationary object, another vehicle, or road drag. Severe abrasion injuries (road rash) down to bone can occur with drag. Severe abrasion injuries can occur when motorcycle riders are slowed after a collision by road drag. Road drag is most often associated with which type of motorcycle impact? A. Head-on collision - Rationale: Road drag can occur in a head-on collision, but is more often associated with an ejection. B. Angular collision - Rationale: Road drag can occur in an angular collision, but is more often associated with an ejection. C. Ejection - Rationale: Correct answer D. Controlled crash - Rationale: Road drag can occur in a controlled crash, but is more often associated with an ejection.

A motorcyclist crashed his bike and has closed deformities to both of his midshaft femurs. He is conscious, but restless; his skin is cool and clammy; and his radial pulses are rapid and weak. The MOST appropriate splinting technique for this patient involves: A. applying rigid board splints. B. applying two traction splints. C. securing him to a long backboard. D. immobilizing his femurs with air splints.

Answer: C Rationale: In this particular case, it is more practical—and less time-consuming—to secure the patient to a long backboard. He is in shock and requires rapid transport. Taking the time to apply traction splints, air splints, or board splints will only delay transport. A motorcyclist crashed his bike and has closed deformities to both of his midshaft femurs. He is conscious, but restless; his skin is cool and clammy; and his radial pulses are rapid and weak. The MOST appropriate splinting technique for this patient involves: A. applying rigid board splints. - Rationale: This causes undue delays in the transport of the patient. B. applying two traction splints. - Rationale: This causes undue delays in the transport of the patient. C. securing him to a long backboard. - Rationale: Correct answer D. immobilizing his femurs with air splints. - Rationale: This causes undue delays in the transport of the patient.

Which of the following statements regarding intra-abdominal bleeding is FALSE? A. Intra-abdominal bleeding often causes abdominal distention. B. Intra-abdominal bleeding is common following blunt force trauma. C. The absence of pain and tenderness rules out intra-abdominal bleeding. D. Bruising may not occur immediately following blunt abdominal trauma.

Answer: C Rationale: Intra-abdominal bleeding is common following blunt trauma to the abdomen. Signs include abdominal distention; rigidity; bruising (may not occur immediately); and, in some cases, pain to palpation. However, unlike gastric juices and bacteria, blood within the abdominal cavity does not provoke an inflammatory response; therefore, the absence of pain and tenderness does not rule out internal bleeding. Which of the following statements regarding intra-abdominal bleeding is FALSE? A. Intra-abdominal bleeding often causes abdominal distention.- Rationale: You may see evidence of abdominal distention. B. Intra-abdominal bleeding is common following blunt force trauma. - Rationale: This is a common condition following blunt trauma. C. The absence of pain and tenderness rules out intra-abdominal bleeding.- Rationale: Correct answer D. Bruising may not occur immediately following blunt abdominal trauma. - Rationale: This may not occur immediately.

A 21-year-old male has a large laceration to his neck. When you assess him, you note that bright red blood is spurting from the left side of his neck. You should immediately: A. apply a pressure dressing to his neck. B. sit the patient up to slow the bleeding. C. place your gloved hand over the wound. D. apply 100% oxygen via nonrebreathing mask.

Answer: C Rationale: Laceration of the carotid artery—as evidenced by bright red blood spurting from the wound—can cause profuse bleeding, profound shock, and death very quickly. You must immediately control the bleeding with the use of direct pressure. Cover the wound with your gloved hand initially and then apply a bulky pressure dressing. After the bleeding has been controlled, apply high-flow oxygen and transport promptly. A 21-year-old male has a large laceration to his neck. When you assess him, you note that bright red blood is spurting from the left side of his neck. You should immediately: A. apply a pressure dressing to his neck. - Rationale: You should apply a bulky dressing to control bleeding. B. sit the patient up to slow the bleeding. - Rationale: Bleeding must be controlled first with direct pressure. C. place your gloved hand over the wound. - Rationale: Correct answer D. apply 100% oxygen via nonrebreathing mask. - Rationale: A nonrebreathing mask is applied after bleeding is controlled.

What is the most common presenting sign of PID? A. Vomiting B. Vaginal discharge C. Lower abdominal pain D. Fever

Answer: C Rationale: Lower abdominal pain is the most common sign of pelvic inflammatory disease. What is the most common presenting sign of PID? A. Vomiting - Rationale: Vomiting is considered to be another sign of PID. B. Vaginal discharge - Rationale: Vaginal discharge is considered to be another sign of PID. C. Lower abdominal pain - Rationale: Correct answer D. Fever - Rationale: Fever is considered to be another sign of PID.

Perfusion is MOST accurately defined as: A. the removal of adequate amounts of carbon dioxide during exhalation. B. the intake of adequate amounts of oxygen during the inhalation phase. C. circulation of blood within an organ with sufficient amounts of oxygen. D. the production of carbon dioxide, which accumulates at the cellular level.

Answer: C Rationale: Perfusion is the circulation of blood within an organ and tissues with sufficient amounts of oxygen and other nutrients. Carbon dioxide is the by-product of normal cellular metabolism; it should be returned to the lungs for removal from the body, it should not accumulate at the cellular level. Perfusion is MOST accurately defined as: A. the removal of adequate amounts of carbon dioxide during exhalation. - Rationale: Removal of carbon dioxide is a part of exhalation, and not perfusion. B. the intake of adequate amounts of oxygen during the inhalation phase. - Rationale: This is a function of ventilation, and not perfusion. C. circulation of blood within an organ with sufficient amounts of oxygen. - Rationale: Correct answer D. the production of carbon dioxide, which accumulates at the cellular level. - Rationale: Carbon dioxide is a normal by-product of cellular metabolism and should not accumulate in the cells.

A 40-year-old man was standing near a building when it exploded. He has multiple injuries, including a depressed skull fracture, severe burns, and an impaled object in his abdomen. His head injury was MOST likely caused by ___________ blast injuries. A. primary B. secondary C. tertiary D. quaternary

Answer: C Rationale: Primary blast injuries are caused by the pressure wave and include ruptured eardrums and hollow organ rupture. Secondary blast injuries are caused by flying debris and include impaled objects and shrapnel injuries. Tertiary blast injuries occur when the victim is thrown into a solid object, resulting in blunt trauma to virtually any part of the body. Quaternary blast injuries include other miscellaneous injuries. A 40-year-old man was standing near a building when it exploded. He has multiple injuries, including a depressed skull fracture, severe burns, and an impaled object in his abdomen. His head injury was MOST likely caused by ___________ blast injuries. A. - primary - Rationale: These injuries include damage eardrums, lungs, and hollow organs. B. secondary - Rationale: These injuries are caused by flying debris and usually involve impalement. C. tertiary - Rationale: Correct answer D. quaternary - Rationale: These injuries include burns, respiratory injuries and crush injuries.

Even when seatbelts are worn properly and the airbags deploy, injury may occur to the: A. chest. B. extremities. C. iliac crests. D. lower ribcage.

Answer: C Rationale: Seatbelts should be positioned over the iliac crests of the pelvis. If they are positioned higher, significant intra-abdominal injury can occur. Even when seatbelts are properly positioned and the airbags deploy, injury to the iliac crests may occur as the locking mechanism of the seatbelt engages during a motor vehicle crash that involves rapid deceleration. Even when seatbelts are worn properly and the airbags deploy, injury may occur to the: A. chest. - Rationale: If the seatbelts are worn too high, then abdominal injuries may occur. B. extremities. - Rationale: The extremities are not likely to be injured if seatbelts are worn properly. C. iliac crests. - Rationale: Correct answer D. lower ribcage. - Rationale: If seatbelts are worn too high, then abdominal injuries may occur.

The AVPU scale is used to monitor a patient's level of consciousness. What does the "P" stand for? A. Pallor B. Pediatric C. Painful D. Pale

Answer: C Rationale: The "P" in the AVPU scale stands for painful. If the patient is responsive to pain they should withdraw from it. The AVPU scale is used to monitor a patient's level of consciousness. What does the "P" stand for? A. Pallor - Rationale: Pallor means that the skin is pale. This has nothing to do with level of consciousness. B. Pediatrics - Rationale: The same AVPU scale is used for adults and pediatrics. C. Painful - Rationale: Correct answer D. Positioning - Rationale: The patient's position may provide clues to the patient's condition but it is not part of the AVPU scale.

The 1-minute Apgar score of a newborn reveals that the baby has a heart rate of 90 beats/min, a pink body but blue hands and feet, and rapid respirations. The baby cries when the soles of its feet are flicked and resists attempts to straighten its legs. You should assign an Apgar score of: A. 4. B. 6. C. 8. D. 9.

Answer: C Rationale: The Apgar score, which is obtained at 1 and 5 minutes after birth, assigns a numeric value to the following five areas: appearance, pulse, grimace, activity, and respirations. A heart rate below 100 beats/min is assigned a 1; a pink body with blue hands and feet is a 1; rapid respirations is a 2; a strong cry in reaction to a painful stimulus is a 2; and resistance against an attempt to straighten the hips and knees is a 2. Added together, the Apgar score for this infant is 8. The 1-minute Apgar score of a newborn reveals that the baby has a heart rate of 90 beats/min, a pink body but blue hands and feet, and rapid respirations. The baby cries when the soles of its feet are flicked and resists attempts to straighten its legs. You should assign an Apgar score of: A. 4 - Rationale: The correct score is 8. B. 6 - Rationale: The correct score is 8. C. 8- Rationale: Correct answer D. 9 - Rationale: The correct score is 8.

A 16-year-old boy was playing football and was struck in the left flank during a tackle. His vital signs are stable; however, he is in severe pain. You should be MOST concerned that he has injured his: A. liver. B. spleen. C. kidney. D. bladder.

Answer: C Rationale: The flanks are located laterally in the back and overlie the kidneys. During football, spearing injuries occur when a player is struck in the flank by another player's helmet. This can result in injury to the kidney ranging from bruising to severe bleeding. Injury to the liver, spleen, and bladder would more likely occur following blunt trauma to the anterior abdomen. A 16-year-old boy was playing football and was struck in the left flank during a tackle. His vital signs are stable; however, he is in severe pain. You should be MOST concerned that he has injured his: A. liver.- Rationale: The liver is associated with an injury to the anterior abdomen and right upper quadrant. B. spleen. - Rationale: The spleen is associated with an injury to the anterior abdomen and left upper quadrant. C. kidney.- Rationale: Correct answer D. bladder. - Rationale: The bladder is associated with an injury to the anterior abdomen and midline in the lower quadrants.

A 20-year-old man has major open facial injuries after his vehicle struck a tree head-on. Which of the following findings within the car would MOST likely explain his injury pattern? A. Deployed airbag B. Bent steering wheel C. Non-intact windshield D. Crushed instrument panel

Answer: C Rationale: The mechanism of injury and condition of the vehicle's interior suggest likely areas of injury. Head and neck injuries are likely to result when the head and face impact the windshield. A 20-year-old man has major open facial injuries after his vehicle struck a tree head-on. Which of the following findings within the car would MOST likely explain his injury pattern? A. Deployed airbag - Rationale: This typically results in abrasions of the face, head, and arms. B. Bent steering wheel - Rationale: This typically indicates the presence of chest injuries. C. Non-intact windshield - Rationale: Correct answer D. Crushed instrument panel - Rationale: This typically indicates the presence of leg and hip injuries.

The purpose of the eustachian tube is to: A. move in response to sound waves. B. transmit impulses from the brain to the ear. C. equalize pressure in the middle ear when external pressure changes. D. house fluid within the inner chamber of the ear and support balance.

Answer: C Rationale: The middle ear is connected to the nasal cavity by the eustachian tube, which permits equalization of pressure in the middle ear when external atmospheric pressure changes. The purpose of the eustachian tube is to: A. move in response to sound waves. - Rationale: This occurs in the tympanic membrane or eardrum. B. transmit impulses from the brain to the ear. - Rationale: Impulses are transmitted from the ear to the brain. C. equalize pressure in the middle ear when external pressure changes. - Rationale: Correct answer D. house fluid within the inner chamber of the ear and support balance. - Rationale: Bony chambers in the inner ear support balance.

A young male was involved in a motor vehicle accident and experienced a closed head injury. He has no memory of the events leading up to the accident, but remembers that he was going to a birthday party. What is the correct term to use when documenting his memory loss? A. Concussion B. Cerebral contusion C. Retrograde amnesia D. Anterograde amnesia

Answer: C Rationale: The term amnesia means loss of memory; it is common in patients who have experienced a cerebral concussion. Amnesia of events leading up to an injury is called retrograde amnesia. Anterograde amnesia—also called posttraumatic amnesia—is the inability to remember events that occurred—or will occur—after the injury. A young male was involved in a motor-vehicle accident and experienced a closed head injury. He has no memory of the events leading up to the accident, but remembers that he was going to a birthday party. What is the correct term to use when documenting his memory loss? A. Concussion - Rationale: This occurs when the brain is jarred inside the skull. B. Cerebral contusion - A young male was involved in a motor-vehicle accident and experienced a closed head injury. He has no memory of the events leading up to the accident, but remembers that he was going to a birthday party. What is the correct term to use when documenting his memory loss? C. cetrograde amnesia - Rationale: Correct answer D. Anterograde amnesia - Rationale: This is the loss of memory relating to events that occurred after the injury.

What is the outermost cavity of a woman's reproductive system? A. Cervix B. Ovaries C. Vagina D. Uterus

Answer: C Rationale: The vagina is the outermost cavity of a woman's reproductive system. A. Cervix - Rationale: The cervix opens into the vagina. Sperm passes through the cervix to the uterus and up the fallopian tubes. B. Ovaries - Rationale: The ovaries are located on each side of the lower abdomen. C. Vagina - Rationale: Correct answer D. Uterus - Rationale: The uterus is the muscular organ where the fetus grows during pregnancy.

A 4-year-old girl fell from a second-story balcony and landed on her head. She is unresponsive; has slow, irregular breathing; has a large hematoma to the top of her head; and is bleeding from her nose. You should: A. immediately perform a full-body scan to detect other injuries, administer high-flow oxygen, and transport at once. B. apply a pediatric-sized cervical collar, administer high-flow oxygen via pediatric nonrebreathing mask, and prepare for immediate transport. C. manually stabilize her head, open her airway with the jaw-thrust maneuver, insert an airway adjunct, and begin assisting her ventilations with a bag-valve mask. D. suction her airway for up to 10 seconds, insert a nasopharyngeal airway, apply a pediatric-sized cervical collar, and administer oxygen via pediatric nonrebreathing mask.

Answer: C Rationale: This child has a severe head injury and is not breathing adequately. You must manually stabilize her head to protect her spine, open her airway with the jaw-thrust maneuver, suction her airway if needed, insert an oropharyngeal airway, and assist her ventilations with a bag-valve mask. The full-body scan is performed after you have performed a primary assessment to detect and correct any life threats. The nasopharyngeal airway is contraindicated for this child; she has a head injury and is bleeding from her nose. A 4-year-old girl fell from a second-story balcony and landed on her head. She is unresponsive; has slow, irregular breathing; has a large hematoma to the top of her head; and is bleeding from her nose. You should: A. immediately perform a full-body scan to detect other injuries, administer high-flow oxygen, and transport at once. - Rationale: A full-body scan is performed after the primary assessment. B. apply a pediatric-sized cervical collar, administer high-flow oxygen via pediatric nonrebreathing mask, and prepare for immediate transport. - Rationale: Assisted ventilations must be started on a patient with slow, irregular respirations. C. manually stabilize her head, open her airway with the jaw-thrust maneuver, insert an airway adjunct, and begin assisting her ventilations with a bag-valve mask. - Rationale: Correct answer D. suction her airway for up to 10 seconds, insert a nasopharyngeal airway, apply a pediatric-sized cervical collar, and administer oxygen via pediatric nonrebreathing mask. - Rationale: A nasopharyngeal airway is contraindicated with potential facial injuries. Ventilations need to be maintained with a bag-valve mask.

You respond to a sick child late at night. The child appears very ill, has a high fever, and is drooling. He is sitting in a tripod position, struggling to breathe. You should suspect: A. croup. B. pneumonia. C. epiglottitis. D. severe asthma.

Answer: C Rationale: This child has all the classic signs of epiglottitis: high fever, drooling, and severe respiratory distress. Epiglottitis is a potentially life-threatening bacterial infection that causes the epiglottis to swell rapidly and potentially obstruct the airway. You respond to a sick child late at night. The child appears very ill, has a high fever, and is drooling. He is sitting in a tripod position, struggling to breathe. You should suspect: A. croup. - Rationale: This is a viral disease characterized by edema of the upper airways, a barking cough, and stridor. B. pneumonia. - Rationale: This is an inflammation of the lungs caused by bacteria, viruses, fungi, and other organisms. C. epiglottitis. - Rationale: Correct answer D. severe asthma. - Rationale: This is a lower airway condition resulting in intermittent wheezing and excess mucus production.

The MOST appropriate way to dress and bandage an open abdominal wound with a loop of bowel protruding from it is to: A. cover the wound with a dry, sterile dressing and apply firm pressure. B. apply a moist, sterile dressing to the wound and apply firm pressure. C. apply a moist, sterile dressing to the wound and secure it with an occlusive dressing. D. carefully replace the protruding bowel into the abdomen and cover the wound.

Answer: C Rationale: Treatment for an abdominal evisceration includes applying a moist, sterile dressing to the wound and covering the moist dressing with an occlusive dressing. Do not replace a protruding bowel back into the wound or apply firm pressure, which may force the bowel back into the wound; these actions increase the risk of infection. The MOST appropriate way to dress and bandage an open abdominal wound with a loop of bowel protruding from it is to: A. cover the wound with a dry, sterile dressing and apply firm pressure. - Rationale: You must use a moist dressing. B. apply a moist, sterile dressing to the wound and apply firm pressure. - Rationale: You should not apply pressure. C. apply a moist, sterile dressing to the wound and secure it with an occlusive dressing. - Rationale: Correct answer D. carefully replace the protruding bowel into the abdomen and cover the wound. - Rationale: Never force a bowel back into the abdominal cavity.

What device is placed directly into the stomach to feed patients? Colostomy Ileostomy Gastrostomy tube Central venous catheter

Answer: C Rationale: A gastrostomy tube is used to feed patients who cannot ingest fluids, food, or medication by mouth. What device is placed directly into the stomach to feed patients? Colostomy Rationale: This allows for elimination of waste. Ileostomy Rationale: This allows for elimination of waste. What device is placed directly into the stomach to feed patients? Gastrostomy tube Rationale: Correct answer Central venous catheter Rationale: This is a venous access device.

After taking Vicodin for 2 years for chronic pain, a 40-year-old woman finds that her usual dosage is no longer effective and goes to the doctor to request a higher dosage. This is an example of: addiction. dependence. tolerance. drug abuse.

Answer: C Rationale: A person who takes a medication for a prolonged period of time often finds that higher doses of the medication are required to achieve the same effect. This is called tolerance. After taking Vicodin for 2 years for chronic pain, a 40-year-old woman finds that her usual dosage is no longer effective and goes to the doctor to request a higher dosage. This is an example of: addiction. Rationale: This is a physiological or psychological dependence on a potentially harmful drug. dependence. Rationale: This is a physical or psychological need to use a drug. After taking Vicodin for 2 years for chronic pain, a 40-year-old woman finds that her usual dosage is no longer effective and goes to the doctor to request a higher dosage. This is an example of: tolerance. ;answer drug abuse. Rationale: This is the deliberate use of an illegal drug or too much of a prescribed drug.

If a hernia is incarcerated and the contents are so greatly compressed that circulation is compromised, the hernia is said to be: reducible. ruptured. strangulated. congenital.

Answer: C Rationale: A strangulated hernia occurs when a hernia is incarcerated and compressed by the surrounding tissues. It is a serious medical emergency and requires immediate surgery to repair the hernia, remove the dead tissue, and return oxygen to the tissues. When the mass can be placed back into the body, it is considered reducible. Hernias are not at risk of rupturing. A congenital hernia is one that is present at birth and is usually found around the umbilicus. If a hernia is incarcerated and the contents are so greatly compressed that circulation is compromised, the hernia is said to be: reduced. Rationale: This is a mass or lump that will disappear back into the body cavity in which it belongs. ruptured. Rationale: This is a mass or lump that bursts from internal pressure. If a hernia is incarcerated and the contents are so greatly compressed that circulation is compromised, the hernia is said to be: strangulated. Rationale: Correct answer hypoxemic. Rationale: This is a decrease in arterial oxygen levels.

The negative effects associated with anaphylactic shock are the result of: severe internal fluid loss. inadequate pumping of the heart. vasodilation and bronchoconstriction. the nervous system's release of adrenaline.

Answer: C Rationale: Anaphylaxis is an extreme allergic reaction that is life threatening and involves multiple organ systems. In severe cases, anaphylaxis can rapidly result in death. One of the most common signs of anaphylaxis is wheezing, a high-pitched, whistling breath sound that is typically heard on expiration, usually resulting from bronchospasm/bronchoconstriction and increased mucus production. The negative effects associated with anaphylactic shock are the result of: severe internal fluid loss. Rationale: The body does not lose fluid; blood pools in the dilated circulatory system, which causes less blood flow back to the heart. inadequate pumping of the heart. Rationale: Inadequate pumping is not the problem; the cardiac output is decreased due to poor return to the heart. The negative effects associated with anaphylactic shock are the result of: vasodilation and bronchoconstriction. Rationale: Correct answer the nervous system's release of adrenaline. Rationale: Adrenaline is the treatment for anaphylaxis.

Which of the following is a physiologic change that occurs during the process of aging? Increased elasticity of the alveoli A gradual decrease in blood pressure A decline in kidney function 10% to 15% increase in brain weight

Answer: C Rationale: As a person gets older, certain anatomic and physiologic changes occur. The alveoli in the lungs become less elastic, even though their overall size increases. Blood pressure gradually increases secondary to the process of arteriosclerosis (hardening of the arteries). A decline in kidney function occurs because of a decrease in the number of nephrons. By the age of 85 years, a 10% reduction in brain weight occurs, which causes an increased risk of head trauma. Which of the following is a physiologic change that occurs during the process of aging? Increased elasticity of the alveoli Rationale: With aging, alveoli lose some of their elasticity. A gradual decrease in blood pressure Rationale: Blood pressure generally increases due to arteriosclerosis. Which of the following is a physiologic change that occurs during the process of aging? A decline in kidney ;Rationale: Correct answer 10% to 15% increase in brain weight Rationale: The brain decreases in weight by 5% to 10%.

What improves a patient's quality of life shortly before death? Home care Hospice care Comfort care Health care

Answer: C Rationale: Comfort care is also called palliative care. Pain medications are provided during a patient's last days so he or she can enjoy time with family and friends. What improves a patient's quality of life shortly before death? Home care Rationale: Home care may improve the patient's quality of life. Hospice care Rationale: Hospice care may improve the patient's quality of life. What improves a patient's quality of life shortly before death? Comfort care Rationale: Correct answer Terminal care Rationale: This is not the correct answer.

APGAR score

Appearance (0-2) Pulse (0-2) Grimace (0-2) Activity (0-2) Respirations (0-2)

The slow onset of progressive disorientation, shortened attention span, and loss of cognitive function is called: senility. delirium. dementia. delusion.

Answer: C Rationale: Dementia is defined as the slow onset of progressive disorientation, shortened attention span, and loss of cognitive function. Alzheimer disease is an example of dementia. In contrast to dementia, delirium is an acutely altered mental status, such as that caused by hypoglycemia. The slow onset of progressive disorientation, shortened attention span, and loss of cognitive function is called: senility. Rationale: Senility causes forgetfulness and confusion. The person is mentally less acute in later life. delirium. Rationale: Delirium is an acutely altered mental status. The slow onset of progressive disorientation, shortened attention span, and loss of cognitive function is called: dementia;Rationale: Correct answer delusion. Rationale: Delusion is a fixed belief that is not shared by others and cannot be changed by reasonable argument.

It is important to remove a drowning victim from the water before laryngospasm relaxes because: the patient will suffer less airway trauma. the risk of severe hypothermia is lessened. less water will have entered the patient's lungs. you can ventilate the patient with laryngospasm.

Answer: C Rationale: Even small amounts of salt or fresh water will irritate the larynx, causing it to spasm (laryngospasm). This is the body's protective mechanism. If the EMT can safely remove the patient from the water before the laryngospasm relaxes, the amount of water that enters the lungs will be minimized. It will also be easier to ventilate the patient. It is important to remove a drowning victim from the water before laryngospasm relaxes because: the patient will suffer less airway trauma. Rationale: A laryngospasm is the closing of the vocal cords. This process will not cause trauma to the airway. the risk of severe hypothermia is lessened. Rationale: Submersion will produce hypothermia with or without the presence of a laryngospasm. It is important to remove a drowning victim from the water before laryngospasm relaxes because: less water will have entered the patient's lungs. Rationale: Correct answer you can ventilate the patient with laryngospasm. Rationale: A laryngospasm is an upper airway obstruction and you will not be able to ventilate until it relaxes.

Food poisoning is almost always caused by eating food that contains: fungi. viruses. bacteria. protozoa.

Answer: C Rationale: Food poisoning is almost always caused by eating food that contains bacteria. Salmonella poisoning and botulism—two common forms of food poisoning—are both caused by bacteria. Food poisoning is almost always caused by eating food that contains: fungi. Rationale: Fungi include mildews, molds, mushrooms, rusts, smuts, and yeasts. viruses. Rationale: Viruses are not considered to be independent living organisms. Viruses need a living host and are not found in food. Food poisoning is almost always caused by eating food that contains: bacteria. ;answer protozoa. Rationale: Protozoa are single-celled organisms.

Known risk factors for Down syndrome include: smoking. traumatic brain injury at birth. increased maternal age. lack of vitamin B.

Answer: C Rationale: Increased maternal age, along with a family history of Down syndrome, are risk factors of Down syndrome. Known risk factors for Down syndrome include: smoking. Rationale: Smoking is a risk factor for many conditions. traumatic brain injury at birth. Rationale: TBI is a risk factor of cerebral palsy. Known risk factors for Down syndrome include: increased maternal age. Rationale: Correct answer lack of vitamin B. Rationale: This is a risk factor for spina bifida.

When caring for a patient with an emotional crisis who is calm and not in need of immediate emergency care, your BEST course of action is to: advise the patient that he or she cannot refuse treatment. leave the patient with a trusted friend or family member. attempt to obtain consent from the patient to transport. apply soft restraints in case the patient becomes violent.

Answer: C Rationale: Just because a patient is experiencing an emotional crisis does not mean that he or she is "mentally incompetent" and cannot refuse EMS treatment and/or transport. You should attempt to obtain consent from any conscious patient unless he or she clearly does not have decision-making capacity (eg, underage, altered mental status, alcohol intoxication). When caring for a patient with an emotional crisis who is calm and not in need of immediate emergency care, your BEST course of action is to: advise the patient that he or she cannot refuse treatment. Rationale: Do this only if the patient clearly does not have decision-making capacity (eg, underage, intoxicated). leave the patient with a trusted friend or family member. Rationale: Attempt to obtain verbal consent for transport to a medical facility. When caring for a patient with an emotional crisis who is calm and not in need of immediate emergency care, your BEST course of action is to: attempt to obtain consent from the patient to transport. Rationale: Correct answer apply soft restraints in case the patient becomes violent. Rationale: Restraints are not often used in situations where a patient might become violent, but they are considered.

What breathing pattern would you MOST likely encounter in a patient with diabetic ketoacidosis (DKA)? Slow and shallow Shallow and irregular Rapid and deep Slow and irregular

Answer: C Rationale: Kussmaul respirations—a rapid and deep breathing pattern seen in patients with DKA—indicates that the body is attempting to eliminate ketones via the respiratory system. A fruity or acetone breath odor is usually present in patients with Kussmaul respirations. What breathing pattern would you MOST likely encounter in a patient with diabetic ketoacidosis (DKA)? Slow and shallow Rationale: Agonal respirations are seen with cerebral anoxia and may have an occasional gasp. Shallow and irregular Rationale: Agonal respirations are seen with cerebral anoxia and may have an occasional gasp. What breathing pattern would you MOST likely encounter in a patient with diabetic ketoacidosis (DKA)? Rapid and deep Rationale: Correct answer Slow and irregular Rationale: Slow and irregular respiration results from increased intracranial pressure and can also have periods of apnea.

Which of the following are components of the Cincinnati Prehospital Stroke Scale? Arm drift, blood pressure, speech Speech, pupil response, arm drift Facial symmetry, speech, arm drift Pupil response, facial droop, speech

Answer: C Rationale: The three components of the Cincinnati Prehospital Stroke Scale are facial symmetry, speech, and arm drift. Both sides of the patient's face should move symmetrically (equally) when he or she smiles. The patient's speech should be easily understandable and without slurring. The patient should be able to hold both arms out in front of his or her body—with eyes closed and palms up—without one arm drifting down to his or her side. Which of the following are components of the Cincinnati Prehospital Stroke Scale? Arm drift, blood pressure, speech Rationale: The scale does not use blood pressure. Speech, pupil response, arm drift Rationale: The scale does not use pupil response. Facial symmetry, speech, arm drift Rationale: Correct answer Pupil response, facial droop, speech Rationale: The scale does not use pupil response.

In which position do most patients with acute abdominal pain prefer to be transported? Sitting, with their head elevated 45° Supine, with their legs elevated 12 inches On their side, with their knees flexed In Fowler's position, with their legs straight

Answer: C Rationale: Most patients with acute abdominal pain prefer to lie on their side with their knees flexed (and usually drawn up into their abdomen). This position takes pressure off the abdominal muscles and may afford them pain relief. The other positions do not allow the pressure to be relieved and may cause further discomfort. In which position do most patients with acute abdominal pain prefer to be transported? Sitting, with the head elevated 45° Rationale: This is also known as semi-Fowler's position. Supine, with their legs elevated 12 inches Rationale: This position will not relieve pressure from the abdomen. In which position do most patients with acute abdominal pain prefer to be transported? On their side, with their knees flexed Rationale: Correct answer In Fowler's position, with their legs straight Rationale: This is when the patient is sitting straight up.

Most patients with this disease also have hydrocephalus. Paralysis Down syndrome Spina bifida Cerebral palsy

Answer: C Rationale: Most patients with spina bifida also have hydrocephalus, which requires the placement of a shunt. Most patients with this disease also have hydrocephalus. Paralysis Rationale: This is not the correct answer. Down syndrome Rationale: This is not the correct answer. Spina bifida Rationale: Correct answer Cerebral palsy Rationale: This is not the correct answer.

Shivering in the presence of hypothermia indicates that the: musculoskeletal system is damaged. nerve endings are damaged, causing loss of muscle control. body is trying to generate more heat through muscular activity. thermoregulatory system has failed and body temperature is falling.

Answer: C Rationale: Shivering in the presence of hypothermia indicates that the body is trying to generate more heat (thermogenesis) through muscular activity. In early hypothermia, shivering is a voluntary attempt to produce heat; as hypothermia progresses, shivering becomes involuntary. Shivering in the presence of hypothermia indicates that the: musculoskeletal system is damaged. Rationale: Hypothermia is not a physical injury. nerve endings are damaged, causing loss of muscle control. Rationale: Hypothermia is not a physical injury. Shivering in the presence of hypothermia indicates that the: body is trying to generate more heat through muscular activity. Rationale: Correct answer thermoregulatory system has failed and body temperature is falling. Rationale: The thermoregulatory system has not failed; it is producing heat and keeping the body warm.

Which of the following patients is at HIGHEST risk for suicide? A 24-year-old woman who is successfully being treated for depression A 29-year-old man who was recently promoted with a large pay increase A 33-year-old man who regularly consumes alcohol and purchased a gun A 45-year-old woman who recently found out her cancer is in full remission

Answer: C Rationale: Situations or indications that place a patient at high risk for suicide include, but are not limited to, recent diagnosis of a serious illness; financial setback; marital discord; death of a loved one; untreated psychiatric illness; recent acquisition of items that can cause death, such as a gun or knife; and chronic alcohol use. Which of the following patients is at HIGHEST risk for suicide? A 24-year-old woman who is successfully being treated for depression Rationale: This woman is not a high risk for suicide. A 29-year-old man who was recently promoted with a large pay increase Rationale: This man is not a high risk for suicide. Which of the following patients is at HIGHEST risk for suicide? A 33-year-old man who regularly consumes alcohol and purchased a gun Rationale: Correct answer A 45-year-old woman who recently found out her cancer is in full remission Rationale: This woman is not a high risk for suicide.

A young male is experiencing signs and symptoms of anaphylactic shock after being stung by a scorpion. His level of consciousness is diminished, his breathing is severely labored, you can hear inspiratory stridor, and his face is cyanotic. The patient has a prescribed epinephrine auto-injector. What should you do first? Assist him in administering his epinephrine. Apply high-flow oxygen via nonrebreathing mask. Provide ventilatory assistance with a bag-valve mask. Elevate his legs and cover him with a warm blanket.

Answer: C Rationale: The patient is not breathing adequately, as noted by his decreased level of consciousness, severely labored breathing, inspiratory stridor, and cyanosis. Therefore, you should first assist his ventilations with a bag-valve mask. He clearly requires epinephrine, but not before restoring adequate breathing first. Regardless of the situation, a patient's airway must be patent and his or her breathing must remain adequate at all times. A young male is experiencing signs and symptoms of anaphylactic shock after being stung by a scorpion. His level of consciousness is diminished, his breathing is severely labored, you can hear inspiratory stridor, and his face is cyanotic. The patient has a prescribed epinephrine auto-injector. What should you do first? Assist him in administering his epinephrine. Rationale: This is part of the treatment, but only after the breathing has been addressed. Apply high-flow oxygen via nonrebreathing mask. Rationale: Respirations need assistance due to labored breathing and a diminished level of consciousness. A young male is experiencing signs and symptoms of anaphylactic shock after being stung by a scorpion. His level of consciousness is diminished, his breathing is severely labored, you can hear inspiratory stridor, and his face is cyanotic. The patient has a prescribed epinephrine auto-injector. What should you do first? Provide ventilatory assistance with a bag-valve mask. Rationale: Correct answer Elevate his legs and cover him with a warm blanket. Rationale: You should treat the patient for shock, but breathing is the first priority.

You are called to a local baseball park for a 23-year-old man with difficulty breathing. He states that he ate a package of peanuts approximately 30 minutes ago and denies any allergies or past medical history. Your assessment reveals widespread urticaria, tachycardia, and a BP of 90/60 mm Hg. You can hear him wheezing, even without a stethoscope. You should be MOST suspicious of a(n): acute asthma attack. mild allergic reaction. anaphylactic reaction. moderate allergic reaction.

Answer: C Rationale: The patient's signs and symptoms indicate an anaphylactic reaction. Signs and symptoms of an anaphylactic reaction include difficulty breathing, urticaria (hives) over large parts of the body, and signs of shock (eg, tachycardia, hypotension). Certain foods, such as shellfish and nuts, may result in a relatively slow onset of symptoms, but the symptoms can become just as severe. You are called to a local baseball park for a 23-year-old man with difficulty breathing. He states that he ate a package of peanuts approximately 30 minutes ago, and denies any allergies or past medical history. Your assessment reveals widespread urticaria, tachycardia, and a BP of 90/60 mm Hg. You can hear him wheezing, even without a stethoscope. You should be MOST suspicious of a(n): acute asthma attack. Rationale: Asthma presents with difficulty breathing, but patients will not have urticaria (hives). You are called to a local baseball park for a 23-year-old man with difficulty breathing. He states that he ate a package of peanuts approximately 30 minutes ago, and denies any allergies or past medical history. Your assessment reveals widespread urticaria, tachycardia, and a BP of 90/60 mm Hg. You can hear him wheezing, even without a stethoscope. You should be MOST suspicious of a(n): mild allergic reaction. Rationale: Mild reactions usually appear with urticaria, itching, and some swelling, but not hypotension and breathing difficulties. You are called to a local baseball park for a 23-year-old man with difficulty breathing. He states that he ate a package of peanuts approximately 30 minutes ago and denies any allergies or past medical history. Your assessment reveals widespread urticaria, tachycardia, and a BP of 90/60 mm Hg. You can hear him wheezing, even without a stethoscope. You should be MOST suspicious of a(n): anaphylactic reaction. Rationale: Correct answer You are called to a local baseball park for a 23-year-old man with difficulty breathing. He states that he ate a package of peanuts approximately 30 minutes ago and denies any allergies or past medical history. Your assessment reveals widespread urticaria, tachycardia, and a BP of 90/60 mm Hg. You can hear him wheezing, even without a stethoscope. You should be MOST suspicious of a(n): moderate allergic reaction. Rationale: The designation is mild or severe (anaphylaxis)—not moderate.

If a patient complains of a severe migraine, how should she be transported? In a brightly lit ambulance so she can see while her vision is impaired With loud sirens so she can get to the hospital as soon as possible Without lights and sirens This patient should not be transported.

Answer: C Rationale: Treatment of a migraine headache is supportive; however, you should always assess the patient for other signs and symptoms that might indicate a more serious condition. Applying high-flow oxygen, if tolerated, may help ease the patient's condition. When possible, provide a darkened and quiet environment because patients are sensitive to light and sound. Do not use lights and sirens during transport. If a patient complains of a severe migraine, how should she be transported? In a brightly lit ambulance so she can see while her vision is impaired Rationale: Migraine patients are sensitive to light. With loud sirens so she can get to the hospital as soon as possible Rationale: Migraine patients are sensitive to loud noises. If a patient complains of a severe migraine, how should she be transported? Without lights and sirens Rationale: Correct answer This patient should not be transported. Rationale: A migraine could indicate a more serious condition.

You respond to a soccer game for a 16-year-old male with severe ankle pain. When you deliver him to the hospital, the physician tells you that he suspects a sprain. This means that: A. there is a disruption of the joint and the bone ends are no longer in contact. B. the patient has an incomplete fracture that passes only partway through the bone. C. stretching or tearing of the ligaments with partial or temporary dislocation of the bone ends has occurred. D. the muscles of the ankle have been severely stretched, resulting in displacement of the bones from the joint.

Answer: C Rationale: A sprain is a joint injury in which there is both partial or temporary dislocation of the bone ends and partial stretching or tearing of the supporting ligaments. Sprains are typically marked by swelling, pain, and ecchymosis. You respond to a soccer game for a 16-year-old male with severe ankle pain. When you deliver him to the hospital, the physician tells you that he suspects a sprain. This means that: A. there is a disruption of the joint and the bone ends are no longer in contact. - Rationale: With a sprain, there will be a partial or temporary dislocation of the bone ends. B. the patient has an incomplete fracture that passes only partway through the bone. - Rationale: With a sprain, there is no fracture associated with the injury. C. stretching or tearing of the ligaments with partial or temporary dislocation of the bone ends has occurred. - Rationale: Correct answer D. the muscles of the ankle have been severely stretched, resulting in displacement of the bones from the joint. - Rationale: A sprain is not an injury to the muscles.

You should discourage a rape or sexual assault victim from doing which of the following? A. Urinating B. Cleaning herself C. Changing clothes D. All of the above

Answer: D Rationale: A victim of sexual assault or rape should be discouraged from showering, urinating, changing clothes, moving bowels, or rinsing out her mouth in order to preserve evidence. You should discourage a rape or sexual assault victim from doing which of the following? A. Urinating - Rationale: The victim should not urinate in order to preserve evidence. B. Cleaning herself - Rationale: The victim should not clean herself in order to preserve evidence. C. Changing clothes - Rationale: The victim should not change her clothes in order to preserve evidence. D. All of the above - Rationale: Correct answer

When caring for a patient with internal bleeding, the EMT must first: A. ensure a patent airway. B. obtain baseline vital signs. C. control any external bleeding. D. take appropriate standard precautions.

Answer: D Rationale: All of the interventions in this question must be performed. However, before providing patient care—whether the patient is bleeding or not—the EMT must first ensure that he or she has taken the appropriate standard precautions. When caring for a patient with internal bleeding, the EMT must first: A. ensure a patent airway. - Rationale: This would be the first step after standard precautions. B. obtain baseline vital signs.- Rationale: This would be the third step after standard precautions, airway, and bleeding control. C. control any external bleeding.- Rationale: This would be the second step after standard precautions and airway. D. take appropriate standard precautions. - Rationale: Correct answer

Other than applying a moist, sterile dressing covered with a dry dressing to treat an abdominal evisceration, an alternative form of management may include: A. placing dry towels over the open wound. B. cleaning the exposed bowel with sterile saline. C. applying the PASG to stop the associated bleeding. D. applying an occlusive dressing, secured by trauma dressings.

Answer: D Rationale: Although the preferred management for an abdominal evisceration includes the application of a moist, sterile dressing covered by a dry dressing, protocols in some EMS systems call for an occlusive dressing, secured by trauma dressings. An occlusive dressing may help prevent the loss of body heat through the abdominal wound. Other than applying a moist, sterile dressing covered with a dry dressing to treat an abdominal evisceration, an alternative form of management may include: A. placing dry towels over the open wound.- Rationale: Treatment is a moist, sterile dressing over the open wound. B. cleaning the exposed bowel with sterile saline. - Rationale: EMS should not clean any exposed abdominal organs. C. applying the PASG to stop the associated bleeding.- Rationale: PASG inflation is contraindicated with an abdominal evisceration. D. applying an occlusive dressing, secured by trauma dressings. - Rationale: Correct answer

Which effects will the application of an ice pack have on a hematoma? A. Vasodilation and increased pain B. Vasodilation and decreased bleeding C. Vasoconstriction and increased swelling D. Vasoconstriction and decreased bleeding

Answer: D Rationale: Applying an ice pack to a closed wound, such as a hematoma, will decrease bleeding, pain, and swelling by causing constriction of the blood vessels. Which effects will the application of an ice pack have on a hematoma? A. Vasodilation and increased pain - Rationale: An ice pack causes vasoconstriction and will reduce pain. B. Vasodilation and decreased bleeding - Rationale: An ice pack causes vasoconstriction and will reduce bleeding. C. Vasoconstriction and increased swelling - Rationale: An ice pack causes vasoconstriction and will reduce swelling. D. Vasoconstriction and decreased bleeding - Rationale: Correct answer

A man involved in a motorcycle crash has multiple abrasions and lacerations. Which of the following injuries has the HIGHEST treatment priority? A. Widespread abrasions to the back with pinkish ooze B. 3" laceration to the forehead with dark red, flowing blood C. Laceration to the forearm with obvious debris in the wound D. 1" laceration to the thigh with spurting, bright red blood

Answer: D Rationale: Bleeding from an artery produces bright red bleeding that spurts with the pulse. The pressure that causes the blood to spurt also makes this type of bleeding difficult to control. Blood loss from an arterial wound is more severe—and thus, more life threatening—than from a venous wound. A man involved in a motorcycle crash has multiple abrasions and lacerations. Which of the following injuries has the HIGHEST treatment priority? A. Widespread abrasions to the back with pinkish ooze - Rationale: Abrasions are painful, but not an immediate life threat. B. 3" laceration to the forehead with dark red, flowing blood- Rationale: Venous bleeding is controlled after arterial bleeding is controlled. C. Laceration to the forearm with obvious debris in the wound- Rationale: There is no indication that this wound is actively bleeding. D. 1" laceration to the thigh with spurting, bright red blood - Rationale: Correct answer

A compression injury that is severe enough to cut off blood flow below the injury is called: A. a contusion. B. a hematoma. C. a local thrombus. D. compartment syndrome.

Answer: D Rationale: Compartment syndrome can occur when a part of the body has been compressed for a prolonged period of time—usually greater than 4 hours. The injured tissue begins to swell, which can impede arterial blood flow and venous return. As a result, the part of the body distal to the compression site becomes hypoxic and metabolic waste products (ie, lactic acid) begin to accumulate. A compression injury that is severe enough to cut off blood flow below the injury is called: A. a contusion. - Rationale: This is a bruise. B. a hematoma. - Rationale: This is blood that has collected within damaged tissue. A hematoma occurs when a large blood vessel is injured. C. a local thrombus. - Rationale: This is a blood clot. D. compartment syndrome. - Rationale: Correct answer

Which of the following would MOST likely result from the third collision in the "three-collision" effect that occurs during a high-speed, frontal impact motor vehicle crash? A. Extensive damage to the automobile B. Flail chest and lower extremity fractures C. Massive external trauma with severe bleeding D. Aortic rupture or compression injury to the brain

Answer: D Rationale: During the third collision, the body's internal organs collide with the inside of the body. These injuries are usually not as obvious, but are often the most life-threatening. Injuries that may result from this include internal injuries of the brain (compression injuries) and aortic tears, resulting in massive internal bleeding. Which of the following would MOST likely result from the third collision in the "three-collision" effect that occurs during a high-speed, frontal impact motor vehicle crash? A. Extensive damage to the automobile - Rationale: This would occur in the first collision. B. Flail chest and lower extremity fractures - Rationale: This would occur in the second collision. C. Massive external trauma with severe bleeding - Rationale: This would occur in the second collision. D. Aortic rupture or compression injury to the brain - Rationale: Correct answer

A distraction injury to the cervical spine would MOST likely occur following: A. a diving accident. B. blunt neck trauma. C. hyperextension of the neck. D. hanging-type mechanisms.

Answer: D Rationale: Excessive traction on the neck, such as what occurs during hanging-type mechanisms, can cause a distraction injury of the cervical spine. Distraction injuries can cause separation of the vertebrae and stretching or tearing of the spinal cord. A distraction injury to the cervical spine would MOST likely occur following: A. a diving accident. - Rationale: This would possibly cause a compression injury. B. blunt neck trauma. - A distraction injury to the cervical spine would MOST likely occur following: C. cyperextension of the neck. - Rationale: This can result in a fracture or neurologic deficit. D. hanging-type mechanisms. - Rationale: Correct answer

Which of the following statements regarding febrile seizures is correct? A. Febrile seizures usually indicate a serious underlying condition, such as meningitis. B. Most febrile seizures occur between the ages of 2 months and 2 years of age. C. Febrile seizures are rarely associated with tonic-clonic activity, but last for more than 15 minutes. D. Febrile seizures usually last less than 15 minutes and often do not have a postictal phase.

Answer: D Rationale: Febrile seizures are the most common seizures in pediatric patients; they are common between the ages of 6 months and 6 years of age. Most pediatric seizures are due to fever alone—hence the name "febrile" seizure. However, seizures and fever may indicate a more serious underlying condition, such as meningitis. Febrile seizures are characterized by generalized tonic-clonic activity and last less than 15 minutes; if a postictal phase occurs, it is generally very short. Which of the following statements regarding febrile seizures is correct? A. Febrile seizures usually indicate a serious underlying condition, such as meningitis. - Rationale: Most febrile seizures are caused by fever, but a fever and seizures may be an indication of a serious underlying condition. B. Most febrile seizures occur between the ages of 2 months and 2 years of age. - Rationale: Most febrile seizures occur between the ages of 6 months and 6 years. C. Febrile seizures are rarely associated with tonic-clonic activity, but last for more than 15 minutes. - Rationale: Febrile seizures last less than 15 minutes. D. Febrile seizures usually last less than 15 minutes and often do not have a postictal phase. - Rationale: Correct answer

During your assessment of a patient who was stabbed, you see an open wound to the left anterior chest. Your MOST immediate action should be to: A. position the patient on the affected side. B. transport immediately. C. assess the patient for a tension pneumothorax. D. cover the wound with an occlusive dressing.

Answer: D Rationale: If you encounter an open chest wound, you must cover it with an occlusive dressing. This will prevent air from moving in and out of the wound. After the dressing is applied, you must monitor the patient for signs of a developing tension pneumothorax. During your assessment of a patient who was stabbed, you see an open wound to the left anterior chest. Your MOST immediate action should be to: A. position the patient on the affected side.- Rationale: This is not the most immediate action. B. transport immediately. - Rationale: Transport should take place once life threats have been managed. C. assess the patient for a tension pneumothorax. - Rationale: You must monitor for signs of a developing pneumothorax. D. cover the wound with an occlusive dressing. - Rationale: Correct answer

Immediately after delivery of the infant's head, you should: A. suction the baby's mouth and then nose. B. suction the baby's nose and then mouth. C. assess the baby's breathing effort and skin color. D. check the position of the umbilical cord.

Answer: D Rationale: Immediately following delivery of the infant's head, you should check the position of the umbilical cord to make sure it is not wrapped around the baby's neck (nuchal cord). If a nuchal cord is not present, suction the infant's mouth and nose. Immediately after delivery of the infant's head, you should: A. suction the baby's mouth and then nose. - Rationale: After EMS has confirmed that the cord is not around the infant's head, this should be performed. B. suction the baby's nose and then mouth. - Rationale: After EMS has confirmed that the cord is not around the infant's head, suctioning of the mouth and then the nose should be performed. C. assess the baby's breathing effort and skin color. - Rationale: This cannot be performed until the entire infant has been delivered completely. D. check the position of the umbilical cord. - Rationale: Correct answer

Peritonitis would MOST likely result following injury to the: A. liver. B. spleen. C. kidney. D. stomach.

Answer: D Rationale: In general, solid organs bleed when injured and hollow organs spill their contents into the abdominal cavity, resulting in peritonitis—inflammation of the intra-abdominal lining. Of the choices listed, the stomach is the only hollow organ. A. liver. - Rationale: The liver typically bleeds into the abdominal cavity. B. spleen. - Rationale: The spleen typically bleeds into the abdominal cavity. C. kidney. - Rationale: The kidneys typically bleed into the retroperitoneal space and not into the abdomen. D. stomach. - Rationale: Correct answer

You should NOT remove an injured football player's helmet if: A. a cervical spine injury is suspected, even if the helmet fits loosely. B. the patient has a patent airway, even if he has breathing difficulty. C. he has broken teeth, but only if the helmet does not fit snugly in place. D. the face guard can easily be removed and there is no airway compromise.

Answer: D Rationale: In general, you should leave a helmet on if it fits snugly and does not allow movement of the head within the helmet, the patient's airway is patent, no airway problems are anticipated, and the patient is breathing without difficulty. If you can easily remove the face guard (often the case with football helmets) and there are no airway problems, do so but leave the helmet on. If the helmet is loose, the airway is in anyway compromised, or the patient has difficulty breathing or is in cardiac arrest, the helmet must be removed. You should NOT remove an injured football player's helmet if: A. a cervical spine injury is suspected, even if the helmet fits loosely. - Rationale: If the helmet allows for movement of the head, it should be removed. B. the patient has a patent airway, even if he has breathing difficulty. - You should NOT remove an injured football player's helmet if: C. ce has broken teeth, but only if the helmet does not fit snugly in place. - Rationale: Broken teeth present a potential for airway obstruction. D. the face guard can easily be removed and there is no airway compromise. - Rationale: Correct answer

Where would you MOST likely find information regarding a patient's wishes to be an organ donor? Select one: A. Voter registration card B. Driver's license C. Insurance card D. Social Security card

B. Driver's license

A young male has a musculoskeletal injury and is unresponsive. You will NOT be able to assess: A. false motion. B. distal pulses. C. capillary refill. D. sensory and motor functions.

Answer: D Rationale: In order to assess sensory and motor functions (eg, Can you feel? Can you move?), the patient must be conscious, alert, and able to follow commands. False motion, distal pulses, and capillary refill are objective findings; therefore, they can be assessed in unresponsive patients. A young male has a musculoskeletal injury and is unresponsive. You will NOT be able to assess: A. false motion. - Rationale: This is an objective finding. B. distal pulses. - Rationale: This is an objective finding. C. capillary refill. - Rationale: This is an objective finding. D. sensory and motor functions. - Rationale: Correct answer

Death from a rollover motor vehicle crash is MOST often secondary to: A. crushing injuries. B. airbag-related trauma. C. multiple collisions to the interior of the car. D. ejection of the patient from the motor vehicle.

Answer: D Rationale: Rollover crashes are the most unpredictable with regard to injuries sustained by the patient. An unrestrained passenger may have struck multiple points within the vehicle. However, the most life-threatening event in a rollover is ejection or partial ejection of the patient from the vehicle. Death from a rollover motor vehicle crash is MOST often secondary to: A. crushing injuries. - Rationale: These injuries occur during ejection or partial ejection. B. airbag-related trauma. - Rationale: Airbags significantly reduce the risk of death in motor vehicle crashes. C. multiple collisions to the interior of the car. - Rationale: This makes the prediction of injury patterns difficult, but is not the most common life-threatening event in a rollover. D. ejection of the patient from the motor vehicle. - Rationale: Correct answer

Which of the following is considered a severe burn? A. Any full-thickness burn B. 20% partial-thickness burn C. 10% full-thickness burn with abrasions D. 5% full-thickness burn with a fracture

Answer: D Rationale: Severe burns include the following: full-thickness burns involving the hands, feet, face, airway, or genitalia; full-thickness burns covering more than 10% of the body's total surface area (BSA); partial-thickness burns covering more than 30% of the BSA; burns involving the respiratory tract (eg, smoke inhalation); burns complicated by fractures; and burns on patients younger than 5 years or older than 55 years that would otherwise be classified as "moderate" burns on younger adults. Which of the following is considered a severe burn? A. Any full-thickness burn - Rationale: A full-thickness burn is severe if it covers more than 10% of the body or involves the hands, face, feet, and genitalia. B. 20% partial-thickness burn - Rationale: This burn must be greater than 30% BSA. C. 10% full-thickness burn with abrasions - Rationale: This burn must be greater than 10% BSA. D. 5% full-thickness burn with a fracture - Rationale: Correct answer

The globe of the eye is also called the: A. lens. B. orbit. C. retina. D. eyeball.

Answer: D Rationale: The globe of the eye is also called the eyeball. The lens, which sits behind the iris, focuses images on the retina—the light-sensitive area at the back of the globe. The globe is located within a bony socket in the skull called the orbit. The globe of the eye is also called the: A. lens. - Rationale: The lens sits behind the iris and focuses images on the light-sensitive area at the back of the globe. B. orbit. - Rationale: The orbit forms the base of the floor of the cranial cavity and contains the eye. C. retina. - Rationale: The retina is the light-sensitive area at the back of the globe. D. eyeball. - Rationale: Correct answer

The need for and extent of newborn resuscitation is based on: A. the 1-minute Apgar score. B. the gestational age of the newborn. C. the newborn's response to oxygen. D. respiratory effort, heart rate, and color.

Answer: D Rationale: The need for and extent of newborn resuscitation is based on respiratory effort, heart rate, and skin color. The Apgar score is not used to determine if resuscitation is needed; the first score is not assigned until the newborn is 1 minute of age. Resuscitation, if needed, should commence immediately. The need for and extent of newborn resuscitation is based on: A. the 1-minute Apgar score. - Rationale: The Apgar score is not used to determine if resuscitation is needed. B. the gestational age of the newborn. - Rationale: A premature gestational age may indicate a greater risk for the infant, but does not indicate if resuscitation is required. C. the newborn's response to oxygen. - Rationale: Oxygen response is evaluated by respiratory rate, heart rate, and color. D. respiratory effort, heart rate, and color. - Rationale: Correct answer

A man is found slumped over the steering wheel, unconscious and making snoring sounds, after an automobile accident. His head is turned to the side and his neck is flexed. You should: A. gently rotate his head to correct the deformity. B. carefully hyperextend his neck to open his airway. C. apply an extrication collar with his head in the position found. D. manually stabilize his head and move it to a neutral, in-line position.

Answer: D Rationale: The patient's snoring sounds indicate an airway problem, which must be corrected or he may die. Manually stabilize his head; carefully move it to a neutral, in-line position; and reassess his breathing. Do not rotate or hyperextend the neck of a patient with a possible spinal injury; the results could be disastrous. A man is found slumped over the steering wheel, unconscious and making snoring sounds, after an automobile accident. His head is turned to the side and his neck is flexed. You should: A. gently rotate his head to correct the deformity. - Rationale: Do not hyperextend the neck of a patient with a possible spinal injury. B. carefully hyperextend his neck to open his airway. - A man is found slumped over the steering wheel, unconscious and making snoring sounds, after an automobile accident. His head is turned to the side and his neck is flexed. You should: C. cpply an extrication collar with his head in the position found. - Rationale: The head must be placed in a neutral position to open the airway. D. manually stabilize his head and move it to a neutral, in-line position. - Rationale: Correct answer

A condition in which a person experiences a loss of appetite is called: ileus. colic. emesis. anorexia.

Answer: D Rationale: Anorexia is defined as a loss of appetite. It is a nonspecific symptom but is often associated with gastrointestinal diseases and abdominal pain. Ileus is the paralysis of the muscular contractions that normally propel material through the intestine. Colic is a severe, intermittent cramping pain. Emesis is the proper medical term for vomiting. A condition in which a person experiences a loss of appetite is called: ileus. Rationale: Ileus is the paralysis of the muscular contractions that normally propel material through the intestine. colic. Rationale: Colic is a severe, intermittent cramping pain. A condition in which a person experiences a loss of appetite is called: emesis. Rationale: Emesis is also known as vomiting. anorexia. Rationale: Correct answer

How does the pediatric anatomy differ from the adult anatomy? A. The trachea is more rigid. B. The tongue is proportionately smaller. C. The epiglottis is less floppy. D. The head is proportionately larger.

Answer: D Rationale: There are several important anatomic differences between pediatric patients and adult patients. The head—specifically the occiput—is proportionately larger. Their tongue and epiglottis are also proportionately larger, and the epiglottis is floppier and more omega-shaped. The airway is narrower at all levels, and the trachea is less rigid and easily collapsible. How does pediatric anatomy differ from adult anatomy? A. The trachea is more rigid. - Rationale: The trachea is less rigid, narrower, and more anterior than an adult trachea. B. The tongue is proportionately smaller. - Rationale: The tongue is proportionally larger than an adult tongue. C. The epiglottis is less floppy. - Rationale: The epiglottis is floppier and shaped differently. D. The head is proportionately larger. - Rationale: Correct answer

You are transporting a woman who is 8 months pregnant. To prevent supine hypotensive syndrome, how should you position this patient? A. On her right side B. Supine C. Semi-Fowler's D. On her left side

Answer: D Rationale: To prevent supine hypotensive syndrome, the patient must be positioned on her left side. This stops the weight of the baby from compressing the inferior vena cava, which can cause low blood pressure. You are transporting a woman who is 8 months pregnant. To prevent supine hypotensive syndrome, how should you position this patient? A. On her right side - Rationale: The patient should be transported on her left side. B. Supine - Rationale: Lying the patient supine will cause hypotension. C. Semi-Fowler's- Rationale: The patient should be transported on her left side. D. On her left side - Rationale: Correct answer

When obtaining a SAMPLE history, which of the following pieces of information is important to obtain? A. Use of a birth control device or birth control pills B. The date of the patient's last menstrual period C. The possibility of pregnancy D. All of the above

Answer: D Rationale: When obtaining a SAMPLE history, the EMT should inquire about the patient's medications. The EMT must ask about the use of birth control pills or birth control devices and ask specifically about the patient's last menstrual period. The EMT should also inquire about the possibility of sexually transmitted diseases and the possibility of pregnancy. When obtaining a SAMPLE history, which of the following pieces of information is important to obtain? A. Use of a birth control device or birth control pills - Rationale: The EMT should also inquire about the possibility of pregnancy and the date of the last menstrual period. B. The date of the patient's last menstrual period - Rationale: The EMT should also inquire about the use of birth control pills and devices and the possibility of pregnancy. C. The possibility of pregnancy - Rationale: The EMT should also inquire about the use of birth control pills and devices and the date of the last menstrual period. D. All of the above - Rationale: Correct answer

When assessing a stab wound, it is important for the EMT to remember that: A. stabbings to an extremity are rarely associated with an exit wound. B. the majority of the internal trauma will be near the path of the knife. C. most stabbings are unintentional and cause less severe internal injury. D. more internal damage may be present than the external wound suggests.

Answer: D Rationale: With low-velocity penetrations, injuries are caused by sharp edges of the object moving through the body and are therefore close to the object's path. Weapons such as knives, however, may have been deliberately moved around internally, causing more internal damage than the external wound suggests. When assessing a stab wound, it is important for the EMT to remember that: A. stabbings to an extremity are rarely associated with an exit wound. - Rationale: The question did not state that the wound was to an extremity. B. the majority of the internal trauma will be near the path of the knife. - Rationale: This is true, but EMS providers must have a high index of suspicion for extended injuries due to movement. C. most stabbings are unintentional and cause less severe internal injury. - Rationale: Any stabbing that penetrates an individual's skin must be considered severe until ruled out by a hospital physician. D. more internal damage may be present than the external wound suggests. - Rationale: Correct answer

If fertilization has not occurred within about ___ days following ovulation, the lining of the uterus begins to separate and menstruation occurs. A. 8 B. 10 C. 12 D. 14

Answer: D Rationale: Women menstruate about 14 days following ovulation. If fertilization has not occurred within about ___ days following ovulation, the lining of the uterus begins to separate and menstruation occurs. A. 8 - Rationale: Women menstruate about 14 days following ovulation. B. 10 - Rationale: Women menstruate about 14 days following ovulation. C. 12 - Rationale: Women menstruate about 14 days following ovulation. D. 14 - Rationale: Correct answer.

Which of the following questions is of LEAST pertinence for the EMT to ask a patient who intentionally overdosed on a medication? "How much do you weigh?" "How much did you ingest?" "What substance did you take?" "Why did you take the medication?"

Answer: D Rationale: Determining what the patient ingested, how much was ingested, and the patient's weight, are all pertinent and have a direct impact on the care that is provided during the acute phase. Why the patient ingested the medication does not have a direct impact on acute care; therefore, it is the least pertinent question to ask. Which of the following questions is of LEAST pertinence for the EMT to ask a patient who intentionally overdosed on a medication? "How much do you weigh?" Rationale: This is a very pertinent question and can impact treatment. "How much did you ingest?" Rationale: This is a very pertinent question and can impact treatment. Which of the following questions is of LEAST pertinence for the EMT to ask a patient who intentionally overdosed on a medication? "What substance did you take?" Rationale: This is a very pertinent question and can impact treatment. "Why did you take the medication?" ;answer

Which of the following effects does drinking alcohol NOT produce? Induction of sleep Slowing of reflexes Inappropriate behavior Increased sense of awareness

Answer: D Rationale: Drinking alcohol (ethyl alcohol) is both a sedative (decreases activity and excitement) and a hypnotic (induces sleep). It dulls the sense of awareness, slows reflexes, and reduces reaction time. It may also cause aggressive or inappropriate behavior and lack of coordination. Which of the following effects does drinking alcohol NOT produce? Induction of sleep Rationale: It is a hypnotic and induces sleep. Slowing of reflexes Rationale: It is a sedative and reduces reaction time. Which of the following effects does drinking alcohol NOT produce? Inappropriate behavior Rationale: It may cause aggressive or inappropriate behavior. Increased sense of awareness ;answer

A patient with a suspected stroke presents with slurred speech that is difficult for you to understand. This is referred to as: aphasia. dysphasia. dysphagia. dysarthria.

Answer: D Rationale: Dysarthria is defined as slurred, poorly articulated speech; it is common in stroke patients. Dysphasia is defined as difficulty speaking; the patient's speech may or may not be slurred. Aphasia is the inability to speak. Dysphagia is defined as difficulty swallowing. A patient with a suspected stroke presents with slurred speech that is difficult for you to understand. This is referred to as: aphasia. Rationale: Aphasia is the inability to produce or understand speech. dysphasia. Rationale: Dysphasia is difficulty in speaking. A patient with a suspected stroke presents with slurred speech that is difficult for you to understand. This is referred to as: dysphagia. Rationale: Dysphagia is difficulty in swallowing. dysarthria. Rationale: Correct answer

The most common trigger of anaphylaxis is: insect stings. chemicals. medications. food.

Answer: D Rationale: Foods such as shellfish and peanuts may be the most common trigger of anaphylaxis. These foods account for 30% of deaths from anaphylaxis, especially in adolescents and young adults. The most common trigger of anaphylaxis is: plants. Rationale: Although plants can cause a severe anaphylactic reaction, they are a less common trigger compared to food. chemicals. Rationale: While several chemicals can cause a severe anaphylactic reaction, they do not lead to as many reactions as food. The most common trigger of anaphylaxis is: medications. Rationale: Medications are the second most common source of anaphylactic reactions. food. Rationale: Correct answer

A type of seizure that is characterized by severe twitching of all the body's muscles and lasts for several minutes or longer is called a(n): partial seizure. absence seizure. tonic-clonic seizure. generalized seizure.

Answer: D Rationale: Generalized seizures are characterized by generalized severe twitching of all of the body's muscles; they often last for several minutes or longer. An absence seizure is characterized by a blank stare; generalized muscle twitching is absent. A type of seizure that is characterized by severe twitching of all the body's muscles and lasts for several minutes or longer is called a(n): partial seizure. Rationale: A partial seizure is broken down into simple (jerking of one part of the body) and complex (changes in behavior and emotion). absence seizure. Rationale: An absence seizure does not involve any changes in motor activity. A type of seizure that is characterized by severe twitching of all the body's muscles and lasts for several minutes or longer is called a(n): tonic-clonic seizure. Rationale: A tonic-clonic seizure exhibits muscle contraction and incontinence. generalized seizure. Rationale: Correct answer

When dealing with hematologic disorders, the EMT must be familiar with the composition of blood. Which of the following is considered a hematologic disease? Sickle cell disease Hemophilia Lou Gehrig's disease Both A and B

Answer: D Rationale: Hematology is the study and prevention of blood-related diseases, such as sickle cell disease and hemophilia. When dealing with hematologic disorders, the EMT must be familiar with the composition of blood. Which of the following is considered a hematologic disease? Sickle cell disease Rationale: Sickle cell disease is a hematologic disorder affecting the red blood cells. Hemophilia Rationale: Hemophilia is a hematologic disorder affecting the blood's ability to clot. When dealing with hematologic disorders, the EMT must be familiar with the composition of blood. Which of the following is considered a hematologic disease? Lou Gehrig's disease Rationale: Lou Gehrig's disease affects the nerve cells in the brain and spinal cord. Both A and B Rationale: Correct answer

In contrast to a hyperglycemic crisis, a hypoglycemic crisis: rarely presents with seizures. presents over a period of hours to days. should not routinely be treated with glucose. usually responds immediately after treatment.

Answer: D Rationale: Hypoglycemic crisis usually responds immediately following treatment with glucose. Patients with hyperglycemic crisis generally respond to treatment gradually, within 6-12 hours following the appropriate treatment. Seizures can occur with both hyperglycemic crisis and hypoglycemic crisis, but are more common in patients with hypoglycemic crisis. In contrast to a hyperglycemic crisis, a hypoglycemic crisis: rarely presents with seizures. Rationale: Hypoglycemic crisis can produce seizures. presents over a period of hours to days. Rationale: Hypoglycemic crisis has a rapid onset of symptoms (possible minutes). In contrast to a hyperglycemic crisis, a hypoglycemic crisis: should not routinely be treated with glucose. Rationale: Hypoglycemic crisis is always treated with glucose. usually responds immediately after treatment. Rationale: Correct answer

When a psychiatric emergency arises, a patient is most likely to exhibit which of the following behaviors? A. Visual or auditory hallucinations B. Any behaviors unacceptable to the patient, family, or community C. Any behaviors that are a violent threat to the patient, EMTs, or others D. Disruptions to activities of daily living

Any behaviors that are a violent threat to the patient, EMTs, or others

You are called to a neatly kept residence for an 80-year-old woman who lives by herself. She burned her hand on the stove and experienced a full-thickness burn. When treating this patient, it is important to note that: there is a high likelihood that she has been abused. isolated full-thickness burns to the hand are not critical burns. this patient should probably be placed in an assisted-living center. slowing of reflexes causes a delayed pain reaction in older people.

Answer: D Rationale: In older patients, the sense of touch decreases due to a loss of the end-nerve fibers. This loss, in conjunction with slowing of the peripheral nervous system, causes a delayed reaction to pain. In this particular scenario, there is no indication that the patient has been abused. Partial- and full-thickness burns to the hands, feet, face, and genitalia are considered critical burns, regardless of the patient's age. You are called to a neatly kept residence for an 80-year-old woman who lives by herself. She burned her hand on the stove and experienced a full-thickness burn. When treating this patient, it is important to note that: there is a high likelihood that she has been abused. Rationale: There is no indication of abuse in this situation. You are called to a neatly kept residence for an 80-year-old woman who lives by herself. She burned her hand on the stove and experienced a full-thickness burn. When treating this patient, it is important to note that: isolated full-thickness burns to the hand are not critical burns. Rationale: Any full-thickness burns of the hands, face, feet, or genitalia are considered critical. You are called to a neatly kept residence for an 80-year-old woman who lives by herself. She burned her hand on the stove and experienced a full-thickness burn. When treating this patient, it is important to note that: this patient should probably be placed in an assisted-living center. Rationale: This is no indication that the patient cannot take care of herself. slowing of reflexes causes a delayed pain reaction in older people;Rationale: Correct answer

Inflicted bruises are commonly found in all of the following areas, EXCEPT: the buttocks. the lower back. the inner thighs. the forearms.

Answer: D Rationale: Inflicted bruises are typically found on the buttocks and lower back, genitalia and inner thighs, cheek or earlobe, upper lip and inside the mouth, and neck. Bruises to these areas should increase your index of suspicion for abuse. Inflicted bruises are commonly found in all of the following areas, EXCEPT: the buttocks. Rationale: This is an area where bruises are typically inflected. the lower back. Rationale: This is an area where bruises are typically inflected. the inner thighs. Rationale: This is an area where bruises are typically inflected. the forearms Rationale: Correct answer

Which of the following may be difficult to perform on a patient with Down syndrome? CPR Pulse oximetry Splinting Intubation

Answer: D Rationale: Intubation may be difficult because patients with Down syndrome often have large tongues and small oral and nasal cavities. Which of the following may be difficult to perform on a patient with Down syndrome? CPR Rationale: This should not be difficult. Pulse oximetry Rationale: This should not be difficult. Splinting Rationale: This should not be difficult. Intubation Rationale: Correct answer

Reflective listening, an assessment technique used when caring for patients with an emotional crisis, involves: asking the patient to repeat his or her statements. simply listening to the patient, without speaking. asking the patient to repeat everything that you say. repeating, in question form, what the patient tells you.

Answer: D Rationale: Reflective listening—a technique in which you repeat, in question form, what the patient tells you—allows the patient to further expand on his or her thoughts; it also helps the EMT gain insight into the patient's situation. Reflective listening, an assessment technique used when caring for patients with an emotional crisis, involves: asking the patient to repeat his or her statements. Rationale: This is considered to be clarification of a response. simply listening to the patient, without speaking. Rationale: This is considered to be active listening. Reflective listening, an assessment technique used when caring for patients with an emotional crisis, involves: asking the patient to repeat everything that you say. Rationale: Simplify and summarize the patient's response when a patient gives confusing or disorganized responses. repeating, in question form, what the patient tells you. Rationale: Correct answer

The assessment of a patient with a hematologic disorder is the same as it is with all other patients an EMT will encounter. In addition to obtaining a SAMPLE history, EMTs should ask which of the following questions? Have you had a crisis before? When was the last time you had a crisis? How did your crisis resolve? All of the above.

Answer: D Rationale: SAMPLE is the mnemonic used in taking the history of all patients. In addition to asking the SAMPLE, EMTs should also ask about past crises. The assessment of a patient with a hematologic disorder is the same as it is with all other patients an EMT will encounter. In addition to obtaining a SAMPLE history, EMTs should ask which of the following questions? Have you had a crisis before? Rationale: You should ask the patient this question. When was the last time you had a crisis? Rationale: You should ask the patient this question. The assessment of a patient with a hematologic disorder is the same as it is with all other patients an EMT will encounter. In addition to obtaining a SAMPLE history, EMTs should ask which of the following questions? How did your crisis resolve? Rationale: You should ask the patient this question. All of the above. Rationale: Correct answer

MOST patients with an acute abdomen present with: dyspnea. diarrhea. hypotension. tachycardia.

Answer: D Rationale: Tachycardia (heart rate > 100 beats/min) is commonly seen in patients with an acute abdomen; it is usually the result of severe pain. Hypotension is not seen in all patients with an acute abdomen; if the patient is hypotensive, you should suspect internal bleeding or a severe infection (sepsis). Many patients with an acute abdomen have increased respirations (tachypnea); however, dyspnea (a feeling of shortness of breath) is not common. MOST patients with an acute abdomen present with: dyspnea. Rationale: Some patients may have increased respirations, but they typically do not have difficulty breathing. diarrhea. Rationale: Diarrhea may be a symptom of some abdominal problems but not in most patients. MOST patients with an acute abdomen present with: hypotension. Rationale: Hypotension is not seen in most patients; when it is present, shock should be suspected. tachycardia. Rationale: Correct answer

Which of the following injuries would MOST likely occur as a direct result of the third collision in a motor vehicle crash? A. Aortic rupture B. Flail chest C. Extremity fractures D. Forehead lacerations

Aortic rupture

A woman called EMS because her 12-year-old son, who had been experiencing excessive urination, thirst, and hunger for the past 36 hours, has an altered mental status and is breathing fast. You should be MOST suspicious for: low blood sugar. hypoglycemia. hypoglycemic crisis. hyperglycemic crisis.

Answer: D Rationale: The child is experiencing a hyperglycemic crisis secondary to severe hyperglycemia. Hyperglycemic crisis is characterized by a slow onset and excessive urination (polyuria), thirst (polydipsia), and hunger (polyphagia). Other signs include rapid, deep breathing with a fruity or acetone breath odor (Kussmaul respirations); a rapid, thready pulse; and an altered mental status. A woman called EMS because her 12-year-old son, who had been experiencing excessive urination, thirst, and hunger for the past 36 hours, has an altered mental status and is breathing fast. You should be MOST suspicious for: low blood sugar. Rationale: Low blood sugar does not cause frequent urination. hypoglycemia. Rationale: Hypoglycemia is low blood sugar. A woman called EMS because her 12-year-old son, who had been experiencing excessive urination, thirst, and hunger for the past 36 hours, has an altered mental status and is breathing fast. You should be MOST suspicious for: hypoglycemic crisis. Rationale: Hypoglycemic crisis does not produce any of these symptoms. hyperglycemic crisis. Rationale: Correct answer

Which of the following is a hollow organ? Liver Kidney Spleen Gallbladder

Answer: D Rationale: The gallbladder is a hollow organ that concentrates and stores bile, which is produced by the liver. Other hollow organs include the stomach and intestines. The liver, spleen, and kidney are all solid organs. Which of the following is a hollow organ? Liver Rationale: The liver is a solid organ. Kidney Rationale: The kidney is a solid organ. Spleen Rationale: The spleen is a solid organ. Gallbladder Rationale: Correct answer

The LEAST common cause of death in patients over 65 years of age is: stroke. diabetes. heart attack. drug overdose.

Answer: D Rationale: The leading causes of death in patients over 65 years of age are heart disease, diabetes, stroke, cancer, pulmonary diseases, and trauma. Drug overdose—intentional or unintentional—is not a leading cause of death in this age group. The LEAST common cause of death in patients over 65 years of age is: stroke. Rationale: This is one of the common causes of death. diabetes. Rationale: This is one of the common causes of death. heart attack. Rationale: This is one of the common causes of death. drug overdose Rationale: Correct answer

When assessing a patient with a behavioral crisis, your primary concern must be: allowing the patient to express himself or herself to you in his or her own words. setting your personal feelings aside and providing needed care. gathering the patient's belongings and taking them to the hospital. whether the patient will cause harm to you or your partner.

Answer: D Rationale: There are many things that you should be concerned with when assessing a patient with a behavioral crisis, including all of the items listed in this question. Your primary concern, however, must be the safety of yourself and your partner. When assessing a patient with a behavioral crisis, your primary concern must be: allowing the patient to express himself or herself to you in his or her own words. Rationale: This is a good technique to use in assessment. setting your personal feelings aside and providing needed care. Rationale: It is important not to allow your own prejudice to interfere with your treatment of patients. When assessing a patient with a behavioral crisis, your primary concern must be: gathering the patient's belongings and taking them to the hospital. Rationale: Good patient skills are utilized in the treatment of every patient. whether the patient will cause harm to you or your partner. Rationale: Correct answer

An acute ischemic stroke is caused by: a ruptured cerebral artery. increased intracranial pressure. an acute rise in a person's blood pressure. a blocked cerebral artery.

Answer: D Rationale: There are two types of stroke—hemorrhagic and ischemic. A hemorrhagic stroke is caused by a ruptured cerebral artery (aneurysm), which causes bleeding within the brain and increased intracranial pressure. An ischemic stroke is caused by a blocked cerebral artery—either from a clot that grows locally (thrombus) or that travels to the brain from another part of the body (embolus). An acute ischemic stroke is caused by: a ruptured cerebral artery. Rationale: This is known as a hemorrhagic stroke. increased intracranial pressure. Rationale: This can be caused by swelling, bleeding, or tumors. An acute ischemic stroke is caused by: an acute rise in a person's blood pressure. Rationale: This is known as a hypertensive crisis. a blocked cerebral artery. Rationale: Correct answer

What do vagal nerve stimulators do? Keep seizures from occurring Keep the airway clear from secretions Act as an alternative treatment to seizure medicine Both A and C

Answer: D Rationale: Vagal nerve stimulators are an alternative treatment to medication for patients with seizures and keep seizures from occurring. What do vagal nerve stimulators do? Keep seizures from occurring Rationale: This is one of the two correct answers. Keep the airway clear from secretion Rationale: This is not the correct answer. Act as an alternative treatment to seizure medication Rationale: This is one of the two correct answers. Both A and C Rationale: Correct answer

A 13-year-old girl is found floating face down in a swimming pool. Witnesses tell you that the girl had been practicing diving. After you and your partner safely enter the water, you should: turn her head to the side and give five back slaps. turn her head to the side and begin rescue breathing. rotate her entire body as a unit and carefully remove her from the pool. rotate the entire upper half of her body as a unit, supporting her head and neck.

Answer: D Rationale: When caring for a patient who is in the water and has possibly been injured, rotate the upper half of the body as a unit, supporting the head and neck, until the patient is face up. Open the airway with the jaw-thrust maneuver and begin artificial ventilation. A 13-year-old girl is found floating face down in a swimming pool. Witnesses tell you that the girl had been practicing diving. After you and your partner safely enter the water, you should: turn her head to the side and give five back slaps. Rationale: You must consider a spinal injury. turn her head to the side and begin rescue breathing. Rationale: Manual stabilization must occur when treating patients with suspected neck injuries. A 13-year-old girl is found floating face down in a swimming pool. Witnesses tell you that the girl had been practicing diving. After you and your partner safely enter the water, you should: rotate her entire body as a unit and carefully remove her from the pool. Rationale: While in the water and placing a patient in the supine position, a controlled rotation of the upper torso will automatically cause the proper rotation of the lower torso. rotate the entire upper half of her body as a unit, supporting her head and neck. Rationale: Correct answer

A woman has frostbite in both feet after walking several miles in a frozen field. Her feet are white, hard, and cold to the touch. Treatment at the scene should include: rubbing her feet gently with your own warm hands. trying to restore circulation by helping her to walk around. removing her wet clothing and rubbing her feet briskly with a warm, wet cloth. removing her wet clothing and covering her feet with dry, sterile dressings.

Answer: D Rationale: When treating a patient with frostbite, you should remove any wet clothing and cover the injured area with dry, sterile dressings. Do not break any blisters, and do not apply heat to try to rewarm the area. A woman has frostbite in both feet after walking several miles in a frozen field. Her feet are white, hard, and cold to the touch. Treatment at the scene should include: rubbing her feet gently with your own warm hands. Rationale: Do not rub or massage the frostbitten area. trying to restore circulation by helping her to walk around. Rationale: Do not allow the patient to stand or walk on a frostbitten foot. A woman has frostbite in both feet after walking several miles in a frozen field. Her feet are white, hard, and cold to the touch. Treatment at the scene should include: C. removing her wet clothing and rubbing her feet briskly with a warm, wet cloth. Rationale: Do not apply something warm or hot. D. removing her wet clothing and covering her feet with dry, sterile dressings. Rationale: Correct answer

Which of the following can cause vaginal bleeding? A. Ectopic pregnancy. B. Spontaneous abortion. C. Trauma. D. All of the above

Answer: D Rationale: Ectopic pregnancy, spontaneous abortion, and trauma can cause vaginal bleeding and should not be overlooked. Which of the following can cause vaginal bleeding? A. Ectopic pregnancy - Rationale: Ectopic pregnancy can cause vaginal bleeding. B. Spontaneous abortion - Rationale: Spontaneous abortion can cause vaginal bleeding. C. - Trauma D. Rationale: Trauma can cause vaginal bleeding. - None of the above Rationale: Correct answer.

A 54-year-old male accidentally shot himself in the leg while cleaning his gun. Your assessment reveals a small entrance wound to the medial aspect of his right leg. The exit wound is on the opposite side of the leg and is actively bleeding. The patient complains of numbness and tingling in his right foot. You should: A. assess distal pulses as well as sensory and motor functions. B. control the bleeding and cover the wound with a sterile dressing. C. manually stabilize the leg above and below the site of injury. D. gently manipulate the injured leg until the numbness dissipates.

B

A fracture is MOST accurately defined as a(n): A. abnormality in the structure of a bone. B. break in the continuity of the bone. C. total loss of function in a bone. D. disruption in the midshaft of a bone.

B

A patient who is suspected of being hypoxic and is breathing adequately should be given supplemental oxygen with a: A. mouth-to-mask device. B. nonrebreathing mask. C. bag-valve mask. D. nasal cannula.

B

During your assessment of a patient with respiratory distress, you hear wheezing when listening to breath sounds. This indicates: A. fluid in the alveoli. B. a lower airway obstruction. C. secretions in the airway. D. swelling of the upper airway.

B

How does CPAP improve oxygenation and ventilation in patients with certain respiratory problems? A. It decreases intrathoracic pressure, which allows more room for lung expansion. B. It forces the alveoli open and pushes oxygen across the alveolar membrane. C. It prevents alveolar collapse by pushing air into the lungs during inhalation. D. It pushes thick, infected pulmonary secretions into isolated areas of the lung.

B

In which of the following situations should the EMT splint an injured limb in the position of deformity? A. If transport time to the hospital is greater than 20 to 30 minutes B. If resistance is encountered or the patient experiences severe pain C. If a traction splint will be used to immobilize the injured extremity D. When distal circulation and neurological functions are absent

B

The musculoskeletal system refers to the: A. involuntary muscles of the nervous system. B. bones and voluntary muscles of the body. C. nervous system's control over the muscles. D. connective tissue that supports the skeleton.

B

When caring for a patient with a possible fracture of the scapula, the EMT should: A. recognize that scapular fractures are life threatening. B. carefully assess the patient for breathing problems. C. assume that minimal force was applied to the back. D. apply rigid board splints across the chest and back.

B

Which of the following musculoskeletal injuries would MOST likely result in deformity? A. Severe strain B. Displaced fracture C. Moderate sprain D. Hairline fracture

B

Which of the following statements regarding normal gas exchange in the lungs is correct? A. The actual exchange of oxygen and carbon dioxide occurs in the capillaries. B. Oxygen and carbon dioxide diffuse across the alveolar walls and capillaries. C. The oxygen content in the alveoli is highest during the exhalation phase. D. Blood that returns to the lungs from the body has low levels of carbon dioxide.

B

You are performing mouth-to-mask ventilations with oxygen connected and set at a flow rate of 15 L/min. What percentage of oxygen is your patient receiving? A. 75% B. 55% C. 65% D. 45%

B

Which of the following suffixes mean "pertaining to"? Select one: A. "-ic" and "-ology" B. "-al" and "-ic" C. "-al" and "-ology" D. "-ology" and "-oma"

B. "-al" and "-ic"

The meaning "around" can have which of the following prefixes? Select one: A. "sub-" and "infra-" B. "peri-" and "circum-" C. "infra-" and "peri-" D. "epi-" and "sub-"

B. "peri-" and "circum-"

Trench collapses usually involve large areas of falling dirt that weigh approximately _______ per cubic foot. Select one: A. 50 lb B. 100 lb C. 150 lb D. 200 lb

B. 100 lb

While the ages can vary, women typically experience menstruation from approximately _________ to ____________ years of age. A. 10; 70 B. 11; 50 C. 20; 60 D. 10; 40

B. 11; 50

If a baby is born at 7:52, the second Apgar score should be calculated at: A. 7:53. B. 7:57. C. 7:59. D. 8:00.

B. 7:57.

You are triaging four patients who were involved in a head-on motor vehicle crash. Which of the following patients should be assigned the highest (red) triage category? Select one: A. A 50-year-old male with an open head injury and no pulse B. A 49-year-old female with diabetes and difficulty breathing C. A 36-year-old female with back pain and numb extremities D. A 29-year-old male with bilaterally closed femur deformities

B. A 49-year old female with diabetes and difficulty breathing.

Which of the following descriptions MOST accurately portrays emergency medical services (EMS)? Select one: A. A system composed exclusively of emergency medical responders (EMRs) and emergency medical technicians (EMTs) who are responsible for providing care to sick and injured patients B. A team of health care professionals who are responsible for providing emergency care and transportation to the sick and injured C. A team of paramedics and emergency physicians who are responsible for providing emergency care to critically injured patients D. A vast network of advanced life support (ALS) providers who provide definitive emergency care in the prehospital setting

B. A team of health care professionals who are responsible for providing emergency care and transportation to the sick and injured

While treating a patient complaining of severe difficulty breathing, the patient asks for your help taking his inhaler. You search through his plastic bag of medications, looking for an inhaler marked with any of the following medication names EXCEPT: A. albuterol B. Advair C. Ventolin D. Alupent

B. Advair (used for COPD patients)

Which of the following is a component of the Cincinnati Prehospital Stroke Scale? A. Blood pressure B. Arm drift C. Family history D. Grip strength

B. Arm drift

Which of the following sets of vital signs would the EMT MOST likely encounter in a patient with acute cocaine overdose? Select one: A. BP, 180/100 mm Hg; pulse, 50 beats/min B. BP, 200/100 mm Hg; pulse, 150 beats/min C. BP, 190/90 mm Hg; pulse, 40 beats/min D. BP, 60/40 mm Hg; pulse, 140 beats/min

B. BP, 200/100 mm Hg; pulse, 150 beats/min

A 5-year-old child has had severe vomiting and diarrhea for 4 days. Which of the following assessment findings would be the MOST indicative of decompensated shock? A. Pulse rate greater than 120/min B. Blood pressure of 70/40 mm Hg C. Capillary refill time of 4 seconds D. Respiratory rate of 30 breaths/min

B. Blood pressure of 70/40 mm Hg

How do poisons typically act to harm the body? Select one: A. By causing a slowing of nearly all bodily functions B. By changing the normal metabolism of cells or by destroying them C. By interfering with normal neurologic function D. By causing burns and damage to either external or internal organs

B. By changing the normal metabolism of cells or by destroying them

The type _____ ambulance is a standard van with a forward-control integral cab body. Select one: A. I B. II C. III D. IV

B. II

Compression injury is most likely due to which of the following? A. Stabbing B. Improperly placed lab belt C. Ejection of unrestrained driver D. Hollow-organ rupture

B. Improperly placed lab belt

Which of the following statements regarding sickle cell disease is correct? Select one: A. Because of their abnormal shape, red blood cells in patients with sickle cell disease are less apt to lodge in a blood vessel. B. In sickle cell disease, the red blood cells are abnormally shaped and are less able to carry oxygen. C. The red blood cells of patients with sickle cell disease are round and contain hemoglobin. D. Sickle cell disease is an inherited blood disorder that causes the blood to clot too quickly.

B. In sickle cell disease, the red blood cells are abnormally shaped and are less able to carry oxygen.

Which of the following is considered a priority when determining what needs to be done during the scene size-up? Select one: A. Rescue operations B. Incident stabilization C. Notifying hospitals D. Establishing operations

B. Incident stabilization

Which of the following is true regarding injury to the kidneys? A. The kidneys are not well protected. B. Injury to the kidneys usually indicates injury to other organs. C. Kidney injuries are rarely caused by blunt trauma. D. Only minimal force is needed to damage the kidneys.

B. Injury to the kidneys usually indicates injury to other organs.

When should you visually inspect the external genitalia on your patient? A. Only when ordered by medical direction B. Only when there is a complaint of severe pain or other injury C. Always during the secondary assessment D. Anytime the patient agrees to treatment and transport

B. Only when there is a complaint of severe pain or other injury

Which of the following statements regarding patients with intellectual disabilities is correct? A. Speaking with the patient's family is the least effective way to determine how much the patient understands. B. Patients with intellectual disabilities are susceptible to the same disease processes as other patients. C. An intellectual disability differs from mental retardation in that it is the result of a congenital abnormality. D. Most patients with intellectual disabilities have normal cognitive function, but abnormal physical features.

B. Patients with intellectual disabilities are susceptible to the same disease processes as other patients

Patient complaints of pain during intercourse, lower abdominal discomfort, and nausea may be indicative of which of the following? A. Bacterial vaginosis B. Pelvic inflammatory disease C. Chlamydia D. Gonorrhea

B. Pelvic inflammatory disease

You are caring for a confused and restless 16-year-old female with blunt chest trauma following a single vehicle accident. She complains of dyspnea and orthopnea. You note a reduced tidal volume, cyanosis and tachycardia. Which of the following should be done first? A. Begin positive pressure ventilations. B. Perform a rapid scan. C. Perform a secondary exam. D. Obtain a pulse oximeter (SaO2) reading.

B. Perform a rapid scan.

In which of the following situations would the EMTs MOST likely utilize a police escort? Select one: A. The weather is treacherous and there are numerous roads washed out. B. The EMTs are unfamiliar with the location, but the police officer knows the area. C. The EMTs are transporting a critical pediatric patient through traffic. D. The call is dispatched as an unresponsive patient with CPR in progress.

B. The EMTs are unfamiliar with the location, but the police officer knows the area.

Which of the following statements regarding the NREMT is correct? Select one: A. EMS training standards are regulated by the NREMT. B. The NREMT provides a national standard for EMS testing. C. The NREMT is the exclusive certifying body for EMTs. D. The NREMT is a governmental agency that certifies EMTs.

B. The NREMT provides a national standard for EMS testing.

Which of the following statements regarding the Salmonella bacterium is correct? Select one: A. The Salmonella bacterium produces toxins that cause food poisoning. B. The Salmonella bacterium itself causes food poisoning. C. Symptoms of salmonellosis appear within 12 hours. D. Refrigeration of food will prevent salmonellosis.

B. The Salmonella bacterium itself causes food poisoning.

Which of the following statements regarding glucose is correct? Select one: A. The brain requires insulin to allow glucose to enter the cells. B. The brain requires glucose as much as it requires oxygen. C. Blood glucose levels decrease in the absence of insulin. D. Most cells will function normally without glucose.

B. The brain requires glucose as much as it requires oxygen.

In which of the following circumstances can the EMT legally release confidential patient information? Select one: A. A police officer requests a copy to place on file B. The patient is competent and signs a release form C. A media representative inquires about the patient D. The family requests a copy for insurance purposes Feedback

B. The patient is competent and signs a release form

Which of the following observations or statements represents the "E" in the GEMS diamond? Select one: A. A patient is assisted with his or her activities of daily living. B. The patient's residence is cold due to a malfunctioning heater. C. Elderly patients present atypically and deserve your respect. D. The patient's medications have not been filled in 2 months.

B. The patient's residence is cold due to a malfunctioning heater.

Why do middle adults commonly experience financial concerns? Select one: A. The majority of middle adults still have small children who live at home with them. B. They are preparing for retirement but must still manage everyday financial demands. C. They are typically receiving social security and must budget with a fixed income. D. Most people in the middle adult age group have chronic illnesses and cannot work.

B. They are preparing for retirement but must still manage everyday financial demands.

You are caring for an unresponsive diabetic patient with deep, rapid respirations. The patient's blood glucose is 480 mg/dL. What is the most likely cause of this patient's hyperventilations? A. This patient is likely hyperventilating due to severe alkalosis. B. This patient is likely hyperventilating due to severe acidosis. C. This patient's hyperventilations will increase CO2 levels in the blood. D. This patient's hyperventilations will rapidly lower the blood glucose level.

B. This patient is likely hyperventilating due to severe acidosis. hyperventilations will help reduce CO2 levels, especially because acidosis is buildup of CO2

Victims of inhaled poisoning will require which of the following? Select one: A. Suctioning to clear the airway B. Transport to an emergency department for evaluation C. Ventilation with a bag-valve mask D. An oropharyngeal or nasopharyngeal device to maintain an airway

B. Transport to an emergency department for evaluation

When is forcible restraint permitted? Select one: A. Only if consent to restrain is given by a family member B. When the patient poses a significant threat to self or others C. Only if law enforcement personnel have witnessed threatening behavior D. Anytime that the EMT feels threatened

B. When the patient poses a significant threat to self or others

Which of the following questions would be LEAST pertinent during the initial questioning of a patient who ingested a substance? Select one: A. How long ago was the substance taken? B. Why was the substance ingested? C. What type of substance was taken? D. How much of the substance was taken?

B. Why was the substance ingested?

A patient with pelvic inflammatory disease will typically complain of _________. A. nausea and vomiting associated with intercourse B. abdominal pain associated with menstruation C. aches and fever associated with urination D. bleeding associated with stress

B. abdominal pain associated with menstruation

Equipment and supplies that are carried on an ambulance should be stored: Select one: A. as directed by the EMS system's medical director. B. according to the urgency and frequency of their use. C. based on recommendations of the health department. D. in locked or secured cabinets in order to prevent theft.

B. according to the urgency and frequency of their use.

You have just delivered a major trauma patient to the hospital. Shortly after departing the hospital, dispatch advises you of another call. The back of the ambulance is contaminated with bloody dressings and is in disarray, and you are in need of airway equipment and numerous other supplies. You should: Select one: A. proceed to the call, functioning only as an emergency medical responder. B. advise the dispatcher that you are out of service and to send another unit. C. have your partner quickly clean the ambulance as you proceed to the call. D. quickly proceed to the call and clean and restock the ambulance afterwards.

B. advise the dispatcher that you are out of service and to send another unit.

According to the Emergency Medical Treatment and Active Labor Act (EMTALA): A. a health care facility has the right to refuse assessment and treatment to a patient, but only if his or her condition is not deemed critical. B. all health care facilities must provide a medical assessment and required treatment, regardless of the patient's ability to pay. C. a patient maintains the legal right to recant his or her consent to emergency treatment, even after signing in to the emergency department. D. all health care facilities are legally obligated to provide assessment and care only if the patient is critically ill or injured.

B. all health care facilities must provide a medical assessment and required treatment, regardless of the patient's ability to pay.

Between each chest compression, you should __________. Select one: A. check for a pulse B. allow full chest recoil C. administer a breath D. remove your hands from the chest

B. allow full chest recoil

During your assessment of a 20-year-old man with a severe headache and nausea, you ask him when his headache began, but he does not answer your question immediately. You should: Select one: A. ask him if he frequently experiences severe headaches and nausea. B. allow him time to think about the question and respond to it. C. tell him that you cannot help him unless he answers your questions. D. repeat your question because he probably did not hear you.

B. allow him time to think about the question and respond to it.

The most reliable indicator of the severity of an allergic reaction is the: A. amount of antigen to which the patient was exposed B. amount of time from exposure to onset of symptoms C. route of exposure D. body surface area affected by rash

B. amount of time from exposure to onset of symptoms

Points of distribution (PODs) are strategically placed facilities where: A. chemical and biologic weapons are stockpiled. B. antidotes, antibiotics, and vaccines are distributed. C. weapons of mass destruction are distributed. D. chemical weapons are manufactured and distributed.

B. antidotes, antibiotics, and vaccines are distributed. Correct

A 60-year-old man is found to be unresponsive, pulseless, and apneic. You should: Select one: A. start CPR and transport immediately. B. begin CPR until an AED is available. C. withhold CPR until he is defibrillated. D. determine if he has a valid living will.

B. begin CPR until an AED is available.

You are attempting to gain access to a patient who was injured when his truck struck another vehicle from behind. The patient is conscious and alert, but is screaming in pain. You try to open the door, but it is locked. You should: Select one: A. break the window and unlock the door. B. ask the patient if he can unlock the door. C. request the rescue team to extricate him. D. use a pry bar to attempt to open the door.

B. ask the patient if he can unlock the door.

An elderly patient reportedly collapsed on a golf course almost 10 in ago. You assess her LOC and determine she is unresponsive. Which of the following should you do first? A. open airway and insert an OPA B. assess pulse and begin CPR if needed C. providing ventilations with high flow oxygen D. attach AED

B. assess pulse and begin CPR if needed if unresponsive, do circulation (where CPR would be found) first

While triaging patients at the scene of a building collapse, you encounter a young child who is conscious, alert, and breathing; has bilateral radial pulses; and has a severely angulated leg, which is not bleeding. According to the JumpSTART triage system, you should: Select one: A. quickly bind his legs together to stabilize the fracture, and continue triaging. B. assign him a delayed (yellow) category and continue triaging the other patients. C. evacuate him to a designated area and assign him a minimal (green) category. D. apply high-flow oxygen, obtain baseline vital signs, and continue triaging.

B. assign him a delayed (yellow) category and continue triaging the other patients.

Your 67-year-old patient recently began taking a new medication. He is complaining of a sudden onset of severe respiratory distress. He quickly becomes semi-conscious and unable to follow verbal commands. His ox sat is 89%. You should: A. insert an oropharyngeal airway B. assist ventilations with a BVM C. apply a continuous positive airway pressure (CPAP) device. D. apply high-flow oxygen via NRB.

B. assist ventilations with a BVM

Unless he or she is critically ill or injured, you should generally begin your assessment of a toddler: A. en route to the hospital. B. at the feet. C. at the head. D. in the ambulance.

B. at the feet.

Because of the complexity of the older patient and the vagueness of his or her complaint, you should: Select one: A. rely exclusively on family members for the medical history. B. attempt to differentiate between chronic and acute problems. C. perform a rapid assessment on all geriatric patients you treat. D. limit your physical examination to the area of pain or injury.

B. attempt to differentiate between chronic and acute problems.

As you enter the residence of a patient who has possibly overdosed, you should: Select one: A. quickly gain access to the patient. B. be alert for personal hazards. C. look for drug paraphernalia. D. observe the scene for drug bottles.

B. be alert for personal hazards.

Your documentation on a sexual assault victim should _______. A. describe the status of the suspect(s) B. be objective and factual C. be subjective and summarize the crime D. include your opinion of the nature of the incident

B. be objective and factual

All of the following are vesicant agents, EXCEPT: A. phosgene oxime. B. sarin. C. lewisite. D. sulfur mustard

B. sarin. Correct

You respond to the home of a 59-year-old man who is unconscious; has slow, shallow breathing; and has a weak pulse. The family states that the patient has terminal brain cancer and does not wish to be resuscitated. They further state that there is a DNR order for this patient, but they are unable to locate it. You should: Select one: A. honor the patient's wishes and withhold all treatment. B. begin treatment and contact medical control as needed. C. transport the patient without providing any treatment. D. decide on further action once the DNR order is produced.

B. begin treatment and contact medical control as needed.

Activated charcoal is given to patients who have ingested certain substances because it: Select one: A. decreases absorption of poisons into the lungs. B. binds to the substance and prevents absorption. C. is a direct antidote for many toxic substances. D. induces vomiting and empties the stomach.

B. binds to the substance and prevents absorption.

Pale skin in a child indicates that the: A. oxygen content in the blood is decreased. B. blood vessels near the skin are constricted. C. child's core body temperature is elevated. D. child is in severe decompensated shock.

B. blood vessels near the skin are constricted.

Cerebral palsy is characterized by poorly controlled ________ movement. A. extremity B. body C. eye D. neck

B. body

When relaying patient information via radio, communications should be: Select one: A. coded and scripted. B. brief and easily understood. C. lengthy and complete. D. spoken in a loud voice.

B. brief and easily understood.

Clouding of the lenses of the eyes is called: Select one: A. retinitis. B. cataracts. C. conjunctivitis. D. glaucoma.

B. cataracts.

If a technical rescue team is required at the scene, but is not present when you arrive, you should: Select one: A. don personal protective equipment and begin the rescue process. B. check with the incident commander to ensure that the team is en route. C. remain with your ambulance until the rescue team arrives at the scene. D. have fire personnel initiate the rescue process if they are at the scene.

B. check with the incident commander to ensure that the team is en route.

The EMT should assess for hypoglycemia in small children with a severe illness or injury because: Select one: A. children overproduce insulin during severe illness or injury. B. children cannot store excess glucose as effectively as adults. C. a child's cells do not uptake glucose as rapidly as adults' do. D. illness or injury causes the pancreas to produce less insulin.

B. children cannot store excess glucose as effectively as adults.

A surgical procedure that creates an opening between the intestine and the surface of the body that allows for elimination of waste products is called a(n): A. gastric stoma. B. colostomy. - C. gastrostomy. D. intestinal shunt.

B. colostomy.

A 2-year-old female has experienced a seizure. When you arrive at the scene, the child is conscious, crying, and clinging to her mother. Her skin is hot and moist. The mother tells you that the seizure lasted approximately 5 minutes. She further tells you that her daughter has no history of seizures, but has had a recent ear infection. You should: A. suspect that the child has meningitis and transport at once. B. cool the child with tepid water and transport to the hospital. C. place the child in cold water to attempt to reduce her fever. D. allow the mother to drive her daughter to the hospital.

B. cool the child with tepid water and transport to the hospital.

Putrefaction is defined as: Select one: A. separation of the torso from the rest of the body. B. decomposition of the body's tissues. C. profound cyanosis to the trunk and face. D. blood settling to the lowest point of the body.

B. decomposition of the body's tissues.

A 49-year-old male presents with confusion, sweating, and visual hallucinations. The patient's wife tells you that he is a heavy drinker and she thinks he had a seizure shortly before your arrival. This patient is MOST likely experiencing: Select one: A. acute schizophrenia. B. delirium tremens. C. alcohol intoxication. D. acute hypovolemia.

B. delirium tremens.

Difficulty breathing and a sunken appearance of the anterior abdominal wall is MOST indicative of a ruptured: A. spleen. B. diaphragm. C. aorta. D. stomach.

B. diaphragm.

The _____________ connect(s) each ovary with the uterus. A. abdomen B. fallopian tubes C. cervix D. egg

B. fallopian tubes

The means by which a terrorist will spread a particular agent is called: A. incubation. B. dissemination. C. weaponization. D. aerosolization

B. dissemination. Correct

A 5-year-old boy has fallen and has a severe deformity of the forearm near the wrist. He has possibly sustained a fracture of the ____________ forearm. Select one: A. proximal B. distal C. dorsal D. superior

B. distal

The Centennial Park bombing during the 1996 Summer Olympics is an example of: A. state-sponsored terrorism. B. domestic terrorism. C. an ecoterrorist attack. D. apocalyptic violence.

B. domestic terrorism. Correct

Your conscious patient has a mild partial airway obstruction. You should: Select one: A. place the patient supine. B. encourage the patient to cough. C. administer back blows. D. perform abdominal thrusts.

B. encourage the patient to cough.

When performing his or her duties, the EMT is generally expected to: Select one: A. contact medical control on every EMS call. B. exercise reasonable care and act prudently. C. function above his or her scope of practice. D. consistently exceed the standard of care.

B. exercise reasonable care and act prudently.

To date, the preferred weapons of mass destruction for terrorists have been: A. chemical weapons. B. explosive weapons. C. biologic weapons. D. nuclear weapons.

B. explosive weapons. Correct

Following delivery of a full-term baby, you have properly cared for the baby and have clamped and cut the umbilical cord. During transport, you note that the mother is experiencing moderate vaginal bleeding. You should: A. carefully insert a sterile trauma dressing into her vagina. B. firmly massage the uterine fundus with a circular motion. C. place her legs together and position her on her left side. D. elevate her legs 6 to 8 inches and cover her with a blanket.

B. firmly massage the uterine fundus with a circular motion.

Bile is produced by the liver and concentrated and stored in the: A. pancreas. B. gallbladder. C. stomach. D. kidneys

B. gallbladder. Correct

Complications associated with chest compressions include all of the following, EXCEPT: Select one: A. a fractured sternum. B. gastric distention. C. rib fractures. D. liver laceration.

B. gastric distention.

Common safety equipment carried on the ambulance includes all of the following, EXCEPT: Select one: A. face shields. B. hazardous materials gear. C. safety goggles. D. turnout gear.

B. hazardous materials gear.

The leading cause of death in the geriatric patient is: Select one: A. hypertension. B. heart disease. C. arthritis. D. altered mental status.

B. heart disease.

Common causes of seizures in children include all of the following, EXCEPT: A. electrolyte imbalances. B. hyperglycemia. C. infection. D. poisonings or ingestion.

B. hyperglycemia.

You are assessing a 70-year-old female who complains of intense thirst, frequent urination, and dizziness. She has a history of type 2 diabetes, heart disease, rheumatoid arthritis, and gout. Her blood glucose reads "high." She is conscious, but confused. Her blood pressure is 92/52 mm Hg, her pulse rate is 130 beats/min and weak, and her respirations are 22 breaths/min and shallow. This patient's clinical presentation is MOST consistent with: Select one: A. diabetic ketoacidosis. B. hyperosmolar hyperglycemic nonketotic syndrome. C. hyperglycemia with moderate dehydration. D. acute renal failure with associated hyperglycemia.

B. hyperosmolar hyperglycemic nonketotic syndrome.

A nuchal cord is defined as an umbilical cord that: A. is lacerated due to a traumatic delivery. B. is wrapped around the baby's neck. C. has abnormally developed blood vessels. D. has separated from the placenta.

B. is wrapped around the baby's neck.

As you and your partner report for duty, you check your ambulance and begin talking about the possibility of a terrorist attack. The MOST effective and appropriate way to determine the likelihood of this happening is to: A. ask your immediate supervisor if he or she has been watching the local news. B. know the current threat level issued by the Department of Homeland Security. C. ascertain the current situation overseas with regard to the number of casualties. D. check with local businesses to see if they have received any terrorist threats.

B. know the current threat level issued by the Department of Homeland Security. Correct

You are treating a patient with respiratory distress. He has a history of multiple myocardial infarctions. Lung sounds indicate pulmonary edema. You note the patient does NOT have any JVD or pedal edema. The patient is most likely experiencing: A. angina B. left heart failure C. right heart failure D. asthma

B. left heart failure pulmonary edema --> left heart failure JVD/pedal edema --> right heart failure

You are dispatched for a homeless person found unresponsive. You find the patient prone on the sidewalk unresponsive to pain. You should first: A. assess the blood glucose level. B. log roll the patient into a supine position. C. auscultate lung sounds. D. expose the patient and check the back for apparent injuries

B. log roll the patient into a supine position. need to do CPR if its indicated

You are ventilating a patient with a stoma. You note air is escaping from the mouth and nose with each breath. You should: A. perform a jaw thrust maneuver. B. manually seal the mouth and nose. C. open the airway using a head tilt-chin lift. D. reduce the tidal volume of each ventilation.

B. manually seal the mouth and nose. reducing tidal volume will lead to inadequate ventilations

As a triage supervisor, you: Select one: A. must prepare patients for transport before they leave the triage area. B. must not begin treatment until all patients have been triaged. C. are responsible for providing initial treatment to all patients. D. should communicate with area hospitals regarding their capabilities.

B. must not begin treatment until all patients hve been triaged.

Basic life support (BLS) is defined as: Select one: A. basic lifesaving treatment that is performed by bystanders while EMS providers are en route to the scene of an emergency. B. noninvasive emergency care that is used to treat conditions such as airway obstruction, respiratory arrest, and cardiac arrest. C. invasive emergency medical interventions such as intravenous therapy, manual defibrillation, and advanced airway management. D. any form of emergency medical treatment that is performed by advanced EMTs, paramedics, physicians, and emergency nurses.

B. noninvasive emergency care that is used to treat conditions such as airway obstruction, respiratory arrest, and cardiac arrest.

The respiratory rate of 30 breaths per minute in an infant is ________. Select one: A. too fast B. normal C. too shallow D. too slow

B. normal

Immediately upon arriving at the scene of an emergency call involving a traumatic injury, you should notify the dispatcher of your arrival and then: Select one: A. quickly gain access to the patient. B. observe the scene for safety hazards. C. determine if additional units are needed. D. carefully assess the mechanism of injury.

B. observe the scene for safety hazards.

You respond to a nursing home for a call about a patient with a history of COPD. You are told that the patient complained of difficulty breathing, so oxygen was applied at 2 LPM by a simple face mask. After several minutes, the patient became drowsy and then unconscious. As you assess the patient, you find him barely breathing and being assisted ventilations with a BVM. After a few minutes, the patient improves and begins waking up. Why did the patient rapidly deteriorate after oxygen was applied? A. patient was give too much oxygen B. patient was allowed to rebreathe carbon dioxide C. the patient could not tolerate the face mask D. none of the above

B. patient was allowed to rebreathe carbon dioxide

Once entrance and access to the patient have been provided, you should: Select one: A. allow extrication to commence. B. perform a primary assessment. C. administer high-flow oxygen. D. begin treating his or her injuries.

B. perform a primary assessment.

When a hollow organ is punctured during a penetrating injury to the abdomen: A. it commonly protrudes through the injury site. B. peritonitis may not develop for several hours. C. it will bleed profusely and rapidly cause shock. D. the abdomen will become instantly distended.

B. peritonitis may not develop for several hours.

The EMT's first priority is __________. Select one: A. rapid response B. personal safety C. treatment and transport D. empathy for all patients Feedback

B. personal safety

Excessive eating caused by cellular "hunger" is called: Select one: A. dyspepsia. B. polyphagia. C. dysphasia. D. polydipsia.

B. polyphagia.

You are considering administration of oral glucose to your diabetic patient. this drug contraindicated when it: A. is not prescribed to the patient B. poses a significant risk of harm C. produces undesired effects D. is likely to relieve the patient's symptoms

B. poses a significant risk of harm oral glucose doesn't need a prescription

The purpose of a ventricular peritoneum shunt is to: A. monitor pressure within the skull in patients with a head injury. B. prevent excess cerebrospinal fluid from accumulating in the brain. C. remove fluid from the abdomen of patients with right-sided heart failure. D. divert excess cerebrospinal fluid to the ventricles of the brain.

B. prevent excess cerebrospinal fluid from accumulating in the brain.

CPR will NOT be effective if the patient is: Select one: A. supine. B. prone. C. horizontal. D. on a firm surface.

B. prone.

After assessing a patient's blood glucose level, you accidentally stick yourself with the contaminated lancet. You should: Select one: A. get immunized against hepatitis as soon as possible. B. report the incident to your supervisor after the call. C. discontinue patient care and seek medical attention. D. immerse your wound in an alcohol-based solution.

B. report the incident to your supervisor after the call

As you are wheeling your patient through the emergency department doors, you receive another call for a major motor vehicle crash. You should: Select one: A. inform the admissions clerk of the situation and then respond at once. B. respond only after giving a verbal patient report to a nurse or physician. C. leave a copy of the run form with a nurse and then respond to the call. D. place the patient in a high-visibility area and then respond to the call.

B. respond only after giving a verbal patient report to a nurse or physician.

In an apparent suicide attempt, a 19-year-old female ingested a full bottle of amitriptyline (Elavil). At present, she is conscious and alert and states that she swallowed the pills approximately 30 minutes earlier. Her blood pressure is 90/50 mm Hg, her pulse is 140 beats/min and irregular, and her respirations are 22 breaths/min with adequate depth. When transporting this patient, you should be MOST alert for: Select one: A. acute respiratory depression. B. seizures and cardiac arrhythmias. C. an increase in her blood pressure. D. a sudden outburst of violence.

B. seizures and cardiac arrhythmias.

You could be sued for ___________ if your radio report to the hospital describes the patient in a manner that injures his or her reputation. Select one: A. assault B. slander C. libel D. negligence

B. slander

You are dispatched to a call for a 4-month-old infant with respiratory distress. While you prepare to take care of this child, you must remember that: Select one: A. assisted ventilations in infants often need to be forceful to inflate their lungs. B. small infants are nose breathers and require clear nasal passages at all times. C. an infant's head should be placed in a flexed position to prevent obstruction. D. the infant's proportionately small tongue often causes an airway obstruction.

B. small infants are nose breathers and require clear nasal passages at all times.

You respond to a call about a stabbing and find your male patient sitting on a bar stool with a broken beer bottle implied in his anterior thigh. This is your patient's only injury, and the bleeding appears minimal. You should: A. remove the object, apply pressure, and dress/bandage the would B. stabilize the object in place with bulky dressings C. apply direct pressure until bleeding stops, and remove object D. none of the above

B. stabilize the object in place with bulky dressings

Symptomatic hypoglycemia will MOST likely develop if a patient: Select one: A. markedly overeats and misses an insulin dose. B. takes too much of his or her prescribed insulin. C. misses one or two prescribed insulin injections. D. eats a regular meal followed by mild exertion.

B. takes too much of his or her prescribed insulin.

You are transporting to a 66-year-old patient with a history of heart problems. The patient chart indicates that he has hepatomegaly. The root of the term indicates that: Select one: A. the problem is unrelated to the patient's heart condition. B. the problem is with the patient's liver. C. the organ involved is enlarged. D. the problem is a direct consequence of the patient's heart disease.

B. the problem is with the patient's liver.

After primary triage, the triage supervisor should communicate all of the following information to the medical branch officer, EXCEPT: Select one: A. the total number of patients that have been triaged. B. the recommended transport destination for each patient. C. recommendations for movement to the treatment area. D. the number of patients in each triage category.

B. the recommended transport destination for each patient.

n determining the potential for a terrorist attack, you should routinely observe all of the following on every call, EXCEPT: A. victim's statements. B. weather conditions. C. the location type. D. the type of call.

B. weather conditions. Correct

When performing CPR on a child, you should compress the chest: Select one: A. to a depth of 1 to 2 inches. B. with one or two hands. C. until a radial pulse is felt. D. 80 to 100 times per minute.

B. with one or two hands.

You respond to a wooded area to help search for a child who has been missing for approximately 24 hours. Which of the following equipment should you leave in the ambulance? Radio Flashlight Jump kit Backboard

Backboard

In order for sweating to be effective it must

Be able to evaporate from the body

In general, musculoskeletal injuries should be splinted before moving the patient unless: A. the patient is in severe pain. B. transport time is less than 15 minutes. C. the patient is clinically unstable. D. deformity and swelling are present.

C

Which of the following patients is breathing adequately? A. A conscious female with facial cyanosis and rapid, shallow respirations B. A conscious male with respirations of 18 breaths/min and reduced tidal volume C. A conscious male with respirations of 19 breaths/min and pink skin D. An unconscious 52-year-old female with snoring respirations and cool, pale skin

C

Which of the following statements regarding compartment syndrome is correct? A. Most cases of compartment syndrome occur following a severe fracture of the pelvis. B. In most cases, compartment syndrome develops within a few minutes after an injury. C. Compartment syndrome typically develops within 6 to 12 hours after an injury. D. Compartment syndrome occurs because of increased pressure within the bone cavity.

C

You are ventilating a patient with a stoma, however, air is escaping from the mouth and nose. To prevent this, you should: A. thrust the jaw forward. B. ventilate with less pressure. C. seal the mouth and nose. D. thoroughly suction the stoma.

C

Which of the following statements is NOT appropriate to document in the narrative section of a PCR? Select one: A. "Significant damage was noted to the front end of the vehicle." B. "After oxygen was administered, the patient's breathing improved." C. "General impression revealed that the patient was intoxicated." D. "The patient admits to smoking marijuana earlier in the day."

C. "General impression revealed that the patient was intoxicated."

What is the normal respiratory rate for an adult? Select one: A. 40-60 breaths/min B. 25-50 breaths/min C. 12-20 breaths/min D. 15-30 breaths/min

C. 12-20 breaths/min

A normal systolic blood pressure for a 30-year-old is between: Select one: A. 60 and 120 mm Hg. B. 80 and 120 mm Hg. C. 90 and 140 mm Hg. D. 70 and 140 mm Hg.

C. 90 and 140 mm Hg.

Which of the following scenarios MOST accurately depicts a posttraumatic stress disorder (PTSD) reaction? Select one: A. An EMT with many years of field experience becomes irritable with her coworkers and experiences headaches and insomnia. B. An EMT is emotionally exhausted and depressed after a school bus crash involving critical injuries and the death of several children. C. An EMT becomes distracted at the scene of a motor vehicle crash involving the same type of car in which a child was previously killed. D. A newly certified EMT becomes extremely nauseated and diaphoretic at the scene of an incident involving grotesque injuries.

C. An EMT becomes distracted at the scene of a motor vehicle crash involving the same type of car in which a child was previously killed.

During your visual inspection of a 25-year-old woman in labor, you see the baby's head crowning at the vaginal opening. What should you do? A. Tell the mother not to push and transport her immediately. B. Maintain firm pressure to the head until it completely delivers. C. Apply gentle pressure to the baby's head as it delivers. D. Place your fingers in the vagina to assess for a nuchal cord.

C. Apply gentle pressure to the baby's head as it delivers.

Under what circumstances is a left ventricular assist device used? A. To ensure that the ventricles contract at an adequate and consistent rate B. To permanently replace the function of one or both of the ventricles C. As a bridge to heart transplantation while a donor heart is being located D. To reduce ventricular pumping force in patients with aortic aneurysms

C. As a bridge to heart transplantation while a donor heart is being located

You are requested out to County Road 93 for a vehicle collision at a rural area known for serious crashes. After driving with lights and sirens for nearly 20 minutes to reach the scene, you arrive at the intersection at the east end of the county. As you pull up, you see two pickup trucks crushed into a mass of twisted, smoking metal. A sheriff's deputy is shouting and waving you over to the passenger side door of one of the demolished trucks. You quickly look down all four roads leading to the scene and note that they are deserted as far as you can see. How would you ensure the proper control of traffic around this scene? Select one: A. Put out flares in a pattern that leads other vehicles safely around the involved vehicles. B. Because the roads were deserted when you arrived, it is not a priority. C. Ask the law enforcement officer to control any traffic. D. Pull completely off the roadway and leave your red emergency lights flashing.

C. Ask the law enforcement officer to control any traffic.

According to the START triage system, what should you do if a patient is found to have a respiratory rate of 24 breaths/min? Select one: A. Administer high-flow oxygen. B. Triage the patient as delayed. C. Assess for bilateral radial pulses. D. Assess his or her neurologic status. Next

C. Assess for bilateral radial pulses.

Select the patient condition that is most likely to require transport to a specialized facility. A. Dyspnea. B. Diabetic emergency. C. Cerebrovascular accident. D. Syncopal episode.

C. Cerebrovascular accident. syncopal --> sudden LOC due to drop in BP

Which of the following conditions would MOST likely lead to pelvic inflammatory disease if left untreated? A. Ovarian cysts B. Genital herpes C. Chlamydia D. Ectopic pregnancy

C. Chlamydia

Which of the following systems is responsible for releasing hormones that regulate body activities? A. Nervous B. Reproductive C. Endocrine D. Skeletal

C. Endocrine Correct

American Heart Association protocols are based on __________. Select one: A. theories ready for field trial B. trends in the EMS community C. evidence-based research D. opinions from a core group of cardiologists

C. Evidence-based research

You are approaching an overturned tanker truck to assess the driver, who appears to be unresponsive. As you get closer to the vehicle, you note the smell of noxious fumes and find that you are in the midst of a vapor cloud. What should you do? Select one: A. Remain where you are and perform a visual assessment of the patient. B. Cover your face with your shirt and quickly extricate the injured driver. C. Exit the area immediately and gather information for the HazMat team. D. Realize that you are in the danger zone and prevent others from entering.

C. Exit the area immediately and gather information for the HazMat team.

You are treating a patient who complains of increasing shortness of breath, swelling of the ankles, and nonproductive cough over the past week. She states that she has been treated for various cardiac problems in the past, but has been unable to afford the medication. As you assess the patient, you find rales in all lung fields. During transport, this patient would be the most comfortable in the _________ position. A. supine B. prone C. Fowler's D. lateral recumbent

C. Fowler's

__________ rays easily penetrate through the human body and require lead or several inches of concrete to prevent penetration. A. Neutron B. Beta C. Gamma D. Alpha

C. Gamma Correct

Which of the following infections, when left untreated, may lead to infection of the blood and brain? A. Chlamydia B. Pelvic inflammatory disease C. Gonorrhea D. Bacterial vaginosis

C. Gonorrhea

You are transporting a 54-year-old male in respiratory arrest. An EMR is driving the ambulance as you and your partner are caring for the patient. Which of the following is the MOST logical way of notifying the hospital? Select one: A. Call the receiving hospital with your cell phone while providing patient care. B. Request that a police officer respond to the hospital to apprise the staff of your arrival. C. Have the driver contact dispatch and relay the patient information to the hospital. D. Wait until you arrive at the hospital and then quickly apprise the staff of the situation.

C. Have the driver contact dispatch and relay the patient information to the hospital.

Which of the following types of questions allow for the most detailed response? Select one: A. Yes or no questions B. Closed-ended questions C. Open-ended questions D. Multiple questions asked at once Feedback

C. Open-ended questions

A patient with hypoglycemia will often present with which of the following signs/symptoms? Select one: A. Warm, red, and dry skin B. Deep, rapid respirations C. Pale, cool, and clammy skin D. Hypertension

C. Pale, cool, and clammy skin

An intoxicated 40-year-old male is found lying face down. How would you document his body's position? Select one: A. Supine B. Recumbent C. Prone D. Dorsal

C. Prone

Which of the following statements regarding blast injuries is correct? A. The gastrointestinal tract is the organ system most sensitive to blast injuries and is the leading cause of death following an explosion. B. Tertiary blast injuries are penetrating or nonpenetrating injuries that result from flying debris, such as ordnance projectiles. C. Solid organs are relatively protected from shock wave injury but may be injured during the secondary or tertiary blast phase. D. Solid organs such as the middle ear, lungs, and gastrointestinal tract are the most susceptible to pressure changes.

C. Solid organs are relatively protected from shock wave injury but may be injured during the secondary or tertiary blast phase. Correct

Which organ lies in the lateral and posterior portion of the left upper quadrant of the abdomen? A. Stomach B. Cecum C. Spleen D. Liver

C. Spleen Correct

Patients with penetrating neck trauma are at greater risk for all the following life threats EXCEPT: A. air embolism B. cervical spine injury C. cardiac tamponade D. hemorrhagic shock

C. cardiac tamponade hemorrhagic shock --> when you lose > 20% blood volume (hypovolemia)

Which of the following describes the Moro reflex? Select one: A. When something touches a neonate's cheek, he or she instinctively turns his or her head toward the touch. B. When the sole of the foot is stroked with a blunt object, the big toe lifts upward and the other toes fan outward. C. The neonate opens his or her arms wide, spreads his or her fingers, and seems to grasp at something after being startled. D. An infant's heart rate decreases secondary to hypoxia because he or she depends heavily on the heart rate to perfuse the body.

C. The neonate opens his or her arms wide, spreads his or her fingers, and seems to grasp at something after being startled.

Which of the following statements regarding HIV is correct? Select one: A. HIV is transmitted exclusively via blood. B. HIV is easily transmittable in the EMS field. C. There is no vaccine against HIV infection. D. HIV is far more contagious than hepatitis B.

C. There is no vaccine against HIV infection.

What is the function of the fallopian tubes? A. To supply blood to the uterine lining B. To produce progesterone and estrogen C. To transport a mature egg to the uterus D. To connect the ovaries

C. To transport a mature egg to the uterus Correct

When questioning the parent of a child who ingested a poisonous substance, which of the following questions would be of LEAST pertinence initially? A. What time did the ingestion occur? B. Do you know what substance was ingested? C. Why did your child ingest the poison? D. Have you noticed any signs or symptoms?

C. Why did your child ingest the poison?

During the natural process of aging, the number of functional cilia in the respiratory system decreases, resulting in: Select one: A. air-trapping within the alveoli. B. baseline respiratory distress. C. a decreased ability to cough. D. an increased risk of COPD.

C. a decreased ability to cough.

You should suspect a kidney injury anytime the patient presents with _______. A. nausea B. abdominal distention C. a hematoma in the flank region D. dyspnea

C. a hematoma in the flank region

Continual reassessment of the scene at a suspected terrorist or weapon of mass destruction incident is MOST important because: A. weather conditions may change quickly. B. bystanders may destroy the evidence. C. a secondary explosive device may detonate. D. terrorists are often at the scene after an attack.

C. a secondary explosive device may detonate. Correct

You respond to a call about a sudden outbreak illness at a packed gym. Dispatch informs you that dozens of people are sick. You quickly glance at your watch to start a mental timeline and recognize the date as the anniversary of a major foreign invasion 5 years ago. Immediately you begin to suspect the cause of illness to be: A. bird flu B. carbon monoxide poisoning C. a terrorist attack D. none of the above

C. a terrorist attack

An infant with severe dehydration would be expected to present with: A. excessive tearing. B. bulging fontanelles. C. absent urine output. D. moist oral mucosa.

C. absent urine output.

You are transporting an elderly woman who has possibly experienced a stroke. She is obviously scared but is unable to talk and cannot move the entire right side of her body. In addition to providing the medical care that she needs, you should: Select one: A. reassure her that after proper treatment in the hospital, she will regain her speech in time. B. maintain eye contact at all times and tell her that there is no need for her to be scared. C. acknowledge that she is scared and tell her that you will take good care of her. D. tell her that you understand why she is scared and that everything will likely be okay.

C. acknowledge that she is scared and tell her that you will take good care of her.

When functioning at the scene of a motor vehicle crash in which a patient will require complex extrication, you should enter the vehicle and provide care to the patient: Select one: A. only if the patient has experienced a major injury. B. after rescue personnel have cut the battery cables. C. after receiving approval from the incident commander. D. only after you believe the vehicle has been stabilized.

C. after receiving approval from the incident commander.

Activated charcoal works by: A. acting as an antidote to common poisons B. inducing vomiting to eliminate poisons C. allowing safe excretion of poisons D. preventing additional clot formation

C. allowing safe excretion of poisons

You are dispatched to a neighborhood park on a warm day for an 8-year-old with difficulty breathing. You find the patient breathing adequately but with some accessory muscle use. You note expiratory wheezes and a persistent cough. You suspect the patient is experiencing: A. exercise induced pulmonary edema. B. a severe airway obstruction. C. an asthma attack. D. heat exhaustion.

C. an asthma attack !accessory muscle use!

You are assessing the victim of a motorcycle accident who complains of severe pain when either hip is flexed. You notice the patient's feet are both rotated externally. You feel crepitus and the patient screams in pain when you perform a pelvic rock. You should: A. reassess pelvic rock every 5 min to monitor for changes B. apply bilateral traction splints C. apply a pelvic wrap or binder and treat for shock D. all of the above

C. apply a pelvic wrap or binder and treat for shock

You respond to a call for a male who put his hand through a window. First responders report a large amount of bright red blood. When you arrive, they are applying direct pressure with three hands and blood is still flowing freely. He appears pale, disoriented, and frightened. After assessing his ABCs, your first treatment should be to: A. apply pressure to an appropriate pressure point B. elevate the extremity C. apply a tourniquet D. apply high-flow oxygen

C. apply a tourniquet

A 54-year-old male experienced an avulsion to his penis when his foreskin got caught in the zipper of his pants. He was able to unzip his pants and remove the foreskin prior to your arrival. Your assessment reveals that he is in severe pain and that the avulsion is bleeding moderately. The MOST appropriate treatment for this patient includes: A. administering 100% oxygen via a nonrebreathing mask. B. requesting a paramedic to administer pain medication. C. applying direct pressure with a dry, sterile dressing. D. covering the avulsion with moist, sterile dressings.

C. applying direct pressure with a dry, sterile dressing.

After eating at a local restaurant, a 20-year-old male complains of blurred vision, difficulty speaking, and difficulty breathing. He is conscious; however, his respirations are profoundly labored and producing minimal tidal volume. You should: A. position him supine and elevate his legs. B. apply oxygen via a nonrebreathing mask. C. assist his ventilations with high-flow oxygen. D. request a paramedic to administer atropine.

C. assist his ventilations with high-flow oxygen. Correct

When driving in emergency mode with lights and siren operating, ambulance operators in most states are permitted to do all of the following EXCEPT: A. exceed the posted speed limit B. park wherever needed C. assume that other drivers will see them and yield D. drive the ambulance the "wrong" way on a one-way street

C. assume that other drivers will see them and yield

Multiple people in a small town began experiencing abdominal cramps, excessive salivation and urination, and muscle twitching shortly after a small crop duster plane made several passes over the community. As you are assessing the patients, you further determine that most of them are bradycardic and have miosis. In addition to high-flow oxygen, the MOST appropriate treatment for these patients includes: A. activated charcoal and glucose. B. amyl nitrate and naloxone. C. atropine and pralidoxime chloride. D. epinephrine and hyperbaric oxygen.

C. atropine and pralidoxime chloride. Correct

In anticipation of receiving a fertilized ovum, the lining of the uterine wall: A. diverts blood flow to the vagina. B. thins and begins to separate. C. becomes engorged with blood. D. sheds and is expelled externally.

C. becomes engorged with blood.

The least reliable method of assessing circulation in the adult patient is to assess: A. blood pressure B. pulse quality C. capillary refill D. skin color

C. capillary refill

Your 24 y.o. male patient lost control of his ATV and hit several trees. On assessment, you find your patient responsive only to painful stimuli with rapid breathing, tachycardia, and diaphoresis. You direct your partner to being packaging the patient as you continue your assessment and note several small puncture wounds around the left chest and flank, bulging neck veins, and BP of 70/52. Your patient's breath sounds are clear and equal bilaterally. The patient's most likely injury is: A. tension pneumothorax B. hemothorax C. cardiac tamponade D. ruptured spleen

C. cardiac tamponade

Upon arriving at the scene of a possible hazardous materials (HazMat) incident involving several patients, you should: Select one: A. retrieve all critical patients. B. rope off the entire perimeter. C. carefully assess the situation. D. divert traffic away from the scene.

C. carefully assess the situation.

You are transporting a patient with blunt abdominal trauma. The patient is unstable and is experiencing obvious signs and symptoms of shock. Your estimated time of arrival at the hospital is less than 10 minutes. After treating the patient appropriately, you should: A. perform a comprehensive secondary assessment. B. forgo the hospital radio report because of his condition. C. closely monitor him and reassess him frequently. D. begin documenting the call on the patient care form.

C. closely monitor him and reassess him frequently.

A condition in which the heart is unable to pump efficiently and fluids accumulate in the lungs, neck veins, or extremities is called: A. myocardial infarction B. cardiogenic shock C. congestive heart failure D. pulmonary embolism

C. congestive heart failure the person has CHF (the condition) and is experiencing cardiogenic shock (what is happening)

During the transport phase of an ambulance call, it is MOST important to: Select one: A. reassess the patient only if he or she deteriorates. B. reassess unstable patients at least every 15 minutes. C. converse with the patient and provide reassurance. D. complete the run form before arrival at the hospital.

C. converse with the patient and provide reassurance.

You are transporting a major trauma patient who you believe to be in hypovolemic shock. During the transport, the patient's ventilations become ineffective, and you begin assisting ventilations with a BVM. On reassessment, you expect to see: A. bradycardia B. decreasing pulse oximetry reading C. decreasing BP D. cyanosis

C. decreasing BP

You are caring for a patient who is unresponsive after falling from a roof. After managing all life threats and assessing airway, breathing and circulation, your next action is to: A. perform a rapid trauma assessment B. obtain vital signs C. determine the patient's priority D. perform an ongoing assessment

C. determine the patient's priority

The primary assessment of an adult with chest pain includes: A. identifying the specific cause of the patient's pain B. ensuring the scene is safe C. determine the transport priority D. reassessing vital signs

C. determine the transport priority

The ____________ is a muscular dome that forms the inferior boundary of the thorax, separating the thorax from the abdomen. A. costovertebral angle B. costal arch C. diaphragm D. mediastinum

C. diaphragm Correct

Infection should be considered a possible cause of an airway obstruction in an infant or child, especially if he or she presents with: A. skin that is cool and dry. B. extreme restlessness. C. drooling or congestion. D. acute respiratory distress.

C. drooling or congestion.

The most common cause of uncomplicated seizures in the pediatric patient is: A. failture to take prescribed meds B. trauma to the patient's head or neck C. fever D. aneurysm

C. fever **febrile seizures**

You respond to the home of a 48 year old male complaining of chest pain. Following the scene size-up and initial exam, what is the proper order of assessment? A. rapid head-to-toe exam, detailed physical exam, and focused history and exam B. detailed physical exam, ongoing assessment, and detailed physical exam C. focused history and exam, ongoing assessment, detailed physical exam, and ongoing assessment D. rapid head-to-toe exam, detailed physical exam, ongoing assessment, and focused history and exam

C. focused history and exam, ongoing assessment, detailed physical exam, and ongoing assessment

The areas of the infant's skull that have not yet fused together are called ________. Select one: A. cranial valleys B. sutures C. fontanelles D. ventricles

C. fontanelles

An urgent move should be used: A. when the patient is in immediate danger on the scene of an accident B. when the patient must be moved quickly to treat life-threatening problems C. for patients who require rapid transport but with spinal immobilization maintained D. without regard for spinal immobilization

C. for patients who require rapid transport but with spinal immobilization maintained

The brain connects to the spinal cord through a large opening at the base of the skull called the: A. vertebral foramen. B. foramen ovale. C. foramen magnum. D. spinous foramen.

C. foramen magnum. Correct

A supervisor who has more than seven people reporting to him or her: Select one: A. is more beneficial to the overall effort than a supervisor with fewer personnel because his or her team can accomplish more tasks. B. should regularly report to the incident commander (IC) to inform him or her of the functions that his or her team is performing. C. has exceeded an effective span of control and should divide tasks and delegate the supervision of some tasks to another person. D. should assign a specific task to each person reporting to him or her and regularly follow up to ensure that the tasks were carried out.

C. has exceeded an effective span of control and should divide tasks and delegate the supervision of some tasks to another person.

Vaccination against the hepatitis A virus is unnecessary if you: Select one: A. received a hepatitis B vaccination. B. have a weak immune system. C. have been infected with hepatitis in the past. D. are older than 35 years of age.

C. have been infected with hepatitis in the past.

During your assessment of a 19-year-old male, you are told that he is being treated with factor VIII. This indicates that: Select one: A. his blood clots too quickly. B. he has thrombophilia. C. he has hemophilia A. D. he has a thrombosis.

C. he has hemophilia A.

After applying a tourniquet, the injury from a patient's leg stops bleeding. This is called: Select one: A. hematemesis. B. hemiplegia. C. hemostasis. D. hemolysis.

C. hemostasis.

You respond to a call for an MVC and find your patient unconscious with obvious signs of shock but no obvious external injuries. You know that the most common type of shock encountered in the trauma patient is: A. neurogenic shock B. cardiogenic shock C. hypovolemic shock D. anaphylactic shock

C. hypovolemic shock (plasma in blood is too low) neurogenic --> occurs with damage to nervous system cardiogenic --> heart can't pump enough blood to go around body anaphylactic --> allergic rxn

Upon arriving at the scene of a motor vehicle crash, you can see three patients, one who is entrapped in his car and two who have been ejected from their vehicle. You should: Select one: A. begin triage to determine injury severity. B. call medical control for further direction. C. immediately request additional resources. D. request law enforcement for traffic control.

C. immediately request additional resources.

The LEAST practical place to store a portable oxygen cylinder is: Select one: A. inside the jump kit. B. near the side or rear door. C. in the driver's compartment. D. on the ambulance stretcher.

C. in the driver's compartment.

A condition in which blood supply to an area of the heart is interrupted or insufficient causing injury to the tissue is called: A. cardiac arrest B. cardiac tamponade C. myocardial infarction D. arrhythmia

C. myocardial infarction arrest --> sudden loss of heart function tamponade --> compression of heart arrhythmia --> irregular heartbeat

The MOST common and usually the most serious ambulance crashes occur at: Select one: A. stop signs. B. stop lights. C. intersections. D. railroad crossings.

C. intersections.

CPR retraining is the MOST effective when it: Select one: A. occurs every 24 months. B. is delivered by computer. C. involves hands-on practice. D. is self-paced and brief.

C. involves hands-on practice.

In contrast to type 1 diabetes, type 2 diabetes: Select one: A. occurs when antibodies attack insulin-producing cells. B. is caused by a complete lack of insulin in the body. C. is caused by resistance to insulin at the cellular level. D. is commonly diagnosed in children and young adults.

C. is caused by resistance to insulin at the cellular level.

A diabetic patient has polydipsia. This means that she: Select one: A. urinates frequently. B. is unable to swallow. C. is excessively thirsty. D. has low blood sugar.

C. is excessively thirsty.

An abdominal aortic aneurysm: Select one: A. is usually not repairable, even if discovered early. B. causes dull pain that often radiates to the shoulders. C. is often the result of hypertension and atherosclerosis. D. can sometimes be palpated as a mass in the groin area.

C. is often the result of hypertension and atherosclerosis.

The stooped posture of some older people, which gives them a humpback appearance, is called: Select one: A. arthritis. B. scoliosis. C. kyphosis. D. miosis.

C. kyphosis.

The __________ and _________ are folds of tissue that surround the urethral and vaginal openings. A. vagina; cervix B. fallopian tubes; uterus C. labia majora; labia minora D. perineum; anus

C. labia majora; labia minora

The physical examination of a sexual assault victim should be: A. as detailed as possible so all injuries can be documented. B. performed in the presence of at least two police officers. C. limited to a brief survey for life-threatening injuries. D. deferred until the patient can be evaluated by a physician.

C. limited to a brief survey for life-threatening injuries.

Placing a pregnant patient in a supine position during the third trimester of pregnancy: A. results in spontaneous urinary incontinence if the bladder is full. B. often causes hypotension secondary to cardiac compression. C. may decrease the amount of blood that returns to the heart. D. is recommended if the patient has severe abdominal discomfort.

C. may decrease the amount of blood that returns to the heart.

An important sign of respiratory distress that is common in infants but less common in adults is: A. supra-sternal retractions B. accessory muscle use C. nasal flaring D. pursed lip breathing

C. nasal flaring

Ambulances today are designed according to strict government regulations based on _____ standards. Select one: A. local B. state C. national D. individual

C. national

Your patient complains of shortness of breath and shows signs of respiratory distress, including cyanosis, tachypnea, and accessory muscle use. The patient's chest rise and fall appears to be adequate. The best device to deliver oxygen to this patient is the: A. nasal cannula B. non visualized airway C. non-rebreather mask D. partial rebreather mask

C. non-rebreather mask

A patient with an isolated concussion injury will likely: A. get progressively worse over time. B. lose consciousness for long periods. C. progressively improve over time. D. require an extended hospital stay.

C. progressively improve over time.

When removing a critically injured patient from his or her vehicle, you should: Select one: A. remove him or her using a short backboard. B. move him or her in one fast, continuous step. C. protect the cervical spine during the entire process. D. release c-spine control to facilitate rapid removal.

C. protect the cervical spine during the entire process.

Law enforcement personnel request your assistance to assess a 31-year-old female who was sexually assaulted. When you arrive at the scene, you find the patient sitting on a curb outside her apartment. She is conscious, alert, and crying. When you ask her what happened, she tells you that she does not want to be treated or transported to the hospital. She further tells you that all she wants to do is clean up. You should: A. advise her that she cannot clean herself up because this will destroy evidence. B. perform a limited hands-on assessment to detect life-threatening injuries. C. provide emotional support and visually assess her for obvious trauma. D. ask her if there is anyone you can contact, such as a friend or relative.

C. provide emotional support and visually assess her for obvious trauma.

While assessing your patient who has a painful, angulated injury to his left lower leg after a skateboarding accident, you notice that the foot is cool, pale and has no pulse. You should: A. pull gentle traction until the limb is completely realigned B. splint in the position found and transport rapidly C. pull gentle traction until the limb's pulses and color improve D. none of the above

C. pull gentle traction until the limb's pulses and color improve

Three days after delivering her baby, a 30-year-old woman complains of a sudden onset of difficulty breathing. Her level of consciousness is decreased and she is tachycardic. The EMT should suspect: A. acute pulmonary edema. B. intrauterine bleeding. C. pulmonary embolism. D. spontaneous pneumothorax.

C. pulmonary embolism.

Extrication is defined as: Select one: A. using heavy equipment to access a patient. B. dismantling an automobile to remove a victim. C. removal from a dangerous situation or position. D. immobilizing a patient before moving him or her.

C. removal from a dangerous situation or position.

If an object is visible in the unconscious patient's airway, you should __________. Select one: A. leave it in place B. continue chest compressions C. remove it D. place the patient on his or her side

C. remove it

Radio systems that receive, amplify, and rebroadcast radio transmissions are known as: A. cell systems B. 800 MHz radios C. repeaters D. MDTs

C. repeaters MDTs are used to connect to a central dispatch office

Pulse oximetry if useful for: A. determining how much oxygen your patient requires B. determining whether ALS is needed C. trending a patient's respiratory status D. assessing a patient's ventilation effort

C. trending a patient's respiratory status

In most cases, cardiopulmonary arrest in infants and children is caused by: Select one: A. a cardiac dysrhythmia. B. severe chest trauma. C. respiratory arrest. D. a drug overdose.

C. respiratory arrest.

The recommended treatment for absorbed or contact poisons includes _________. Select one: A. neutralizing the poisonous substance with a chemical B. transporting the patient to the emergency department with the poisonous substance in place C. safely removing or diluting the poisonous substance D. collecting and transporting the poisonous substance to the emergency department for identification

C. safely removing or diluting the poisonous substance

Some women experience ___________ during ovulation. A. hypotension B. severe abdominal pain C. slight cramping D. severe bleeding

C. slight cramping

All of the following are hollow abdominal organs, EXCEPT the: A. bladder. B. stomach. C. spleen. D. ureters.

C. spleen.

Lighting at a scene, establishing a tool and equipment area, and marking for a helicopter landing all fall under: Select one: A. logistics operations. B. EMS operations. C. support operations. D. law enforcement.

C. support operations.

Returning the emergency unit to service is part of the _________. Select one: A. transfer phase B. hazard-control phase C. termination phase D. support phase

C. termination phase

You are caring for a 56-year-old male patient complaining of abdominal pain. Your service has recently switched to an electronic PCR system (ePCR). When completing the ePCR, it is important to be aware that: Select one: A. a written patient care record will need to be completed on arrival at the hospital. B. the ePCR does not contain the same level of information as the written version and your verbal report should be expanded. C. the ePCR allows patient information to be transmitted directly to the receiving hospital's computers. D. the ePCR is relatively unsecured and should not contain patient-specific information.

C. the ePCR allows patient information to be transmitted directly to the receiving hospital's computers.

A 77-year-old female presents with an acute onset of altered mental status. Her son is present and advises that she has a history of hypertension, atrial fibrillation, type 2 diabetes, and glaucoma. He further advises that she takes numerous medications and that she is normally alert. When you assess this patient, it is important to note that: Select one: A. her mental status is likely the result of hypoglycemia and you should give her sugar. B. because of her age and medical history, you should suspect Alzheimer disease. C. the patient is experiencing delirious behavior, which suggests a new health problem. D. dementia typically presents as an acute onset of deterioration of cognitive function.

C. the patient is experiencing delirious behavior, which suggests a new health problem.

You are caring for a driver who struck a light pole. She admits to drinking alcohol but orders you not to tell anyone. You should report the information to __________. Select one: a. the state motor vehicle department b. law enforcement personnel c. the receiving nurse or doctor d. your medical director

C. the receiving nurse or doctor

The term "pericardiocentesis" means: Select one: A. surgical repair of the sac around the heart. B. a surgical opening made in the heart. C. the removal of fluid from around the heart. D. narrowing of the arteries supplying the heart.

C. the removal of fluid from around the heart.

You are providing care to a 61-year-old female complaining of chest pain that is cardiac in origin. Your service utilizes a multiplex communication system. You wish to transmit the patient's electrocardiogram to the hospital. In order to accomplish this, you must: Select one: A. wait until you reach the hospital to transmit the information. B. send the electrocardiogram from the back of the ambulance. C. use the multiplex system to transmit the information . D. be aware that only audio or data transmissions can be sent at any one time.

C. use the multiplex system to transmit the information .

A medical transport helicopter is incoming and you are responsible for setting up the landing zone (LZ). You should: Select one: A. use yellow caution tape to mark of the LZ perimeter. B. ask bystanders to stand at all four corners of the LZ. C. use weighted cones to mark all four corners of the LZ. D. place four flares 100 feet apart in an "X" pattern.

C. use weighted cones to mark all four corners of the LZ.

Basic theory of extrication states that when performing disentanglement at an MVC, the objective is to remove the: A. patient from the vehicle B. hazards from the extrication scene C. vehicle from around the patient D. patient from the hazardous area

C. vehicle from around the patient *definition of disentanglement*

You receive a call to a residence where a man found his wife unresponsive on the couch. The patient's respiratory rate is 8 breaths/min, her breathing is shallow, her heart rate is 40 beats/min, and her pulse is weak. The husband hands you an empty bottle of hydrocodone (Vicodin), which was refilled the day before. You should: Select one: A. perform a rapid head-to-toe exam. B. apply oxygen via a nonrebreathing mask. C. ventilate her with a BVM. D. contact the poison control center.

C. ventilate her with a BVM.

Effective therapeutic communication skills require _________. Select one: A. correct use of complex medical terminology B. English-speaking patients or family members C. verbal and nonverbal communication techniques D. supervision by advanced life support personnel

C. verbal and nonverbal communication techniques

A school-age child looking for approval from his peers and society is demonstrating what kind of reasoning? Preconventional reasoning Conventional reasoning Postconventional reasoning Trust and mistrust

Conventional reasoning

A 56-year-old man has an acute myocardial infarction. Which of the following blood vessels became blocked and led to his condition? Coronary veins Coronary arteries Pulmonary veins Pulmonary arteries

Coronary arteries

A 33-year-old male sustained an abdominal evisceration to the left lower quadrant of his abdomen after he was cut with a large knife. After appropriately managing his ABCs and assessing him for other life-threatening injuries, how you should care for his wound? A. Cover it with moist, sterile gauze and secure with an occlusive dressing. B. Cover the exposed bowel and keep his legs in a straight position. C. Carefully replace the exposed bowel into the abdomen and transport. D. Irrigate it with sterile water and cover it with a dry dressing.

Cover it with moist, sterile gauze and secure with an occlusive dressing. Correct

A 22-year-old female was ejected from her car after striking a tree head-on. As you approach her, you note obvious closed deformities to both of her femurs. She is not moving and does not appear to be conscious. You should: A. assess for a carotid pulse and assist her ventilations. B. administer oxygen and perform a rapid assessment. C. apply manual stabilization to both of her femurs. D. stabilize her head and perform a primary assessment.

D

A man was found unresponsive in his bed at home. There is no evidence of injury and the patient's medical history is not known. The patient's face is cyanotic, yet the pulse oximeter reads 98%. Which of the following would MOST likely explain this? A. Severe pulmonary edema B. His extremities are cold C. Increased body temperature D. Carbon monoxide poisoning

D

During your secondary assessment of a 30-year-old male who fell 25 feet, you note crepitus when palpating his pelvis. Your partner advises you that the patient's blood pressure is 80/50 mm Hg and his heart rate is 120 beats/min and weak. After completing your assessment, you should: A. perform a focused physical exam with emphasis on the pelvis. B. log roll the patient onto a long backboard and transport at once. C. defer spinal immobilization and transport to a trauma center. D. stabilize the pelvis with a pelvic binder and protect the spine.

D

Irregular respirations characterized by an increasing rate and depth of breathing followed by periods of apnea are called: A. agonal respirations. B. ataxic respirations. C. eupneic respirations. D. Cheyne-Stokes respirations.

D

Patients may experience allergic reactions to which of the following substances? a. Insect or animal bites and stings b. Plant material c. Food and medications d. All of these answers are correct

D

When a foreign substance invades the body, the body will __________ A. protect itself B. go on alert C. attempt to inactivate the foreign substance D. All of these answers are correct

D

Which of the following musculoskeletal injuries would pose the greatest threat to a patient's life? A. An amputated extremity B. Bilateral femur fractures C. Nondisplaced long bone fractures D. Pelvic fracture with hypotension

D

Which of the following patients should you place in the recovery position? A. A 40-year-old conscious female with a possible neck injury and regular respirations B. A 24-year-old unconscious female who overdosed and has a reduced tidal volume C. A 19-year-old conscious male with a closed head injury and normal respirations D. A 31-year-old semiconscious male with low blood sugar and adequate breathing

D

Which of the following statements regarding breathing adequacy is correct? A. A patient with slow respirations and adequate depth will experience an increase in minute volume. B. Patients with a grossly irregular breathing pattern usually do not require assisted ventilation. C. The single most reliable sign of breathing adequacy in the adult is his or her respiratory rate. D. Patients breathing shallowly may require assisted ventilation despite a normal respiratory rate.

D

With regard to musculoskeletal injuries, the zone of injury is defined as the: A. exact part of the bone or joint that was disrupted. B. part of the body that sustained secondary injury. C. area of obvious deformity over the site of impact. D. area of soft-tissue damage surrounding the injury.

D

Children of which age group are considered toddlers? Select one: A. Over 6 years B. 1 month to 1 year C. 3-6 years D. 1-3 years

D. 1-3 years

A mother who is pregnant with her first baby is typically in the first stage of labor for approximately: A. 8 hours. B. 4 hours. C. 10 hours. D. 16 hours.

D. 16 hours.

The human body should be functioning at its optimal level between the ages of: Select one: A. 18 and 22 years. B. 21 and 30 years. C. 25 and 35 years. D. 19 and 25 years.

D. 19 and 25 years.

The incubation period for Ebola can be up to: A. 72 hours. B. 1 week. C. 6 months. D. 21 days.

D. 21 days. Correct

Hydroplaning of the ambulance on wet roads would MOST likely occur at speeds of greater than _____ mph. Select one: A. 10 B. 15 C. 20 D. 30

D. 30

When arriving at the scene of a cave-in or trench collapse, response vehicles should be parked at least _____ away from the scene. Select one: A. 50 feet (15 m) B. 150 feet (46 m) C. 250 feet (76 m) D. 500 feet (152 m)

D. 500 feet (152 m)

The average pulse rate of persons between 19 and 60 years of age is typically: Select one: A. 80 beats/min. B. 60 beats/min. C. 90 beats/min. D. 70 beats/min.

D. 70 beats/min.

The normal blood glucose level is between: Select one: A. 30 and 150 mg/dL. B. 60 and 80 mg/dL. C. 160 and 200 mg/dL. D. 80 and 120 mg/dL.

D. 80 and 120 mg/dL.

The JumpSTART triage system is intended to be used for children younger than _____ years or who appear to weigh less than _____. Select one: A. 5; 50 lb B. 6; 70 lb C. 7; 90 lb D. 8; 100 lb

D. 8; 100 lb

Which of the following would provide the EMT with the BEST cover in a situation involving active gunfire? Select one: A. Stacked empty barrels B. Behind a car door C. A large cluster of shrubs D. A concrete barricade

D. A concrete barricade

Anaphylaxis is defined as the presence of signs of an allergic reaction combined with: A. airway compromise B. respiratory distress C. shock D. A or C

D. A or C

Which of the following general statements regarding consent is correct? Select one: A. Patients who are intoxicated are generally allowed to refuse treatment. B. All patients older than 18 years can legally refuse treatment or transport. C. Expressed consent is valid only if given in writing by a family member. D. A patient can consent to transport but can legally refuse to be treated.

D. A patient can consent to transport but can legally refuse to be treated.

The written or electronic run report should be finished A. before transferring care to the ER B. after the patient's diagnosis is known C. within 24 hrs of completing the run D. ASAP after the run

D. ASAP after the run

Which of the following statements about use of the automated external defibrillator (AED) is correct? A. The AED should not be used on children less than 1 year of age. B. The AED should not be used on children weighing less than 90 pounds. C. Adult AED pads should never be used on pediatric patients. D. Adult AED pads can be used on pediatric patients if pediatric pads are not available

D. Adult AED pads can be used on pediatric patients if pediatric pads are not available

Causes of vaginal bleeding include which of the following? A. Ectopic pregnancy B. Vaginal trauma C. Spontaneous abortion D. All of these answers are correct.

D. All of these answers are correct.

During gynecologic emergencies, in addition to the standard SAMPLE questions, the EMT should attempt to determine which of the following? A. The amount of time since of the patent's last menstrual cycle B. The possibility that the patient may be pregnant C. If the patient is sexually active D. All of these answers are correct.

D. All of these answers are correct.

In cases of sexual assault, it is important to ensure that you do which of the following? A. Immediately write down your observations of the scene before assessing the patient. B. Call for a female EMT or ALS care provider. C. Discuss with law enforcement any suspicions that you may have about the perpetrator. D. All of these answers are correct.

D. All of these answers are correct.

The ambulance inspection should include checks of: Select one: A. fuel level. B. brake fluid. C. wheels and tires. D. All of these answers are correct.

D. All of these answers are correct.

You respond to a multiple-vehicle collision. You and your partner are reviewing dispatch information en route to the scene. You will be at a major intersection of two state highways. As you approach the scene, you review the guidelines for sizing up the scene. The guidelines include: Select one: A. looking for safety hazards. B. evaluating the need for additional units or other assistance. C. evaluating the need to stabilize the spine. D. All of these answers are correct.

D. All of these answers are correct.

Which of the following conditions is the diabetic patient at an increased risk of developing? Select one: A. Alcoholism B. Hepatitis B C. Depression D. Blindness

D. Blindness

Which of the following statements regarding sudden infant death syndrome (SIDS) is correct? A. Certain cases of SIDS are predictable and therefore preventable. B. SIDS is most commonly the result of an overwhelming infection. C. The cause of death following SIDS can be established by autopsy. D. Death as a result of SIDS can occur at any time of the day or night.

D. Death as a result of SIDS can occur at any time of the day or night.

Which of the following components are needed to prove negligence? Select one: A. Breach of duty, injury/damages, abandonment, and causation B. Abandonment, breach of duty, damages, and causation C. Duty to act, abandonment, breach of duty, and causation D. Duty to act, breach of duty, injury/damages, and causation

D. Duty to act, breach of duty, injury/damages, and causation

Prehospital patient care decisions should be based on __________. Select one: A. the opinions of physicians B. an EMS agency's fiscal resources C. the experiences of EMTs D. EMS research

D. EMS research

The determination that prompt surgical care in the hospital is more important than performing time-consuming procedures in the field on a major trauma patient is based MOSTLY on: Select one: A. regional trauma guidelines. B. local protocols. C. the lead EMT's decision. D. EMS research.

D. EMS research

Which type of consent is involved when a 39-year-old mentally competent female with a severe headache asks you to take her to the hospital? Select one: A. Informed B. Formal C. Implied D. Expressed

D. Expressed

Which of the following duties or responsibilities does NOT fall within the realm of the medical branch of the incident command system? Select one: A. Triage B. Transport C. Treatment D. Extrication

D. Extrication

Which of the following is NOT true of the lymphatic system? A. It relies on muscle movement to circulate lymph. B. It circulates waste products of metabolism away from cells. C. It circulates oxygen, nutrients and hormones to the cells. D. Its vessels closely parallel the major arteries in the body.

D. Its vessels closely parallel the major arteries in the body. Correct

Which of the following statements regarding autism is correct? A. The majority of patients with autism do not speak at all. B. Autism affects females four times greater than males. C. Impairment of motor activity is a classic sign of autism. D. Most cases of autism are diagnosed by 3 years of age.

D. Most cases of autism are diagnosed by 3 years of age.

A patient has a large accumulation of blood in the sac surrounding the heart. Which type of shock would this condition cause? A. Neurogenic B. Cardiogenic C. Hypovolemic D. Obstructive

D. Obstructive Correct

CPR is in progress on a pregnant woman. Shortly after manually displacing her uterus to the left, return of spontaneous circulation occurs. Which of the following would MOST likely explain this? Select one: A. Displacement of her uterus caused blood to flow backward, which increased blood flow to her heart. B. Increased blood flow to her heart caused her ventricles to stop fibrillating, which restored her pulse. C. Displacement of her uterus allowed her lungs to expand more fully, which restored her pulse. D. Pressure was relieved from her aorta and vena cava, which improved chest compression effectiveness.

D. Pressure was relieved from her aorta and vena cava, which improved chest compression effectiveness.

Which of the following conditions would MOST likely be encountered in a patient with cerebral palsy? A. Type 2 diabetes B. Paralysis C. Brain tumors D. Seizure disorder

D. Seizure disorder

Which of the following is a specific example of the Mobile Integrated Healthcare (MIH) model? Select one: A. EMTs respond to a woman who complains of high fever B. Paramedics pick up a patient from an acute care clinic C. EMTs assess a patient whose lung disease is "acting up" D. The paramedic administers a patient's flu vaccination

D. The paramedic administers a patient;s lu vaccination

Which of the following statements regarding abdominal eviscerations is correct? A. The organs should be replaced carefully to avoid heat loss. B. Most eviscerations occur to the left upper quadrant. C. Adherent material is preferred when covering an evisceration. D. The protruding organs should be kept warm and moist.

D. The protruding organs should be kept warm and moist.

The EMT should expect that a patient who was exposed to cyanide will have: A. an abnormally slow pulse rate. B. an abnormally slow respiratory rate. C. skin that is cherry red and hot. D. a normal pulse oximetry reading.

D. a normal pulse oximetry reading. Correct

To help protect patients, EMS agencies are required to have __________. Select one: A. public forums with their medical director B. online access to patient records C. an anonymous reporting system D. a privacy officer to answer questions

D. a privacy officer to answer questions

An 84-year-old male fell a week ago and has been bedridden since then. Today, he presents with an altered mental status. His skin is pale and cold and his respirations are rapid and shallow. The EMT should suspect: Select one: A. hypovolemic shock. B. acute hyperglycemia. C. a subdural hematoma. D. a systemic infection.

D. a systemic infection.

Down syndrome is a genetic defect that occurs as the result of: A. a sperm that contains 24 chromosomes. B. a separation of chromosome 21. C. an extra pair of chromosomes. D. a triplication of chromosome 21. Correct

D. a triplication of chromosome 21.

CPR should be initiated when: Select one: A. rigor mortis is obvious. B. signs of putrefaction are present. C. the carotid pulse is very weak. D. a valid living will is unavailable.

D. a valid living will is unavailable.

You are dispatched to a residence for a child with respiratory distress. The patient, an 18-month-old female, is tachypneic, has sternal retractions, and is clinging to her mother. Her skin is pink and dry, and her heart rate is 120 beats/min. The MOST appropriate treatment for this child includes: A. allowing the child to remain with her mother and applying a nasal cannula. B. requesting a paramedic ambulance to insert an advanced airway device. C. separating the child from her mother and providing ventilatory assistance. D. administering blow-by oxygen and transporting the child with her mother.

D. administering blow-by oxygen and transporting the child with her mother.

General adaptation syndrome is characterized by which of the following phases? Select one: A. Reaction and resistance, euphoria, and physical exhaustion B. Delayed reaction, alarm response, and physical recovery C. Immediate reaction, psychological exhaustion, and recovery D. Alarm response, reaction and resistance, and recovery

D. alarm response, reaction and resistance, and recovery

Esophageal varices typically occurs in patients with a history of: A. nose bleeds. B. appendicitis. C. epiglottitis. D. alcohol abuse.

D. alcohol abuse.

When decontaminating the back of your ambulance after a call, you should: Select one: A. use a bleach and water solution at a 1:2 dilution ratio to thoroughly wipe all surfaces. B. clean all surfaces and patient contact areas with a mixture of alcohol and water. C. spray the contaminated areas and then immediately wipe them dry with a towel. D. allow surfaces to air dry unless otherwise indicated in the product directions.

D. allow surfaces to air dry unless otherwise indicated in the product directions.

Common causes of depression in the elderly include all of the following, EXCEPT: Select one: A. prescription medication use. B. alcohol abuse and dependence. C. chronic medical conditions. D. an acute onset of dementia.

D. an acute onset of dementia.

The ___________ of the heart is the inferior portion of the ventricles. Select one: A. dorsum B. base C. septum D. apex

D. apex

A NRB should be administered to patients who: A. have slow, shallow respirations B. have a reduced tidal volume C. are breathing inadequately D. are breathing adequately with signs of hypoxia

D. are breathing adequately with signs of hypoxia hypoxia --> oxygen isn't getting to tissues

During your monthly internal quality improvement (QI) meeting, you review several patient care reports (PCRs) with the staff of your EMS system. You identify the patient's name, age, and sex, and then discuss the treatment that was provided by the EMTs in the field. By taking this approach to the QI process, you: Select one: A. acted appropriately but must have each EMT sign a waiver stating that he or she will not discuss the cases with others. B. adequately safeguarded the patient's PHI because the cases were discussed internally. C. violated the patient's privacy because you should have discussed the information only with the EMTs involved. D. are in violation of HIPAA because you did not remove the PHI from the PCR beforehand.

D. are in violation of HIPAA because you did not remove the PHI from the PCR beforehand.

You respond to a call about a patient with multiple soft tissue injuries after a domestic disturbance. As you enter the scene, you see that the patient has wounds on each of her arms. One arm is spurting bright red blood. The other arm slowly oozes dark blood from a very shallow wound. You identify these as __________ and __________ hemorrhages respectively. A. venous; arterial B. arterial; venous C. venous; capillary D. arterial; capillary

D. arterial; capillary arterial --> usually bright red and usually spurts venous --> usually dark red, from deep wound

Common factors that influence how a patient reacts to the stress of an illness or injury include all of the following, EXCEPT: Select one: A. history of chronic disease. B. fear of medical personnel. C. mental disorders. D. assessment by the EMT.

D. assessment by the EMT

Which of the following interventions is used by all levels of EMS providers? Select one: A. Multilumen airways B. Automatic transport ventilators C. Needle decompression D. Automated external defibrillator

D. automates external defibrillator

Your 5 y.o. patient is unresponsive. A carotid pulse is present at 40 beats per minutes. Breaths are shallow at six per min. You should immediately: A. apply an NRB B. begin artificial ventilation C. place the patient in the recovery position D. begin chest compressions

D. begin chest compressions CPR is indicated for unresponsive children with pulse under 60

By the 20th week of pregnancy, the uterus is typically at or above the level of the mother's: A. pubic bone. B. superior diaphragm. C. xiphoid process. D. belly button.

D. belly button.

The root "melan/o" means: Select one: A. red. B. gray. C. white. D. black.

D. black.

You arrive at an accident scene to find an alternative-fuel vehicle leaking an unknown substance. You should immediately _________. Select one: A. search the vehicle for occupants B. disconnect the car battery C. apply retardant to the leaking fuel D. call for additional resources and keep bystanders away from the vehicle

D. call for additional resources and keep bystanders away from the vehicle

The waste products of aerobic metabolism include: A. uric acid and nitrogen. B. ATP and glucose. C. glucose and lactic acid. D. carbon dioxide and water.

D. carbon dioxide and water. Correct

Several attempts to adequately open a trauma patient's airway with the jaw-thrust maneuver have been unsuccessful. You should: Select one: A. try opening the airway by lifting up on the chin. B. suction the airway and reattempt the jaw-thrust maneuver. C. tilt the head back while lifting up on the patient's neck. D. carefully perform the head tilt-chin lift maneuver.

D. carefully perform the head tilt-chin lift maneuver.

"Chondro" in the word chondritis means: Select one: A. joint. B. rib. C. tendon. D. cartilage.

D. cartilage.

When operating at a hazardous materials incident, responders and equipment assigned solely to EMT-B should stage in the: A. hot zone B. transport area C. treatment area D. cold zone

D. cold zone

You are called to attend to an elderly patient with an extensive medical history who is now in cardiac arrest. The patient's family tells you that the patient has a DNR order. There is no paperwork available but the patient does have a MedicAlert bracelet indicating Do Not Resuscitate. You should: Select one: A. confirm the patient's identity and honor the DNR order. B. initiate resuscitation in the absence of paperwork. C. transport with minimal care. D. confirm the patient identity and then confirm the DNR by calling the MedicAlert Foundation number on the bracelet.

D. confirm the patient identity and then confirm the DNR by calling the MedicAlert Foundation number on the bracelet.

Your presence is requested by law enforcement to assess a 33-year-old female who was sexually assaulted. The patient is conscious and obviously upset. As you are talking to her, you note an impressive amount of blood on her clothes in the groin area. Her blood pressure is 98/58 mm Hg, her pulse is 130 beats/min, and her respirations are 24 breaths/min. You should: A. arrange for a rape crisis center representative to speak with the patient. B. allow her to change her clothes and take a shower before you transport. C. visualize the vaginal area and pack the vagina with sterile dressings. D. control any external bleeding, administer oxygen, and transport at once.

D. control any external bleeding, administer oxygen, and transport at once.

A viral infection that may cause obstruction of the upper airway in a child is called: A. asthma. B. epiglottitis. C. bronchitis. D. croup.

D. croup.

Without practice, your CPR skills will __________. Select one: A. become part of your muscle memory B. improve over time C. come back automatically when needed D. deteriorate over time

D. deteriorate over time

A patient with an altered mental status; high blood glucose levels; and deep, rapid breathing may have a condition known as __________. Select one: A. hyperosmolar hyperglycemic nonketotic coma B. hypoglycemic crisis C. hyperglycemic crisis D. diabetic ketoacidosis

D. diabetic ketoacidosis

A 19-year-old female has just been extricated from her severely damaged car. She is on a long backboard and has been moved to a place of safety. As your partner maintains manual stabilization of her head, you perform a rapid assessment. The patient is unresponsive, has slow and shallow respirations, and has bilaterally closed femur deformities. You should: Select one: A. stabilize her legs with long board splints. B. apply oxygen via a nonrebreathing mask. C. obtain baseline vital signs and transport at once. D. direct your partner to begin ventilatory assistance.

D. direct your partner to begin ventilatory assistance.

Pathophysiology is the study of the functional changes that occur when the body reacts to a particular: A. medication. B. assessment. C. protocol. D. disease.

D. disease. Correct

Which term describes how ambulance operators must act in regard to other drivers when responding to an emergency? A. negligence B. compliance C. duty to act D. due regard

D. due regard (aka safe driving)

The primary duty of the triage division is to: Select one: A. begin basic treatment. B. establish zones for categorized patients. C. communicate with the treatment division. D. ensure that every patient receives a primary assessment.

D. ensure that every patient receives a primary assessment.

You and your partner arrive at the scene of a fire at a large office complex. Witnesses tell you that they heard a loud explosion shortly before the building caught fire. You should: A. tell the witnesses that you suspect that the explosion was the work of a terrorist. B. take standard precautions and begin searching for critically injured patients. C. carefully document the witnesses' statements and report them immediately. D. ensure that your ambulance is parked upwind and uphill from the building.

D. ensure that your ambulance is parked upwind and uphill from the building. Correct

Burns in children are commonly caused by all of the following, EXCEPT: A. hot items on a stovetop. B. exposure to caustic chemicals. C. scalding water in a bathtub. D. entrapment in a structural fire.

D. entrapment in a structural fire.

You are hired at the local EMS service. Your duties include station cleanup and checking the unit for mechanical problems. You should also check all medical equipment and supplies: Select one: A. after every call. B. after every emergency transport. C. every 12 hours. D. every day.

D. every day.

Most cases of anthrax begin with: A. respiratory distress. B. signs of shock. C. pulmonary edema. D. flulike symptoms.

D. flulike symptoms. Correct

Signs of a sudden severe upper airway obstruction include all of the following, EXCEPT: Select one: A. grasping the throat. B. acute cyanosis. C. inability to speak. D. forceful coughing.

D. forceful coughing.

When a patient stiffens the muscles of the abdomen, it is known as _______. A. instability. B. crepitus. C. distention. D. guarding.

D. guarding.

When caring for a female child who has possibly been sexually abused, you should: A. immediately report your suspicions to the parents. B. encourage the child to urinate and take a shower. C. carefully examine the genitalia for signs of injury. D. have a female EMT remain with her if possible.

D. have a female EMT remain with her if possible.

If there are downed power lines near a vehicle involved in a crash, you should: Select one: A. attempt to move the power lines yourself. B. touch the power lines with an object to see if there is active electricity. C. have the patient slowly exit the vehicle. D. have the patient remain in the vehicle.

D. have the patient remain in the vehicle.

A by-product of involuntary muscle contraction and relaxation is: A. oxygen. B. nitrogen. C. lactic acid. D. heat.

D. heat. Correct

Most poisonings occur via the __________ route. Select one: A. injection B. absorption C. inhalation D. ingestion

D. ingestion

You are transporting a 42-year-old male who experienced blunt abdominal trauma. He is receiving oxygen at 12 L/min via a nonrebreathing mask and full spinal precautions have been applied. During your reassessment, you note his level of consciousness has decreased and his respirations have become shallow. You should: A. suction his oropharynx to ensure it is clear of secretions and then increase the oxygen flow rate to 15 L/min. B. reassess his vital signs and then notify the receiving hospital of the change in his clinical status. C. perform a comprehensive secondary assessment to determine why his clinical status has changed. D. insert an airway adjunct if he will tolerate it and begin assisting his ventilations with a BVM.

D. insert an airway adjunct if he will tolerate it and begin assisting his ventilations with a BVM.

The amniotic fluid serves to: A. remove viruses from the fetus. B. transfer oxygen to the fetus. C. assist in fetal development. D. insulate and protect the fetus.

D. insulate and protect the fetus.

Maintaining a cushion of safety when operating an ambulance means: Select one: A. remaining in the far right-hand lane when transporting a critical patient and refraining from passing other motorists on the left side. B. driving at the posted speed limit, regardless of the patient's condition, and routinely using your lights and siren when driving on a freeway. C. driving about 2 to 3 seconds behind any vehicles in front of you and exceeding the posted speed limit by no more than 20 to 25 mph. D. keeping a safe distance between your ambulance and the vehicles in front of you and remaining aware of vehicles potentially hiding in your mirrors' blind spots.

D. keep a safe distance between your ambulance and the vehicles in front of you and remaining aware of vehicles potentially hiding in your mirrors' blind spots.

Talking about an elderly patient in front of him or her to other members of the family: Select one: A. is usually beneficial because the patient's cognitive skills are typically impaired. B. often causes the patient to become paranoid and untrusting of your help. C. will anger the patient and result in his or her refusal to accept care or transport. D. may cause the patient to think that he or she has no say in making decisions.

D. may cause the patient to think that he or she has no say in making decisions.

The topographic term used to describe the location of body parts that are closer toward the midline of the body is: Select one: A. midclavicular. B. midaxillary. C. lateral. D. medial.

D. medial.

While rescuer one is finishing his or her fifth cycle of 30 compressions, rescuer two should: Select one: A. give two breaths and prepare to start compressions. B. suction the patient's mouth and give two more ventilations. C. assess for a carotid pulse for 15 seconds. D. move to the opposite side of the patient's chest.

D. move to the opposite side of the patient's chest.

In adults, the most common cause of a single, generalized seizure without complications is: A. fever B. trauma C. infection D. none of the above

D. none of the above

Which of the following patients should have a NPA placed? A. a patient who has had difficulty breathing for the past three hours B. a patient with snoring respirations after being hit in the face with a baseball bat C. a patient with a bloody nose after a rollover motor vehicle collision (MVC) D. none of the above

D. none of the above

You are caring for a 12 y.o. who was injured at the neighborhood skate park. He was not wearing a helmet and is responsive to pain. You note snoring respirations and deformity to the left ankle. You should: A. suction the airway and insert an OPA B. secure the patient to a long board and transport C. immediately splint the injured ankle D. open the airway using a jaw-thrust maneuver

D. open the airway using a jaw-thrust maneuver

Heroin is an example of a(n): Select one: A. hypnotic. B. sympathomimetic. C. cholinergic. D. opioid.

D. opioid.

When you arrive at the scene where there is a potential for hazardous materials exposure: Select one: A. turn off your warning light. B. do not waste time waiting for the scene to be marked and protected. C. park your unit downhill of the scene. D. park your unit uphill of the scene.

D. park your unit uphill of the scene.

The type and severity of wounds sustained from incendiary and explosive devices primarily depend on the: A. type of material used to manufacture the device. B. pressure that is generated from the explosion itself. C. size of the structure that was involved in the explosion. D. patient's distance from the epicenter of the explosion.

D. patient's distance from the epicenter of the explosion. Correct

For every emergency request, the dispatcher should routinely gather and record all of the following information, EXCEPT the: Select one: A. nature of the call. B. caller's phone number. C. location of the patient(s). D. patient's medical history.

D. patient's medical history.

Online medical control requires __________. Select one: A. the presence of an advanced-level provider B. a physician's presence on the scene of the call C. written protocols approved by medical control D. phone or radio contact with the medical director

D. phone or radio contact with the medical director

Poor maintenance of home, poor personal care, and dietary neglect are all possible indicators of ____________ elder abuse. Select one: A. financial B. psychological C. emotional D. physical

D. physical

A 26-year-old female who is 34 weeks pregnant is experiencing a seizure. Her husband tells you that her blood pressure has been high and that she had been complaining of a headache for the past few days. You should: A. elevate her legs to improve brain perfusion and keep her warm. B. administer oral glucose for presumed hypoglycemia and transport. C. insert an oral airway and ventilate her with a bag-valve mask. D. place her on her side, administer high-flow oxygen, and transport.

D. place her on her side, administer high-flow oxygen, and transport.

You are called for a 24-year-old female who is 28 weeks pregnant. You find the patient supine in bed complaining of severe weakness, dizziness, and nausea. Which of the following is your first priority? A. ask the patient if she is expecting twins. B. assess the patient's blood glucose level. C. determine the patient's SAMPLE history. D. place the patient on her left side.

D. place the patient on her left side. take the weight of the uterus off the inferior vena cavae

Your 22-year-old patient is in active labor. Upon visual inspection, you note that the infant's leg is protruding from the vagina. Appropriate management of this situation includes: A. carefully attempting to push the infant's leg off of the umbilical cord. B. gently pulling on the infant's leg in an attempt to facilitate delivery. C. placing the mother in a recumbent position and rapidly transporting. D. placing the mother supine with her head down and pelvis elevated.

D. placing the mother supine with her head down and pelvis elevated.

A pregnant trauma patient may lose a significant amount of blood before showing signs of shock because: A. blood is shunted to the uterus and fetus during major trauma. B. pregnant patients can dramatically increase their heart rate. C. pregnancy causes vasodilation and a lower blood pressure. D. pregnant patients have an overall increase in blood volume.

D. pregnant patients have an overall increase in blood volume.

Because the depth of an open abdominal wound is often difficult to determine: A. the abdomen must be vigorously palpated. B. vital signs should be monitored frequently. C. the EMT must perform a thorough exam. D. prompt transport to the hospital is essential.

D. prompt transport to the hospital is essential.

Which of the following vessels carries deoxygenated blood? A. aorta B. renal artery C. pulmonary veins D. pulmonary arteries

D. pulmonary arteries only arteries that carry deoxygenated blood

Upon arriving at the scene of a motor vehicle crash, you note that two small cars collided head-on; the occupants are still in their vehicles. The fire department is in the process of stabilizing the vehicles and law enforcement personnel are directing traffic. After parking the ambulance at a safe distance, you and your partner should: Select one: A. report to the incident commander. B. quickly begin the triage process. C. assist with vehicle stabilization. D. put on high-visibility safety vests.

D. put on high-visibility safety vests.

Blood levels of medications may rise in the elderly, sometimes to toxic levels. This is MOST likely due to: Select one: A. splenic dysfunction. B. intentional overdose. C. pancreatic failure. D. renal insufficiency.

D. renal insufficiency.

The ability to take appropriate action with little direction is known as __________. Select one: A. time management B. diplomacy C. self-confidence D. self-motivation

D. self-motivation

An attack on an abortion clinic would MOST likely be carried out by a(n): A. violent religious group. B. extremist political group. C. doomsday cult. D. single-issue group.

D. single-issue group. Correct

While en route to a call for a patient in cardiac arrest, you approach a stopped school bus with its red warning lights flashing. You should: Select one: A. pass the bus only after all the children have exited. B. back up and take an alternate route to the scene. C. slowly and carefully pass the bus on the left side. D. stop and wait until the warning lights stop flashing.

D. stop and wait until the warning lights stop flashing.

The MOST serious consequence of drug or alcohol abuse among EMS personnel is: Select one: A. punitive action and the loss of a job. B. tension among coworkers and supervisors. C. low morale and frequently missed shifts. D. substandard or inappropriate patient care.

D. substandard or inappropriate patient care.

You are treating an unconscious patient with gurgling, atonal respirations. After performing a jaw-thrust to open the airway manually, your first treatment priority is to: A. begin ventilations with a BVM B. complete the initial assessment C. place a non visualized airway D. suction the oropharynx

D. suction the oropharynx

Older adults frequently ___________. Select one: A. are able to increase their muscle mass B. are unable to overcome medical problems C. experience a decrease in body fat as they age D. take multiple medications

D. take multiple medications

When assessing an 8-year-old child, you should: A. refrain from taking a blood pressure. B. rely solely on the parent for information. C. use a toe-to-head assessment approach. D. talk to the child, not just the caregiver.

D. talk to the child, not just the caregiver.

When caring for a patient who is visually impaired, it is important to: A. allow a service dog to remain with the patient at all times, even if the patient is critically ill. B. leave items such as canes and walkers at the residence if the patient will be carried on a gurney. C. stand to the side of the patient when speaking if his or her peripheral vision is impaired. D. tell him or her what is happening, identify noises, and describe the situation and surroundings

D. tell him or her what is happening, identify noises, and describe the situation and surroundings.

Your patient was stabbed with an unknown weapon in the left upper chest. First responders applied an occlusive dressing and secured it with tape on all four sides. Upon your arrival, the patient is now unconscious with bulging neck veins, tracheal deviation to the right, poor chest rise on the left and barely palpable pulses. You recognize this injury as _____________ and your first action should be to __________. A. cardiac tamponade; begin CPR B. simple pneumothorax; apply oxygen via NRB C. cardiac tamponade; begin artificial ventilation with a BVM D. tension pneumothorax; remove or loosen one side of dressing

D. tension pneumothorax; remove or loosen one side of dressing

Atherosclerosis is defined as ________. Select one: A. constriction of the blood vessels B. the buildup of plaque inside blood vessels C. dilation of the arteries D. the blockage of a coronary artery

D. the blockage of a coronary artery

The ability to handle multiple tasks based on their priority is called __________. Select one: A. patient advocacy B. patient empathy C. integrity D. time management

D. time management

If the victim of a toxicologic emergency vomits, an EMT should _________. Select one: A. have the patient enclose the vomitus in a container and take it to the emergency department. B. not approach the vomitus. C. safely dispose of the vomitus as soon as possible and decontaminate the vehicle as necessary. D. use appropriate personal protective equipment and examine the vomitus for pill fragments or other clues for patient care.

D. use appropriate personal protective equipment and examine the vomitus for pill fragments or other clues for patient care.

Your patient has been exposed to a chemical that over-stimulates the parasympathetic nervous system. You would expect this patient to exhibit: A. dilated pupils B. dry mouth C. hypertension D. vomiting

D. vomiting b/c parasympathetic does digestive system (rest and digest)

The EMT's responsibility during search-and-rescue operations is to _________. Select one: A. assist in the search on foot B. direct other incoming EMS units C. assume medical command D. wait at the staging area until the patient is located

D. wait at the staging area until the patient is located.

The scene size-up begins _____________ and is complete _____________. A. when you arrive at the patient's side; after you have considered all possible hazards B. when you arrive at the scene; when the call is complete C. when the ambulance is moving towards the call; after you have considered all possible hazards D. when the call is dispatched; when the call is complete

D. when the call is dispatched; when the call is complete

Your pediatric patient has spontaneous respirations. This means the patient is breathing: A. at a normal rate B. with adequate tidal volume C. without difficulty D. without assistance

D. without assistance

You respond to a call for "man down". On the scene, you find two police officers and a man lying down on a sidewalk. The man is not responding to vocal stimuli. As you begin your assessment, the patient beings moving spontaneously, but he doesn't seem to understand what you're telling him, and he can't answer any of your questions. You may treat this patient: A. only after obtaining consent from a family member B. only if the patient can verbalize his full consent C. after receiving permission from medical direction D. without obtaining anyone else's consent

D. without obtaining anyone else's consent

Airbags are designed to:

Decrease the severity of deceleration injuries

Age related changes in the renal system

Decreased ability to clear the waste from the body and conserve fluids when they are needed

A. right atrium.

Deoxygenated blood from the body returns to the: Select one: A. right atrium. B. right ventricle. C. left atrium. D. left ventricle.

Which of the following medications blocks the release of histamines? A. Albuterol (Ventolin) B. Acetaminophen (Tylenol) C. Diphenhydramine (Benadryl) D. Epinephrine (Adrenalin)

Diphenhydramine (Benadryl)

Which of the following situations would MOST likely disqualify a person for EMS certification? A misdemeanor at the age of 17 Driving under the influence of alcohol Possessing a valid driver's license from another state A mild hearing impairment that is corrected with hearing aids

Driving under the influence of alcohol

Following a stab wound to the left anterior chest, a 25 year old male presents with a decreased level of consciousness and signs of shock. Which of the following additional assessment findings should increase your index of suspicion for a cardiac tamponade? A. Widening pulse pressure B. A rapid, irregular pulse C. Diminished breath sounds D. Engorged jugular veins

Engorged jugular veins

Which of the following should be the EMT's highest priority? Controlling severe bleeding Maintaining a patient's airway Ensuring patient safety Ensuring personal safety

Ensuring personal safety

A 30 yr old male presents with acute shortness of breath, widespread hives, and facial swelling. He denies any past medical history and takes no medication. During your assessment you hear wheezing over all the lung fields, his BP is 90/50, heart rate 110. In addition to giving him high flow oxygen the most important treatment for this patient is

Epinephrine

The hormone responsible for the actions of the sympathetic nervous system is

Epinephrine

Three ambulances respond to a golf course where a group of six golfers were struck by lighting. Two of the golfers are conscious and alert with superficial skin burns (Group 1). The next two golfers have minor fractures and appear confused (Group 2). The last two golfers are in cardiac arrest (Group 3). According to reverse triage, which group of golfers should be treated FIRST? Group 1 Group 2 Group 3 Groups 1 and 2

Group 3 should be tagged as deceased ;Answer: C Rationale: The process of triaging multiple patients who were struck by lightning differs from standard triage; it is called "reverse triage." If the patients are alive at the scene, survival is likely. Delayed cardiac arrest following a lightning strike is uncommon. If the patients are in cardiac arrest, there is a good chance that they can be resuscitated with early, high-quality CPR and defibrillation. Therefore, Group 3 should be treated first. Three ambulances respond to a golf course where a group of six golfers were struck by lighting. Two of the golfers are conscious and alert with superficial skin burns (Group 1). The next two golfers have minor fractures and appear confused (Group 2). The last two golfers are in cardiac arrest (Group 3). According to reverse triage, which group of golfers should be treated FIRST? Group 1 Rationale: Delayed cardiac arrest following a lightening strike is uncommon. This group should not deteriorate. Group 2 Rationale: Delayed cardiac arrest following a lightening strike is uncommon. This group should not deteriorate. Three ambulances respond to a golf course where a group of six golfers were struck by lighting. Two of the golfers are conscious and alert with superficial skin burns (Group 1). The next two golfers have minor fractures and appear confused (Group 2). The last two golfers are in cardiac arrest (Group 3). According to reverse triage, which group of golfers should be treated FIRST? Group 3 Rationale: Correct answer Groups 1 and 2; Group 3 should be tagged as deceased Rationale: Group 3 has a good chance of surviving with quality CPR and defibrillation.

In late adults, the amount of air left in the lungs after expiration of the maximum amount of air

Increases which hampers the fusion of gasses because of the stagnate air that remains in the aveoli

cystitis

Inflammation of the bladder.

cholecystitis

Inflammation of the gallbladder.

pancreatitis

Inflammation of the pancreas.

peritonitis

Inflammation of the peritoneum.

Which of the following routes of medication administration has the fastest effect? Oral Intravenous Subcutaneous Intramuscular

Intravenous

Which of the following statements about the patient care report (PCR) is true? It is not a legal document in the eyes of the law. It cannot be used for patient billing information. It helps ensure efficient continuity of patient care. It is for use only by the prehospital care provider.

It helps ensure efficient continuity of patient care.

Which of the following statements regarding the vitreous humor is correct? A. It is a clear fluid that is produced by the lacrimal glands and cannot be replaced if it is lost. B. It is a clear, watery fluid surrounding the eye and can be replaced if it is lost. C. It is a clear, jellylike fluid near the back of the eye that cannot be replaced if it is lost. D. It is a clear, watery fluid that is located in front of the lens and can be replaced if it is lost.

It is a clear, jellylike fluid near the back of the eye that cannot be replaced if it is lost. Correct

Which of the following statements about helicopters is true? It is possible that the main rotor blade will dip to within 4 feet of the ground. A helicopter is considered "hot" when it is on the ground and the rotors are still. If the helicopter must land on a grade, you should approach it from the uphill side. If you must go from one side of the helicopter to the other, the best way is to duck under the body.

It is possible that the main rotor blade will dip to within 4 feet of the ground.

Which of the following assessment findings is most indicative of a cardiovascular problem

Jugular vein distension

What is the first rule of lifting? Twist slowly when you lift Keep your back in a straight position Bend at the waist to pick something up Use your arms to do most of the lifting

Keep your back in a straight position

A young female presents with costovertebral angle tenderness. She is conscious and alert with stable vital signs. Which of the following organs is MOST likely causing her pain? A. Pancreas B. Liver C. Kidney D. Gallbladder

Kidney

Which toxicity level would you assign a hazardous material spill that could cause a person temporary damage or residual injury unless prompt medical treatment is given? Level 1 Level 2 Level 3 Level 4

Level 2

A 29-year-old pregnant woman has had severe vomiting for the last 2 days. Today, she is vomiting large amounts of blood. Her skin is cool and pale and she is tachycardic. The EMT should suspect: A. acute pancreatitis. B. esophagitis. C. Mallory-Weiss tear. D. esophageal varices.

Mallory-Weiss tear.

A. resistance of the body to the flow of electricity.

Most AEDs are set up to adjust the voltage based on the impedance, which is the: Select one: A. resistance of the body to the flow of electricity. B. direction that the electrical flow takes in the body. C. actual amount of energy that the AED will deliver. D. distance between the two AED pads on the chest.

While examining a woman in labor, you see the umbilical cord protruding from the vagina. You should

Move the infants head away from the chord

Upon entering the residence of a geriatric patient with a medical or trauma complaint, the EMT should:

Observe for conditions that may make the resident unsafe

Which of the following statements regarding parenteral medications is correct? Select one: A. Tylenol is an example of a parenteral medication because it is taken orally. B. Compared to enteral medications, parenteral medications have fewer side effects. C. Parenteral medications are absorbed by the body through the digestive system. D. Parenteral medications are absorbed more quickly than enteral medications.

Parenteral medications are absorbed more quickly than enteral medications

What is the second stage of response in the stress response known as general adaptation syndrome? Recovery Exhaustion Alarm Reaction and resistance

Reaction and resistance

During a HazMat incident, you are working in the treatment area. As patients are removed from the danger zone, you should:

Remain where you are and have the patients brought to you

You are assessing a 13 month old female who is running a fever and has been vommiting. While you are performing a physical examination of this child, you will most likely find that she

Responds to her name but is fearful of your presnece

Which types of motor vehicle collisions present the greatest potential for multiple impacts? A. Rotational and rollover B. Lateral and rollover C. Frontal and rotational D. Rear-end and rotational

Rotational and rollover

When assessing a patient with an infectious disease, what is the first action you should perform? Size up the scene and take standard precautions. Obtain a SAMPLE history. Hand the patient off to a paramedic. Cover your mouth and nose with your hand.

Size up the scene and take standard precautions.

The EMTS role at the scene of a search and rescue attempt is to what

Stand by at the command post until the person is located

Which of the following statements regarding the acute abdomen is correct? A. The parietal peritoneum is typically the first abdominal layer that becomes inflamed or irritated. B. The initial pain associated with an acute abdomen tends to be vague and poorly localized. C. The most common cause of an acute abdomen is inflammation of the gallbladder and liver. D. An acute abdomen almost always occurs as the result of blunt trauma to solid abdominal organs.

The initial pain associated with an acute abdomen tends to be vague and poorly localized.

A patient with atherosclerotic heart disease experiences chest pain during exertion because

The lumen of the coronary artery is narrowed and cannot accommodate increased blood flow

Which of the following statements regarding patient weight distribution is correct? A. The majority of a horizontal patient's weight is in the torso. B. The EMT at the patient's head will bear the least amount of weight. C. A semi-sitting patient's weight is equally distributed on both ends. D. Most of the patient's weight rests on the foot end of the stretcher.

The majority of a horizontal patient's weight is in the torso.

peritoneum

The membrane lining the abdominal cavity (parietal peritoneum) and covering the abdominal organs (visceral peritoneum).

Which of the following is NOT a guideline for safe ambulance driving? Always use your siren if you have the emergency lights on. Always exercise due regard for person and property. Use one-way streets whenever possible. Go with the flow of the traffic.

Use one-way streets whenever possible.

hematemesis

Vomited blood.

B. angina.

When, for a brief period of time, heart tissues do not get enough oxygen, the pain is called: Select one: A. acute myocardial infarction. B. angina. C. ischemia. D. coronary artery disease.

B. myocardial oxygen demand exceeds supply.

Which of the following is NOT a common sign or symptom associated with malfunction of an implanted cardiac pacemaker? Select one: A. A rapid heart rate B. Syncope or dizziness C. Heart rate less than 60 beats/min D. Generalized weakness

B. Dependent edema

Which of the following signs is commonly observed in patients with right-sided heart failure? Select one: A. Labored breathing B. Dependent edema C. Pulmonary edema D. Flat jugular veins

Which of the following is correct about the secondary assessment for a high-priority patient? A. The secondary assessment must be performed prior to transport. B. Never perform a secondary assessment on a high-priority patient. C. Never contact medical direction before completing the secondary assessment. D. You may not have time to complete a secondary assessment.

You may not have time to complete a secondary assessment.

A disease that can be spread from person-to-person is known as: an infectious disease. a communicable disease. a transmittable disease. a spreadable disease.

a communicable disease.

emphysema

a condition in which the air sacs of the lungs are damaged and enlarged, causing breathlessness. causes shortness of breath (dyspnea)

symphysis

a joint with limited motion

A sudden onset of difficulty breathing, sharp chest pain, and cyanosis that persists despite supplemental oxygen is MOST consistent with: severe pneumonia. myocardial infarction. a pulmonary embolism. a spontaneous pneumothorax.

a pulmonary embolism.

At the site of the injury, signs and symptoms of an animal or insect bite or sting may include pain, localized heat, and a wheal, which appears as __________. A. a series of small, raised pustules B. a raised, well-defined area of the skin C. a large area of red, blotchy skin D. an uncomfortable depression in the skin

a raised, well-defined area of the skin

A flail chest occurs when: A. a segment of the chest wall is detached from the thoracic cage. B. a segment of fractured ribs bulges during the inhalation phase. C. more than three ribs are fractured on the same side of the chest. D. multiple ribs are fractured on both sides of the thoracic cage.

a segment of the chest wall is detached from the thoracic cage.

What type of dressing should be applied to a sucking chest wound?

a three-sided occlusive dressing

An advance directive is: a set of specific guidelines that clearly defines the different types of consent. a formal list that defines by state law whether a patient has decision-making capacity. a written document that specifies the care you should provide if the patient is unable to make decisions. a verbal order given to you by a dying patient's family regarding whether treatment should be provided.

a written document that specifies the care you should provide if the patient is unable to make decisions.

You are dispatched to a state park for a young female experiencing an allergic reaction. Your assessment reveals that her breathing is severely labored and her blood pressure is very low. You carry epinephrine auto-injectors on your ambulance and have been trained and approved by your medical director to administer them. As your partner gives the patient supplemental oxygen, you attempt to contact medical control but do not have a signal from your cell phone. You should: Select one: A. immediately load the patient into the ambulance, begin transport, and reattempt to contact medical control when you receive a cell signal. B. notify dispatch and request that a paramedic unit respond to the scene so they can administer epinephrine to the patient. C. place her in a supine position, keep her warm, begin transport to the hospital, and request a paramedic intercept en route. D. administer epinephrine to the patient, begin immediate transport, and attempt to contact medical control en route to the hospital.

administer epinephrine to the patient, begin immediate transport, and attempt to contact medical control en route to the hospital.

A 59-year-old male presents with sudden-onset severe lower back pain. He is conscious and alert, but very restless and diaphoretic. Your assessment reveals a pulsating mass to the left of his umbilicus. You should: A. administer oxygen and prepare for immediate transport. B. vigorously palpate the abdomen to establish pain severity. C. request a paramedic unit to give the patient pain medication. D. place the patient in a sitting position and transport at once.

administer oxygen and prepare for immediate transport.

Vector-borne transmission of an infectious organism occurs via:

animals or insects

A 19-year-old female was stung multiple times on the legs by fire ants. She states that she is allergic to fire ants, but does not carry her own epinephrine. The patient is conscious and alert and complains of pain to the area of the bites. Her blood pressure is 122/70 mm Hg, her pulse is 100 beats/min and strong, and her respirations are 18 breaths/min and unlabored. You should: A. position her legs well above the level of her heart. B. advise her to see her physician as soon as possible. C. administer oxygen and transport her to the hospital. D. request a paramedic unit to administer epinephrine.

administer oxygen and transport her to the hospital

In addition to severe bleeding, the MOST life-threatening complication associated with an open neck injury is: A. an air embolism. B. nerve fiber damage. C. a spinal fracture. D. an ischemic stroke.

an air embolism. Correct

gastroenteritis

an infection with associated diarrhea, nausea, and vomiting usually due to contaminated food or water common cause of shock in children

approaching vehicles

approach from the sides, not the front

A young male sustained a gunshot wound to the abdomen during an altercation. As your partner is assessing and managing his airway, you should control the obvious bleeding and then: A. assess for an exit wound. B. obtain baseline vital signs. C. auscultate bowel sounds. D. apply a cervical collar.

assess for an exit wound

third stage of labor

begins once baby is delivered and ends with delivery of the placenta

When caring for a patient whose arm is covered with a dry chemical, you should: A. use forceful streams of water to remove the chemical. B. brush away the chemical before flushing with water. C. deactivate the chemical with a 5% vinegar solution. D. quickly irrigate the arm with large amounts of water.

brush away the chemical before flushing with water. Correct

Placards are used on: buildings. individual packages. storage lockers. storage papers.

buildings.

atherosclerosis

buildup of plaque in a blood vessel

The three major parts of the brain are the: A. cerebrum, cerebellum, and brain stem. B. cerebellum, medulla, and occiput. C. brain stem, midbrain, and spinal cord. D. midbrain, cerebellum, and spinal cord.

cerebrum, cerebellum, and brain stem

The largest part of the brain is the: cerebrum. brain stem. cerebellum. foramen magnum.

cerebrum.

spinal cord

cervical --> C1-C7 thoracic --> T1-T12 lumbar --> L1-L5 sacrum --> 5 fused vertebrae coccyx --> 4 fused vertebrae

All of the following are factors that increase the risk for developing MRSA, EXCEPT: antibiotic therapy. prolonged hospital stays. exposure to an infected patient. close contact with wild birds.

close contact with wild birds.

Prefixes can indicate: color. conditions. body parts. procedures.

color.

When applied to a patient, a metered-dose inhaler will: deliver the same dose each time it is used. be ineffective when given to patients with asthma. deliver a different dose each time it is used. Deliver the drug to the lungs over a period of 6 to 8 hours.

deliver the same dose each time it is used.

hypothermia

develops when core temp falls below that needed to maintain homeostasis signs/symptoms: pale or cyanosis skin, shivering, loss of coordination, altered LOC, vitals (bradycardia, bradypnea, hypotension)

trench-foot

develops when feet have prolonged exposure to cold water

The thoracic cavity is separated from the abdominal cavity by the: A. anterior rib cage. B. intercostal margin. C. costovertebral angle. D. diaphragm.

diaphram

orthopnea

difficulty breathing lying down

conduction (body losing heat)

direct transfer of heat though contact with a colder structure ex. bare feet on a cold floor

intramuscular route

directly into muscle rapid absorption, not as fast as intravenous or intraosseus Ex. EpiPen

A colleys fracture involves a fracture of the

distal radius

frontal plane

divides body anterior and posterior

sucking chest wound / open pneumothorax

during inhalation, the hole in the chest will draw air into the pleural space cover hole with a 3-sided occlusive dressing to prevent air from entering the chest cavity

Anaphylaxis is MOST accurately defined as a(n): A. moderate allergic reaction that primarily affects the vasculature. B. severe allergic reaction that typically resolves without treatment. C. allergic reaction that causes bronchodilation and vasoconstriction. D. extreme allergic reaction that may affect multiple body systems.

extreme allergic reaction that may affect multiple body systems

irreversible shock

final stage of shock patient will die

red diamond on Hazmat sign

fire hazard

cardiac tamponade (obstructive shock)

fluid accumulates within the pericardial sac and compresses the heart signs/symptoms: jugular vein distention, narrowing pulse pressures, hypotension

cardiac/pericardial tamponade

fluid in the pericardium (the sac around the heart) builds up and results in compression of the heart

supplemental oxygen

goal is to maintain a pulse oximetry of > 94%

According to the American College of Surgeons Committee on Trauma (ACS-COT), an adult trauma patient should be transported to the highest level of trauma center if he or she: A. has a bleeding disorder or takes anticoagulant medications and has any blunt or penetrating injury. B. has a systolic blood pressure of less than 110 mm Hg or a heart rate greater than 110 beats/min. C. was involved in a motor vehicle crash in which another patient in the same vehicle was killed. D. has a GCS score of less than or equal to 13 with a mechanism attributed to trauma.

has a GCS score of less than or equal to 13 with a mechanism attributed to trauma

respiration (body losing heat)

in a cold environment, exhaled air has been warmed by the body

constriction of blood vessels

increases SVR and causes increase in BP

jaw-thrust maneuver

indication: patient with AMS and suspected c-spine injury CONTRAINDICATION: conscious patients

Beck's triad

indicative of cardiac tamponade JVD, muffled heart sounds, narrowing pulse pressure

rigid suction catheter

known as "tonsil tip" or Yankauer best suited for suctioning the oral airway

cerebrum

largest part of brain controls thought, memory, and the senses

dead/dying patients

last patient priority black tagged

decompensated shock

late or "progressive" shock body cannot compensate for the hypovolemic state and blood pressure falls

Approximately 25% of severe injuries to the aorta occur during: A. rollover collisions. B. rear-end collisions. C. frontal collisions. D. lateral collisions.

lateral collisions

When pulling a patient you should do all of the following except

lean forward another 15-20 inches

Quality control in an EMS system is the ultimate responsibility of the: paramedic. lead EMT. medical director. EMS administrator.

medical director.

Your patient is complaining of fever, headache, stiffness of the neck, and red blotches on his skin. He most likely has: tuberculosis. hepatitis B. SARS. meningitis.

meningitis.

The best indicator of brain function is the patients

mental status

Proper removal of a critically injured patient from an automobile involves: moving the patient in one fast, continuous step. utilizing no more than two personnel to avoid crowding. moving the patient in smooth, slow, controlled steps. removing the patient by grasping the immobilization device.

moving the patient in smooth, slow, controlled steps.

online orders

must call med control

If a patient with a chest injury is only able to inhale small amounts of air per breath, he or she: A. will eliminate more carbon dioxide than if he or she were breathing deeply. B. will maintain adequate minute volume if his or her respiratory rate stays the same. C. must increase his or her respiratory rate to maintain adequate minute volume. D. often breathes at a slower rate because of lung damage caused by the injury.

must increase his or her respiratory rate to maintain adequate minute volume.

basal skull fracture

occur at base of skull CSF may leak from nose or ears signs: battle's signs (bruising behind ears) and raccoon eyes (bruising under eyes)

diabetic ketoacidosis

occurs more frequently with type I diabetes blood glucose levels >350 mg/dL brain cells are able to use the glucose, but the rest of the body's cells are starving

flail chest

occurs when a portion of the thorax becomes separated from the rest of the thorax caused by fracture of at least 2 consecutive ribs patient may exhibit paradoxical motion of the chest

prolapsed cord

occurs when the cord is the presenting part in the birth canal

ectopic pregnancy

occurs when the egg is implanted outside the uterus, usually in the fallopian tube presents as severe abdominal pain with/out vaginal bleeding

To protect a restrained patient and prevent him from using leverage to break free, the EMT should secure: A. both arms above the head B. only the patient's torso C. one arm above the head D. both arms at the patient's sides

one arm above the head.

When assessing an elderly patient who fell, it is important to remember that: A. any fall in the elderly is considered to be high-energy trauma. B. elderly patients who fall usually have a secondary head injury. C. osteoporosis can cause a fracture as a result of a fall from a standing position. D. bilateral hip fractures usually occur when an elderly person falls.

osteoporosis can cause a fracture as a result of a fall from a standing position

In ___________ administration, you are administering medication to yourself or your partner. Select one: A. patient-assisted B. peer-assisted C. EMT-administered D. paramedic-administered

peer-assisted

negligence

person must prove EMT has duty to act, breech of duty, damage, and causation

absence seizures

petit mal seizures patient doesn't interact with environment, but there is no convulsive activity

latissimus dorsi

posterior chest muscle

A 30-year-old woman with a history of alcoholism presents with severe upper abdominal pain and is vomiting large amounts of bright red blood. Her skin is cool, pale, and clammy; her heart rate is 120 beats/min and weak; and her blood pressure is 70/50 mm Hg. Your MOST immediate action should be to: A. rapidly transport her to the hospital. B. keep her supine and keep her warm. C. protect her airway from aspiration. D. give her high-flow supplemental oxygen.

protect her airway from aspiration.

narrow blood pressure

pulse pressure below 25% of systolic indicates possible hypoperfusion, tension pneumothorax, pericardial tamponade example: 80/62

three primary causes of shock

pump (heart) problems: myocardial infarction, cardiac trauma pipe (blood vessel) problems: anaphylaxis, spinal trauma, infection fluid (blood volume) problems: bleeding, vomiting, diarrhea

obstructive shock

pump problem caused by mechanical obstruction of the heart muscle

cardiogenic shock

pump problem heart muscle cannot pump effectively, causing a backup of fluid, pulmonary edema, and hypotension signs/symptoms: hypotension, probably cardiac history, chest pain, respiratory distress, pulmonary edema, and AMS

penetrating wound (open injury)

puncture wound

Proper guidelines for correct reaching include all of the following, EXCEPT: avoiding twisting your back. avoiding hyperextension of your back. keeping the back in a locked-in position. reaching no more than 30 inches in front of your body.

reaching no more than 30 inches in front of your body.

A 31-year-old female is experiencing an acute asthma attack. She is conscious and alert, but in obvious respiratory distress. After assisting her with her prescribed MDI, you should: Select one: A. administer another treatment in 30 seconds if she is still in distress. B. reassess the patient and document her response to the medication. C. check the drug's expiration date to ensure that it is still current. D. contact medical control and apprise him or her of what you did.

reassess the patient and document her response to the medication

After administering a nitroglycerin tablet to a patient, the EMT should: check the expiration date of the nitroglycerin. reassess the patient's blood pressure within 5 minutes. instruct the patient to chew the tablet until it is dissolved. ensure that the nitroglycerin is prescribed to the patient.

reassess the patient's blood pressure within 5 minutes.

You are caring for a 68-year-old man with sudden onset of left-sided paralysis and slurred speech. His airway is patent, his respirations are 14 breaths/min with adequate depth, and his oxygen saturation is 98%. Treatment for this patient should include: A. oral glucose gel and transport. B. ventilatory assistance and transport. C. recovery position and transport. D. high-flow oxygen and transport.

recovery position and transport

insulin shock

referes to severe hypoglycemia

A device that receives a low-frequency signal and then transmits it at a relatively higher frequency is called a: duplex. scanner. repeater. receiver.

repeater.

A 56-year-old male has an incomplete avulsion to his right forearm. After controlling any bleeding from the wound, you should: A. carefully probe the wound to determine if the bleeding is venous or arterial. B. replace the avulsed flap to its original position and cover it with a sterile dressing. C. carefully remove the avulsed flap and wrap it in a moist, sterile trauma dressing. D. thoroughly irrigate the wound with sterile water and cover it with a sterile dressing.

replace the avulsed flap to its original position and cover it with a sterile dressing. Correct

An 8-year-old male was bitten by a stray dog. He has a large laceration to the back of his left hand, which your partner covered with a sterile dressing and bandage. In addition to transporting the child to the hospital, you should: A. report the incident to the appropriate authorities. B. administer oxygen via a nonrebreathing mask. C. advise the child that he will need rabies shots. D. ask the child's father to try to locate the dog.

report the incident to the appropriate authorities. Correct

You and your partner are ventilating an apneic adult when you notice that his stomach is becoming distended. You should: suction his airway for up to 15 seconds. reposition his head. increase the rate and volume of your ventilations. decrease your ventilation rate but use more volume.

reposition his head.

Quid pro quo, a type of sexual harassment, occurs when the harasser:

requests sexual favors in exchange for something else

You are attending to a 22-year-old female patient who has overdosed. The patient is unresponsive in an upstairs bedroom. The most appropriate way to bring the patient downstairs is: A. secured to a stair chair with the strongest provider at the head end. B. secured to a stair chair with the strongest provider at the foot end. C. secured to a fracture board with the strongest provider at the head end. D. secured to a fracture board with the strongest provider at the foot end.

secured to a fracture board with the strongest provider at the head end.

glucagon

serves to increase blood glucose levels

A 70-year-old man recently had a heart attack and now complains of severe difficulty breathing, especially when lying flat. He is coughing up pink, frothy secretions. This patient is MOST likely experiencing: acute right heart failure. severe left heart failure. an acute onset of bronchitis. an acute pulmonary embolism.

severe left heart failure.

Signs of adequate breathing in the adult include all of the following, EXCEPT: pink, warm, dry skin. shallow chest rise. symmetrical chest movement. a respiratory rate of 16 breaths/min.

shallow chest rise.

agonal breaths

shallow, ineffective gasps

During your assessment of a patient who was shot in the abdomen, you notice a large entrance wound with multiple small puncture wounds surrounding it. This wound pattern is MOST consistent with a: A. handgun. B. .357 magnum. C. shotgun. D. .22-caliber pistol.

shotgun. Correct

pertinent negatives

signs or symptoms you have reason to suspect but the patient denies having

simple face mask

similar to a NRB but without an oxygen reservoir flow rate: 6-10 LPM delivers 40-60% oxygen

automatic implanted cardiac defibrillators (AICD)

similar to an AED, but is placed under the skin and is connected directly to the heart

You place a patient in the semi-Fowler's position for transport. This means the patient is: lying on his or her back. lying on his or her stomach. sitting at a 45-degree angle. sitting at a 90-degree angle.

sitting at a 45-degree angle.

When assessing a patient who was exposed to a vesicant agent, you should expect to encounter: skin blistering. loss of hearing. vomiting blood. profound bradycardia.

skin blistering.

bradypnea

slow breathing

The five hazards most commonly associated with a structural fire are: smoke, oxygen deficiency, high ambient temperatures, toxic gases, and building collapse. smoke, oxygen deficiency, inhalation of tar particles, injury from breaking glass, and building collapse. smoke, high ambient temperatures, toxic gases, electric shock, and inhalation of tar particles. oxygen deficiency, high ambient temperatures, toxic gases, electric shock, and injury from breaking glass.

smoke, oxygen deficiency, high ambient temperatures, toxic gases, and building collapse.

A patient with neurogenic shock would be LEAST likely to present with: tachypnea. hypotension. tachycardia. altered mentation.

tachycardia.

Elevation of the ribcage during inhalation occurs when

the intercostal muscles contract

spontaneous breathing

the movement of gas in and out of the lungs that is produced in response to an individual's respiratory muscles

Kehr's sign

the occurrence of acute pain in the tip of the shoulder due to the presence of blood or other irritants in the peritoneal cavity when a person is lying down and the legs are elevated pain in left shoulder is indicative of ruptured spleen

Which of the following symptoms would lead the EMT to believe that a patient's headache is caused by sinus congestion? A. The pain is worse when bending over B. There is associated neck stiffness C. The headache began suddenly D. There is numbness in the extremities

the pain is worse when bending over

tension pneumothorax

the progressive build-up of air within the pleural space lungs sounds are absent over the affected area tracheal deviation towards the *unaffected* side

osteoporosis

the progressive loss of bone density over time

During the scene size-up, you should routinely determine all of the following, EXCEPT: the mechanism of injury or nature of illness. the ratio of pediatric patients to adult patients. whether or not additional resources are needed. if there are any hazards that will jeopardize safety.

the ratio of pediatric patients to adult patients.

The left cerebral hemisphere controls: A. the right side of the face. B. heart rate and pupil reaction. C. breathing and blood pressure. D. the right side of the body.

the right side of the body

The MOST prominent landmark on the anterior surface of the neck is the: mastoid process. cricoid cartilage. thyroid cartilage. cricothyroid membrane.

thyroid cartilage

Factors that have the GREATEST impact on the severity of radiation exposure include: age and overall health. gender and wind speed. the method of dispersal. time, distance, and shielding.

time, distance, and shielding.

frostbite

tissue is frozen, which frequently leads to permanent damage can lead to gangrene (tissue death)

anatomical differences in children

tongue is larger in proportion to airway airway is more easily obstructed head is larger in proportion to body

stratum corneal layer of epidermis

top epidermal layer and consists of dead skill cells

lower airway

trachea carina bronchi bronchioles alveoli

During insertion of an oropharyngeal airway into an unconscious patient, she begins to vomit. The first thing you should do is: turn the patient on her side. remove the airway at once. suction the patient's mouth. use a smaller-sized oral airway.

turn the patient on her side.

Your 12-year-old patient can speak only two or three words without pausing to take a breath. He has a serious breathing problem known as: nasal flaring. two- to three-word dyspnea. labored breathing. shallow respirations.

two- to three-word dyspnea.

When ventilating an apneic adult with a bag-valve mask, you should squeeze the bag: until it is empty. over a period of 2 seconds. at a rate of 20 breaths/min. until visible chest rise is noted.

until visible chest rise is noted.

The goal of invaders such as bacteria and viruses is to __________. A. cause an immune response B. kill their human host C. spread disease and destruction D. use a human body as a home

use a human body as a home

To facilitate a safe and coordinated move, the team leader should: A. be positioned at the feet so the team can hear. B. never become involved in the move, only direct the move. C. use preparatory commands to initiate any moves. D. speak softly but clearly to avoid startling the patient.

use preparatory commands to initiate any moves.

continuous positive airway pressure (CPAP)

used to improve ventilatory efficiency in spontaneously breathing patients in respiratory distress

When you are obtaining medical history from the family of a suspected stroke patient, it is MOST important to determine: A. when the patient last appeared normal. B. if there is a family history of a stroke. C. if the patient has been hospitalized before. D. the patient's overall medication compliance.

when the patient last appeared normal


Conjuntos de estudio relacionados

TLE 9: MATERIALS, TOOLS, EQUIPMENT and TESTING DEVICES

View Set

BYUI GEOG 120 Exam 2 (Europe-Oceania)

View Set

Additional Peds Questions (Exam 1)

View Set

The Concept Of Acceptance And Empathy In The Social Worker-Client/Client System Relationship

View Set

Chapter 29: Management of Patients with Nonmalignant Hematologic Disorders

View Set

acc exam 3 concept questions ch.10

View Set

Το εσωτερικό ενός Η/Υ

View Set

A&P I Chapters 12, and 13 Lab - Nervous System

View Set